Вы находитесь на странице: 1из 391

Page 1 of 391

ANSWERS TO BAR
EXAMINATION QUESTIONS
IN
CIVIL LAW
ARRANGED BY TOPIC



(1975 2004)





Edited and Arranged by:


Janette Laggui-Icao and
Alex Andrew P. Icao
(Silliman University College of Law)


From the ANSWERS TO BAR EXAMINATION QUESTIONS
by the UP LAW COMPLEX


July 26, 2005


Page 2 of 391




This work is not intended for sale or commerce. This work is
freeware. It may be freely copied and distributed. It is primarily intended
for all those who desire to have a deeper understanding of the issues
touched by the Philippine Bar Examinations and its trend. It is specially
intended for law students from the provinces who, very often, are
recipients of deliberately distorted notes from other unscrupulous law
schools and students. Share to others this work and you will be richly
rewarded by God in heaven. It is also very good karma.
We would like to seek the indulgence of the reader for some Bar
Questions which are improperly classified under a topic and for some
topics which are improperly or ignorantly phrased, for the authors are just
Bar Reviewees who have prepared this work while reviewing for the Bar
Exams under time constraints and within their limited knowledge of the
law. We would like to seek the readers indulgence for a lot of
typographical errors in this work.

The Authors
July 26, 2005

Page 3 of 391



Table of Contents



Persons .......................................................................................................... 11
Conflict of laws............................................................................................ 25
Adoption......................................................................................................... 52
Family Code.................................................................................................. 58
Succession.................................................................................................. 112
Donations ................................................................................................... 157
Modes of Acquiring Ownership ........................................................... 167
Property ....................................................................................................... 169
LTD................................................................................................................. 211
Prescription................................................................................................. 245
Contracts .................................................................................................... 250
Estoppel ....................................................................................................... 263
Natural obligations ................................................................................... 265
Obligations ................................................................................................. 265
Trusts ............................................................................................................ 297
Sales ............................................................................................................. 301
Lease............................................................................................................. 324
Common carriers ...................................................................................... 335
Agency.......................................................................................................... 339
Partnership.................................................................................................. 346
Commodatum & Mutuum....................................................................... 352
Deposit.......................................................................................................... 356
Surety............................................................................................................ 358
Antichresis
Mortgage
Pledge ........................................................................................................... 363
Quasi-contracts ......................................................................................... 367
Torts and damages................................................................................... 372
Aleatory contracts; gambling............................................................... 390
Compromises and arbitration............................................................... 390



Page 4 of 391

Detailed Table of Contents


Persons .......................................................................................................... 11
civil register................................................................................................... 12
conclusive presumption of knowledge of the law.......................................... 13
decisions of inferior courts ............................................................................ 14
duty of judges to render judgment ................................................................ 14
effect of obiter and dissenting opinion; decisions of the SC.......................... 14
effectivity of laws........................................................................................... 14
emancipation ................................................................................................ 15
equity ............................................................................................................16
human relations ............................................................................................ 16
ignorance of the law vs mistake of fact ........................................................ 17
independent civil action ................................................................................ 17
juridical capacity vs capacity to act ............................................................... 18
natural persons............................................................................................. 19
prejudicial question....................................................................................... 19
presumption of simultaneous death.............................................................. 20
prospectivity of laws; exceptions................................................................... 22
restrictions on capacity to act........................................................................ 22
use of surnames ........................................................................................... 23
waiver ........................................................................................................... 23
Conflict of laws............................................................................................ 25
Art. 15, 16 & 17 ............................................................................................ 25
cognovit; borrowing statute; characterization................................................ 32
effect of divorce granted to former Filipinos.................................................. 32
effect of divorce secured abroad................................................................... 33
forum non conveniens & lex loci contractus & long arm statute.................... 34
laws governing contracts .............................................................................. 36
laws governing contracts of carriage ............................................................ 36
laws governing divorce ................................................................................. 37
laws governing marriages............................................................................. 38
laws governing real and personal property ................................................... 40
laws governing succession ........................................................................... 40
laws governing wills ...................................................................................... 43
nationality theory........................................................................................... 44
naturalization ................................................................................................ 45
operation of foreign laws upon resident ........................................................46
processual presumption................................................................................ 47
renvoi doctrine .............................................................................................. 47
significant relationships theory...................................................................... 49
theory of effective nationality ........................................................................ 49
torts; prescriptive period ............................................................................... 49
Adoption......................................................................................................... 52
qualifications of adopter................................................................................ 52
successional rights of adopted child ............................................................. 55
who are considered as natural parents......................................................... 56
Family Code.................................................................................................. 58
Annulment
effects; requisites before remarriage ..................................................... 59
fraud ......................................................................................................60
grounds..................................................................................................62
judicial declaration................................................................................. 62
Page 5 of 391
Legal Separation; prescription of actions............................................... 63
proper party ........................................................................................... 63
psychological incapacity ........................................................................ 64
art. 26 ........................................................................................................... 64
declaration of nullity: annulment: legal separation: separation of property
grounds......................................................................................................... 66
divorce; void marriages................................................................................. 67
donations by reason of marriage; effect of declaration of nullity ................... 67
effect of divorce; void marriages...................................................................68
emancipation ................................................................................................ 69
family ............................................................................................................ 69
family home .................................................................................................. 70
legal separation ............................................................................................ 71
grounds..................................................................................................71
proper party ........................................................................................... 72
nullity; annulment; legal separation; grounds................................................ 72
parental authority.......................................................................................... 73
paternity and filiation..................................................................................... 75
presumptive legitime..................................................................................... 80
property regime of unions without marriage.................................................. 81
property relations.......................................................................................... 83
charges.................................................................................................. 91
conjugal partnership pf gains................................................................. 92
dissolution of partnership; custody of children....................................... 93
donations between spouses .................................................................. 94
marriage settlements ............................................................................. 94
separation of property............................................................................ 96
requisites of marriage ................................................................................... 97
retroactive application; vested rights........................................................... 103
rights and obligations of husband and wife................................................. 103
special parental authority; liability of teachers............................................. 105
support........................................................................................................ 105
void marriages ............................................................................................106
legal personality to seek declaration of nullity...................................... 109
property regime ................................................................................... 110
psychological incpacity ........................................................................ 110
Succession
acceptance, repudiation, collation............................................................... 112
amount of successional rights..................................................................... 112
barrier between illegitimate and legitimate relatives ................................... 112
capacity to inherit; conditional devise ......................................................... 113
collation ...................................................................................................... 114
disinheritance.............................................................................................. 115
donation of a spouses share...................................................................... 120
incapacity.................................................................................................... 121
intestate heirs ............................................................................................. 121
intestate proceedings; jurisdiction............................................................... 123
intestate succession ................................................................................... 123
joint wills ..................................................................................................... 136
legal separation; effect................................................................................ 136
legitime ....................................................................................................... 136
order of succession and sharing; right of representation, institution, accretion
.................................................................................................................... 138
partition....................................................................................................... 139
preterition.................................................................................................... 139
probate ....................................................................................................... 141
renunciation; compromise........................................................................... 144
Page 6 of 391
representation............................................................................................. 144
reserva troncal ............................................................................................ 145
right of representation................................................................................. 148
testate succession ...................................................................................... 148
transmission of rights to succession ........................................................... 149
when death takes place; pesumptive legitime............................................. 150
wills.............................................................................................................151
codicil................................................................................................... 151
formalities ............................................................................................ 151
revocation............................................................................................ 154
testamentary intent .............................................................................. 155
witnesses to holographic wills.............................................................. 156
Donations
Donation & Sales ownership of the thing donated ...................................... 157
Annulment................................................................................................... 157
donations mortis causa............................................................................... 160
donees civil personality.............................................................................. 161
effect of illegal and immoral conditions ....................................................... 161
kinds ...........................................................................................................162
mortis causa; formalities ............................................................................. 163
requisited for perfection .............................................................................. 163
revocation ................................................................................................... 164
valid acceptance......................................................................................... 165
Modes of Acquiring Ownership ........................................................... 167
Property
accession.................................................................................................... 169
chattel mortgage over immovables............................................................. 173
co-ownership .............................................................................................. 173
easements .................................................................................................. 180
good faith/ bad faith .................................................................................... 189
hidden treasure........................................................................................... 196
immovable properties; chattel mortgage over immovables......................... 198
nuisance ..................................................................................................... 199
possession vs occupation........................................................................... 201
possession; 559.......................................................................................... 201
possession; squatting ................................................................................. 204
real vs personal property ............................................................................ 205
rights of a property owner; limitations ......................................................... 207
usufruct ....................................................................................................... 207
LTD................................................................................................................. 211
acquisition of lands; citizenship requirement .............................................. 211
Act 3344 ..................................................................................................... 212
amendment of entries in a certificate of title ............................................... 212
annotation of lis pendens............................................................................ 213
annotations; classification........................................................................... 213
foreshore lands........................................................................................... 214
homestead.................................................................................................. 214
increase of area by accession .................................................................... 217
indefeasibility of title.................................................................................... 217
IPV
adverse claims ...................................................................................... 217
buyer in good faith; laches..................................................................... 218
collateral attack ..................................................................................... 218
constructive trust ................................................................................... 219
effect of entry in day book; acquisitive prescription; laches................... 219
effect of laches ...................................................................................... 220
Page 7 of 391
foreclosure of registered lands; prescription.......................................... 220
forged deed as a root or title.................................................................. 221
forgery; innocent purchaser for value.................................................... 223
fraud in the procurement of patent ........................................................ 224
free patents; jurisdiction of Director of Lands ........................................ 225
government lands.................................................................................. 225
innocent purchaser for value ................................................................. 226
lis pendens ............................................................................................ 227
mirror principle....................................................................................... 227
notice of lis pendens; transferee pendente lite ...................................... 228
reconveyance........................................................................................ 229
registration as the operative act of conveying the land.......................... 230
remedies; prescriptive period ................................................................ 230
remedies; reconveyance ....................................................................... 231
reopening of a decree of registration..................................................... 232
sale of unregistered lands ..................................................................... 236
unregistered land................................................................................... 236
use of fraud in aplication of title; prescriptive period for reconveyance . 237
judicial confirmation of imperfect title.......................................................... 238
judicial reconstitution of title........................................................................ 239
ministerial duty of the Register of deeds to register .................................... 240
procedure; consulta .................................................................................... 241
reclamation of foreshore lands.................................................................... 241
redemption.................................................................................................. 242
scope of registration ................................................................................... 242
torrens vs recording.................................................................................... 243
writ of possession ....................................................................................... 243
Prescription................................................................................................. 245
acquisitive; movable ................................................................................... 246
just title ....................................................................................................... 246
laches ......................................................................................................... 247
real rights.................................................................................................... 248
Contracts
annulment of contracts; capacity to sue...................................................... 250
annulment of contracts; prescriptive period ................................................ 250
consensual vs real contracts....................................................................... 251
consent; invitation to bid ............................................................................. 251
consideration; validity ................................................................................. 252
form of contracts......................................................................................... 252
inducing another to violate a contract ......................................................... 253
innominate contracts................................................................................... 253
obligatory nature of contracts...................................................................... 254
perfection of contracts; obligations with a period ........................................ 255
privity of contract......................................................................................... 255
reformation of instruments .......................................................................... 256
rescission of contracts; capacity to sue ...................................................... 256
Statute of Frauds ........................................................................................ 257
stipulation pour atrui ................................................................................... 259
void contracts; in pari delicto principle ........................................................ 260
void contracts; usurious interests................................................................ 260
void vs voidable contracts........................................................................... 261
void/unenforceable contracts...................................................................... 261
voidable contracts....................................................................................... 261
Estoppel ....................................................................................................... 263
Natural obligations ................................................................................... 265
Obligations
Page 8 of 391
alternative/ facultative obligations............................................................... 265
alternative/facultative obligations................................................................ 265
civil vs natural oblig..................................................................................... 266
conditional obligations ................................................................................ 267
exemption from liability due to fortuitous events; exceptions ...................... 270
extinguishment of a cause of action............................................................ 270
extinguishment
assignment of rights ............................................................................ 270
compensation ...................................................................................... 271
compensation vs payment ............................................................ 272
compensation: payment: confusion: set-off .................................. 273
condonation......................................................................................... 273
dation................................................................................................... 274
dation in payment vs assignment ................................................. 275
extraordinary inflation or deflation........................................................ 275
loss ......................................................................................................276
loss; impossible service................................................................ 277
novation............................................................................................... 277
payment............................................................................................... 281
consignation, set-off ..................................................................... 283
application .................................................................................... 284
consignation, when applicable...................................................... 284
fortuitous events ......................................................................................... 285
joint/ solidary liability................................................................................... 286
loss of the thing due.................................................................................... 290
loss of the thing due; force majeure............................................................ 290
nature and effect of obligations................................................................... 291
obligation to deliver a determinate thing; effect of loss ............................... 291
obligation to deliver a generic thing ............................................................ 292
obligation to give; obligation to do............................................................... 293
obligations with a penal clause ................................................................... 294
obligations with a period ............................................................................. 295
suspensive period dependent upon will of debtor................................ 297
Trusts ............................................................................................................ 297
Sales
Sales & Donation; ownership of the thing sold............................................ 301
Art. 1592..................................................................................................... 301
assignment of credit.................................................................................... 302
conditional sale vs absolute sale ................................................................ 302
contract of sale vs agency to sell ................................................................ 303
contract of sale vs contract to sell ............................................................... 303
contract to sell............................................................................................. 303
contract to sell vs contract of sale............................................................... 304
double sales................................................................................................ 304
effect of oral sale ........................................................................................ 306
equitable mortgage..................................................................................... 307
Maceda law................................................................................................. 310
option contract ............................................................................................ 312
option; earnest money; Art. 1592................................................................ 312
pacto de retro; when not ............................................................................. 313
perfected sale ............................................................................................. 314
Recto law.................................................................................................... 315
redemption (conventional and legal)........................................................... 317
redemption; legal; by co-owners.......................................................... 318
right of first refusal ...................................................................................... 319
right of repurchase...................................................................................... 320
tradition....................................................................................................... 320
Page 9 of 391
transfer of ownership .................................................................................. 321
vendors lien................................................................................................ 322
who bears risk of loss ................................................................................. 322
Lease............................................................................................................. 324
Lease of urban lands .................................................................................. 325
contract for a piece of work......................................................................... 326
contract for a piece of work; liability of architects/contractors................ 326
contract of labor .......................................................................................... 327
effect of death of lesee ............................................................................... 327
ejectment .................................................................................................... 327
extinguishment............................................................................................ 328
implied new lease ....................................................................................... 328
lease of rural lands ..................................................................................... 329
lease with a term......................................................................................... 329
liability for hidden defects ........................................................................... 330
option to buy ............................................................................................... 331
reduction of rent.......................................................................................... 331
rights and obligations of leasee and lessor ................................................. 332
sublease ..................................................................................................... 332
sublease; delay in payment of rentals ................................................... 333
Common carriers ...................................................................................... 335
Agency.......................................................................................................... 339
agency vs sale............................................................................................ 339
appointment of sub-agent ........................................................................... 339
authority to sell does not include authority to collect................................... 340
commission................................................................................................. 340
coupled with an interest .............................................................................. 340
general vs special agency .......................................................................... 342
liability of an agent ...................................................................................... 342
to render an account ............................................................................. 343
powers of the agent .................................................................................... 343
termination; death or principal: double sales .............................................. 344
termination; effect of death of agent ........................................................... 344
Partnership.................................................................................................. 346
conveyance of a partners share dissolution............................................... 346
dissolution................................................................................................... 346
effect of death of partner............................................................................. 347
obligations of a partner ............................................................................... 347
partner by estoppel ..................................................................................... 348
partner vs lender (debtor) ........................................................................... 348
revocation of managers power; liability of an industrial partner.................. 349
who can form partnerships; spouses; corporations..................................... 350
Commodatum & Mutuum....................................................................... 352
Commodatum............................................................................................. 352
Commodatum vs usufruct...................................................................... 352
Commodatum; liabilties of a bailee....................................................... 353
Mutuum
vs commodatum.................................................................................... 353
interests................................................................................................. 353
Deposit.......................................................................................................... 356
Surety............................................................................................................ 358
recovery of deficiency ................................................................................. 358
Antichresis
vs pledge and mortgage ............................................................................. 359
Mortgage
Page 10 of 391
Chattel mortgage ........................................................................................360
chattel mortgage vs pledge......................................................................... 360
effect of loss of thing mortgaged upon principal oblig................................. 361
pactum commissorium................................................................................361
real & chattel; future loans .......................................................................... 362
real estate mortgage vs sale with repurchase............................................. 362
right of redemption vs equity of redemption................................................ 363
Pledge ........................................................................................................... 363
effect of loss of thing pledged upon principal obligation: immediate
demandability of principal oblig................................................................... 364
vs mortgage; antichresis............................................................................. 365
use of the thing pledged ............................................................................. 365
Quasi-contracts ......................................................................................... 367
Negotiorium gestio...................................................................................... 367
1474; reasonable value .............................................................................. 370
solutio indebiti ............................................................................................. 370
Torts and damages................................................................................... 372
abuse of right .............................................................................................. 373
acts contrary to morals ............................................................................... 374
actual and moral damages ......................................................................... 374
breach of contract ....................................................................................... 375
collapse of structures.................................................................................. 375
common carriers......................................................................................... 376
damages..................................................................................................... 376
damages arising from death of unborn child............................................... 377
damages; moral damages & atty fees ........................................................ 378
death indemnity .......................................................................................... 378
defense; due diligence in selection............................................................. 379
filing of separate civil action; need for reservation ...................................... 379
human relations .......................................................................................... 380
insurance; subrogation ............................................................................... 382
loss of an unborn child................................................................................ 382
product liability............................................................................................ 383
quasi- delict................................................................................................. 383
vicarious liability.......................................................................................... 384
defenses; fortuitous event........................................................................... 385
liability of employer for damage caused by employees; defense of due diligence
.................................................................................................................... 385
motor vehicle mishaps; solidary liability of owner who was in the vehicle... 386
primary liability vs subsidiary liability of employers ..................................... 387
vicarious liability.......................................................................................... 388
Aleatory contracts; gambling............................................................... 390
Compromises and arbitration............................................................... 390


Page 11 of 391
00; Civil law vs common law; trend and governance
1997 No. 1:
How would you compare the Civil Law system in its governance and trend with that of the
Common Law system?
Answer:
As regards "governance":
Governance in Civil Law is codal, statutory and written law. It is additionally derived from
case law. Common law is basically derived from case law.
As regards "trend":
Civil law is now tending to rely more and more on decisions of the courts explaining the
laws. Common law is now codifying laws more and more. So they are now merging towards similar
systems.
Additional Answers:
1. Common law refers to the traditional part of the law as distinct from legislation; it refers
to the universal part of law as distinct from particular local customs (Encyclopedia Americana, Vol.
7).
On the other hand, civil law is understood lo be that branch of law governing the relationship
of persons in respect of their personal and private interests as distinguished from both public and
international laws.
In common law countries, the traditional responsibility has for the most part been with the
judges; in civil law countries, the task is primarily reposed on the lawmakers. Contemporary
practices, however, so indicate a trend towards centralizing that function to professional groups that
may indeed, see the gradual assimilation in time of both systems. [Vitug, Civil. Law and
Jurisprudence, p. XX)
2. In Civil Law. the statutes theoretically take precedence over court decisions interpreting
them; while in Common Law, the court decisions resolving specific cases are regarded as law rather
than the statutes themselves which are, at the start, merely embodiments of case law. Civil Law is
code law or written law, while Common Law is case law. Civil Law adopts the deductive method -
from the general to the particular, while the Common Law uses the inductive approach - from the
particular to the general. Common Law relies on equity. Civil Law anchors itself on the letter of the
law. The civilists are for the judge-proof law even as the Common Law Is judge-made law. Civil Law
judges are merely supposed to apply laws and not interpret them.
00; Civil law; definition; sources
1977 No. I-a
What is civil law? What are the sources of the New Civil Code of the Philippines?
Answer
Civil Law is defined as the mass of precepts which determines and regulates those relations
of assistance, authority and obedience existing among members of a family as well as among
members of a society for the protection of private interests.
The sources of the Civil Code of the Philippines are:
(1) Civil Code of Spain of 1889;
(2) Codes and laws of other countries, such as Spain, the various states of the United
States, especially California and Louisiana, France, Argentina, Germany, Mexico, Switzerland,
England and Italy;
(3) Judicial decisions of the Supreme Courts of the Philippines, of various states of the
United States, of Spain, and of other countries;
(4) Philippine laws or statutes, such as the Code of Civil Procedure (Act No. 190), the
Rules of Court, the Marriage Law (Act No. 3613), and the Divorce Law (Act No. 2710);
(5) Works of jurists of various nations;
(6) Filipino customs and traditions; and
(7) The Code Commission itself.
00; Persons
Page 12 of 391
1988 No 1;
(c) As a rule, once the criminal action has been commenced, the civil action
for damages arising from the offense charged shall be suspended until the final
termination of the criminal action. What are the exceptions to said rule as provided
by the Civil Code?
Answer:
(c) The exceptions are as follows:
(1) Where the civil action is based on an obligation not arising from the act
or omission complained of as a felony, such as when the basis of the civil action is
culpa contractual, culpa aquiliana, etc. (Arts. 31, 2177, CC).
(2) Where the law grants to the injured party the right to institute a civil action
which is entirely separate and independent from the criminal action, such as when
the action is based on (a) interferences by public officers or employees or by private
individuals with civil rights and liberties; (b) defamation; (c) fraud; (d) physical
injuries; or (e) refusal or neglect of a city or municipal police officer to "render aid or
protection in case of danger to life or property (Arts. 32,33, 34, CC).
(3) Where the question to be resolved in the civil action is prejudicial to the
criminal action (Art. 36, CC).
Alternative Answers to: No. 1 (a) and (b)
(a) A prejudicial question is a question which arises in a civil suit, the
resolution of which is determinative of the guilt or innocence of the accused.
Its essential elements, as prescribed by Section 5 of Rule 111 are: (a) the
civil action involves an issue raised in the criminal action; and (b) the resolution of
such issue determines whether or not the criminal action may proceed.
Its effect upon a criminal case is to suspend the criminal case if one has
already been commenced.
(b) Considering that the acquittal is not based on reasonable doubt but on a
positive finding of innocence (he was not negligent) the civil action can no longer
prosper.
The civil liability arising from the act complained of as a felony is barred by
the finding of innocence. However, the civil action for damages based on an
obligation ex-contractu is not deemed barred because of article 31 of the Civil Code.
00; Persons; civil register
1975 No. VIII
An illegitimate child of a Chinese father and a Filipino mother was registered
in the Civil Registry as a Chinese. She filed a petition for the correction of the entry
to make her citizenship read 'Filipino" in view of the absence of marriage between
her parents. Will the petition prosper? Why?
Answer
Not the petition will not prosper. While ostensibly, the action seeks a mere
correction of an entry in the Civil Registry, it in effect requests the judicial declaration
of Philippine citizenship. The correction is not merely of a clerical error and therefore
cannot be made through a petition for the correction of an entry in the civil register.
In a line of decisions the Supreme Court has consistently refused to entertain
petitions of this nature. (China v. Republic, 27 SCRA 447)
00; Persons; civil register; correction of entries
Page 13 of 391
1987 No. 3:
Celso Lim would like to correct an allegedly wrong entry in the birth certificate
of one of his sons, Celso Jr., describing the latter as a Chinese national and not as a
Filipino. Among the evidence Celso Lim had are his own birth certificate describing
him as a Filipino, the birth certificate of his other children, all describing them as
Filipinos and a court decision describing his father (Celso, Jr.'s grandfather) as a
Filipino.
(a) What action must Celso Lim take to ensure the effective correction of
the allegedly wrong entry in his son's birth certificate?
(b) Who should be made parties to such action or proceeding?
Answer:
a, Celso must file a petition in an adversary proceeding to correct the
erroneous entry. In the case of Republic v. Valencia, (L-32181, March 5, 1986), it
was ruled that not only clerical errors can be the subject-matter of the petition, but
even the controversial entries can be corrected.
b. Under Rule 108 of the Rules of Court, he should notify the Civil Registrar
and all the persons affected or who have an interest in the subject matter of the
petition, including the Solicitor General.
00; Persons; civil registry
1981 No. 18
(c) True or false? -- The records of a person's birth, as kept by the Civil
Registrar, is a public record and may be inquired into by a person interested.
Answer
(c) False. According to the Child and Youth Welfare Code, the records of a
person's birth shall be kept strictly confidential and no information relating thereto
shall be issued except in certain cases enumerated in the law. (See Art. 7).
00; Persons; conclusive presumption of knowledge of the law
1985 No. 1
A) Although far from reality, everyone is conclusively presumed to know the
law pursuant to Article 3 of the New Civil Code which states that "ignorance of the
law excuses no one from compliance therewith."
Discuss the underlying philosophy behind said pro vision.
Answers:
A) 1. The philosophy of the law is founded upon public policy. The rule
excludes ignorance as an excuse for not complying with the law. Acts contrary to
law should not have the same effects as when done conformably with the law
merely because of ignorance,
2. The philosophy of the law is founded upon public policy. The rule excludes
ignorance as an excuse for not complying with the law. Acts contrary to law
should not have the same effects as when done conformably with the law merely
because of ignorance. It is based on necessity and expediency.
3. Ignorance of the law does not excuse compliance with the law of the land
(Art. 3, Civil Code) whether civil or penal and whether substantive or remedial. This
principle is founded not only on expediency and public policy but on necessity;
otherwise, an easy shelter from a disregard of the law may easily and conveniently
be available.
Page 14 of 391
4. Ignorance of the law does not excuse compliance with the law of the land
(Art. 8, Civil Code) whether civil or penal and whether substantial or remedial
(People vs. Malvar, 21 SCRA 1119). This principle is founded not only on
expediency and public policy but on necessity (see Zulueta vs. Zulueta, 1 Phil 256);
otherwise an easy shelter from a disregard of the law may easily and conveniently
be available.
00; Persons; decisions of inferior courts
1994 No. 4:
1) Are decisions of the Court of Appeals considered laws?
Alternative Answers:
1) a) No, but decisions of the Court of Appeals may serve as precedents
for inferior courts on points of law not covered by any Supreme Court decision, and
a ruling of the Court of Appeals may become a doctrine. (Miranda u. Imperial 77
Phil. 1066).
b) No. Decisions of the Court of Appeals merely have persuasive, and
therefore no mandatory effect. However, a conclusion or pronouncement which
covers a point of law still undecided may still serve as judicial guide and it is possible
that the same maybe raised to the status of doctrine. If after it has been subjected to
test in the crucible of analysis, the Supreme Court should find that it has merits and
qualities sufficient for its consideration as a rule of jurisprudence (Civil Code, Paras).
00; Persons; duty of judges to render judgment
1977 No. XX-b
According to the Civil Code, no judge or court shall decline to render
judgment by reason of the silence, obscurity or insufficiency of the laws. How should
the judge or court then decide the controversy before him?
Answer
The judge shall apply the custom of the place. This can easily be inferred
from the Civil Code (Arts. 9, et seq., Civil Code). In default thereof, he shall apply the
general principles of law and justice.
00; Persons; effect of obiter and dissenting opinion; decisions of the SC
1994 No. 4:
2) What are the binding effects of an obiter dictum and a dissenting opinion?
3) How can a decision of the Supreme Court be set aside?
Alternative Answers:
2) None. Obiter dictum and opinions are not necessary to the determination
of a case. They are not binding and cannot have the force of official precedents. It
is as if the Court were turning aside from the main topic of the case to collateral
subjects: a dissenting opinion affirms or overrules a claim, right or obligation. It
neither disposes nor awards anything it merely expresses the view of the dissenter.
(Civil Code, Paras]
3) A decision of a division of the Supreme Court maybe set aside by the
Supreme Court sitting en banc, a Supreme Court decision may be set aside by a
contrary ruling of the Supreme Court itself or by a corrective legislative act of
Congress, although said laws cannot adversely affect those favored prior to the
Supreme Court decision. [Civil Code, Paras).
00; Persons; effectivity of laws
Page 15 of 391
1990 No 14:
After a devastating storm causing widespread destruction in four Central
Luzon provinces, the executive and legislative branches of the government agreed
to enact a special law appropriating P1 billion for purposes of relief and rehabilitation
for the provinces. In view of the urgent nature of the legislative enactment, ft is
provided in its effectivity clause that it shall take effect upon approval and after
completion of publication in the Official Gazette and a newspaper of general
circulation in the Philippines. The law was passed by the Congress on July 1, 1990.
signed into law by the President on July 3, 1990, and published in such newspaper
of general circulation on July 7, 1990 and in the Official Gazette on July 10, 1990.
(a) As to the publication of said legislative enactment, is there sufficient
observance or compliance with the requirements for a valid publication? Explain
your answer.
(b) When did the law take effect? Explain your answer.
(c) Can the executive branch start releasing and disbursing funds
appropriated by the said law the day following its approval? Explain your answer.
Answer:
(a) Yes, there is sufficient compliance. The law itself prescribes the
requisites of publication for its effectivity, and all requisites have been complied with.
(Article 2, Civil Code)
(b) The law takes effect upon compliance with all the conditions for
effectivity, and the last condition was complied with on July 10, 1990. Hence, the"
law became effective on that date.
(c) No. It was not yet effective when it was approved by Congress on July 1,
1990 and approved by the President on July 3, 1990. The other requisites for its
effectivity were not yet complete at the time.
00; Persons; emancipation
1986 No. 2:
Mr, Mamirapal, 19 years old and married, seeks to do the following:
1. Lend his money at interest
2. Donate a piece of his land to his infant son.
3. Sell his car.
4. Sue his neighbor for damages.
The land, money, and car are Mamirapal's separate property derived from his
own income. He is not sure whether he can legally do all these things without his
father's consent and assistance. He comes to you for advice. What advice will you
give him?
Answer:
I will advice Mamirapal: (1) to lend his money at interest without securing his
father's consent or assistance; (2) to donate a piece of his land to his infant son. but
with father's consent; (3) to sell his car without father's consent or assistance; and
(4) to sue his neighbor for damages, but with his father's assistance. Although
Mamirapal is already emancipated, his emancipation is not perfect or plenary in
character. Under the law, he cannot borrow money or alienate or encumber real
property without the consent of his father. Neither can he sue or be sued without the
assistance of his father. It is obvious that only Nos. (2) and (4) fall within the purview
of these exceptions.
Page 16 of 391
(Note - The above answer is based upon Art. 399 of the Civil Code).
Answer The C.C. provides that an emancipated minor cannot borrow
money, alienate or encumber real property without the consent of his parent or
guardian. Neither can he sue or be sued without the assistance of his parent or
guardian. Therefore, the answer to questions:
1) Yes, he can legally do it alone;
2) No, because donation is a form of alienation;
3) Yes, as property is movable;
4) He needs the assistance of his father.
00; Persons; equity
2003 No I
It is said that equity follows the law What do you understand by this phrase,
and what are its basic implications?
Suggested Answer:
Equity Follows the law means that courts exercising equity jurisdiction are
bound by rules of law and have no arbitrary discretion to disregard them. (Arsenal v
IAC, 143 SCRA 40 [1986]). Equity is applied only in the absence of but never
against statutory law. (Toyota Motor Phil. V CA 216 SCRA 236 [1992]). 5%
00; Persons; human relations
1978 No. I-a
B, a married man, by force and intimidation, succeeded in having sexual
intercourse with A, single, without her consent. As a result, A became pregnant. A
sued B for damages, Is B liable for damages? Why?
Answer
B is liable for damages. From the factual setting stated in the problem, it is
clear that B's act is contrary to law and that he has willfully caused damage to A. As
a matter of fact, he has committed the crime of rape. According to the Civil Code,
every person who, contrary to law, willfully or negligently causes damage to another,
shall indemnify the latter for the same. Consequently, B is liable for compensatory
damages, moral damages, exemplary damages and attorney's fees.
(NOTE: The above answer is based on Art. 20 of the Civil Code and on the
corresponding provisions of the Code on damages, such as Arts. 2202 and 2204
with respect to compensatory damages, Arts. 2219 with respect to moral damages,
Arts. 2230 and 2234 with respect to exemplary damages, and Art. 2208 with respect
to attorney's fees.)
Alternative Answer
B is liable for damages. It must be observed that his act which has caused
damage to A may be considered as a crime or as a quasi-delict. It is now well-
settled doctrine in this jurisdiction that his act gives rise to two separate and
independent liabilitiesfirst, the civil liability arising from crime (culpa criminal) and
second, the civil liability arising from quasi-delict (culpa aquilana). These two
liabilities may be enforced by A against B simultaneously or successively. What is
barred by the law is double recovery. Whether based on the crime or on the quasi-
delict, B is liable for compensatory damages, moral damages, exemplary damages
and attorney's fees.
Page 17 of 391
(NOTE: The above answer is based on Elcano v. Hill, 77 SCRA 98. It has
also been suggested that an answer based on Article 21 of the Civil Code should
also be accepted as a correct answer.)
00; Persons; ignorance of the law vs mistake of fact
1996 No. 1:
1) Is there any difference in their legal effect between ignorance of the law
and ignorance or mistake of fact?
Answer;
Yes, there is a difference. While ignorance of the law is not an excuse for not
complying with it, ignorance of fact eliminates criminal intent as long as there is no
negligence (Art, NCC). In addition, mistake on a doubtful or difficult question of law
may be the basis of good faith (Art. 526. NCC). Mistake of fact may, furthermore,
vitiate consent in a contract and make it voidable (Art. 1390. NCC).
Alternative Answer:
Yes. ignorance of the law differs in legal effect from Ignorance or mistake of
fact. The former does not excuse a party from the legal consequences of his
conduct while the latter does constitute an excuse and is a legal defense.
00; Persons; independent civil action
1983 No. 1
After an altercation with B, A hit B's car with a piece of wood, breaking the
windshield. When C came along and tried to pacify A, the latter stabbed the former,
causing injuries which would have been fatal were it not for timely medical attention.
A was charged with frustrated homicide and malicious mischief. Two
separate civil actions for damages were also filed against him, one by B and the
other by C.
A moved for the suspension of the civil actions until after the termination of
the criminal cases.
Resolve with reasons.
Answer
It should be denied in so far as the civil action for damages arising from B's
injuries is concerned. In cases of physical injuries, which have been held to include
homicide, whether consummated or frustrated, a civil action for damages, entirely
separate and distinct from the criminal action may be brought by the injured party
and shall proceed independently of the criminal proceeding.
It should, however, be sustained as to the civil action to recover indemnity for
the damage to the taxi, it not being one of the cases specified in the Code in which
an independent civil action may be maintained. For this reason, it must be held in
abeyance until the termination of the criminal action.
00; Persons; independent civil actions
1976 No. I-a
During a brawl in a basketball game, A Attacks B with a bottle and causes
physical injuries. If B files a civil case against A for damages and during its
pendency a criminal case is filed against A, should the civil case be suspended
meanwhile? Explain.
(Abellana vs. Marave, 57 SCRA 106)
Answer
Page 18 of 391
No, because the civil action for damages arising from physical injuries is an
independent civil action and is entirely separate and distinct from the criminal action.
(Article 33)
00; Persons; independent civil actions
1976 No. I-b
If a criminal case is filed first, may the civil case be filed during the pendency
of the criminal case or later, even without a reservation? Explain.
Answer
Yes. The civil case may be filed during the pendency of the criminal case
even without a reservation, it being an independent, civil action. It is a substantive
right and cannot be affected or modified by the Rules of Court. (Abellana v. Marave,
57 SCRA 106)
2. If the criminal case is no longer pending, there are two possibilities. There
is either an acquittal or conviction.
If there is an acquittal based on a positive finding of innocence with a
pronouncement that the facts from which the civil case might arise do not exist, then
the civil action cannot be filed; but if it is an acquittal based on reasonable doubt,
then the civil action may still prosper.
If the criminal cage resulted in conviction, whether damages were awarded or
not, the civil case may no longer prosper, if any of the following circumstances took
place:
(a) If a private prosecutor appeared in the criminal case, which is an
indication that the civil action was instituted together with the criminal case; or
(b) If the injured party in the civil case actively participated and/or
intervened in the criminal case. (Manio vs. Gaddi, 44 SCRA 198 (1972)
00; Persons; independent civil actions
1976 No. I-c
Does Article 33 of the Civil Code on separate civil action for damages arising
from injuries require that there be a reservation in the criminal case to file a separate
civil action? Explain
Answer
Best Answer:
No, because Article 33 provides that it is separate and distinct from the
criminal action and may proceed independently of the criminal action. It is a
substantive right and cannot be rendered nugatory by the Rules of Court. The power
of the Supreme Court to promulgate rules is limited to pleadings, practice and
procedure. (Abellana vs-Marave, supra)
Alternative Answer
1. Yes, because Section 2, Rule 111 in relation to Article 33 requires a
reservation in order to take out the civil case from the jurisdiction of the criminal
court. The civil liability resulting from damages and the law accord the injured party
an option to reserve or not to reserve. (Padua v. Robles)
2. Yes, because the general rule is that when a criminal action is filed, the
civil action is impliedly instituted in the same criminal case,
00; Persons; juridical capacity vs capacity to act
1996 No. 1:
Page 19 of 391
2) Distinguish juridical capacity from capacity to act, Answer:
Juridical capacity is the fitness to be the subject of legal relations while
capacity to act Is the power or to do acts with legal effect. The former is inherent in
every natural person and is lost only through death while the latter is merely
acquired and may be lost even before death (Art. 37, NCC).
Alternative Answer;
Juridical capacity, as distinguished from capacity to act: (a) the former is
passive while the latter is active, (b) the former is inherent in a person while the
latter is merely acquired, (c) the former is lost only through death while the latter
may be lost through death or restricted by causes other than death, and Id) the
former can exist without capacity to act while the latter cannot exist without juridical
capacity.
00; Persons; natural persons
1999 No I.
Elated that her sister who had been married for five years was pregnant for
the first time, Alma donated P100,000.00 to the unborn child. Unfortunately, the
baby died one hour after delivery. May Alma recover the P100.000.00 that she had
donated to said baby before it was born considering that the baby died? Stated
otherwise, is the donation valid and binding? Explain. (5%)
ANSWER:
The donation is valid and binding, being an act favorable to the unborn child,
but only if the baby had an intra-uterine life of not less than seven months and pro-
vided there was due acceptance of the donation by the proper person representing
said child. If the child had less than seven months of intra-uterine life, it is not
deemed born since it died less than 24 hours following its delivery, in which ease the
donation never became effective since the donee never became a person, birth
being determinative of personality.
ALTERNATIVE ANSWER:
Even if the baby had an intra-uterine life of more than seven months and the
donation was properly accepted, it would be void for not having conformed with the
proper form. In order to be valid, the donation and acceptance of personal property
exceeding five thousand pesos should be in writing. (Article 748, par. 3)
00; Persons; prejudicial question
1988 No 1;
(a) What is' a prejudicial question? What are its elements? What is its effect
upon a criminal action?
Answer:
(a) A prejudicial question is a question which arises in a case, the resolution
of which is a logical antecedent of the issue involved in said case, and the
cognizance of which pertains to another tribunal (People vs. Aragon 94 Phil. 357;
Jimenez vs. Aceria 22 SCRA 1380).
It has two elements. They are: First, that it must be determinative of the guilt
or innocence of the accused in the criminal case; and second, jurisdiction to try said
question must be lodged in another tribunal (Ibid.)
Its effect upon a criminal case is to suspend it if one has already been
commenced (Article 36, CC). This is of course, the reverse of the ordinary rule of
procedure. The reason for this is that the resolution of the question is determinative
of the guilt or innocence of the accused in the criminal case.
Page 20 of 391
00; Persons; prejudicial questions
1997 No. 3:
In the context that the term is used in Civil Law, state the (a) concept, (b)
requisites and (c) consequences of a prejudicial question,
Answer:
(a) Concept
A prejudicial question is one which must be decided first before a criminal
action may be instituted or may proceed because a decision therein is vital to the
judgment in the criminal case. In the case of People vs. Adelo Aragon (L-5930, Feb.
17, 1954), the Supreme Court defined it as one which arises in a case, the
resolution of which question is a logical antecedent of the issues involved in said
case and the cognizance of which pertains to another tribunal (Paras, Vol. 1, Civil.
Code Annotation, 1989 ed. p, 194).
(b) Requisites
1, The prejudicial question must be determinative of the case before the
court.
2. Jurisdiction to try said question must be lodged in another tribunal.
Additional Answer:
1. The civil action involves an issue similar or intimately related to the issue
raised in the criminal action, and
2. the resolution of such issue determines whether or not the criminal action
may proceed.
(c) Consequences
The criminal case must be suspended. Thus, in a criminal case for damages
to one's property, a civil action that involves -the ownership of said property should
first be resolved (De Leon vs. Mabanag. 38 Phil. 202)
00; Persons; presumption of simultaneous death
1998 No III.
Jaime, who is 65, and his son, Willy, who is 25, died in a plane crash. There
is no proof as to who died first. Jaime's only surviving heir is his wife, Julia, who is
also Willy's mother. Willy's surviving heirs are his mother, Julia and his wife, Wilma.
1. In the settlement of Jaime's estate, can Wilma successfully claim that
her late husband, Willy had a hereditary share since he was much younger than his
father and, therefore, should be presumed to have survived longer? [3%]
2. Suppose Jaime had a life insurance policy with his wife, Julia, and his
son, Willy, as the beneficiaries. Can Wilma successfully claim that one-half of the
proceeds should belong to Willy's estate? |2%J
Answer:
1. No, Wilma cannot successfully claim that Willy had a hereditary share in
his father's estate. Under Art. 43, Civil Code, two persons "who are called to
succeed each other" are presumed to have died at the same time, in the absence of
proof as to which of them died first. This presumption of simultaneous death applies
in cases involving the question of succession as between the two who died, who in
this case are mutual heirs, being father and son.
Answer:
Page 21 of 391
2. Yet, Wilma can invoke the presumption of survivorship and claim that one-
half of the proceeds should belong to Willy's estate, under Sec. 3 (jj) par. 5 Rule
131, Rules of Court, as the dispute does not involve succession. Under this
presumption, the person between the ages of 15 and 60 years is deemed to have
survived one whose age was over 60 at the time of their deaths. The estate of Willy
endowed with juridical personality stands in place and stead of Willy, as beneficiary.
00; Persons; presumption of simultaneous death
1999 No II.
Mr. and Mrs. Cruz, who are childless, met with a serious motor vehicle
accident with Mr. Cruz at the wheel and Mrs. Cruz seated beside him, resulting in
the instant death of Mr. Cruz. Mrs. Cruz was still alive when help came but she also
died on the way to the hospital. The couple acquired properties worth One Million (PI
,000,000.00) Pesos during their marriage, which are being claimed by the parents of
both spouses in equal shares. Is the claim of both sets of parents valid and why?
(3%)
(b) Suppose in the preceding question, both Mr. and Mrs. Cruz were already
dead when help came, so that no-body could say who died ahead of the other,
would you answer be the same to the question as to who are entitled to the
properties of the deceased couple? (2%)
ANSWER:
(a) No, the claim of both parents is not valid. When Mr. Cruz died, he was
succeeded by his wife and his parents as his intestate heirs who will share his estate
equally. His estate was 0.5 Million pesos which is his half share in the absolute
community amounting to 1 Million Pesos. His wife, will, therefore, inherit O.25
Million Pesos and his parents will inherit 0.25 Million Pesos.
When Mrs. Cruz died, she was succeeded by her parents as her intestate
heirs. They will inherit all of her estate consisting of her 0.5 Million half share in the
ab-solute community and her 0.25 Million inheritance from her husband, or a total of
0.750 Million Pesos.
In sum, the parents of Mr. Cruz will inherit 250,000 Pesos while the parents of
Mrs. Cruz will inherit 750,000 Pesos.
(b) This being a case of succession, in the absence of proof as to the time
of death of each of the spouses, it is presumed they died at the same time and no
transmission of rights from one to the other is deemed to have taken place.
Therefore, each of them is deemed to have an estate valued at P500,000,00, or
one-half of their conjugal property of PI million. Their respective parents will thus
inherit the entire PI Million in equal shares, OT P5OO,000.00 per set of parents.
00; Persons; presumption of simultaneous death
2000 No III.
b) Cristy and her late husband Luis had two children, Rose and Patrick, One
summer, her mother-in-law, aged 70, took the two children, then aged 10 and 12,
with her on a boat trip to Cebu. Unfortunately, the vessel sank en route, and the
bodies of the three were never found. None of the survivors ever saw them on the
water. On the settlement of her mother-in-law's estate, Cristy files a claim for a share
of her estate on the ground that the same was inherited by her children from their
grandmother in representation of their father, and she Inherited the same from them.
Will her action prosper? (2%)
SUGGESTED ANSWER:
Page 22 of 391
No, her action will not prosper. Since there was no proof as to who died first,
all the three are deemed to have died at the same time and there was no
transmission of rights from one to another, applying Article 43 of the New Civil Code.
ALTERNATIVE ANSWER:
No, her action will not prosper. Under Article 43 of the New Civil Code,
inasmuch as there is no proof as to who died first, all the three are presumed to
have died at the same time and there could be no transmission of rights among
them. Her children not having inherited from their grandmother. Cristy has no right to
share in her mother-in-law's estate. She cannot share in her own right as she is not
a legal heir of her mother-in-law. The survivorship provision of Rule 131 of the Rules
of Court does not apply to the problem. It applies only to those cases where the
issue involved is not succession.
00; Persons; prospectivity of laws; exceptions
1977 No, I-b
Give five (5) exceptions to the rule that laws shall have no retroactive effects.
Answer
The following are the exceptions to the rule that laws shall have no
retroactive effect:
(1) When the law itself expressly provides for its retroactivity (Art. 4, CC).
(2) When the law is penal insofar as it favors the accused who is not a
habitual criminal, even though at the time of the enactment of such law final
sentence has already been rendered (Art. 22, Rev. Penal Code).
(3) When the law is procedural so long as it does not affect or change
vested rights (Aguillon vs. Dir. of Lands, 17 Phil. 560).
(4) When the law creates new substantive rights (Arts. 2253, 2263,
CC; Bona vs. Briones, 38 Phil. 276).
(5) When the law Is curative in character in the sense that the purpose for its
enactment is to cure defects or imperfections in judicial or administrative
proceedings.
00; Persons; restrictions on capacity to act
1985 No. 1
B) Among the restrictions on capacity to act are minority, insanity, deaf-
mutism, prodigality and civil interdiction, all of which do not exempt the incapacitated
person from "certain obligations."
Explain and discuss the said rule and illustrate its application by specific
examples.
Answers:
B) 1. Such incapacitated person is not exempt from all obligations except
those arising from contracts. In other words, he is liable if the obligation arises from
law, from delicts, from quasi-delicts.
2. The general rule may be illustrated as follows: An incapacitated person is
not exempt from civil liability except those arising from contracts; so he can be liable
for a crime committed by him. He may not be criminally liable but he is a civilly liable.
One may be insane but does not exempt him from his legal obligation to support his
family,
Page 23 of 391
3. A minor under 15 may commit a crime. He might have committed it
without discernment and therefore would be free or exempt from the civil liability.
With regard to property relations, a 5-year-old child may inherit property and the fact
that the new owner of the property is incapacitated does not exempt the property
and himself from the burden of easement.
4. The rule is based upon lack of voluntariness of the act due to physical or
mental defects, and upon the necessity of protection by the State because of
disabilities. Despite incapacity, a minor may be liable for crime. An insane person
must recognize legal easements on his property. A person civilly interdicted may be
liable for quasi-delict.
5. Minority, Insanity, deaf-mutism, prodigality and civil interdiction are
mere restrictions on capacity to act and do not necessarily exempt the incapacitated
person from certain obligations. Among such obligations are those arising from his
acts or from property relations. Accordingly, a minor may be estopped by his
misrepresentations; and an infant may be held civilly liable for his tortious conduct, a
rule that is more preferable than to let the guiltless victim suffer the loss.
6. Minority, insanity, deaf-mutism, prodigality and civil interdictions are mere
restrictions on capacity to act and do not necessarily exempt the incapacitated
person from certain obligations. Among such obligations an those arising from his
acts or from property relations (Arts. 38-39, Civil Code). Accordingly, a minor may be
estopped by his misrepresentation (Mercado vs. Espiritu, 37 Phil. 215); and
an infant may be held liable for his tortious conduct, a rule that is preferable than to
let the guiltless victim suffer the lost (See Magtibay vs. Tiangco, 74 Phil. 676),
00; Persons; use of surnames
1975 No. VII
A mother who had a natural child subsequently married a man other than the
child's father. The child filed a petition to change his surname from the father's name
to that of the step-father who has no objection thereto. May the petition be granted?
Why?
Answer
Yes, the petition may be granted. While it is true that a natural child shall
employ the surname of the recognizing parent or that of the father if acknowledged
by both parents, this does not mean that such child is prohibited by law from taking
another surname for justifiable reasons. The purpose of the law in allowing a change
of name is to give a person an opportunity to improve his personality and promote
his best interests. To allow the child here to change his name would eliminate the
stigma of illegitimacy which he would continue to bear if his surname were to be that
of his illegitimate father. Furthermore, the stepfather is agreeable to the child's using
his surname.
The court, therefore, in the sound exercise of its discretion may grant the
petition. (Calderon v. Republic, 19 SCRA 721)
00; Persons; waiver
1978 No. V-b
A was a consistent scholar and honor student for three (3) years in Manuel L.
Quezon (MLQ) University and he enjoyed free tuition privileges. In his fourth year,
he decided to study in a University in Davao as his father died and he had to stay
with his mother. He needed the transcripts of his records in MLQ University, but
MLQ refused to issue them until he had refunded the whole amount of tuition fees
given to him for three (3) years of his stay, alleging that he had signed an agreement
Page 24 of 391
beforehand and waiving his right to transfer to another university without having
refunded the cash equivalent of his scholarship. A was forced to refund the amount
as he did not want to be late for his enrollment in the Davao University. Later, he
sued for its return. Can A recover the amount he refunded to MLQ University?
Reasons for your answer.
Answer
Yes, A can recover the amount he refunded to MLQ University. The waiver
signed by A is contrary to public policy, and therefore, null and void. Scholarship
grants are awarded in recognition of merit and not to attract and keep brilliant
students in school for their propaganda value. To look at such grants as a business
scheme designed to increase the business potential of an educational institution is
not only inconsistent with sound public policy bat also good morals.
(NOTE: The above answer is based on Cui vs. Arellano University, L-15127,
May 30, 1961 applying Art. 1306 of the New Civil Code.)
00; Persons; waiver of rights
2004 No. IX
B. DON, an American businessman, secured parental consent for the
employment of five minors to play certain roles in two movies he was producing at
home in Makati. They worked at odd hours of the day and night, but always
accompanied by parents or other adults. The producer paid the children talent fees
at rates better than adult wages.
But a social worker, DEB, reported to OSWD that these children often missed
going to school. They sometimes drank wine, aside from being exposed to drugs.
In some scenes, they were filmed naked or in revealing costumes. In his defense,
DON contended all these were part of artistic freedom and cultural creativity. None
of the parents complained, said DON. He also said they signed a contract
containing a waiver of their right to file any complaint in any office or tribunal
concerning the working conditions of their children acting in the movies.
Is the waiver valid and binding? Why or why not? Explain. (5%)
00; Persons; waiver of rights
1977 No. XX-a
Rights may be waived In what cases may waiver be prohibited and declared
null and void?
Answer
Waiver is prohibited when it is contrary to law, public order, public policy,
morals or good customs, or prejudicial to a third person with a right recognized by
law, (Art. 6, Civil Code).
Page 25 of 391
01; Conflict of laws
1981 No. 18
(b) True or false? -- A woman loses her citizenship when she marries a
foreigner and under the national law of the husband she automatically acquires his
citizenship by marriage.
Answer
(b) False. Under the 1973 Constitution, a female citizen of the Philippines
who marries an alien shall retain her citizenship. Consequently, she does not
necessarily acquire her husband's nationality.
01; Conflict of laws; Art. 15
1998 No II.
Francis Albert, a citizen and resident of New Jersey, U.S.A., under whose law
he was still a minor, being only 20 years of age, was hired by ABC Corporation of
Manila to serve for two years as its chief computer programmer. But after serving for
only four months, he resigned to join XYZ Corporation, which enticed him by offering
more advantageous terms. His first employer sues him in Manila for damages
arising from the breach of his contract of employment. He sets up his minority as a
defense and asks for annulment of the contract on that ground. The plaintiff disputes
this by alleging that since the contract was executed in the Philippines under whose
law the age of majority is 18 years, he was no longer a minor at the time of
perfection of the contract.
1, Will the suit prosper? [3%]
2. Suppose XYZ Corporation is impleaded as a co-defendant, what would
be the basis of its liability, if any? [2%]
Answer:
1. The suit will not prosper under Article 15, Civil Code, New Jersey law
governs Francis Albert's capacity to act, being his personal law from the standpoint
of both his nationality and his domicile. He was, therefore, a minor at the time he
entered into the contract.
Alternative Answer:
1. The suit will not prosper. Being a U.S. national, Albert's capacity to enter
into a contract is determined by the law of the State of which he is a national, under
which he to still a minor. This is in connection with Article 15 of the Civil Code which
embodies the said nationality principle of lex patriae. While this principle intended to
apply to Filipino citizens under that provision, the Supreme Court in Recto v. Harden
is of the view that the status or capacity of foreigners is to be determined on the
basis of the same provision or principle, i.e., by U.S. law in the present problem.
Plaintiffs argument does not hold true, because status or capacity is not
determined by lex loci contractus but by lex patriae.
Another Answer:
1. Article 17 of the Civil Code provides that the forms and solemnities of
contracts, wills and other public instruments shall be governed by the laws of the
country in which they are executed.
Since the contract of employment was executed in Manila, Philippine law
should govern. Being over 18 years old and no longer a minor according to
Philippine Law, Francis Albert can be sued. Thus, the suit of ABC Corporation
against him for damages will prosper*
Page 26 of 391
Answer:
2. XYZ Corporation, having enticed Francis Albert to break his contract
with the plaintiff, may be held liable for damages under Art. 1314, Civil Code.
Alternative Answer:
2. The basis of liability of XYZ Corporation would be Article 28 of the Civil
Code which states that:
"Unfair competition in agricultural, commercial, or industrial enterprises or in
labor through the use of force, intimidation, deceit, machination or any other unjust,
oppressive or highhanded method shall give rise to a right of action by the person
who thereby suffers damage."
Another Answer:
2. No liability arises. The statement of the problem does not in any way
suggest intent, malice, or even knowledge, on the part of XYZ Corporation as to the
contractual relations between Albert and ABC Corporation.
01; Conflict of laws; Art. 15; legal capacity
1995 No, 2:
3. What law governs the capacity of the Filipino to buy the land? Explain your
answer and give its legal basis.
Answer:
Philippine law governs the capacity of the Filipino to buy the land. In addition
to the principle of lex rei sitae given above. Article 15 of the NCC specifically
provides that Philippine laws relating to legal capacity of persons are binding upon
citizens of the Philippines no matter where they are.
01; Conflict of laws; Art. 15; legal capacity; capacity to contract
1995 No, 2:
2. What law governs the capacity of the Japanese to sell the land? Explain
your answer and give its legal basis.
Answer:
Japanese law governs the capacity of the Japanese to sell the land being his
personal law on the basis of an interpretation of Art. 15, NCC.
Alternative Answers;
a) Since capacity to contract is governed by the personal law of an
individual, the. Japanese seller's capacity should be governed either by his national
law (Japanese law) or by the law of his domicile, depending upon whether Japan
follows the nationality or domiciliary theory of personal law for its citizens.
b) Philippine law governs the capacity of the Japanese owner in selling the
land. While as a general rule capacity of persons is governed by the law of his
nationality, capacity concerning transactions involving property is an exception.
Under Article 16 of the NCC. the capacity of persons in transactions involving title to
property is governed by the law of the country where the property is situated.
Since the property Is in the Philippines, Philippine law governs the capacity of the
seller.
01; Conflict of laws; Art. 16
2001 No I
Page 27 of 391
Alex was born a Filipino but was a naturalized Canadian citizen at the time of
his death on December 25,1998. He left behind a last will and testament in which he
bequeathed all his properties, real and personal, In the Philippines to his
acknowledged illegitimate Fillpina daughter and nothing to his two legitimate Filipino
sons. The sons sought the annulment of the last will and testament on the ground
that it deprived them of their legitimes but the daughter was able to prove that there
were no compulsory heirs or legitimes under Canadian law. Who should prevail?
Why? (5%)
SUGGESTED ANSWER
The daughter should prevail because Article 16 of the New Civil Code
provides that intestate and testamentary succession shall be governed by the
national law of the person whose succession is under consideration.
01; Conflict of laws; Art. 16
1995 No, 2:
While in Afghanistan, a Japanese by the name of Sato sold to Ramoncito, a
Filipino, a parcel of land situated in the Philippines which Sato inherited from his
Filipino mother.
1. What law governs the formality in the execution of the contract of sale?
Explain your answer and give its legal basis.
Answer:
Under Art. 16 par. 1, NCC, real property is subject to the law of the country
where it is situated. Since the property is situated in the Philippines, Philippine law
applies. The rule of lex rei sitae in Article 16 prevails over lex loci contractu in Article
17 of the NCC.
Alternative Answer:
Afghanistan law governs the formal requirements of the contract since the
execution is in Afghanistan. Art. 17 of the Civil Code provides that the forms and
solemnities of contracts, wills, and other public Instruments shall be governed by the
laws of the country in which they are executed. However, if the contract was
executed before the diplomatic or consular officials of the Republic of the Philippines
in Afghanistan, Philippine law shall apply.
01; Conflict of laws; Art. 16
1985 No. 11
A, a Filipino, 18 years of age, married and residing in a foreign country,
having run out of money, borrowed P60,000.00 from B and to secure its payment,
executed a real mortgage on a house and lot owned by him in Manila. Under the law
of the country where he resides., he is deemed to be of age and the real estate
mortgage is valid. The loan not having been paid on maturity, B brought an action in
Manila to foreclose the mortgage and recover what is due him.
a) May the real estate mortgage be foreclosed? Reasons.
b) May B recover the loan of P50.000.00 due him as an unsecured debt and
have the mortgaged property levied upon and sold for the satisfaction thereof?
Discuss.
Answers:
(a) and (b)
1. Since A is 18 years of age and is a minor in our country, his real property
is governed by our law, and therefore, the mortgage is voidable and can be
Page 28 of 391
enforced subject to the defense of minority. The law provides that an emancipated
minor cannot borrow money. By analogy with the ruling in the case of a conveyance
of conjugal real property by the husband without the consent of the wife that held
that conveyance void, the mortgage is likewise void.
2. Laws relating to family rights and duties or to the status, condition and
legal capacity of persons are binding upon the citizens of the Philippines even
though living abroad.
A is a minor emancipated by marriage. Hence, he cannot borrow money or
alienate or encumber real property without the consent of his father, mother or
guardian. The real estate mortgage cannot be foreclosed and there can be no
recovery because both contracts were not validly executed.
3. The law provides that an emancipated minor cannot borrow money or
alienate or encumber real property without the consent of his father or mother, or
guardian (Art. 399). Applying by analogy the ruling in the case of conveyance of
conjugal real property by the husband without the wife's consent that held that
conveyance void (Garcia re. Court of Appeals), the loan and the real estate
mortgage contracted by A should be declared void. The real estate mortgage may
not thus be foreclosed.
For the reason addressed above, the rules on void contracts would apply that
may generally negate recovery by either party thereunder (Art, 1412, Civil Code),
4. The loan is merely voidable under Art. 1390 being a contract where one is
incapable of giving consent. Until annulled, the loan, as well as the real estate
mortgage are binding between the parties.
When & contract is voidable because of such incapacity, the incapacitated is
"not obliged to make any restitution except insofar as he has benefited by the thing
or price received by him" (Art. 1399, Civil Code).
01; Conflict of laws; Art. 17; contracts contrary to public policy
1996 No 18:
Alma was hired as a domestic helper in Hongkong by the Dragon Services,
Ltd., through its local agent. She executed a standard employment contract
designed by the Philippine Overseas Workers Administration (POEA) for overseas
Filipino workers. It provided for her employment for one year at a salary of
US$1,000.00 a month. It was submitted to and approved by the POEA. However,
when she arrived In Hongkong, she was asked to sign another contract by Dragon
Services, Ltd. which reduced her salary to only US$600.00 a month. Having no
other choice. Alma signed the contract but when she returned to the Philippines, she
demanded payment of the salary differential of US$400.00 a month. Both Dragon
Services, Ltd. and its local agent claimed that the second contract is valid under the
laws of Hongkong, and therefore binding on Alma.
Is their claim correct? Explain. Answer:
Their claim is not correct. A contract is the law between the parties but the
law can disregard the contract if it is contrary to public policy. The provisions of the
1987 Constitution on the protection of labor and on social justice (Sec. 10. Art II)
embody a public policy of the Philippines. Since the application of Hongkong law in
this case is in violation of that public policy, the application shall be disregarded by
our Courts. (Cadalin v. POEA. 238 SCRA 762)
Alternative Answers;
a) Their claim is not correct. Assuming that the second contract is binding
under Hongkong law, such second contract Is invalid under Philippine law which
Page 29 of 391
recognizes as valid only the first contract. Since the case is being litigated in the
Philippines, the Philippine Court as the forum will not enforce any foreign claim
obnoxious to the forum's public policy. There is a strong public policy enshrined in
our Constitution on the protection of labor. Therefore, the second contract shall be
disregarded and the first contract will be enforced. (Cadalin v. POEA, 238 SCRA
762).
b) No, their claim is not correct. The second contract executed In Hongkong,
partakes of the nature of a waiver that is contrary to Philippine law and the public
policy governing Filipino overseas workers. Art. 17, provides that our prohibitive
laws concerning persons, their acts, or their property or which have for their object
public order, public policy and good customs shall not be rendered ineffective by
laws or conventions agreed upon in a foreign country. Besides. Alma's consent to
the second contract was vitiated by undue influence, being virtually helpless and
under financial distress in a foreign country, as indicated by the given fact that she
signed because she had no choice. Therefore, the defendants claim that the
contract is valid under Hongkong law should be rejected since under the doctrine of
processual presumption a foreign law is deemed similar or identical to Philippine law
in the absence of proof to the contrary, and such is not mentioned in the problem as
having been adduced.
01; Conflict of laws; Art. 17; labor contracts
1991 No 7;
A. The Japan Air Lines (JAL), a foreigner corporation licensed to do
business in the Philippines, executed in Manila a contract of employment with
Maritess Guapa under which the latter was hired as a stewardess on the aircraft
plying the Manila-Japan-Manila route. The contrast specifically provides that (1) the
duration of the contract shall be two (2) years, (2) notwithstanding the above
duration, JAL may terminate the agreement at any time by giving her notice in
writing ten (10) days in advance, and (3) the contract shall be construed as
governed under and by the laws of Japan and only the court in Tokyo, Japan shall
have the jurisdiction to consider any matter arising from or relating to the contract.
JAL dismissed Maritess on the fourth month of her employment without giving
her due notice. Maritess then filed a complaint with the Labor Arbiter for
reinstatement, backwages and damages. The lawyer of JAL contends that neither
the Labor Arbiter nor any other agency or court In the Philippines has jurisdiction
over the case in view of the above provision (3) of the contract which Maritess
voluntarily signed. The contract Is the law between her and JAL.
Decide the issue.
B. Where under a State's own conflicts rule that domestic law of another
State should apply, may the courts of the former nevertheless refuse to apply the
latter? If so, under what circumstance?
Answer,
A, Labor Legislations are generally intended as expressions of public policy
on employer-employee relations. The contract therefore, between Japan Air Lines
(JAL) and Maritess may apply only to the extent that its provisions are not
inconsistent with Philippine labor laws intended particularly to protect employees.
Under the circumstances, the dismissal of Maritess without complying with
Philippine Labor law would be invalid and any stipulation in the contract to the
contrary is considered void. Since the law of the forum in this case is the Philippine
law. the issues should-be resolved in accordance with Philippine law.
B. The third paragraph of Art. 17 of the Civil Code provides that:
Page 30 of 391
"Prohibitive laws concerning persons, their acts or property, and those which
have for their object public order, public policy and good customs shall not be
rendered ineffective by laws or judgments promulgated, or by determinations or
conventions agreed upon in a foreign country."
Accordingly, a state's own conflict of laws rule may, exceptionally be
inapplicable, given public policy considerations by the law of the forum.
Going into the specific provisions of the contract in question, I would rule as
follows:
1. The duration of the contract is not opposed to Philippine law and it can
therefore be valid as stipulated;
2. The second provision to the effect that notwithstanding duration, Japan
Air Lines (JAL) may terminate her employment is invalid, being inconsistent with our
Labor laws;
3. That the contract shall be construed as governed under and by the laws
of Japan and only the courts of Tokyo, Japan shall have jurisdiction, is invalid as
clearly opposed to the aforecited third paragraph of Arts. 17 and 1700 of the Civil
Code, which provides:
"Art. 1700. The relations between capital and labor are not merely
contractual. They are so impressed with public interest that labor contracts must
yield to the common good. Therefore, such contracts are subject to the special laws
on labor unions, collective bargaining, strikes and lockouts, closed shop, wages,
working conditions, hours of labor and similar subjects."
Alternative Answer;
A. When a contract has a foreign element such as in the factual setting stated
in the problem where one of the parties is a foreign corporation, the contract can be
sustained as valid particularly the stipulation expressing that the contract is
governed by the laws of the foreign country. Given this generally accepted principle
of international law, the contract between Maritess and JAL is valid and it should
therefore be enforced.
01; Conflict of laws; Arts 15, 16 & 17
1998 No I.
Juan is a Filipino citizen residing in Tokyo, Japan. State what laws govern:
1. His capacity to contract marriage in Japan, [ 1%]
2. His successional rights as regards his deceased Filipino father's property
in Texas, U.S.A. [1%]
3. The extrinsic validity of the last will and testament which Juan executed
while sojourning in Switzerland. [2%]
4. The intrinsic validity of said will. (1%) Answer:
1. Juan's capacity to contract marriage is governed by Philippine
law - i.e., the Family Code -pursuant to Art. 15, Civil Code, which provides that our
laws relating to, among others, legal capacity of persons are binding upon citizens of
the Philippines even though living abroad.
Answer:
2. By way of exception to the general rule of lex rei sitae prescribed by the
first paragraph of Art. 16. Civil Code, a person's successional rights are governed by
the national law of the decedent (2nd par.. Art. 16). Since Juan's deceased father
was a Filipino citizen, Philippine law governs Juan's successional rights.
Page 31 of 391
Another Answer:
2. Juan's successional rights are governed by Philippine law, pursuant to
Article 1039 and the second paragraph of Article 16, both of the Civil Code. Article
1039, Civil Code, provides that capacity to succeed shall be governed by the "law of
the nation" of the decedent, i.e.. his national law. Article 16 provides In paragraph
two that the amount of successional rights, order of succession, and intrinsic validity
of testamentary succession shall be governed by the "national law" of the decedent
who is identified as a Filipino in the present problem.
Answer:
3. The extrinsic validity of Juan's will is governed by (a) Swiss law, It being
the law where the will was made (Art. 17. 1st par. Civil Code), or (b) Philippine law,
by implication from the provisions of Art. 816, Civil Code, which allows even an alien
who is abroad to make a will in conformity with our Civil Code.
Answer:
4. The intrinsic validity of his will is governed by Philippine law, it being his
national law. (Art. 16, Civil Code)
01; Conflict of laws; Arts 15, 16, 17
2002 No III
Felipe and Felisa, both Filipino citizens, were married in Malolos, Bulacan on
June 1, 1950. In 1960 Felipe went to the United States, becoming a U.S. citizen in
1975. In 1980 the obtained a divorce from Felisa, who was duly notified of the
proceedings. The divorce decree became final under California Law. Coming back
to the Philippines in 1982, Felipe married Sagundina, A Filipino Citizen. In 2001,
Filipe, then domiciled in Los Angeles, California, died, leaving one child by Felisa,
and another one by Sagundina. He left a will which he left his estate to Sagundina
and his two children and nothing to Felisa.
Sagundina files a petition for the probate of Felipes will. Felisa questions the
intrinsic validity of the will, arguing that her marriage to Felipe subsisted despite the
divorce obtained by Felipe because said divorce is not recognized in the Philippines.
For this reason, she claims that the properties and that Sagundina has no
successional rights.
A. Is the divorce secured by Felipe in California recognizable and valid in
the Philippines? How does it affect Felipes marriage to Felisa? Explain. (2%).
B. What law governs the formalities of the will? Explain. (1%)
C. Will Philippine law govern the intrinsic validity of the will? Explain. (2%)
SUGGESTED ANSWER:
A. (1.) The divorce secured by Felipe in California is recognizable and
valid in the Philippines because he was no longer a Filipino at that time he secured
it, Aliens may obtain divorces abroad which may be recognized in the Philippines
provided that they are valid according to their national law (Van Dorn V. Romillo, Jr.,
139 SCRA 139 [1985]; Quita v. Court of Appeals, 300 SCRA 406 [1998]; Llorente v.
Court of Appeals, 345 SCRA 595 [2000] ).
(2). With respect to Felipe the divorce is valid, but with respect to Felisa it is
not. The divorce will not capacitate Felisa to remarry because she and Felipe were
both Filipinos at the time of their marriage. However, in DOJ Opinion No. 134 series
of 1993, Felisa is allowed to remarry because the injustice sought to be corrected by
Article 26 also obtains in her case.
Page 32 of 391
B. The foreigner who executes his will in the Philippines may observed the
formalities described in:
1. The Law of the country of which he is a citizen under Article 817 of the
New Civil Code, or
2. the law of the Philippines being the law of the place of execution under
Article 17 of the New Civil Code.
C. Philippine law will not govern the intrinsic validity of the will. Article 16
of the New Civil Code provides that intrinsic validity of testamentary provisions shall
be governed by the National Law of the person whose succession is under
consideration. California law will govern the intrinsic validity of the will.
01; Conflict of laws; cognovit; borrowing statute; characterization
1994 No 1;
In Private International Law (Conflict of Laws) what is:
1} Cognovit?
2) A borrowing statute?
3) Characterization?
Alternative Answers;
1) a) Cognovit is a confession of judgment whereby a portion of the
complaint is confessed by the defendant who denies the rest thereof (Philippine law
Dictionary, 3rd Ed.) (Ocampo v. Florenciano, L-M 13553, 2/23/50),
b) Cognovit Is a "statement of confession" Oftentimes, it is referred to as a
"power of attorney" or simply as a "power", it is the written authority of the debtor
and his direction to the clerk of the district court, or justice of the peace to enter
judgment against the debtor as stated therein. (Words and Phrases, vol. 7, pp. 115-
166).
c) Cognovit is a plea in an action which acknowledges that the defendant did
undertake and promise as the plaintiff in its declaration has alleged, and that it
cannot deny that it owes and unjustly detains from the plaintiff the sum claimed by
him in his declaration, and consents that judgment be entered against the defendant
for a certain sum. [Words and Phrases, vol. 7, pp. 115-166).
d) Cognovit is a note authorizing a lawyer for confession of judgment by
defendant.
2) "Borrowing Statute" - Laws of the state or jurisdiction used by another
state in deciding conflicts questioned involved in the choice of law (Black's Law
Dictionary, 5th ed. 1979).
3) a) "Characterization" is otherwise called "classification" or "qualification." It
is the process of assigning a disputed question to its correct legal category (Private
International Law, Salonga).
b) "Characterization" is a process in determining under what category a
certain set of facts or rules fall. (Paras, Conflict of Laws, p. 94. 1984 ed.)
01; Conflict of laws; effect of divorce granted to former Filipinos
1997 No. 2:
In 1977, Mario and Clara, both Filipino citizens, were married in the
Philippines. Three years later, they went to the United States of America and
established their residence in San Francisco, California. In 1987, the couple applied
for, and were granted, U.S. citizenship. In 1989, Mario, claiming to have been
Page 33 of 391
abandoned by Clara, was able to secure a decree of divorce in Reno, Nevada,
U.S.A.
In 1990, Mario returned to the Philippines and married Juana who knew well
Mario's past life.
(a) Is the marriage between Mario and Juana valid?
(b) Would the renvoi doctrine have any relevance to the case?
Answer;
(a) Yes, because Phil law recognizes the divorce between Mario and Clara
as valid.
(b) No, (pls see renvoi)
01; Conflict of laws; effect of divorce secured abroad
1978 No. X
Ana and Basilio, both Filipino citizens and of legal age, were married in 1950
in Antique but they never lived together. Ana subsequently left the Philippines and
secured a divorce in Nevada, United States in 1953 on the ground of extreme
mental cruelty. In 1955, Ana sought papal dispensation of the marriage and then
married Carl, an American, in Nevada. She lived with him in California and begot
children. She acquired American citizenship in 1959.
1. Will the divorce decree obtained abroad be accorded validity in the
Philippines? What law supports your answer?
2. What is the status of the marriage of Ana and Carl under our laws?
Reasons for your answer.
Answer
1. The divorce decree obtained abroad is not accorded validity in the
Philippines. The following laws support this answer:
(a) The Civil Code states that laws relating to family rights and duties, or to
the status, condition and legal capacity of persona are binding upon citizens of the
Philippines, even though living abroad. It is clear that the divorce decree, which
affects the status and conditions of Ana and Basilio, is not valid under Philippine
laws.
(b) It is a well-settled rule in this jurisdiction that absolute divorce is contrary
to public policy. The Civil Code states that a declaration of public policy cannot be
rendered ineffective by a judgment promulgated in a foreign country.
(NOTE: The above answer is based on Arts. I5 and 17, par. 3, Civil Code and
on decided cases.)
2. The marriage of Ana and Carl is void under the Philippine laws for the
following reasons:
(a) Ana is still married to Basilio under Philippine laws. The decree of
absolute divorce which she obtained in Nevada is not valid in the Philippines for the
reasons stated above. The papal dispensation which she also obtained is not
also valid under Philippine laws.
(b) True, the Civil Code provides that a marriage celebrated outside of the
Philippines in accordance with the laws in force in the country where it is celebrated,
and valid there as such, shall also be valid in this country. But there are exceptions
to this rule. This rule cannot be applied if the marriage is bigamous, polygamous, or
incestuous as determined by Philippine laws. The marriage of Ana and Carl is
certainly bigamous as determined by Philippine laws.
Page 34 of 391
(NOTE: The above answer is based on Arts. 83 and 71 of the Civil Code.)
01; Conflict of laws; effect of divorce secured abroad
1983 No. 19:
A, a Filipino woman, and B, an American, were married and initially lived in
Manila. They later established their domicile in Texas, of which state B is a citizen,
and there obtained an absolute divorce in accordance with the laws of Texas. A
afterwards returned to the Philippines and married a Filipino.
Evaluate the validity of the divorce and the subsequent marriage of A, citing
reasons.
Answer
B will succeed. The divorce granted in Texas cannot be recognized in the
Philippines, in so far as A is concerned. She is a Filipino and is bound by local law
which does not sanction an absolute divorce. B, therefore, remains to be her
husband notwithstanding the divorce.
However, the contrary view is not without legal basis. By adopting the
principle of nationality in Art. 15, under which one's personal law is the law of his
nation, the Civil Code accepts, that an alien woman married to a Filipino may validly
obtain a divorce abroad in accordance with her national law. By recognizing the
validity of such a divorce, it in effect admits its effectiveness in respect of both
spouses. No other solution is possible except the contrary one which, as the
problem above posed shows, leaves one party married to a man or woman who has
ceased to be a spouse and gives rise to bizarre social problems as well as intricate
legal questions.
01; Conflict of laws; forum non conveniens & lex loci contractus
2002 No XIII.
Felipe is a Filipino citizen. When he went to Sydney for vacation, he met a
former business associate, who proposed to him a transaction which took him to
Moscow. Felipe brokered a contract between Sydney Coals Corp. (Coals), an
Australian firm, and Moscow Energy Corp. (Energy), a Russian firm, for Coals to
supply coal to Energy on a monthly basis for three years. Both these firms were not
doing, and still do not do, business in the Philippines. Felipe shuttled between
Sydney and Moscow to close the contract. He also executed in Sydney a
commission contract with Coals and in Moscow with Energy, under which contracts
he was guaranteed commissions by both firms based on a percentage of deliveries
for the three-year period, payable in Sydney and in Moscow, respectively, through
deposits in accounts that he opened in the two cities. Both firms paid Felipe his
commission for four moths, after which they stopped paying him. Felipe learned from
his contacts, who are residents of Sydney and Moscow, that the two firms talked to
each other and decided to cut him off. He now files suit in Manila against both Coals
and Energy for specific performance.
A. Define or explain the principle of lex loci contractus. (2%)
B. Define or explain the rule of forum non conveniens (3%)
C. Should the Philippine court assume jurisdiction over the case? Explain.
(5%)
SUGGESTED ANSWER:
A. Lex loci contractus may be understood in two senses, as follows:
Page 35 of 391
(1) It is the law of the place where contracts, wills, and other public
instruments are executed and governs their forms and solemnities, pursuant to the
first paragraph, Article 17 of the New Civil Code; or
(2) It is the proper law of the contract; e.i., the system of law intended to
govern the entire contract, including its essential requisites, indicating the law of the
place with which the contract has its closest connection or where the main elements
of the contract converge. As illustrated by Zalamea v. Court of Appeals (228 SCRA
23 [1993]), it is the law of the place where the airline ticket was issued, where the
passengers are nationals and residents of, and where the defendant airline
company maintained its office.
ALTERNATIVE ANSWER:
A. Under the doctrine of lex loci contractus, as a general rule, the law of
the place where a contract is made or entered into governs with respect to its nature
and validity, obligation and interpretation. This has been said to be the rule even
though the place where the contract was made is different from the place where it is
to be performed, and particularly so, if the place of the making and the place of
performance are the same (United Airline v. CA, G.R. No. 124110, April 20, 2001).
SUGGESTED ANSWERS:
B. Forum non conveniens means that a court has discretionary authority
to decline jurisdiction over a cause of action when it is of the view that the action
may be justly and effectively adjudicated elsewhere.
C. No, the Philippine courts cannot acquire jurisdiction over the case of
Felipe. Firstly, under the rule of forum non conveniens, the Philippine court is not a
convenient forum as all the incidents of the case occurred outside the Philippines.
Neither are both Coals and Energy doing business inside the Philippines. Secondly,
the contracts were not perfected in the Philippines. Under the principle of lex loci
contractus, the law of the place where the contract is made shall apply. Lastly, the
Philippine court has no power to determine the facts surrounding the execution of
said contracts. And even if a proper decision could be reached, such would have no
biding effect on Coals and Energy as the court was not able to acquire jurisdiction
over the said corporations. (Manila Hotel Corp. v. NLRC. 343 SCRA 1, 13-14[2000])
01; Conflict of laws; forum non-conveniens; long-arm statute
1994 No. 2:
1) What is the doctrine of Forum non conveniens?
2) What is a "long arm statute"? Alternative Answers:
1) a) Forum non conveniens is a principle in Private International Law that
where the ends of justice strongly indicate that the controversy may be more suitably
tried elsewhere, then jurisdiction should be declined and the parties relegated to
relief to be sought in another forum. (Moreno. Philippine Law Dictionary, p. 254,
1982 ed.).
b) Where in a broad sense the ends of justice strongly indicate that the
controversy may be more suitably tried elsewhere, then jurisdiction should be
declined and the parties relegated to relief to be sought in another forum. (Handbook
on Private International Law, Aruego).
c) Forum non conveniens means simply that a court may resist imposition
upon Its jurisdiction even when jurisdiction Is authorized by the letter of a general
venue statute. (Salonga. Private International Law. p, 51. 1967 ed.)
d) Forum non conveniens is a doctrine whereby a court of law having full
Jurisdiction over a case brought in a proper venue or district declines to determine
Page 36 of 391
the case on its merits because Justice would be better served by the trial over the
case in another jurisdiction. (Webster's Dictionary)
(2} a) Long arm statute is a legislative act which provides for personal
jurisdiction, via substituted service or process, over persons or corporations which
are non-residents of the state and which voluntarily go into the state, directly or by
agent or communicate with persons in the state for limited purposes, inactions which
concern claims relating to performance or execution of those purposes (Black's Law
Dictionary, 5th Ed. 1979).
b) Long arm statute refers simply to authorized substituted service.
01; Conflict of laws; laws governing contracts
1992 No 14:
X and Y entered into a contract in Australia, whereby it was agreed that X
would build a commercial building for Yin the Philippines, and in payment for the
construction, Y will transfer and convey his cattle ranch located In the United States
in favor of X.
What law would govern:
a) The validity of the contract?
b) The performance of the contract?
c) The consideration of the contract?
Answer:
(a) The validity of the contract will be governed by Australian law, because
the validity refers to the element of the making of the contract in this case.
(Optional Addendum:"... unless the parties agreed to be bound by another
law".}
(b) The performance will be governed by the law of the Philippines where the
contract Is to be performed.
(c) The consideration will be governed by the law of the United States where
the ranch is located.
(Optional Addendum: In the foregoing cases, when the foreign law would
apply, the absence of proof of that foreign law would render Philippine law
applicable under the "eclectic theory".)
01; Conflict of laws; laws governing contracts of carriage
1995 No. 10:
On 8 December 1991 Vanessa purchased from the Manila office of Euro-Aire
an airline ticket for its Flight No. 710 from Dallas to Chicago on 16 January 1992.
Her flight reservation was confirmed. On her scheduled departure Vanessa checked
in on time at the Dallas airport. However, at the check-in counter she discovered that
she was waitlisted with some other passengers because of intentional overbooking,
a Euro-Aire policy and practice. Euro-Alre admitted that Vanessa was not advised of
such policy when she purchased her plane ticket. Vanessa was only able to fly two
days later by taking another airline.
Vanessa sued Euro-Aire In Manila for breach of contract and damages. Euro-
Aire claimed that it cannot be held liable for damages because Its practice of
overbooking passengers was allowed by the U.S. Code of Federal Regulations.
Vanessa on the other hand contended that assuming that the U.S. Code of Federal
Regulations allowed Intentional overbooking, the airline company cannot invoke the
U.S. Code on the ground that the ticket was purchased in Manila, hence, Philippine
Page 37 of 391
law should apply, under which Vanessa can recover damages for breach of contract
of carriage.
Decide. Discuss fully. Answer;
Vanessa can recover damages under Philippine law for breach of contract of
carriage, Philippine law should govern as the law of the place where the plane
tickets were bought and the contract of carriage was executed. In Zalamea v. Court
of Appeals (G.R No. 104235, Nov. 10, 1993) the Supreme Court applied Philippine
law in recovery of damages for breach of contract of carriage for the reason that it is
the law of the place where the contract was executed.
Alternative Answer:
If the violation of the contract was attended with bad faith, there is a ground to
recover moral damages. But since there was a federal regulation which was the
basis of the act complained of, the airline cannot be in bad faith. Hence, only actual
damages can be recovered. The same is true with regards to exemplary damages.
01; Conflict of laws; laws governing divorce
1987 No. 1:
Alma, a Filipino citizen went to the United States on a tourist visa. Wanting to
legalize her stay and obtain permanent employment, she married John, an American
citizen, for a fee, with the understanding that after a year, John would divorce her.
As agreed upon the two obtained a divorce in Reno, Nevada.
(a) Suppose that after the divorce Alma consults you on the question of
how she can now marry her childhood sweetheart Rene, in the Philippines
preferably, or if that cannot to done, in some other country where Alma and Rene
are prepared to go so that they can be joined in wedlock and live the life they had
dreamed about. What advice will you give Alma. Explain.
(b) Suppose on the other hand that Alma and John decided to give their
marriage a try. They had seven years of marriage. Alma eventually became an
American citizen, but the marriage soured and ended up in a divorce, just the same.
This time Alma wants a marriage for keeps, so she comes back to the
Philippines to Rene who, ever faithful, has waited for seven long years. Can she and
Rene contract a valid marriage? Explain.
Answer:
(First Assumption)
a. Assuming that the marriage of Alma and John is valid, then the divorce
obtained by them is void because of Article 15 of the Civil Code. Hence a marriage
between Alma and Rene in the Philippines or elsewhere will be bigamous. The
advice to Alma, therefore, is for her not to marry Rene.
b. Since Alma became an American citizen and presumably she had that
citizenship at the time of the divorce, if the divorce is valid under the American law or
the law of her nationality at that time, then she would be released from her marriage
with John. Alma and Rene can contract a valid marriage when she comes back to
the Philippines.
(Second Assumption)
a. Assuming that the marriage of Alma and John is fictitious, there having
been no real intent to enter into the marriage, and there having been no consent at
all since Alma's intention was merely to legalize her stay and obtain permanent
employment, the marriage between Alma and John is void. On this premise, my
Page 38 of 391
advice to Alma would be that she can now marry her childhood sweetheart Rene in
the Philippines or in any other country for that matter.
b. Assuming, upon the other hand, that Alma and John did intend to marry
and give their marriage a try, and Alma eventually became an American citizen, the
divorce would be valid. Hence, she and Rene can contract a valid marriage upon
her return to the Philippines.
01; Conflict of laws; laws governing marriages
1992 No 1:
In 1989, Maris, a Filipino citizen, married her boss Johnson, an American
citizen, In Tokyo in a wedding ceremony celebrated according to Japanese laws.
One year later, Johnson returned to his native Nevada, and he validly obtained in
that state an absolute divorce from his wife Maris.
After Maris received the final judgment of divorce, she married her childhood
sweetheart Pedro, also a Filipino citizen, in a religious ceremony in Cebu City,
celebrated according to the formalities of Philippine law. Pedro later left for the
United States and became naturalized as an American citizen. Maris followed Pedro
to the United States, and after a serious quarrel, Marts filed a suit and obtained a
divorce decree issued by the court in the state of Maryland.
Maris then returned to the Philippines and in a civil ceremony celebrated in
Cebu City according to the formalities of Philippine law, she married her former
classmate Vincent likewise a Filipino citizen.
a) Was the marriage of Maris and Johnson valid when celebrated? Is their
marriage still validly existing now? Reasons.
Answer:
(a) The marriage of Mans and Johnson was valid when celebrated because
all marriages solemnized outside the Philippines (Tokyo) in accordance with the
laws in force in the country where they are solemnized (Japan), and valid there as
such, are also valid in the Philippines.
Their marriage no longer validly subsists, because it has been dissolved by
the absolute divorce validly obtained by Johnson which capacitated Maris to remarry
(Art. 26. Family Code).
01; Conflict of laws; laws governing marriages celebrated abroad
1975 No. 1
X and Y, both Filipino citizens and cousins within the fourth degree of
consanguinity entered into a marriage contract in Hongkong. Assuming that the
marriage is valid in Hongkong, is such marriage valid in the Philippines? Why?
Answer:
No, the marriage will be considered void in the Philippines. Under Article 71
of the Civil Code, all marriages performed outside the Philippines in accordance with
the laws in force in the country where they were performed, and valid there as such,
shall also be valid in the country, except bigamous, polygamous, or incestuous
marriages as determined by Philippine Law. A marriage between first cousins is
incestuous and void ab initio under Article 81 of the Civil Code.
Moreover, under Article 15 of the Civil Code, laws relating to family rights and
duties, or to the status, condition and legal capacity of persons are binding upon
citizens of the Philippines, even though living abroad. Article 81(3), which provides
that marriages between collaterals within the fourth civil degree are void ab initio is a
Page 39 of 391
law relating to the legal capacity of persons and is thus binding upon Filipinos, even
though they are abroad.
01; Conflict of laws; laws governing marriages; forms and solemnities
2003 No II
Gene and Jane, Filipino, met and got married in England while both were
taking up post-graduate courses there. A few years after their graduation, they
decided to annul their marriage. Jane filed an action to annul her marriage to Gene
in England on the ground of latters sterility, a ground for annulment of marriage in
England. The English court decreed the marriage annulled. Returning to the
Philippines, Gene asked you whether or not he would be free to marry his former
girlfriend. What would your legal advice be? 5%
Suggested Answer:
No, Gene is not free to marry his former girlfriend. His marriage to Jane is
valid according to the forms and solemnities of British law, is valid here (Article 17,
1st par., NCC). However, since Gene and Jane are still Filipinos although living in
England, the dissolution of their marriage is still governed by Philippine law (Article
15, NCC). Since, sterility is not one of the grounds for the annulment of a marriage
under Article 45 of the Family Code, the annulment of Genes marriage to Jane on
that ground is not valid in the Philippines (Article 17, NCC)
Alternative Answer:
Yes, Gene is free to marry his girlfriend because his marriage was validly
annulled in England. The issue of whether or not a marriage is voidable, including
the grounds therefore, is governed by the law of the place where the marriage was
solemnized (lex loci celebrationis). Hence, even if sterility is not a ground to annul
the marriage under the Philippine law, the marriage is nevertheless voidable
because sterility makes the marriage voidable under English law. Therefore,
annulment of the marriage in England is valid in the Philippines.
01; Conflict of laws; laws governing marriages; forms and solemnities
1989 No. 1:
(1) Robert and Evelyn, both Filipinos, met in Los Angles, California. They
agreed to get married on June 10, 1989. On June 7, 1989, Robert flew to New York
due to an urgent business matter but intended to return to Los Angeles on June 9,
1989, in time for the wedding. The business emergency of Robert, however, lasted
longer than he expected so that he failed to return to Los Angeles as planned. In
order not to postpone the wedding, Robert immediately called his brother Val who
was also residing at Los Angeles to stand as his proxy at the wedding, which the
latter did. Is the marriage of Robert and Evelyn valid in the Philippines? Give your
reasons,
Answer:
If the marriage was performed in accordance with the laws of California and
valid there, then the marriage is likewise valid in the Philippines.
Alternative Answer:
Since the problem does not state the California law on marriage by proxy, the
presumption in Private International Law is that the California law is the same as the
Philippine law. Therefore, the marriage would be void.
(2) While "X", an Associate Justice of the Court of Appeals, was vacationing
in Cebu City, he was requested to solemnize the marriage of Serge and Joan in the
residence of Serge's parents. "X" could not refuse the request of both the parents of
Page 40 of 391
the couple because they were his relatives. On the day set for the wedding, there
were so many visitors at the residence of Serge's parents so that "X" decided to
solemnize the marriage at the kiosk of the public plaza located nearby, Is the
marriage of Serge and Joan valid? Give your reasons.
Answer:
Yes, because the requirement that the marriage be solemnized in a public
place is not an essential requisite of the law.
01; Conflict of laws; laws governing real and personal property
1988 No. 12:
(a) Pursuant to private international law or conflict of laws, to what law is real
property as well as personal property subject? Are there any exceptions to the rule?
If there are, name them.
Answer:
(a) Real property as well as personal property is subject to the law of the
country where it is situated (Art. 16, par. 1, CC). There are, however, two exceptions
to this rule. They are: (1) under the second paragraph of Art. 16, which declares
that testamentary and intestate succession, both with respect to the order of
succession, the amount of successional rights and the intrinsic validity of
testamentary provisions shall be regulated by the national law of the decedent; and
(2) under No. 2 Art. 124, which declares that if the husband is a foreigner and the
wife is a Filipino, their property relations shall be governed by the husband's national
law, without prejudice to the provisions of the Civil Code with regard to immovable
property.
01; Conflict of laws; laws governing succession
1986 No. 13:
Mr. Burnside, a citizen of the State of California but domiciled in the
Philippines, made a will in Manila providing that his estate should be distributed in
accordance with Philippine law. At the time of his death, Burnside's estate consisted
of bank accounts in various Philippine banks.
Is the testamentary provision valid? Explain.
What law would govern if Burnside had not made such a testamentary
provision. Explain.
Answer:
The testamentary provision is not valid. According to the Civil Code, there are
four aspects of succession which are governed by the national law of the decedent if
he is a foreigner. They are: first, the order of succession; second, the amount of
successional rights; third, the intrinsic validity of testamentary provisions; and fourth,
the capacity to succeed. It is obvious that the proviso in Burnside's will providing that
his estate shall be distributed in accordance with Philippine Law is contrary to the
explicit mandate of the Civil Code. Therefore, it is void.
If Burnside had not made the above-stated testamentary provision, applying
the doctrine of single renvoi, the law of the Philippines shall be applied. Under the
law of the Philippines, the national law of Burnside shall govern. That means the law
of California. Now, what does the law of California say? According to the conflicts-
rule-law of California, the internal law of Burnside's domicile shall govern and not the
law of California. So, the case is referred back (renvoi) to the internal law of the
Philippines. The law of the Philippines shall, therefore, be applied in the distribution
of Burnside's estate,
Page 41 of 391
(Note: .The answer to the first question is based upon Arts. 16, par. 2, and
1039 of the Civil Code and upon Bellis vs. Bellis, 20 SCR A 358. The answer to the
second question is based upon Art. 16, par. 2, of the Civil Code and upon Aznar vs.
Christensen-Garcia, 7 SCRA 95.)
Answer - The validity of the testamentary provision would depend on whether
the law of California allows such choice.
If Burnside had not made such testamentary provision, Philippine law would
govern, as in the case of Aznar, it was found that California adopts the domiciliary
theory. As the decedent is domiciled in the Philippines, the Philippine law on
succession will apply.
Answer - The laws of the country of which a person is a citizen shall govern
the intrinsic validity of his will as to the order of succession and amounts to be
distributed to his heirs. The testamentary provision is therefore invalid. However, the
provision shall be given effect not by reason of its validity but because of Philippine
laws which have to govern property situated in the Philippines.
Because his estate consisted only of bank accounts in Philippine banks,
Philippine law should still govern.
01; Conflict of laws; laws governing succession
1988 No. 12:
(b) What are the four aspects of succession which are governed by the
national law of the decedent if he is a foreigner?
Answer:
(b) The four aspects of succession which are governed by the national law of
the decedent if he is a foreigner are: first, the order of succession; second, the
amount of successional rights; third, the intrinsic validity of testamentary provisions;
and fourth, the capacity to succeed. (Arts, 16, par. 2, 1039, CC; sec Bellis vs. Bellis.
20 SCRA 358.)
01; Conflict of laws; laws governing succession
1976 No. V-b
Which law shall govern the successional rights? Explain.
Answer
Article 16, paragraph 2 provides that successional rights shall be regulated by
the national law of the person whose succession is under consideration. Since B is a
citizen of Texas, the law that should govern successional rights should be the law of
Texas. However, if the law of Texas recognizes the domiciliary rule that
successional rights shall be governed by the law of the domicile of the decedent,
then, Philippine Law on Succession shall apply.
01; Conflict of laws; laws governing succession
1980 No. VI
(b) "U", an American citizen who used to stay in New York, married "V", a
Filipina. They lived in Manila, "U" died leaving a will disposing of his real and
personal properties both in Manila and New York according to the laws of New York.
The will was presented for probate in Manila, "V" assailed the validity of the will
claiming that the properties of "U" must be distributed according to Philippine laws
where "U" was domiciled at the time of his death.
How would you resolve the issue?
Answer
Page 42 of 391
(b) The will itself should be admitted to probate.
It must be observed that the issue raised by "V" involves the intrinsic validity
of testamentary provisions. It is well-settled that a probate court cannot, as a general
rule, inquire into the validity of testamentary provisions. Its area of inquiry is limited.
It can only inquire into the following: (a) the due execution of the will; and (b) the
testamentary capacity of the testator.
There are, however, some exceptions to the above rule. Assuming then that
the case at bar falls under one of these exceptions, as far as the contention of "V" is
concerned, we must distinguish. Thus
If under New York law, there is no conflicts law rule declaring that it will be
the decedent's domiciliary law that will govern, then "V" is not correct. This is so
because according to our Civil Code, it will be the .decedent's national law that will
govern. (Art. 16, par. 2, Civil Code).
However, if under New York law, there is a conflicts law rule declaring that it
will be the decedent's domicilliary law that will govern, then "V" is correct. It is now a
settled doctrine in this jurisdiction that when the Civil Code says that it will be the
decedent's national law that will govern, it refers not only to the internal law of the
decedent's state with respect to succession but even to the conflicts law rule. So, the
case is referred back (renvoi) to the internal law of the Philippines where the
decedent was domiciled at the time of his death. Consequently, as far as the
distribution of "U's" properties is concerned, it should be the law of the Philippines
that will govern and not the law of New York, (Agnar vs. Christensen-Garcia, 7
SCRA 95; Bellis vs. Bellis, 20 SCRA S58.)
(Note: If the bar candidate attacks the problem directly by invoking the
provision of the second paragraph of Art. 16 of the Civil Code, the Committee
respectfully recommends that the answer should be considered as correct. If the
candidate attacks the problem from the procedural point of view, basing his answer
on Art. 838 of the Civil Code in relation to the Rules of Court, the Committee
respectfully recommends that due credit should -be given.)
01; Conflict of laws; laws governing succession of aliens
1984 No, 20
A, a foreigner, married B, a Filipino, and settled in the Philippines. They begot
a son, C. Under the laws of A's country, the system of complete separation of
property is followed, and the husband can freely dispose of his properties without
restriction.
During the marriage, A bought a large number of shares of Philex Mining
Company out of his salary as a World Bank consultant.
Upon A's death, it was found that he left a will leaving all his Philex shares
the only property acquired during his marriage exclusively to his brother D, totally
omitting B and C. B and C, therefore, opposed the will on the ground of the
preterition of C, among others. Additionally, B claimed that 1/2 of the Philex shares
of stock should pertain to her as her conjugal share.
Is the will valid? If you were the judge, how would you rule on the issues
raised by B and C?
Answer:
A. Furnished by Office of Justice Palma
Under Article 16 of the Civil Code, in testamentary succession, the intrinsic
validity of testamentary shall be regulated by the national law of A's country. Thus,
since the laws of A's country allow him to freely dispose of his property, the
Page 43 of 391
testamentary dispositions in favor of his brother would be valid even though he
totally omits his wife and son. B, is also not entitled to one-half of the Philex shares.
Under Art. 124, the law of the husband's country shall govern their property
relations, which, in this case, was the system of complete separation of property.
Since the Philex shares were acquired by A out of his salary, the shares belonged
exclusively to him.
B. Comments and Suggested Answer
We agree with the answer of the Bar Examiner, However, if the bar candidate
will hold that the will is valid on the ground that the questions of preterition and
intrinsic validity of testamentary provisions are questions which a probate court
cannot determine, and then, he finally resolves the issues of preterition and intrinsic
validity of testamentary provisions correctly, said answers should be considered a
correct answer.
01; Conflict of laws; laws governing sucession of aliens
1995 No. 15:
Michelle, the French daughter of Penreich, a German national, died in Spain
leaving real properties in the Philippines as well as valuable personal properties in
Germany.
1. What law determines who shall succeed the deceased? Explain your
answer and give its legal basis.
2. What law regulates the distribution of the real properties in the
Philippines? Explain your answer and give its legal basis.
3. What law governs the distribution of the personal properties in Germany?
Explain your answer and give its legal basis.
Answer:
Assuming that the estate of the decedent is being settled in the Philippines)
1. The national law of the decedent (French law) shall govern in determining
who will succeed to his estate. The legal basis is Art. 16 par. 2, NCC.
Alternative Answer:
French law shall govern the distribution of his real properties in the
Philippines except when the real property is land which may be transmitted to a
foreigner only by hereditary succession.
2. The distribution of the real properties in the Philippines shall be governed
by French law. The legal basis is Art. 16, NCC).
3. The distribution of the personal properties in Germany shall be governed
by French law. The legal basis is Art. 16, NCC).
01; Conflict of laws; laws governing wills executed abroad
1975 No. II
A, while traveling in Tokyo, Japan executed his will before a diplomatic official
of the Philippines. Only two witnesses signed the attestation clause. Upon his return
to the Philippines A filed a petition to probate the will. The petition is opposed on the
ground that the will is not attested by three witnesses. Assuming that in Japan only
two witnesses are required to attest a will, may the will of A be admitted to probate?
Why?
Answer
Page 44 of 391
No, A's will may not be admitted to probate in the Philippines. Although under
Article 815, a Filipino in a foreign country is authorized to make a will in any form
established by the law of the country in which he may be. Article 17 of the Civil Code
is more explicit and provides that when acts, such as will are executed in a foreign
country before diplomatic or consular officials of the Philippines, the solemnities
established by Philippine laws shall be observed in their execution. Article 17,
paragraph 2, is more of a special provision while Article 815 is but a reiteration of the
general principle of lex loci celebraciones embodied in the first paragraph of Article
17. One of title solemnities required by our laws regarding wills is the attestation and
subscription of at least three witnesses. (Art 805, par. 1)
01; Conflict of laws; laws governing wills of an alien
1985 No. 10:
C) An Englishman, who had resided in the Philippines for a long time,
executed a will in France, disposing of his real and personal properties in the
Philippines.
What law governs the validity of his will? Discuss.
Answers:
C) 1. With respect to the intrinsic validity of the will, it is English law in force
at the time of his death. That rule is absolute without any exception. With respect
however to the extrinsic or formal validity of the will, it is the law of England or of
France or of the Philippines in force at the time of the execution of the will
2. Article 16Real property as well as personal property is subject to the law
of the country where it is situated. However, the order of succession, the amount of
successional rights and the intrinsic validity of the testamentary provisions shall be
regulated by the national law of the person whose succession is under
consideration, whatever may be the nature of the property and regardless of the
country wherein said property may be found.
3. Article 16 and Article 1039 provide as follows:
"Art. 16. Real property as well as personal property is subject to the law of
the country where it is situated.
"However, intestate and testamentary Successions, both with respect to the
order of succession and to the amount of successional rights and to the intrinsic
validity of testamentary provisions, shall be regulated by the national law of the
person whose succession is under consideration, whatever may be the nature of the
property and regardless of the country wherein said property may be found,"
"Art. 1039. Capacity to succeed is governed by the law of the nation of the
decedent."
4. The extrinsic validity of a will of an alien is governed by the law of the
place where the will is executed, where he resides, where he is a national or by the
Civil Code (Arts, 17 and 816). The intrinsic validity of the will, however, shall be
governed by the law of England since the nationality law governs (Art. 16, Civil
Code).
01; Conflict of laws; nationality theory
2004 No. VII
A. PH and LV are HK Chinese. Their parents are now Filipino citizens who
live in Manila. While still students in MNS State, they got married although they are
first cousins. It appears that both in HK and in MNS State first cousins could marry
legally.
Page 45 of 391
They plan to reside and set up business in the Philippines. But they have
been informed, however, that the marriage of first cousins here is considered void
from the beginning by reason of public policy. They are in a dilemma. They dont
want to break Philippine law, much less their marriage vow. They seek your advice
on whether their civil status will be adversely affected by Philippine domestic law?
What is your advice? (5%)
01; Conflict of laws; nationality theory; effect of divorce secured abroad
1981 No. 4:
Spouses, husband "H" and wife "W", Filipinos, with a child, decided that "H",
a doctor, would go to the U.S. to find employment there and then "W" would join
him. When in the U.S., "H" wrote that to establish U.S. residence, he would have to
obtain a divorce, marry an American girl, and once a U.S. resident, divorce his
American wife and then remarry "W" "W" agreed.
Four years later, "H", now a U.S. resident and having divorced his American
wife, returns to the Philippines and finds that his wife, "W", has married a wealthy
man in a ceremony celebrated in Hongkong and is happily living with him in Manila.
a) Was the marriage and divorce obtained by "H" in the U.S. valid?
b) Was the marriage of "W" in Hongkong valid?
Answer
(a) Both divorce and marriage are not valid in the Philippines.
The divorce is not valid for the following reasons:
(1) We adhere to the nationality theory. The Civil Code declares that laws
relating to family rights and duties, or the status, condition and legal capacity of
persona are binding upon citizens of the Philippines even though living abroad.
Divorce certainly affects the status and condition of persons. Since absolute divorce
is not recognized in this country except among Muslims, therefore, the decree of
absolute divorce obtained by "H" is not valid
(2) It is well-settled that absolute divorce is contrary to public policy.
According to the Civil Code, this declaration of public policy cannot be rendered
ineffective by a judgment promulgated in a foreign country,
The marriage of "H" to an American woman is not also valid. True, we adhere
to the principle of locus regit actum a marriage valid where celebrated is also valid
in the Philippines, but excepted from this rule are bigamous, polygamous and
incestuous marriages as determined by Philippine law. Since the divorce obtained
by "H" from "W" is not valid in this country as stated above, the subsequent marriage
to an American woman is dearly bigamous as determined by Philippine law.
Therefore, the marriage is not valid.
(Note: The above answers are based on Arts. 15,17, par. 3, and 71 of the
Civil Code and on decided cases.)
(b) Using the same line of reasoning, "W's" marriage to another man in
Hongkong is not also valid. Since the decree of absolute divorce obtained by her
husband "H" in the U.S. is not valid under Philippine law for the reasons stated
above, her marriage in Hongkong to another man is clearly bigamous as determined
by Philippine law. Therefore, it is not valid.
(Note: The above answer is based on Arts. 16 and 71 of the Civil Code.)
01; Conflict of laws; naturalization
2003 No III
Page 46 of 391
Miss Universe, from Finland, came to the Philippines on a tourist visa. While
in this country, she fell in love with and married a Filipino doctor, Her tourist visa
having been expired and after the maximum extension allowed therefore, the
Bureau of Immigration and Deportation (BID) is presently demanding that she
immediately leave the country but she refuses to do so, claiming that she is already
a Filipino Citizen by her marriage to a Filipino citizen. Can the BID still order the
deportation of Miss Universe? Explain. 5%
Suggested Answer:
Yes, the BID can order the deportation of Miss Universe. The marriage of an
alien woman to a Filipino does not automatically make her a Filipino Citizen. She
must first prove in an appropriate proceeding that she does not have any
disqualification for Philippine citizenship. (Yung Uan Chu v. Republic of the
Philippines, 158 SCRA 593 [1988]). Since Miss Universe is still a foreigner, despite
her marriage to a Filipino doctor, she can be deported upon expiry of her allowable
stay in the Philippines.
Another suggested Answer:
No, the Bureau of Immigration cannot order her deportation. An alien woman
marrying a Filipino, native-born or naturalized, becomes ipso facto a Filipino if she is
not disqualified to be a citizen of the Philippines (Mo Ya Lim v Commission of
Immigration, 41 SCRA 292 [1971]), (Sec 4, Naturalization Law). All that she has to
do is prove in the deportation proceeding the fact of her marriage and that she is not
disqualified to become a Filipino Citizen.
Another suggested Answer:
It depends. If she is disqualified to be a Filipino citizen, she may be deported.
If she is not disqualified to be a Filipino citizen, she may not be deported. An alien
woman who marries a Filipino citizen becomes one. The marriage of Miss Universe
to the Filipino doctor did not automatically make her a Filipino citizen. She still has
to prove that she is not disqualified to become a citizen.
01; Conflict of laws; operation of foreign laws upon resident
1977 No, I-c
Cite five (5) provisions of the New Civil Code which recognize the operation
of foreign laws upon residents of the Philippines.
Answer
Some of the provisions of the Civil Code which sanction the operation of
foreign laws upon Philippine residents are:
(a) Art. 16, paragraph 2, which states that intestate and testamentary
successions, both with respect to the order of succession and to the amount of
successional rights and to the intrinsic validity of testamentary provisions, shall be
regulated by the national law of the person whose succession is under
consideration.
(b) Art. 17, paragraph 1, which states that the forms and solemnities of
contracts, wills, and other public instruments shall be governed by the laws of the
country in which they are executed.
(c) Art. 66, which states that when either or both of the contracting parties
(to a marriage contract) are citizens or subjects of a foreign country, it shall be
necessary, before a marriage license can be obtained, to provide themselves with a
certificate of legal capacity to contract marriage, to be issued by their respective
diplomatic or consular officials.
Page 47 of 391
(d) Art. 71, which states that all marriages performed outside the
Philippines in accordance with the laws in force in the country where they were
performed, and valid there as such, shall also be valid in this country, except
bigamous, polygamous, or incestuous marriages as determined by Philippine law.
(e) Art. 124, No. (2), which states that if the husband is a foreigner and the
wife is a citizen of the Philippines, (as far as their property relations are concerned),
the laws of the husband's country shall be followed without prejudice to the
provisions of the Civil Code with regard to immovable property.
(f) Art. 815, which states that when a Filipino is in a foreign country, he is
authorized to make a will in any of the forms established by the law of the country in
which he may be. Such will may be probated in the Philippines.
(g) Art. 816, which states that the will of an alien who is abroad produces
effect in the Philippines if made with the formalities prescribed by the law of the
place in which he resides, or according to the formalities observed in his country, or
in conformity with those which the Civil Code of the Philippines prescribes.
(h) Art. 817, which states that a will made in the Philippines by a citizen or
subject of another country, which is executed in accordance with the law of the
country of which he is a citizen or subject, and which might be proved and allowed
by the law of his own country, shall have the same effect as if executed according to
the laws of the Philippines.
(i) Art. 1039, which states that capacity to succeed is governed by the law of
the nation of the decedent.
(j) Art. 1753, which states that the law of the country to which the goods are
to be transported shall govern the liability of the common carrier for their loss,
destruction, or deterioration.
01; Conflict of laws; processual presumption
1976 No. V-c
May our courts take judicial notice of the laws of Texas, U.S.A.? Explain.
Answer
No, because our courts may take judicial notice only of local laws, arid foreign
laws must be pleaded and proved in evidence. However, in the absence of proof of
what the foreign law is, it shall be presumed that the foreign law is the same as the
Philippine law. (Miciano vs. Brimo)
01; Conflict of laws; renvoi doctrine
1997 No. 2:
In 1977, Mario and Clara, both Filipino citizens, were married in the
Philippines. Three years later, they went to the United States of America and
established their residence in San Francisco, California. In 1987, the couple applied
for, and were granted, U.S. citizenship. In 1989, Mario, claiming to have been
abandoned by Clara, was able to secure a decree of divorce in Reno, Nevada,
U.S.A.
In 1990, Mario returned to the Philippines and married Juana who knew well
Mario's past life.
(a) Is the marriage between Mario and Juana valid?
(b) Would the renvoi doctrine have any relevance to the case?
Answer;
Page 48 of 391
(a) Yes, because Phil law recognizes the divorce between Mario and Clara
as valid.
(b) No, The renvoi doctrine is relevant in cases where one country applies
the domiciliary theory and the other the nationality theory, and the issue involved is
which of the laws of the two countries should apply to determine the order of
succession, the amount of successional rights, or, the intrinsic validity of
testamentary provisions. Such issue is not involved in this case.
Alternative Answer;
Yes. "Renvoi" - which means "referring back" is relevant because here, we
are applying U.S. law to Mario, being already its citizen, although the formalities of
the second marriage will be governed by Philippine law under the principle of lex loci
celebrationis.
01; Conflict of laws; renvoi doctrine
1988 No. 12:
(c) A, a citizen of California, U.S.A. but domiciled in the Philippines, died
testate in Manila, survived by two acknowledged natural children, B and C. In his
will, he left more than P500,000.00 to B and only P3,000.00 to C. It is admitted that
under the Civil Code of California, the domiciliary law of the decedent shall govern
questions involving the validity of testamentary provisions, C, who is contesting the
validity of the disposition in favor of B now contends that the Philippine laws with
respect to succession are applicable. Is this correct? Give your reasons.
Answer:
(c) Yes, this is correct. The doctrine of renvoi is applicable in the instant
case. Although the Civil Code in Art, 16 states that the intrinsic validity of
testamentary provisions shall be regulated by the decedent's national law,
nevertheless, the Civil Code of California declares that the decedent's domiciliary
law shall govern. Hence, the question shall be referred back to the decedent's
domicile. In other words, the laws of the Philippines with respect to succession shall
govern. Consequently, in the partition of the estate, C shall be given a share which
must not be less than his legitime. (Aznar vs. Garcia, 7 SCRA 93).
01; Conflict of laws; renvoi doctrine
1976 No. V-a
A and B, husband and wife respectively, are both citizens of Texas, U.S.A.
and domiciled in Manila. B dies in Quezon City, leaving properties in Makati, Rizal.
Illustrate how the renvoi doctrine operates.
Answer
According to the Civil Code which follows the Nationality Theory (Article 16),
successional rights are governed by the law of the nation of the decedent; but
according to the domiciliary theory, successional rights are determined by the law of
his domicile. B, being an American citizens but domiciled in Manila, there lies the
conflict. If the Texas law follows the domiciliary theory, therefore, the Texas law will
refer the matter to Philippine Law, the situs of B's domicile, in which case the
Philippine law will apply. This is how the renvoi doctrine operates.
01; Conflict of laws; renvoi doctrine; characterization
1977 No. XIX-a
When the conflicts rule of the forum refers a matter to a foreign law for
decision, is the reference to the corresponding conflicts rule of that foreign law, or is
the reference to the purely internal rules of the foreign system?
Page 49 of 391
Answer
The reference is to the corresponding conflicts rule of the foreign law. This
was the answer of the Supreme Court in the Christensen case (Aznar vs.
Christensen-Garcia) where it accepted the renvoi. If such conflicts rule says that the
internal law of the decedent's domicile shall govern, and he was domiciled in the
Philippines at the time of his death, the case is referred back (renvoi) to the internal
law of the Philippines. The Philippine court must, therefore, apply the law of the
Philippines.
01; Conflict of laws; significant relationships theory
1994 No. 20:
Able, a corporation domiciled in State A, but, doing business in the
Philippines, hired Eric, a Filipino engineer, for its project in State B. In the contract of
employment executed by the parties in State B, it was stipulated that the contract
could be terminated at the company's will, which stipulation is allowed in State B.
When Eric was summarily dismissed by Able, he sued Able for damages in the
Philippines.
Will the Philippine court apply the contractual stipulation?
Alternative Answers:
a) Using the "significant relationships theory", there are contacts significant
to the Philippines. Among these are that the place of business is the Philippines, the
employee concerned is a Filipino and the suit was filed in the Philippines, thereby
justifying the application of Philippine law. In the American Airlines case the Court
held that when what is involved is paramount state Interest such as the protection of
the rights of Filipino laborers, the court can disregard choice of forum and choice of
law. Therefore the Philippine Court should not apply the stipulation in question.
b) No, lex fori should be applied because the suit is filed in Philippine courts
and Eric was hired in the Philippines. The Philippine Constitution affords full
protection to labor and the stipulation as to summary dismissal runs counter to our
fundamental and statutory laws.
01; Conflict of laws; theory of effective nationality
1977 No. XIX-b
A was considered a Chinese citizen under Nationalist China laws and a
Japanese under the laws of Japan. He died in Manila, leaving properties in the
Philippines. Before his death, he was domiciled in Japan, How should a Philippine
Court adjudicate the successional rights to his estate?
Answer
The Philippine Court should adjudicate the successional rights to A's estate in
accordance with Japanese law. This solution is in conformity with the theory of
effective nationality and the conflict of nationality laws of the Hague Convention. It is
obvious that A himself preferred Japanese law to Chinese law because he was not
only a citizen but also a domiciliary of Japan. True, we adhere to the nationality
theory (Art. 16, Civil Code), but here, the domiciliary theory merely comes to the
rescue of the nationality theory.
01; Conflict of laws; torts; prescriptive period
2004 No. VII
B. In a class suit for damages, plaintiffs claimed they suffered injuries from
torture during martial law. The suit was filed upon President EMs arrival on exile in
HI, a U.S. state. The court in HI awarded plaintiffs the equivalent of P100 billion
Page 50 of 391
under the U.S. law on alien tort claims. On appeal, EMs Estate raised the issue of
prescription. It argued that since said U.S. law is silent on the matter, the court
should apply: (1) HIs law setting a two-year limitation on tort claims; or (2) the
Philippine law which appears to require that claims for personal injury arising from
martial law be brought within one year.
Plaintiffs countered that provisions of the most analogous federal statute, the
Torture Victims Protection Act, should be applied. It sets ten years as the period for
prescription. Moreover, they argued that equity could toll the statute of limitations.
For it appeared that EM had procured Constitutional amendments granting himself
and those acting under his direction immunity from suit during his tenure.
In this case, has prescription set in or not? Considering the differences in the
cited laws, which prescriptive period should be applied: one year under Philippine
law, two years under HIs law, ten years under U.S. federal law, or none of the
above? Explain. (5%)
01; Conflicts of law; Art. 15
2004 No. II
A. Distinguish briefly but clearly between: 5. Domiciliary theory and
nationality theory of personal law. (5%)
01; Conflicts of Law; art. 16; capacity to succeed
1991 No 8:
Jacob, a Swiss national, married Lourdes, a Filipina, in Berne, Switzerland.
Three years later, the couple decided to reside in the Philippines. Jacob
subsequently acquired several properties in the Philippines with the money he
inherited from his parents. Forty years later. Jacob died intestate, and is survived by
several legitimate children and duly recognized illegitimate daughter Jane, all
residing in the Philippines.
(a) Suppose that Swiss law does not allow illegitimate children to inherit, can
Jane, who is a recognized illegitimate child, inherit part of the properties of Jacob
under Philippine law?
(b) Assuming that Jacob executed a will leaving certain properties to Jane as
her legitime in accordance with the law of succession in the Philippines, will such
testamentary disposition be valid?
Answer:
A. Yes. As stated in the problem. Swiss law does not allow illegitimate
children to inherit Hence, Jane cannot inherit the property of Jacob under Philippine
law.
B. The testamentary disposition will not be valid if it would contravene Swill
law; otherwise, the disposition would be valid. Unless the Swiss law is proved, it
would be presumed to be the same as that of Philippine law under the doctrine of
processual presumption.
01; Conflicts of Law; art. 17; laws governing wills executed abroad
1993 No. 3;
A, a Filipino, executed a will in Kuwait while there as a contract worker.
Assume that under the laws of Kuwait, it is enough that the testator affix his
signature to the presence of two witnesses and that the will need not be
acknowledged before a notary public.
May the will be probated in the Philippines? Answer:
Page 51 of 391
Yes. Under Articles 815 and 17 of the Civil Code, the formality of the
execution of a will is governed by the law of the place of execution. If the will was
executed with the formalities prescribed by the laws of Kuwait and valid there as
such, the will is valid and may be probated in the Philippines.
Page 52 of 391
02; Adoption; qualification of adopter
1977 No. II-b
Spouses H and W filed a petition to adopt S, the 4-year old brother of W. The
City Fiscal objected, because "the adoption would result in an incongruity, where S
would be the son of his own sister". Should the petition be granted?
Answer
Yes, the petition should be granted assuming that all of the requisites of a
valid adoption arc present and that the adoption is for the best interest of S, There is
no law which prohibits it. True, such adoption would result in a dual relationship
between W and S, but this should not prevent the adoption. One is by nature, while
the other is by legal fiction. (Santos, Jr. vs. Rep., 21 SCRA 379.)
02; Adoption; qualifications of adopter
1996 No. 6:
2) Tess, a former Fillpina, now a naturalized American, and Gary, her
American husband, filed a petition for adoption of her three-year old nephew, one of
the eleven children of her destitute sister in Tondo. She had been supporting the
child since his birth, and being childless, she and her husband have come to love
him as their own son. They are both well-employed in the United States.
If you were the judge, will you grant the adoption? Explain.
Answer:
No, I will not grant the adoption. Tess and Gary must adopt jointly under Art.
185 of the Family Code. They do not fall in any of the exceptions to this rule. In
addition, both Tess and Gary must be qualified to adopt. While Tess is qualified to
adopt under Article l84[3](a) of the FC, Gary is not so qualified because he Is an
alien and he does not fall under any of the exceptions. Hence, the adoption must be
denied (Toledano v. CA. 233 SCRA 9; Republic v. CA. 22 7 SCRA 401).
02; Adoption; qualifications of adopter
2000 No V.
Sometime in 1990, Sarah, born a Filipino but by then a naturalized American
citizen, and her American husband Tom, filed a petition In the Regional Trial Court
of Makati, for the adoption of the minor child of her sister, a Filipina. Can the petition
be granted? (5%)
SUGGESTED ANSWER;
It depends. If Tom and Sarah have been residing in the Philippines for at least
3 years prior to the effectivity of RA 8552, the petition may be granted. Otherwise,
the petition cannot be granted because the American husband is not qualified to
adopt.
While the petition for adoption was filed in 1990, it was considered refiled
upon the effectivity of RA 8552, the Domestic Adoption Act of 1998. This is the law
applicable, the petition being still pending with the lower court.
Under the Act, Sarah and Tom must adopt jointly because they do not fall in
any of the exceptions where one of them may adopt alone. When husband and wife
must adopt jointly, the Supreme Court has held in a line of cases that both of them
must be qualified to adopt. While Sarah, an alien, is qualified to adopt under Section
7(b)(i) of the Act for being a former Filipino citizen who seeks to adopt a relative
within the 4th degree of consanguinity or affinity, Tom, an alien, is not qualified
because he is neither a former Filipino citizen nor married to a Filipino. One of them
not being qualified to adopt, their petition has to be denied. However, if they have
Page 53 of 391
been residents of the Philippines three years prior to the effectivity of the Act and
continues to reside here until the decree of adoption is entered, they are qualified to
adopt the nephew of Sarah under Section 7(b) thereof, and the petition may be
granted.
ALTERNATIVE ANSWER;
Since the petition was filed before the effectivity of the Domestic Adoption Act
of 1998, the Family Code is the law applicable.
Under the FC, Sarah and Tom must adopt jointly because they do not fall in
any of the exceptions where one of them may adopt alone. Under a long line of
cases decided by the Supreme Court, when husband and wife must adopt Jointly,
both of them must be qualified to adopt. While Sarah is qualified to adopt under
Article 184(3) (a) for being a former Filipino citizen who seeks to adopt a relative by
consanguinity, Tom is not. He is not a former Filipino citizen and neither is he
married to a Filipino. One of them not being qualified to adopt, the petition must be
denied.
02; Adoption; qualifications of adopter
2001 No II
A German couple filed a petition for adoption of a minor Filipino child with the
Regional Trial Court of Makati under the provisions of the Child and Youth Welfare
Code which allowed aliens to adopt. Before the petition could be heard, the Family
Code, which repealed the Child and Youth Welfare Code, came into effect.
Consequently, the Solicitor General filed a motion to dismiss the petition, on the
ground that the Family Code prohibits aliens from adopting. If you were the judge,
how will you rule on the motion? (5%)
SUGGESTED ANSWER
The motion to dismiss the petition for adoption should be denied. The law that
should govern the action is the law in force at the time of filing of the petition. At that
time, it was the Child and Youth Welfare Code that was in effect, not the Family
Code. Petitioners have already acquired a vested right on their qualification to adopt
which cannot be taken away by the Family Code. (Republic v. Miller G.R. No.
125932, April 21, 1999, citing Republic v. Court of Appeals, 205 SCRA 356)
ALTERNATIVE ANSWER
The motion has to be granted. The new law shall govern their qualification to
adopt and under the new law, the German couple is disqualified from adopting. They
cannot claim that they have already acquired a vested right because adoption is not
a right but a mere privilege. No one acquires a vested right on a privilege.
[Note: If the examinee based his answer on the current law, RA 8552, his
answer should be considered correct. This question is based on the repealed
provision of the Family Code on Adoption.]
02; Adoption; qualifications of adopter
2003 No VII.
Lina, a former Filipina who became an American citizen shortly after her
marriage to an American husband, would like to adopt in the Philippines, jointly with
her husband, one of her minor brothers. Assuming that all the required consents
have been obtained, could the contemplated joint adoption in the Philippine
prosper? Explain.
SUGGESTED ANSWER:
Page 54 of 391
Yes, Lina and her American husband can jointly adopt a minor brother of Lina
because she and her husband are both qualified to adopt. Lina, as a former Filipino
citizen, can adopt her minor brother under Sec. 7(b)(i) of RA 8552 (Domestic
Adoption Act of 1998), or under Art. 184 (3)(1) of the Family Code. The alien
husband can now adopt under Sec. 7(b) of RA8552. The Supreme Court has held in
several cases that when husband and wife are required to adopt jointly, each one of
them must be qualified to adopt in his or her own right (Republic v. Toledano, 233
SCRA 9 (1994). However, the American husband must comply with the
requirements of the law including the residency requirement of three (3) years.
Otherwise, the adoption will not be allowed.
02; Adoption; qualifications of adopters
1976 No. IV-b
If A is a non-resident Italian, may he adopt a Filipino child? Explain.
Answer
It depends, if he is qualified by the law of Italy to adopt, then he may so adopt
provided that the Philippines has not broken off diplomatic relations with Italy; but if
he is so disqualified, then he may not adopt.
02; Adoption; qualifications of adopters
1976 No. IV-c
Who may not adopt?
Answer
Under Article 28, P.D. 603, the following persons may not adopt :
(a) A married person without the written consent of the spouse;
(b) The guardian with respect to the ward prior to final approval of his
accounts;
(c) Any person who has been convicted of & crime involving moral turpitude;
(d) An alien who is disqualified to adopt according to the laws of his own
country or one with whose government the Republic of the Philippines has broken
diplomatic relations.
02; Adoption; qualifications of the adopter
1976 No IV-a
A wishes to adopt B but has certain doubts on the following matters: May A
adopt B if A has already 3 legitimate children? Explain.
Answer
Yes, under Article 27 of P.D. 603, any person of age and in full possession of
his civil rights may adopt: Provided, (1) That he is in a position to support and care
for his legitimate, legitimated, acknowledged natural children by legal fiction, or other
illegitimate children, in keeping with the means, both material and otherwise, of the
family; and (2) The children by nature, 14 years and above of the adopting parents
give written consent to the adoption.
02; Adoption; qualifications of the adopters
1995 No. 3:
In 1980 spouses Felisa and George, both Filipino citizens, migrated to the
United States. Six years later they became American citizens. In 1989 they jointly
filed a petition before the Regional Trial Court of Malabon seeking to adopt Gildah
Page 55 of 391
the 10-year old daughter of Helen, Felisa's younger sister. The government opposed
the petition on the ground that Felisa and George were disqualified since they were
already American citizens.
1, How will you resolve the petition? Explain.
2. Will your answer be the same if George were a natural-born American
citizen? Explain.
3. Will your answer be the same if Felisa were the Illegitimate parent of
Gilda? Explain.
4. Going back to the basic facts, suppose Felisa acquired her American
citizenship during the pendency of the petition for adoption, will your answer be the
same as in Question No 1?
Explain.
Answer;
1. The petition should be denied because George is not qualified to adopt.
As husband and wife, they have to adopt Jointly under Article 185 of the Family
Code. Their case does not fall in any of the exceptions where a spouse may adopt
alone. In Republic u. Toledano, (233 SCRA 9), the Court ruled that both spouses
must be qualified to adopt when required by law to adopt Jointly. Being aliens, Fellsa
and George are, as a rule, disqualified to adopt under Art. 184 of the FC. While
Fellsa falls In one of the exceptions to this rule, being a former Filipino who seeks to
adopt a relative by consanguinity, George does not. He does not seek to adopt his
relative by consanguinity, or a legitimate child of his spouse and neither Is his
spouse a Filipino. One of the spouses being disqualified to adopt, the petition has to
be denied.
Alternative Answer;
Since the adopters are former Filipino citizens and the child sought to be
adopted is a relative by consanguinity of one of them, and since the rule of joint
adoption by spouses Is duly complied with, the petition should be granted.
2. The answer will be the same if George were a natural-born American.
He will still not fall in any of the exceptions to the disqualification of aliens.
Alternative Answer;
No, my answer will be different because in that case, while Fellsa Is qualified
to adopt, the petition for joint adoption cannot be granted. It should be converted into
a petition only by Felisa. It cannot be granted as a joint petition but can be granted
as an individual petition.
3. No. the answer will be different. In such a case, Felisa may adopt alone.
Her case falls under the exception to the rule in Art, 185 requiring husband and wife
to adopt jointly, because she seeks to adopt her own illegitimate child. She is
qualified to adopt alone under Art, 184 because she is a former Filipino citizen who
seeks to adopt a relative by consanguinity. Hence, the court may decree the
adoption of Gilda by Felisa.
4. Yes, the answer will be the same as in No. 1. The adopter must be
qualified to adopt not only on the date of filing of the case, but also on the date of
judgment.
02; Adoption; successional rights of adopted child
2004 No. VIII
A. A Filipino couple, Mr. and Mrs. BM, Jr., decided to adopt YV, an orphan
from St. Claires orphanage in New York City. They loved and treated her like a
Page 56 of 391
legitimate child for they have none of their very own. However, BM, Jr., died in an
accident at sea, followed to the grave a year later by his sick father, BM, Sr. Each
left a sizable estate consisting of bank deposits, lands and buildings in Manila. May
the adopted child, YV, inherit from BM, Jr.? May she also inherit from BM, Sr.? Is
there a difference? Why? Explain. (5%)
02; Adoption; successional rights of adopted child
1979 No. XV
H died intestate leaving his legal wife, W, and his legally adopted son AS. In
the proceedings for the settlement of his estate, M the widowed mother of H,
intervened and claims for a share in the estate of H, AS opposes the claim of M
contending that since under the law he is given the same rights as if he were a
legitimate child, he excludes M from the estate of H. Should this opposition be
sustained? Why?
Answer
The opposition of AS should not be sustained. True, under our law, an
adopted child shall be entitled to the same successional rights as a legitimate child,
but then there is an exception. According to the law, if the adopter is survived by
legitimate parents or ascendants and by an adopted child, the latter shall not have
more successional rights than an acknowledged natural child. This merely means
that the adopted child cannot exclude the legitimate parents or ascendants in the
succession and that his legitimate or legal share shall be the same as that to which
an acknowledged natural child would be entitled. In the instant case, since H died
intestate, the share of M will therefore be 1/2; W, 14-; and AS, 1/4. (Art. 1000, Civil
Code; Del Rosario vs, Conanan, 76 SCRA 136).
02; Adoption; who are considered as natural parents
1981 No. 3
"M", an unwed mother, gave her child for adoption to a childless couple, "B-
C", for which "B-C" paid "M" P20.000.00. In the civil register of births, the father was
listed as "father unknown."
Two years later, after "B-C" learned to love the child as their own and
adoption proceedings commenced with the required publication, "F", father of the
child appeared to oppose the adoption and to seek custody of the child. "M" sided
with "B-C" claiming that "F" had abandoned her when he learned that she was
pregnant and declaring that she wanted "B-C" to keep the child
a) Could "F" frustrate the adoption and custody of the child? Explain.
b) Could "B-C" recover the P20,000.00 they had paid from either "F" or "M"?
Reasons.
Answer
(a) "F" cannot frustrate the adoption and custody of the child.
The Child and Youth Welfare Code, which is now the governing law on
adoption, expressly states that the written consent of the maternal parents is
necessary. Correlated with pertinent provisions of both the Welfare Code and the
Civil Code, the words "natural parents" can refer only to the parent or parents with
parental authority. Thus, if the child is spurious, the one with parental authority
would be the mother; if the child is natural, the one with parental authority would be
the parent who is the first to' recognize the child. In the instant case, it is obvious
that "M", not "F", has parental authority over the child. Consequently, her written
consent to the adoption would be sufficient. Besides, "F" has clearly abandoned the
Page 57 of 391
child. Even on that score alone, he could not resist the adoption and custody of the
child by "B-C"
(Note: The above answer is based on Art. 31 of PD 603 and on Arts. 288 and
311, par. 2, of the Civil Code.)
(b) "B-C" cannot recover the P20,000.00 which they had paid to "M" from
either "F" or "M", The reason is obvious. They bought the child from "M"; the latter
sold the child to them. Under the law, "M" is criminally liable. The principle of pari
delicto (insofar as the money is concerned) is now applicable. The law will not aid
either party to an illegal agreement; it leaves them where they are.
(Note: The above answer is based on Art. 14-11 of the Civil Code in relation
to Art. 59 of the Child and Youth Welfare Code.)
Page 58 of 391
02; Family Code
1987 No. 2:
A to was the registered owner of a passenger jeepney, which was involved in
a collision accident with a vegetable truck, resulting in the death of four passengers
and injuries to three. At the time of the accident, Ato was legally married to Maria but
was cohabiting with Tonia in a relationship akin to that of husband and wife.
Could the heirs of the dead passengers and the injured persons recover
damages from:
(a) Ato?
(b) Maria?
(c) Tonia?
Explain each case. Answer:
a. Ato - Yes. Insofar as the dead passengers are concerned, the heirs can
recover damages on the basis of culpa contractual. If the injured persons are also
passengers, Ato is likewise liable on the same basis of culpa contractual. However,
if the injured persons are not passengers, then the liability for damages of Ato will
be on the basis of a quasi-delict.
b. Maria - In view of the ruling in Juaniza v. Jose (89 SCRA 306) that the
passenger jeepney acquired by the husband during an illicit cohabitation with
the paramour is conjugal property, Maria is liable to the same extent as Ato insofar
as the conjugal property in the marriage between Ato and Maria could be
answerable. But as regards her paraphernal property, Maria cannot be held
answerable.
c. Tonia - No, In Juaniza v. Jose the paramour of the owner of the passenger
jeepney that figured in an accident was held to be not a co-owner, and therefore not
liable for damages. Article 144 is inapplicable.
02; Family Code
1987 No. 16:
Manny and Nita, husband and wife, decided to separate by mutual
agreement. They had a contract prepared, signed it and had it notarized, providing
for their separation and for the extra-judicial liquidation of their conjugal assets. They
likewise agreed to live separately and that if either spouse should find a more
compatible partner, the other would raise no objection and would refrain from taking
any judicial action against the other.
Determine the validity of each of the provisions of the agreement. Explain
briefly.
Answer:
1. The provision for their separation is void.
2. The provision for the extra-judicial liquidation of their conjugal assets is
void.
3. The agreement to live separately is void.
4. The agreement that should either spouse find a more compatible partner,
the other would raise no objection and would refrain from taking any judicial action
against the other is void.
Page 59 of 391
The aforementioned stipulations are all void because they are contrary to law,
morals, good custom, public order and public policy. The specific provision of law is
Article 221 of the Civil Code.
02; Family Code; annulment; effects; requisites before remarriage
1990 No 10:
The marriage of H and W was annulled by the competent court. Upon finality
of the judgment of nullity. H began looking for his prospective second mate. He fell
in love with a sexy woman S who wanted to be married as soon as possible, i.e.,
after a few months of courtship. As a young lawyer, you were consulted by H,
(a) How soon can H be joined in lawful wedlock to his girlfriend S? Under
existing laws, are there certain requisites that must be complied with before he can
remarry? What advice would you give H?
(b) Suppose that children were born from the union of H and W, what would
be the status of said children? Explain your answer.
(c) If the subsequent marriage of H to S was contracted before
compliance with the statutory condition for its validity, what are the rights of the
children of the first marriage (i.e., of H and W) and of the children of the subsequent
marriage (of H and S)?
Answer:
(a) H, or either spouse for that matter, can marry again after complying with
the provisions of Article 52 of the Family Code, namely, there must be a partition
and distribution, of the properties of the spouses, and the delivery of the children's
presumptive legitimes. which should be recorded in the appropriate civil registry and
registries of property. H should be so advised.
Alternative Answer: for (a)
The following are the requisites prescribed by law and I the advice to H is to
comply with them, namely:
(1) If either spouse contracted the marriage in bad faith, his or her share of
the net profits of the community property
: or conjugal partnership property shall be forfeited in favor of the common
children or, if there are none, the children of the guilty spouse by a previous
marriage or, in default of children, the innocent spouse;
(2) Donations by reason of marriage shall remain valid except that if the
donee contracted the marriage in bad faith, such donations made to said donee are
revoked by operation
of law;
(3) The spouse who contracted the subsequent marriage in bad faith shall be
disqualified to inherit from the innocent spouse by testate and intestate succession;
(4) If both spouses of the subsequent marriage acted in bad faith all
donations by reason of marriage and testamentary dispositions made by one in
favor of the other are revoked by operation of law.
(5) The judgment of annulment of the marriage, the partition and distribution
of the properties of the spouses, and the delivery of the children's presumptive
legitimes shall be recorded in the appropriate civil registry and registers of property,
(Articles 53. 52, 43. 44. Family Code).
Page 60 of 391
(b) The children born from the union of H and W would be legitimate children
if conceived or born before the decree of annulment of the marriage (under Art. 45 of
the Family Code) has become final and executory (Art. 54, Family Code}.
(c) The children of the first marriage shall be considered legitimate children if
conceived or born before the Judgment of annulment of the marriage of H and W
has become final and executory. Children conceived or born of the subsequent
marriage shall likewise be legitimate even if the marriage of H and S be null and void
for failure to comply with the requisites of Article 52 of the Family Code (Article 53,
Family Code).
As legitimate children, they have the following rights;
(1) To bear the surnames of the father and the mother in conformity with the
provisions of the Civil Code on Surnames;
(2) To receive support from their parents, their ascendants, and in proper
cases, their brothers and sisters, in conformity with the provisions of this Code on
Support; and
(3) To be entitled to the legitime and other successional rights granted to
them by the Civil Code (Article 174, Family Code).
02; Family Code; annulment; fraud
1986 No. 3:
After a whirlwind courtship of two weeks, Marikit, starry-eyed and captivated,
got married to Mr. Masanting. Soon after the honeymoon, however, Marikit
discovers that Masanting was not the knight in shining armor she thought she
married. She received official information that Masanting had been dishonorably
discharged from the army for desertion. She also learned that Masanting had a
string of liaisons with all kinds of women during his army career. What infuriated
Marikit was that Masanting had concealed all of these from her and, in fact, had
woven tales of gallantry on the battlefield and of deep religious conviction which
made him lead a pure life. Promptly upon discovering the truth about Masanting and
within the first year of their marriage, Marikit sues to annul the marriage on grounds
of deception and fraud. Will her action prosper? Explain,
Answer:
Her action for annulment will not prosper.
The fraud and deceit do not constitute fraud as a ground for annulment of a
marriage. The C.C. provides that no other misrepresentation or deceit as to
character, rank, fortune or chastity other than those enumerated shall constitute
fraud as well as a ground for annulment of marriage.
Answer - Marikit's action will not prosper. According to the Civil Code, any of
the following shall constitute fraud which will entitle a contracting party to ask for the
annulment of the contract of marriage:
(1) misrepresentation as to the identity of the contracting parties;
(2) non-disclosure of the previous conviction of the other party of a crime
involving moral turpitude, and the penalty imposed was imprisonment for two years
or more;
(3) concealment by the wife of the fact that at the time of the marriage, she
was pregnant by a man other than her husband.
No other misrepresentation with respect to character, rank, fortune or chastity
shall entitle a contracting party to ask for the annulment of the marriage.
Page 61 of 391
It is clear that the misrepresentation employed by Masanting are merely
deceptions with respect to character and chastity. Therefore, there is no ground for
Marikit's asking for the annulment of her marriage to Masanting.
(Note The above answer is based on Art. 86 of the Civil Code.)
02; Family Code; annulment; fraud
1975 No. IV
The plaintiff, a first year law student, met the defendant in March, 1968. After
several meetings, they became engaged on September 19, 1968 and were married
on November 28, 1968. After living together for only 88 days, the defendant gave
birth to a child of nine (9) months on February 23, 1969. As a result, the plaintiff
abandoned the defendant and then filed a suit for annulment of marriage alleging
that his consent to. the marriage was secured by the assurance of the defendant
that she was a virgin. Decide the case.
Answer
Annulment should not be granted. To constitute fraud as a ground for
annulment, the wife must have concealed the fact that at the time of marriage, she
was pregnant by another man. The wife having given birth less than 3 months after
the marriage was already 6 months pregnant at the time of the marriage so that the
husband knowing of the pregnancy of the wife at the time of marriage is deemed to
have acknowledged the paternity of the child. (Art. 258)
02; Family Code; annulment; fraud
1976 No. II-b
A and B married. May it be annulled if B concealed the fact that she was no
longer a virgin at the time of the marriage or if A became impotent afterwards.
Explain.
Answer
The marriage cannot be annulled on either ground because concealment of
non-virginity does not constitute fraud. It is concealment by the wife that at the time
of marriage she was pregnant by a man other than her husband, which constitute
fraud. Misrepresentation as to chastity does not constitute a ground for annulment.
Impotence, to be considered a ground for annulment, must exist at time of the
marriage. As A became impotent after the marriage, it is not a ground for annulment.
02; Family Code; annulment; fraud
1978 No. II-a
1. Is the non-disclosure to a wife by her husband of pre-marital relationship
with another woman a ground for annulment of marriage? Give reasons for your
answer.
2. Is the fact of pregnancy of the wife at the time of the marriage by a man
other than her husband a ground for annulment of marriage? Explain fully.
Answer
1. The non-disclosure to a wife by her husband of pre-marital relationship
with another woman is not a ground for annulment of marriage. True, under our law
on marriage, fraud is a ground for annulment. However, it has a very technical and
limited meaning. There are only three instances of fraud enumerated by the law
which will justify or entitle a spouse in proceeding against the other for the
annulment of their marriage and the above-mentioned non-disclosure is not one of
them. Besides, and this is decisive, the law itself declares that no other
Page 62 of 391
misrepresentation or deceit as to chastity shall constitute such fraud as will give a
ground for the annulment of marriage.
2. The fact of pregnancy of the wife at the time of the marriage by a man
other than the husband is not in itself a ground for annulment of marriage. The law is
explicit. It is the concealment by the wife of the fact that at the time of the marriage,
she was pregnant by a man other than her husband which is a ground for
annulment. So, if there was no concealment or even the possibility of concealment,
such as when the wife was already six months pregnant at the time of the
celebration of the marriage, there would be no ground for annulment. At such an
advanced stage of pregnancy, concealment would be impossible.
(NOTE: The above answers are based on Articles 85, No. 5, and 86 of the
Civil Code and on Buccat vs. Buccat, 72 Phil. 19.)
02; Family Code; annulment; grounds
1991 No 4:
A. One of the grounds for annulment of marriage is that either party, at the
time of their marriage was afflicted with a sexually-transmissible disease, found to
be serious and appears incurable. Two (2) years after their marriage, which took
place on 10 October 1988, Bethel discovered that her husband James has a
sexually-transmissible disease which he contracted even prior to their marriage
although James did not know it himself until he was examined two [2) years later
when a child was already born to them. Bethel sues James for annulment of their
marriage. James opposes the annulment on the ground that he did not even know
that he had such a disease so that there was no fraud or bad faith on his part.
Decide.
B. Suppose that both parties at the time of their marriage were similarly
afflicted with sexually-transmissible diseases, serious and Incurable, and both knew
of their respective infirmities, can Bethel or James sue for annulment of their
marriage?
Answer;
A. The marriage can be annulled. because good faith is not a defense when
the ground is based upon sexually-transmissible disease on the part of either party.
B. Yes. the marriage can still be annulled because the feet that both of them
are afflicted with sexually-transmissible diseases does not efface or nullity the
ground.
Alternative Answer;
B. No, the marriage can no longer be annulled, because the fact that both
were afflicted and that both knew of their respective infirmities constitutes a waiver
of that ground.
02; Family Code; annulment; judicial declaration
1993 No. 19:
Maria and Luis, both Filipinos, were married by a Catholic priest in Lourdes
Church, Quezon City in 1976, Luis was drunk on the day of his wedding. In fact, he
slumped at the altar soon after the ceremony. After marriage, Luis never had a
steady job because he was drunk most of the time. Finally, he could not get
employed at all because of drunkenness. Hence, it was Maria who had to earn a
living to support herself and her child begotten with Luis. In 1986, Maria filed a
petition in the church matrimonial court in Quezon City to annul her marriage with
Page 63 of 391
Luis on the ground of psychological incapacity to comply with his marital obligation.
Her petition was granted by the church matrimonial court.
1) Can Maria now get married legally to another man under Philippine laws
after her marriage to Luis was annulled by the church matrimonial court? Explain.
2) What must Maria do to enable her to get married lawfully to another man
under Philippine laws?
Answers;
1) No, Maria cannot validly contract a subsequent marriage without a court
declaration of nullity of the first marriage. The law does not recognize the church
declaration of nullity of a marriage.
2) To enable Maria to get married lawfully to another man. she must obtain
a judicial declaration of nullity of the prior marriage under Article 36 Family Code.
02; Family Code; annulment; Legal Separation; prescription of actions
1996 No. 5:
2) Bert and Baby were married to each other on December 23,1988. Six
months later, she discovered that he was a drug addict. Efforts to have him
rehabilitated were unsuccessful.
Can Baby ask for annulment of marriage, or legal separation? Explain.
Answer;
No, Baby cannot ask for annulment of her marriage or for legal separation
because both these actions had already prescribed.
While concealment of drug addiction existing at the time of marriage
constitutes fraud under Art. 46 of the FC which makes the marriage voidable under
Art. 45 of the FC, the action must, however, be brought within 5 years from the
discovery thereof under Article 47(3), FC, Since the drug addiction of Bert was
discovered by Baby In June 1989, the action had already prescribed in June of
1994. Although drug addiction is a ground for legal separation under Art. 55(5) and
Art. 57 of the FC requires that the action must be brought within 5 years from the
occurrence of the cause. Since Bert had been a drug addict from the time of the
celebration of the marriage, the action for legal separation must have been brought
not later than 23 December 1993. Hence, Baby cannot, now, bring the action for
legal separation.
02; Family Code; annulment; proper party
1995 No. 14:
Yvette was found to be positive for HIV virus, considered sexually
transmissible, serious and incurable. Her boyfriend Joseph was aware of her
condition and yet married her. After two (2) years of cohabiting with Yvette, and in
his belief that she would probably never be able to bear him a healthy child, Joseph
now wants to have his marriage with Yvette annulled. Yvette opposes the suit
contending that Joseph is estopped from seeking annulment of their marriage since
he knew even before their marriage that she was afflicted with HIV virus.
Can the action of Joseph for annulment of his marriage with Yvette prosper?
Discuss fully.
Answer;
No Joseph knew that Yvette was HIV positive at the time of the marriage. He
is, therefore, not an injured party. The FC gives the right to annul the marriage only
to an injured party. [Art. 47 (5), FC]
Page 64 of 391
Alternative Answer:
The action for annulment can prosper because the prescriptive period of five
(5) years has not yet lapsed. [Art. 45 (6), FC].
02; Family Code; annulment; proper party
1990 No 13:
D and G, age 20 and 19, respectively, and both single, eloped and got
married to each other without parental consent in the case of G, a teenaged student
of an exclusive college for girls. Three years later, her parents wanted to seek
judicial annulment on that ground. You were consulted and asked to prepare the
proper complaint. What advice would you give G's parents? Explain your answer.
Answer:
G himself should file the complaint under Article 45 of the Family Code, and
no longer the parents because G is already 22 years of age.
02; Family Code; annulment; psychological incapacity
1996 No. 3:
On April 15, 1983, Jose, an engineer, and Marina, a nurse, were married to
each other in a civil ceremony in Boac. Marinduque. Six months after their marriage,
Jose was employed in an oil refinery in Saudi Arabia for a period of three years.
When he returned to the Philippines. Marina was no longer living in their house, but
In Zamboanga City, working in a hospital. He asked her to come home, but she
refused to do so, unless he agreed not to work overseas anymore because she
cannot stand living alone. He could not agree as in fact, he had signed another three
year contract. When he returned In 1989, he could not locate Marina anymore. In
1992. Jose filed an action served by publication Ina newspaper of general
circulation. Marina did not file any answer, A possible collusion between the parties
was ruled out by the Public Prosecutor. Trial was conducted* and Marina neither
appeared nor presented evidence In her favor.
If you were the judge, will you grant the annulment. Explain.
Answer:
As judge, I will not grant the annulment. The facts do not show any taint of
personality disorder on the part of the wife Marina so as to lend substance to her
husband's averment of psychological Incapacity within the meaning of Art 36 of the
Family Code. In Santos vs. CA (240 SCRA 20), this particular ground for nullity of
marriage was held to be limited only to the most serious cases of personality
disorders (dearly demonstrative of utter sensitivity or inability to give meaning and
significance to the marriage. Marina's refusal to come home to her husband unless
he agreed not to work overseas, far from being indicative of an insensitivity to the
meaning of marriage, or of a personality disorder, actually shows a sensitive
awareness on her part of the marital duty to live together as husband and wife. Mere
refusal to rejoin her husband when he did not accept the condition imposed by her
does not furnish any basis for concluding that she was suffering from psychological
Incapacity to discharge the essential marital obligations.
Mere intention to live apart does not fall under Art. 36, FC. Furthermore, there
is no proof that the alleged psychological incapacity existed at the time of the
marriage.
02; Family Code; art. 26
1999 No IV.
Page 65 of 391
Ben and Eva were both Filipino citizens at the time of their marriage in 1967,
When their marriage turned sour, Ben went to a small country in Europe, got himself
naturalized there, and then divorced Eva in accordance with the law of that country,
Later, he returned to the Philippines with his new wife.
Eva now wants to know what action or actions she can file against Ben. She
also wants to know if she can likewise marry again. What advice can you give her?
{5%)
ANSWER:
Considering that Art. 26(2nd par.) contemplates a divorce between a
foreigner and a Filipino, who had such respective nationalities at the time of their
marriage, the divorce in Europe will not capacitate the Filipino wife to remarry. The
advice we can give her is either to file a petition for legal separation, on the ground
of sexual infidelity and of contracting a bigamous marriage abroad, or to file a
petition to dissolve the conjugal partnership or absolute community of property as
the case maybe.
ALTERNATIVE ANSWER:
Eva may file an action for legal separation on the grounds of sexual infidelity
of her husband and the contracting by her husband of a bigamous marriage abroad.
She may remarry. While a strict interpretation of Article 26 of the Family Code
would capacitate a Filipino spouse to remarry only when the other spouse was a
foreigner at the time of the marriage, the DOJ has issued an opinion (Opinion 134 s.
of 1993) that the same injustice sought to be cured by Article 26 is present in the
case of spouses who were both Filipino at the time of the marriage but one became
an alien subsequently. Said injustice is the anomaly of Eva remaining married to her
husband who is no longer married to her. Hence, said Opinion makes Article 26
applicable to her case and the divorce obtained abroad by her former Filipino
husband would capacitate her to remarry. To contract a subsequent marriage, all
she needs to do is present to the civil registrar the decree of divorce when she
applies for a marriage license under Article 13 of the Family Code.
02; Family Code; art. 26
1996 No. 5:
1) Flor and Virgillo were married to each other in Roxas City in 198O. In
1984, Flor was offered a teaching Job in Canada, which she accepted. In 1989, she
applied for and was granted Canadian citizenship. The following year, she sued for
divorce from Virgilio in a Canadian court. After Virgilio was served with summons,
the Canadian court tried the case and decreed the divorce. Shortly thereafter, Flor
married a Canadian.
Can Virgilio marry again in the Philippines? Explain. Answer;
No, Virgilio cannot validly remarry. His case is not covered by Article 26 of the
Family Code, For said Article to be applicable, the spouse who filed for divorce must
be a foreigner at the time of the marriage. Since both of them were Filipinos at the
time of the marriage, the divorce obtained by Flor did not capacitate Virgilio to
remarry. The fact that Flor was already an alien at the time she obtained the divorce
does not give Virgilio the capacity to remarry under Philippine Law.
Alternative Answers;
a) Yes, Virgilio can validly remarry. Art. 26 of the FC, merely States the alien
spouse without taking into consideration his or her nationality at the time of the
marriage. While his case is not covered by the letter of Article 26 FC, it is, however,
covered by the spirit of said Article, The injustice to the Filipino spouse sought to be
Page 66 of 391
cured by said Article Is present in this case. (Department of Justice Opinion No. 134
Series of 1993).
b) Although the marriage originally involved Filipino citizens, it eventually
became a marriage between an alien and a Filipino after Flor became a Canadian
citizen. Thus, the divorce decree was one obtained by an alien spouse married to a
Filipino. Although nothing is said about whether such divorce did capacitate Flor to
remarry, that fact may as well be assumed since the problem states that she married
a Canadian shortly after obtaining the divorce. Hence, Virgillo can marry again
under Philippine law, pursuant to Art. 26. FC which applies because Flor was
already an alien at the time of the divorce.
02; Family Code; declaration of nullity: annulment: legal separation:
separation of property grounds
2003 No VI
Which of the following remedies, i.e., (a) declaration of nullity of marriage, (b)
annulment of marriage, (c) legal separation, and/or (d) separation of property, can
an aggrieved spouse avail himself/herself of-
(i) If the wife discovers after the marriage that her husband has AIDS.
(ii) If the wife goes (to) abroad to work as a nurse and refuses to come
home after the expiration of her three-year contract there.
(iii) If the husband discovers after the marriage that his wife has been a
prostitute before they got married.
(iv) If the husband has a serious affair with his secretary and refuses to
stop notwithstanding advice from relatives and friends.
(v) If the husband beats up his wife every time he comes home drunk. 5%
SUGGESTED ANSWER:
(i) Since AIDS is a serious and incurable sexually-transmissible disease,
the wife may file an action for annulment of the marriage on this ground whether
such fact was concealed or not from the wife, provided that the disease was present
at the time of the marriage. The marriage is voidable even though the husband was
not aware that he had the disease at the time of marriage.
(ii) If the wife refuses to come home for three (3) months from the
expiration of her contract, she is presumed to have abandoned the husband and he
may file an action for judicial separation of property. If the refusal continues for more
than one year from the expiration of her contract, the husband may file the action for
legal separation under Art. 55 (10) of the Family Code on the ground of
abandonment of petitioner by respondent without justifiable cause for more than one
year. The wife is deemed to have abandoned the husband when she leaves the
conjugal dwelling without any intention of returning (Article 101, FC). The intention
not to return cannot be presumed during the 30year period of her contract.
(iii) If the husband discovers after the marriage that his wife was a
prostitute before they got married, he has no remedy. No misrepresentation or
deceit as to character, health, rank, fortune or chastity shall constitute fraud as legal
ground for an action for the annulment of marriage (Article 46 FC).
(iv) The wife may file an action for legal separation. The husbands sexual
infidelity is a ground for legal separation 9Article 55, FC). She may also file an action
for judicial separation of property for failure of her husband to comply with his martial
duty of fidelity (Article 135 (4), 101, FC).
Page 67 of 391
(v) The wife may file an action for legal separation on the ground of
repeated physical violence on her person (Article 55 (1), FC). She may also file an
action for judicial separation of property for failure of the husband to comply with his
marital duty of mutual respect (Article 135 (4), Article 101, FC). She may also file an
action for declaration of nullity of the marriage if the husbands behavior constitute
psychological incapacity existing at the time of the celebration of marriage.
02; Family Code; divorce; void marriages
1992 No 1:
In 1989, Maris, a Filipino citizen, married her boss Johnson, an American
citizen, In Tokyo in a wedding ceremony celebrated according to Japanese laws.
One year later, Johnson returned to his native Nevada, and he validly obtained in
that state an absolute divorce from his wife Maris.
After Maris received the final judgment of divorce, she married her childhood
sweetheart Pedro, also a Filipino citizen, in a religious ceremony in Cebu City,
celebrated according to the formalities of Philippine law. Pedro later left for the
United States and became naturalized as an American citizen. Maris followed Pedro
to the United States, and after a serious quarrel, Marts filed a suit and obtained a
divorce decree issued by the court in the state of Maryland.
Maris then returned to the Philippines and in a civil ceremony celebrated in
Cebu City according to the formalities of Philippine law, she married her former
classmate Vincent likewise a Filipino citizen.
b) Was the marriage of Maris and Pedro valid when celebrated? Is their
marriage still valid existing now? Reasons.
c) Was the marriage of Marts and Vincent valid when celebrated? Is their
marriage still validly existing now? Reasons.
d) At this point in time, who is the lawful husband of Marts? Reasons.
Answer:
(b) The marriage of Maris and Pedro was valid when celebrated because
the divorce validly obtained by Johnson in Manila capacitated Maris to marry Pedro.
The marriage of Maris and Pedro is still validly existing, because the marriage
has not been validly dissolved by the Maryland divorce [Art. 26, Family Code).
(c) The marriage of Maris and Vincent is void ab initio because it is a
bigamous marriage contracted by Maris during the subsistence of her marriage with
Pedro (Art 25 and 41, Family Code).
The marriage of Maris and Vincent does not validly exist because Article 26
does not apply. Pedro was not a foreigner at the time of his marriage with marts and
the divorce abroad (in Maryland) was initiated and obtained not by the alien spouse,
but by the Filipino spouse. Hence, the Maryland divorce did not capacitate Marts to
marry Vincent.
(d) At this point in time, Pedro is still the lawful husband of Maris because
their valid marriage has not been dissolved by any valid cause (Art. 26. Family
Code),
02; Family Code; donations by reason of marriage; effect of declaration of
nullity
1996 No. 6:
1) On the occasion of Digna's marriage to George, her father gave her a
donation propter nuptias of a car. Subsequently, the marriage was annulled because
of the psychological immaturity of George.
Page 68 of 391
May Digna's father revoke the donation and get back the car? Explain,
Answer:
No, Digna's father may not revoke the donation because Digna was not in
bad faith, applying Art. 86(3) of the Family Code.
Alternative Answer;
a) Yes, the donation is revocable. Since the ground for the annulment of the
marriage is the psychological immaturity of George, the judgment was in the nature
of a declaration of nullity under Art. 36 of the FC and, therefore, the donation may be
revoked under Art. 86( 1) of the FC for the reason that the marriage has been
judicially declared void ab initio.
b) No, the donation cannot be revoked.
The law provides that a donation by reason of marriage may be revoked by
the donor if. among other cases, the marriage is judicially declared void ab initio
[par. (1) Art. 86. Family Code], or when the marriage is annulled and the donee
acted in bad faith [par. (3), Id.]. Since the problem states that the marriage was
annulled and there is no intimation of bad faith on the part of the donee Digna. the
conclusion is that the donor cannot revoke the donation.
c) Yes, the donation can be revoked.
The ground used in dissolving the marriage was the psychological immaturity
of George, which is not a ground for annulment of marriage. If this term is equated
with psychological Incapacity as used In Art. 36 of the Family Code, then it is a
ground for declaration of nullity of the marriage. Consequently, par. (1) of Art. 86,
FC, is the applicable law. Since Art. 86 of the FC makes no qualification as to who
furnished the ground or who was in bad faith in connection with the nullification of
the marriage, the conclusion is that Digna's father may revoke the donation and get
back the car.
02; Family Code; effect of divorce; void marriages
1980 No. II
(b) "E" and "F", Filipino citizens, were married in the Philippines. Later, they
separated. "E", the husband, went to San Francisco, California, to live there
permanently. He obtained a divorce in California from "F" on grounds of desertion
and mental cruelty. Thereafter, he married "G", a Filipina, who did not know that "E"
was previously married. "E" and "G" had two children "H" and "I". They came back to
the Philippines where "E" died.
Discuss (1) the validity of the divorce obtained by '"E" in California; (2) the
validity of the marriage of "E" to "G"; and (3) the legal status of "H" and "I".
Answer
(b) 1. The decree of absolute divorce obtained by "'E" in California is not valid
in the Philippines for the following reasons:
(a) Absolute divorce is not recognized in the Philippines. According to
the Civil Code, laws resulting to family rights and duties, or to the status,
condition, and legal capacity of persons are binding upon citizens of the Philippines,
even though living abroad (Art. 15).
(b) Well-settled is the rule that absolute divorce is contrary to public policy.
According to the Civil Code, this declaration of public policy cannot be rendered
nugatory by the decree of absolute divorce obtained by "E" in California (Art. 17,
par. 3).
Page 69 of 391
2. The marriage of "E" to "G" is not valid. It is void from the very beginning by
reason of a prior subsisting marriage (Art. 83, par. l Civil Code), From the point of
view of Philippine law, since the decree of absolute divorce obtained by "E" in
California is not valid, he is still married to "F".
3. "H" and "I" are natural children by legal fiction. The reason is that they are
children born of a marriage which is void (Art. 89, Civil Code).
02; Family Code; emancipation
1993 No. 15:
Julio and Lea, both 18 years old, were sweethearts. At a party at the house of
a mutual friend. Lea met Jake, also 18 years old, who showed interest in her. Lea
seemed to entertain Jake because she danced with him many times. In a fit of
jealousy. Julio shot Jake with his father's 38 caliber revolver which, before going to
the party he was able to get from the unlocked drawer Inside his father's bedroom.
Jake died as a result of the lone gunshot wound he sustained. His parents sued
Julio's parents for damages arising from quasi-delict. At the time of the incident,
Julio was 18 years old living with his parents. Julio's parents moved to dismiss the
complaint against them claiming that since Julio was already of majority age, they
were no longer liable for his acts.
1) Should the motion to dismiss be granted? Why?
2) What is the liability of Julio's parents to Jake's parents? Explain your
answer.
Answer:
1) No, the Motion to Dismiss should not be granted. Article 236 of the Family
Code as amended by Republic Act 6809, provides in the third paragraph that
"nothing in this Code shall be construed to derogate from the duty or responsibility of
parents and guardians for children and wards below twenty-one years of age
mentioned in the second and third paragraphs of Article 2180 of the Civil Code".
2) The liability of Julio's parents to Jake's parents arises from quasi-delict
(Arts. 2176 and 2180 Civil Code) and shall cover specifically the following:
a) P50,000.00 for the death of the son;
b) such amount as would correspond to lost earning capacity; and
c) moral damages.
02; Family Code; family
1991 No 1:
A. How does the 1987 Constitution strengthen the family as an Institution?
B. Do the Constitutional policy on the family and the provision that marriage
is the foundation of the family and shall be protected by the State bar Congress from
enacting a law allowing divorce in the Philippines?
Answer:
A. Sec, 2, Article II of the Constitution provides that:
The State recognizes the sanctity of family life and shall protect and
strengthen the family as a basic autonomous social institution. It shall equally protect
the life of the mother and the life of the unborn from conception. The natural and
primary right and duty of parents in the rearing of the youth for civic efficiency and
the development of moral character shall receive the support of the Government.
Section I, Article XV, further provides that:
Page 70 of 391
The State recognizes the Filipino family as the foundation of the nation.
Accordingly, it shall strengthen its solidarity and actively promote its total
development.
(Note: The Committee recommends that a citation of either one of the
provisions be credited as a complete answer).
Answer;
B, No, the Constitutional policy, as well as the supporting provision, does
not amount to a prohibition to Congress to enact a law on divorce. The Constitution
only meant to help the marriage endure, to "strengthen its solidarity and actively
promote its total development."
Alternative Answer;
B. Yes. Congress is barred from enacting a law allowing divorce, since
Section 2 of Article XV provides:
"Sec. 2. Marriage, as an inviolable social institution, is the foundation of the
family and shall be protected by the State."
Since marriage is "Inviolable", it cannot be dissolved by an absolute divorce.
02; Family Code; family home
1989 No. 4:
(2) What is "Family Home" and when is it deemed constituted? Who are the
beneficiaries thereof?
Answer:
The "Family Home" is the dwelling house where the husband, the wife, and
their family including the unmarried head of the family reside and the land on which
it is situated.
The "Family Home" is deemed constituted on a house and lot from the time it
is occupied as a family residence.
The beneficiaries of a family home are:
(1) The husband and wife, or an unmarried person who is the head of a
family; and
(2) Their parents, ascendants, descendants, brothers and sisters, whether
the relationship be legitimate or illegitimate, who are living in the family home and
who depend upon the head of the family for legal support.
02; Family Code; family home
1994 No. 13:
In 1991, Victor established judicially out of conjugal property, a family home
in Manila worth P200.000.00 and extrajudicially a second family home in Tagaytay
worth P50.000.00. Victor leased the family home in Manila to a foreigner. Victor and
his family transferred to another house of his in Pasig.
Can the two family homes be the subject of execution on a judgment against
Victor's wife for non-payment of the purchase in 1992 of household appliances?
Answer;
The two (2) so-called family homes can be the subject of execution. Neither
of the abodes are considered family homes because for purposes of availing the
benefits under the Family Code, there can only be one (1) family home which is
defined as the "dwelling house" where the husband and the wife and their family
Page 71 of 391
actually "reside" and the land on which it is situated. (Arts. 152 and 161, Family
Code)
02; Family Code; legal separation
2002 No II.
C. If drug addiction, habitual alcoholism, lesbianism or homosexuality
should occur only the marriage, would this constitute grounds for a declaration of
nullity or for legal separation, or would they render the marriage voidable? (1%).
SUGGESTED ANSWER:
C. In accordance with law, if drug addiction, habitual alcoholism,
lesbianism or homosexuality should occur only during the marriage, they:
1. Will not constitute as ground for declaration of nullity (Art. 36, Family
Code);
2. Will constitute as grounds for legal separation (Art. 56, FC) and
3. will not constitute as grounds to render the marriage voidable
(Art.45and 46, FC)
02; Family Code; legal separation
1989 No. 4:
{1} Cadio and Corona contracted marriage on June 1, 1982. A few days after
the marriage, Corona discovered that Cadio was a homosexual. As homosexuality
was not a ground for legal separation under the Civil Code, there was nothing that
Corona could do but bear with her problem. The couple, however, stated to live
separately. With the enactment of the Family Code, Corona decided to be legally
separated from Cadio based on the new ground of homosexuality. Corona brought
her action for legal separation on September 15, 1988. Will the action prosper? Give
your reasons.
Answer:
Yes, the action will prosper because the "cause" arose only on August 3,
1988, the effectivity of the Family Code and the action had not yet prescribed.
Alternative Answer:
The action will prosper. The offense of homosexuality as a continuing offense
can be a ground for legal separation. The prescriptive period of five years will apply
only when the offense has a fixed period of time and, therefore, the date of its
occurrence can be computed.
02; Family Code; legal separation; grounds
1975 No. V
What are the grounds for legal separation?
Answer
Under Article 97 of the Civil Code, a petition for legal separation may be filed:
(1) For adultery on the part of the wife and for concubinage on the part of
the husband as defined in the Penal Code; and
(2) An attempt by one spouse against the life of the other.
02; Family Code; legal separation; grounds; prescriptive period
1994 No 12;
Page 72 of 391
Rosa and Ariel were married in the Catholic Church of Tarlac, Tarlac on
January 5. 1988. In 1990, Ariel went to Saudi Arabia to work. There, after being
converted into Islam, Ariel married Mystica, Rosa learned of the second marriage of
Ariel on January 1, 1992 when Ariel returned to the Philippines with Mystica. Rosa
filed an action for legal separation on February 5, 1994,
1) Does Rosa have legal grounds to ask for legal separation?
2) Has the action prescribed?
Alternative Answers;
1) a) Yes, the abandonment of Rosa by Ariel for more than one (1) year is a
ground for legal separation unless upon returning to the Philippines, Rosa agrees to
cohabit with Ariel which is allowed under the Muslim Code. In this case, there is
condonation.
b) Yes. The contracting of a subsequent bigamous marriage whether in the
Philippines or abroad is a ground for legal separation under Article 55(7) of the
Family Code. Whether the second marriage is valid or not, Ariel having converted
into Islam, is immaterial.
2) No. Under Article 57 of the Family Code, the aggrieved spouse must file
the action within five (5) years from the occurrence of the cause. The subsequent
marriage of Ariel could not have occurred earlier than 1990, the time he went to
Saudi Arabia. Hence, Rosa has until 1995 to bring the action under the Family
Code.
02; Family Code; legal separation; proper party
1979 No. III
PB, a good for nothing gigolo, married RH, a rich old maid who inherited
considerable properties from both her deceased parents. Even after the marriage,
PB continued his philandering ways. When RH caught him having scandalous
relations with another woman, they had a terrible quarrel, so RH filed an action for
legal separation. After the evidence for both parties have already been submitted but
before the court had rendered a decision, RH died in an automobile accident leaving
no heirs but her husband and another spinster sister. PB then moved for the
dismissal of the action so the only sister of RH filed an opposition thereto and
prayed that she be substituted for RH contending that if the action were to be
dismissed, PB would be able to inherit a sizeable sum from the estate of the
deceased. Should the opposition and prayer for substitution be sustained?
Answer
The opposition and prayer for substitution cannot be sustained. An action for
legal separation which involves nothing more than the bed-and-board separation of
the spouses is purely personal. The Civil Code recognizes this. Being personal in
character it follows that the death of a party to the action causes the death of the
action.
Even if the action involves property rights, the abatement will still apply. A
review of the effects of legal separation and the corresponding rights thereunder
shows that they are solely the effects of legal separation; hence, they cannot survive
the death of the plaintiff if it occurs prior to the decree. (Lapuz Sy vs. Eugemio, 43
SCRA 1771),
02; Family Code; nullity; annulment; legal separation; grounds
1997 No. 5;
Page 73 of 391
Under what conditions, respectively, may drug addiction be a ground, if at all,
(a) for a declaration of nullity of marriage, (b) for an annulment of the marriage
contract, and (c) for legal separation between the spouses?
Answer:
(a) Declaration of nullity of marriage:
1. The drug addiction must amount to psychological incapacity to comply
with the essential obligations of marriage;
2. It must be antecedent (existing at the time of marriage),, grave and
incurable:
3. The case must be filed before August 1. 1998. Because if they got
married before August 3, 1998, it must be filed before August 1, 1998.
(b) Annulment of the Marriage Contract:
1. The drug addiction must be concealed; 2. It must exist at the time of
marriage;
3. There should be no cohabitation with full knowledge of the drug addiction;
4. The case is filed within five (5) years from discovery.
(c) Legal Separation;
1. There should be no condonation or consent to the drug addiction;
2. The action must be filed within five (5) years from the occurrence of the
cause.
3. Drug addiction arises during the marriage and not at the time of marriage.
02; Family Code; parental authority
2004 No. II
A. Distinguish briefly but clearly between: 2. Substitute parental authority
and special parental authority.
02; Family Code; parental authority; custody
1979 No. I
Boy and Girlie were married when "both were barely 20 years old, Boy being
then merely a student, while Girlie worked as a clerk in a business firm. They lived
with Boy's parents. Boy stopped schooling so he did not finish college and did not
have any job, but had tried but failed in all kinds of business. On the contrary he
incurred huge gambling debts. Girlie on the other hand had been continuously
promoted and rose to become executive secretary of the company where she
worked, receiving P5,000.00 a month salary. They begot one son to whom Boy's
parents became very attached. When the child was 6 years old, Boy's parents
brought the child with them to Baguio for the summer vacation. At this time, because
of the suspicious conduct of Girlie, Boy accused her of having an affair with another
man so they had a terrible quarrel. Girlie therefore went to live with her own parents.
After Boy's parents returned from Baguio with the child, Girlie went to fetch him but
Boy and his parents refused to surrender the child to her, saying that she is unfit to
have custody of the child because of her affair with another man. After serious
efforts for an amicable settlement failed, Girlie filed an action for custody of the child.
Will her action prosper? Why?
Answer
Girlie's action will prosper. True, the child is already six years old, and
therefore, the principle of custodial preference for the mother is not applicable.
Page 74 of 391
According to the law, this principle is applicable only to children under five years of
age (Child and Youth Welfare Code), But then, in cases involving custody of
children, the welfare or interest of the child is always paramount. With this as
criterion, the custody of the child should be awarded to the more suitable parent.
Morally and materially, Girlie has a definite edge over her husband Boy. She is self-
reliant. She had been continuously promoted and rose to become secretary of the
company where she works, and is now receiving P5,000.00 a month salary. That
she is having an affair with another man is baseless. There is no evidence which will
support it. Boy, on the other hand, is not a suitable guardian of his child. He is
addicted to gambling; he has no job; obviously, he has no character to speak of.
True, the parents of Boy are attached to the child, but if custody is awarded to Boy, it
will be Boy and not his parents who will exercise parental authority over the person
and property of the child.
Alternative Answer
Assuming that Girlie is having an affair with another man and that this can be
established by competent evidence, it is submitted that Girlie's action will not
prosper. It must be observed that the child is already six years old. Therefore, he
can be separated from his mother. Under the Child and Youth Welfare Code, the
principle of custodial preference for the mother is applicable only to children under
five years of age. Hence the paramount criterion to consider is the welfare of the
child. True, Boy is a gambler and has no job. But his parents who are well-to-do and
are attached to the child can supply the deficiency. Besides, Boy is still young. He
can still reform; he can still find a job. Furthermore, as between a father who is a
gambler and a mother who is immoral, preference should be given to the former.
02; Family Code; parental authority; custody
1981 No. 4:
Spouses, husband "H" and wife "W", Filipinos, with a child, decided that "H",
a doctor, would go to the U.S. to find employment there and then "W" would join
him. When in the U.S., "H" wrote that to establish U.S. residence, he would have to
obtain a divorce, marry an American girl, and once a U.S. resident, divorce his
American wife and then remarry "W" "W" agreed.
Four years later, "H", now a U.S. resident and having divorced his American
wife, returns to the Philippines and finds that his wife, "W", has married a wealthy
man in a ceremony celebrated in Hongkong and is happily living with him in Manila.
c) Could "H" obtain custody of the child?
d) Could "H" charge "W" with bigamy or adultery? Explain each of your
answers?
Answer
(c) If the child is under five years old, "H" cannot obtain custody over him.
According to the law, no child under five years of age shall be separated from the
mother unless the court finds compelling reasons to do so. ."Compelling reasons"
refer to reasons of health rather than reasons of morality. However, if the child is
already five years old or over, then it is the best interest of the child that will be
considered, taking into account the respective resources and the social and moral
situations of the contending parents.
(Note: The above answer is based on Art. 17, par. 3 PD 603 and on decided
cases, notably Unson vs. Navarro, L-52242, Nov. 17,1980.)
(d) "H" can charge "W" with adultery but not with bigamy. Undoubtedly, all of
the elements of adultery as deemed and punished in the Revised Penal Code are
Page 75 of 391
present. It is different in the case of bigamy. Since the bigamy was committed
outside of our territorial jurisdiction, it is not triable by our courts.
(Note: The above answer is based on the Revised Penal Code, especially
Art. 2 of said Code. See also, Salonga's Private International Law and Paras'
Conflict of Laws.)
02; Family Code; parental authority; rescission of adoption
1994 No, 7:
In 1975, Carol begot a daughter Bing, out of wedlock. When Bing was ten
years old, Carol gave her consent for Bing's legal adoption by Norma and Manuel,
which was granted by the court in 1990. In 1991, Carol learned that Norma and
Manuel were engaged in a call-girl-ring that catered to tourists. Some of the girls
lived with Norma and Manuel. Carol got Bing back, who in the first place wanted to
return to her natural mother.
1) Who has a better right to the custody of Bing. Carol or Norma?
2) Aside from taking physical custody of Bing, what legal actions can Carol
take to protect Bing? Alternative Answers;
1) a) It depends on whether or not Bing was at least 18 years old at the
time Carol asserts the prerogative to take custody of Bing. If she was at least 18
years old. then she is no longer under parental authority and neither Carol nor
Norma can assert the prerogative to take custody. However, if she was less than 18
years old, then Norma has a better right since the adoption by Norma of Bing
terminates the parental authority of Carol over Bing.
b) The natural mother, Carol, should have the better right in light of the
principle that the child's welfare is the paramount consideration in custody rights.
Obviously, Bing's continued stay in her adopting parents' house, where interaction
with the call girls is inevitable, would be detrimental to her moral and spiritual
development. This could be the reason for Bing's expressed desire to return to her
natural mother. It should be noted, however, that Bing is no longer a minor, being 19
years of age now. It is doubtfu1 that a court can still resolve the question of custody
over one who is sui juris and not otherwise incapacitated.
2) a) On the assumption that Bing is still a minor or otherwise
incapacitated, Carol may petition the proper court for resolution or rescission of the
decree of adoption on the ground that the adopting parents have exposed, or are
exposing, the child to corrupt influence, tantamount to giving her corrupting orders or
examples. She can also ask for the revesting in her of parental authority over Bing.
If. however. Bing Is already 19 years of age and therefore no longer a minor, it is not
Carol but Bing herself who can petition the court for judicial rescission of the
adoption, provided she can show a ground for disinheritance of an ascendant.
b) Carol may file an action to deprive Norma of parental authority under
Article 231 of the Family Code or file an action for the rescission of the adoption
under Article 191 in relation to Article 231 (2) of the Family Code.
02; Family Code; paternity and filiation
1999 No VI.
(a) Two (2) months after the death of her husband who was shot by unknown
criminal elements on his way home from office,( Rose married her childhood
boyfriend, and seven (7) months after said marriage, she delivered a baby. In the
absence of any evidence from Rose as to who is her child's father, what status does
the law give to said child? Explain. (2%)
Page 76 of 391
(b) Nestor is the illegitimate son of Dr. Perez. When Dr. Perez died, Nestor
intervened in the settlement of his father's estate, claiming that he is the illegitimate
son of said deceased, but the legitimate family of Dr. Perez is denying Nestor's
claim. What evidence or evidences should Nestor present so that he may receive his
rightful share in his father's estate? (3%)
ANSWER:
(a) The child is legitimate of the second marriage under Article 168(2) of the
Family Code which provides that a "child born after one hundred eighty days
following the celebration of the subsequent marriage is considered to have been
conceived during such marriage, even though It be born within three hundred days
after the termination of the former marriage."
(b) To be able to inherit, the illegitimate filiation of Nestor must have been
admitted by his father In any of the following: (1) the record of birth appearing in the
civil register, (2) a final judgment, (3) a public document signed by the father, or (4) a
private handwritten document signed by the lather (Article 17S in relation to Article
172 of the Family Code).
02; Family Code; paternity and filiation
2004 No. III
A. RN and DM, without any impediment to marry each other, had been living
together without benefit of church blessings. Their common-law union resulted in
the birth of ZMN. Two years later, they got married in a civil ceremony. Could ZMN
be legitimated? Reason. (5%)
02; Family Code; paternity and filiation
1989 No. 5:
(1) What are the grounds for impugning the legitimacy of a child?
Answer:
Legitimacy of a child may be impugned only on the following grounds:
(1) That it was physically impossible for the husband to have sexual
intercourse with his wife within the first 120 days of the 300 days which immediately
preceded the birth of the child because of:
(a) the physical incapacity of the husband to have sexual intercourse with
his wife;
(b) the fact that the husband and wife were living separately in such a way
that sexual intercourse was not possible; or
(c) serious illness of the husband, which absolutely prevented sexual
intercourse:
(2) That it is proved that for biological or other scientific reasons, the child
could not have been that of the husband, except in the instance provided in the
second paragraph of Art. 164; or
(3) That in case of children conceived through artificial insemination, the
written authorization or ratification of either parent was obtained through mistake,
fraud, violence, Intimidation, or undue influence.
{4) Felix, a Filipino doctor of medicine, married Monique, an Italian nurse, in
1985. It was later discovered that Monique cannot bear a child so that the couple
decided to adopt one. Can they jointly adopt Marie, the 19-year old niece of
Monique? Explain.
Answer:
Page 77 of 391
Since the child to be adopted is an Italian citizen, the joint adoption cannot be
effected. Had the child been a relative by consanguinity of the Filipino spouse, the
adoption would have been valid under the Philippine law.
02; Family Code; paternity and filiation
1995 No. 7:
Abraham died intestate on 7 January 1994 survived by his son Braulio.
Abraham's older son Carlos died on 14 February 1990.
Danilo who claims to be an adulterous child of Carlos intervenes in the
proceedings for the settlement of the estate of Abraham in representation of Carlos.
Danilo was legally adopted on 17 March 1970 by Carlos with the consent of the "
latter's wife.
1. Under the Family Code, how may an illegitimate filiation be proved?
Explain.
2. As lawyer for Danilo, do you have to prove Danilo's illegitimate filiation?
Explain.
3. Can Danilo inherit from Abraham in representation of his father Carlos?
Explain.
Answer:
1. Under Art. 172 in relation to Art. 173andArt. 175 of the FC, the filiation of
illegitimate children may be established in the same way and by the same evidence
as legitimate children. Art. 172 provides that the filiation of legitimate children is
established by any of the following: (1) the record of birth appearing in the civil
register or a final Judgment; or (2) an admission of legitimate filiation in a public
document or a private handwritten instrument and signed by the parent concerned.
In the absence of the foregoing evidence, the legitimate filiation shall be proved by:
(1) the open and continuous possession of the status of a legitimate child; or (2) any
other means allowed by the Rules of Court and special laws.
2. No. Since Danilo has already been adopted by Carlos, he ceased to be
an illegitimate child. An adopted child acquires all the rights of a legitimate child
under Art, 189 of the FC.
3. No, he cannot. Danilo cannot represent Carlos as the latter's adopted
child in the inheritance of Abraham because adoption did not make Danilo a
legitimate grandchild of Abraham. Adoption is personal between Carlos and Danilo.
He cannot also represent Carlos as the latter's illegitimate child because in such
case he is barred by Art. 992 of the NCC from inheriting from his illegitimate
grandfather Abraham.
Alternative Answer;
An adopted child's successional rights do not include the right to represent
his deceased adopter in the inheritance of the latter's legitimate parent, in view of
Art. 973 which provides that in order that representation may take place, the
representative must himself be capable of succeeding the decedent. Adoption by
itself did not render Danilo an heir of the adopter's legitimate parent. Neither does
his being a grandchild of Abraham render him an heir of the latter because as an
illegitimate child of Carlos, who was a legitimate child of Abraham, Danilo is
Incapable of succeeding Abraham under Art. 992 of the Code.
02; Family Code; paternity and filiation
1979 No. IV
Page 78 of 391
SL, a widower, died intestate leaving a big estate. In due course, his only
legitimate son, BL, executed an affidavit extrajudicially adjudicating unto himself title
to all the properties of the estate on the basis of which the properties were
registered in his name. CL, claiming to be an acknowledged natural child, filed an
action demanding for a share of the properties. As proof of her claim of being an
acknowledged natural child, she presented her marriage contract when she was
married at the age of 16, wherein it is stated that her father, SL, had given his
consent to said marriage. Will her action prosper? Why?
Answer
CL's action will not prosper. Under the Civil Code, voluntary recognition is
effected either by the record of birth, or by a will or by a statement before a court of
record, or by means of any authentic writing. By authentic writing, the law refers to
any genuine instrument, whether public or private, written in the handwriting of the
maker or duly signed by the maker in his handwriting Obviously, in this sense, a
marriage contract is not an authentic writing within the meaning of the law.
Therefore, CL is not an acknowledged natural child of SL. It would be different if she
had presented as additional evidence of her status the written consent of SL to her
marriage. That would have been sufficient to establish the fact of voluntary
recognition effected by means of an authentic writing. Her action in such a case will
then prosper.
02; Family Code; paternity and filiation
1982 No. 2
"A" and "B", man and woman not related to each other, both single and of
age, had an illicit relation. A child "C" was born out of that relation. Subsequently "A"
married "X". Notwithstanding the marriage, "A" and "B" continued their illicit relation,
and two years later, another child "D" was born to them. After the death of "X", "A"
married "B".
(a) What is the legal status of the children "C" and "D"?
(b) After the marriage, "A" and "B" recognized "C" and "D". What effect has
such recognition on the legal status of "C" and "D"? Reasons.
Answer
(a) "C" is a natural child, while "D" is an illegitimate child not natural (spurious
child).
According to the Civil Code, children born outside wedlock of parents, who, at
the time of the conception of the former, were not disqualified by any impediment to
marry each other are natural. It is clear that "C" falls within the purview of this
definition. However, in the case of "D", it is different. Since he was conceived at a
time when his father "A" was already married to "X", he is clearly an adulterous child.
He is, therefore, an illegitimate child not natural (spurious child).
(Note: The above answer is based on Arts. 269 and 287 of the Civil Code.
The Committee respectfully recommends that if the bar candidate attacks the
problem from the point of view of the two requisites which must be present in order
that child shall be considered as a natural child, it should be considered a correct
answer.)
(b) "C" is now promoted to the category of a legitimated child. "D", on the
other hand, is still an illegitimate child not natural (spurious child).
All of the requisites of legitimation are present in the case of "C". In the first
place, he is a natural child; in the second place, there was a .subsequent marriage
of the parents to each other; and in the third place, he was recognized by both of his
Page 79 of 391
parents as their child after the celebration of their marriage. Consequently, he is now
a legitimated child.
However, in the case of "D", being a spurious, and not a natural, child, he can
never be legitimated. Consequently, he is still an illegitimate child not natural
(spurious child).
(Note: The above answer is based on Arts, 269, 270 and 271 of the Civil
Code.)
02; Family Code; paternity and filiation
1983 No. 3
Out of the illicit relations between A, a married man, and B, an unmarried
woman, a child, C, was born two months before the death of A's wife. A month after
his wife's death, A married B.
What is C's status? Why?
Answer
C is a spurious child, its parents being incapable of marrying each other at
the time of its conception. Their subsequent marriage cannot, therefore, legitimize C.
02; Family Code; paternity and filiation
1985 No. 2
A and B were married on January 1, 1980, Two weeks later, on their way
home from honeymoon, the car A was driving turned turtle. A died instantly while B
was unharmed. A month thereafter, B had illicit relations with C. On October 15,
1980, B gave birth to X. In X's birth certificate, B declared that X's father is C.
Resolve the issue of X's paternity with reasons.
Answers:
1. X is presumed to be the legitimate child of A because X was born after
180 days from the celebration of the marriage and within 300 days after the
dissolution of the marriage due to the death of A. The declaration of B (the mother)
has no legal effect

2. A child born after 180 days following the celebration of a marriage, and
before 300 days following its dissolution, or the separation of the spouses, is quasi-
conclusively presumed to be legitimate. Against this presumption, no evidence shall
be admitted other than that of the physical impossibility of the husband having
access to his wife within the first 120 days (period of conception) of the 300 days
which preceded the birth of the child. (Macadangdang vs. Court of Appeals, L-
40542, 12 Sept. 1980). This physical impossibility may be caused by
1) The impotence of the husband;
2) The fact of the husband and wife living separately in such a way that
access was not possible; or
3) The serious illness of the husband.
(Art. 255, Civil Code).
None of the above exclusionary instances are shown. Accordingly, X must be
considered a legitimate child of A and B.
02; Family Code; paternity and filiation; rights of legitimate children
1990 No 8:
Page 80 of 391
B and G (college students, both single and not disqualified to marry each
other) had a romantic affair, G was seven months in the family way as of the
graduation of B. Right after graduation B went home to Cebu City. Unknown to G, B
had a commitment to C (his childhood sweetheart) to marry her after getting his
college degree. Two weeks after B marriage In Cebu City, G gave birth to a son E in
Metro Manila.
After ten years of married life in Cebu, B became a widower by the sudden
death of C in a plane crash. Out of the union of B and C, two children, X and Y were
born. Unknown to CT while on weekend trips to Manila during the last 5 years of
their marriage, B Invariably visited G and lived at her residence and as a result of
which, they renewed their relationship. A baby girl F was born to B and G two years
before the death of C. Bringing his family later to Manila, B finally married G.
Recently. G died.
What are the rights of B's four children: X and Y of his first marriage; and E
and F, his children with G? Explain your answer.
Answer:
Under the facts stated, X and Y are legitimate children of B and G. E is the
legitimate children of B and G. E is the legitimated child of B&G. F is the illegitimate
child of B and C. As legitimate children of B and C, X and Y have the following
rights:
(1) To bear the surnames of the father and the mother, in conformity with the
provisions of the Civil Code on Surnames;
(2) To receive support from their parents, their ascendants, and in proper
cases, their brothers and sisters, in-conformity with the provisions of the Family
Code on Support; and
(3) To be entitled to the legitime and other successional rights granted to
them by the Civil Code. (Article 174, Family Code). E is the legitimated child of B
and G. Under Art. 177 of the Family Code, only children conceived and born outside
of wedlock of parents who, at the time of the conception of the former, were not
disqualified by any impediment to marry each other may be legitimated. E will have
the same rights as X and Y. F is the illegitimate child of B and G. F has the right to
use the surname of G, her mother, and is entitled to support as well as the legitime
consisting of 1/2 of that of each of X, Y and E. (Article 176, Family Code)
02; Family Code; presumptive legitime
1999 No V
What do you understand by "presumptive legitime", in what case or cases
must the parent deliver such legitime to the children, and what are the legal effects
in each case if the parent fails to do so? (5%)
ANSWER
Presumptive legitime is not defined in the law. Its definition must have been
taken from Act 2710, the Old Divorce Law, which required the delivery to the
legitimate children of "the equivalent of what would have been due to them as their
legal portion if said spouse had died intestate immediately after the dissolution of the
community of property." As used in the Family Code, presumptive legitime is
understood as the equivalent of the legitimate children's legitimes assuming that the
spouses had died immediately after the dissolution of the community of property.
Presumptive legitime is required to be delivered to the common children of
the spouses when the marriage is annulled or declared void ab initio and possibly,
when the conjugal partnership or absolute community is dissolved as in the case of
Page 81 of 391
legal separation. Failure of the parents to deliver the presumptive legitime will make
their subsequent marriage null and void under Article 53 of the Family Code.
02; Family Code; property regime of unions without marriage
1998 No V.
In 1973. Mauricio, a Filipino pensioner of the U.S. Government, contracted a
bigamous marriage with Erlinda, despite the fact that his first wife, Carol, was still
living. In 1975. Mauricio and Erlinda jointly bought a parcel of riceland, with the title
being placed Jointly in their names. Shortly thereafter, they purchased another
property (a house and lot) which was placed in her name alone as the buyer. In
1981, Mauricio died, and Carol promptly filed an action against Erlinda to recover
both the riceland and the house and lot, claiming them to be conjugal property of the
first marriage. Erlinda contends that she and the late Mauricio were co-owners of the
riceland; and with respect to the house and lot, she claims she is the exclusive
owner. Assuming she fails to prove that she had actually used her own money in
either purchase, how do you decide the case? [5%]
Answer;
Carol's action to recover both the riceland and the house and lot is well-
founded. Both are conjugal property, in view of the failure of Erlinda, the wife In a
bigamous marriage, to prove that her own money was used In the purchases made.
The Supreme Court In a case applied Art. 148. Family Code, despite the fact that
the husband's death took place prior to the effectivity of said law. However, even
under Art. 144, Civil Code, the same conclusion would have been reached in view of
the bigamous nature of the second marriage.
Another Answer:
Under Article 148 of the Family Code, which applies to bigamous marriages,
only the properties acquired by both parties through their actual joint contribution of
money, property or industry shall be owned by them in common in proportion to their
respective contributions. Moreover, if one of the parties is validly married to another,
his share In the co-ownership shall accrue to the absolute community /conjugal
partnership existing in such valid marriage.
Thus, in this case, since Erlinda failed to prove that she used her own money
to buy the riceland and house and lot, she cannot claim to be the co-owner of the
riceland nor the exclusive owner of the house and lot. Such properties are
Mauricio's. And since his share accrues to the conjugal partnership with Carol, Carol
can validly claim such properties to the exclusion of Erlinda. (Art. 144, Civil Code).
02; Family Code; property regime of unions without marriage
2000 No II.
For five years since 1989, Tony, a bank Vice-president, and Susan, an
entertainer, lived together as husband and wife without the benefit of marriage
although they were capacitated to many each other. Since Tony's salary was more
than enough for their needs, Susan stopped working and merely "kept house".
During that period, Tony was able to buy a lot and house in a plush subdivision.
However, after five years, Tony and Susan decided to separate.
a) Who will be entitled to the house and lot ? (3%) SUGGESTED
ANSWER:
Tony and Susan are entitled to the house and lot as co-owners in equal
shares. Under Article 147 of the Family Code, when a man and a woman who are
capacitated to marry each other lived exclusively with each other as husband and
wife, the property acquired during their cohabitation are presumed to have been
Page 82 of 391
obtained by their joint efforts, work or industry and shall be owned by them in equal
shares. This is true even though the efforts of one of them consisted merely in his or
her care and maintenance of the family and of the household.
b) Would it make any difference if Tony could not marry Susan because he
was previously married to Alice from whom he is legally separated? (2%)
SUGGESTED ANSWER;
Yes, it would make a difference. Under Article 148 of the Family Code, when
the parties to the cohabitation could not marry each other because of an
impediment, only those properties acquired by both of them through their actual joint
contribution of money, property, or Industry shall be owned by them in common in
proportion to their respective contributions. The efforts of one of the parties in
maintaining the family and household are not considered adequate contribution in
the acquisition of the properties.
Since Susan did not contribute to the acquisition of the house and lot, she has
no share therein. If Tony cohabited with Susan after his legal separation from Alice,
the house and lot is his exclusive property. If he cohabited with Susan before his
legal separation from Alice, the house and lot belongs to his community or
partnership with Alice.
02; Family Code; property regime of unions without marriage
1992 No, 2:
In 1989, Rico, then a widower forty (40) years of age, cohabited with Cora, a
widow thirty (30) years of age. While living together, they acquired from their
combined earnings a parcel of riceland.
After Rico and Cora separated, Rico lived together with Mabel, a maiden
sixteen (16) years of age. While living together. Rico was a salaried employee and
Mabel kept house for Rico and did full-time household chores for him. During their
cohabitation, a parcel of coconut land was acquired by Rico from his savings.
After living together for one (1) year, Rico and Mabel separated. Rico then
met and married Letty, a single woman twenty-six (26) years of age. During the
marriage of Rico and Letty, Letty bought a mango orchard out of her own personal
earnings.
a) Who would own the riceland, and what property regime governs the
ownership? Explain.
b) Who would own the coconut land, and what property regime governs the
ownership? Explain.
c) Who would own the mango orchard, and what property regime governs
the ownership? Explain.
Answer:
(a) Rico and Cora are the co-owners of the riceland. The regime is that of co-
ownership (Art. 147, Family Code, first paragraph).
(Optional Addendum: However, after Rico's marriage to Letty, the half
interest of Rico in the riceland will then become absolute community property of Rico
and Letty.)
(b) Rico is the exclusive owner of the coconut land. The regime is a
sole/single proprietorship (Art. 148. Family Code, first paragraph is applicable, and
not Art. 147 Family Code).
(Optional Addendum: However, after Rico's marriage to Letty, the coconut
land of Rico will then become absolute community property of Rico and Letty.)
Page 83 of 391
(c) Rico and Letty are the co-owners. The regime is the Absolute
Community of Property (Arts, 75,90and9l, Family Code).
02; Family Code; property regime of unions without marriage
1978 No. II-b
A and B lived together publicly as husband and wife for fifteen (15) years in a
house in Bel Air Subdivision acquired during that time. A died intestate. His
legitimate wife C and his two (2) legitimate children sought to include said house in
the estate of the deceased. B objected on the ground that it was acquired during the
period of their cohabitation.
1. What rules govern the relationship of A and B with regard to property
acquired by them during the period of their cohabitation?
2. What conditions must be shown before B can claim any right over property
acquired during that relationship?
Answer
1. The relationship of A and B with regard to property acquired by either or
both of them through their work or industry or their wages and salaries shall be
governed by the rules on co-ownership.
2. B must show that she and A were living together as husband and wife
without getting married; that the property was acquired by either or both of them
through their work or industry or their wages and salaries during the period of her
cohabitation with A; and that she and A had really contributed to the acquisition of
the property.
(NOTE: The above answers are based on Article 144 of the Civil Code and
on Yaptinchay vs. Torres, 28 SCR A 489.)
02; Family Code; property relations
1997 No. 4:
Luis and Rizza, both 26 years of age and single, live exclusively with each
other as husband and wife without the benefit of marriage, Luis is gainfully
employed, Rizza is not employed, stays at home, and takes charge of the household
chores.
After living together for a little over twenty years, Luis was able to save from
his salary earnings during that period the amount of P200,000.00 presently
deposited in a bank. A house and lot worth P500,000.00 was recently purchased for
the same amount by the couple. Of the P500.000.00 used by the common-law
spouses to purchase the property, P200.000.00 had come from the sale of palay
harvested from the hacienda owned by Luis and P300.000,00 from the rentals of a
building belonging to Rizza. In fine, the sum of P500.000.00 had been part of the
fruits received during the period of cohabitation from their separate property, A car
worth P100.000.00. being used by the common-law spouses, was donated Just
months ago to Rizza by her parents.
Luis and Rizza now decide to terminate their cohabitation, and they ask you
to give them your legal advice on the following:
(a) How, under the law. should the bank deposit of P200,000.00t the house
and lot valued at P500.000.00 and the car worth P100.000.00 be allocated to them?
(b) What would your answer be (to the above question) had Luis and Rizza
been living together all the time, ie., since twenty years ago, under a valid marriage?
Answer:
Page 84 of 391
a) Art. 147 of the Family Code provides in part that when a man and a
woman who are capacitated to marry each other, live exclusively with each other as
husband and wife without the benefit of marriage or under a void marriage, their
wages and salaries shall be owned by them In equal shares and the property
acquired by both of them through their work or Industry shall be governed by the
rules of co-ownership.
In the absence of proof to the contrary, properties acquired while they lived
together shall be presumed to have been obtained by their Joint efforts, worker
Industry, and shall be owned by them In equal shares. A party who did not
participate in the acquisition by the other party of any property shall be deemed to
have contributed Jointly in the acquisition thereof if the former's efforts consisted in
the care and maintenance of the family and of the household.
Thus:
1) the wages and salaries of Luis in the amount of P200,000.00 shall be
divided equally between Luis and Rizza.
2) the house and lot valued at P500.000.00 having been acquired by both
of them through work or industry shall be divided between them in proportion to their
respective contribution, in consonance with the rules on co-ownership. Hence, Luis
gets 2\5 while Rizza gets 3\5 of P500.000.00.
3) the car worth P100,000.00 shall be exclusively owned by Rizza, the same
having been donated to her by her parents.
(b) The property relations between Luis and Rizza, their marriage having
been celebrated 20 years ago (under the Civil Code) shall be governed by the
conjugal partnership of gains, under which the husband and wife place in a common
fund the proceeds, products, fruits and income from their separate properties and
those acquired by either or both spouses through their efforts or by chance, and
upon dissolution of the marriage or of the partnership, the net gains or benefits
obtained by either or both spouse shall be divided equally between them (Art. 142.
Civil Code).
Thus:
1) The salary of Luis deposited in the bank in the amount of P200.000.00
and the house and lot valued at P500,000.00 shall be divided equally between Luis
and Rizza.
2) However, the car worth P1OO.000,00 donated to Rizza by her parents
shall be considered to her own paraphernal property, having been acquired by
lucrative title (par. 2, Art. 148, Civil Code).
02; Family Code; property relations
1975 No. VI
The husband sold with right of repurchase a parcel of land belonging to the
conjugal partnership without his wife's consent. Later the wife consented to the
extension of the period of conventional redemption agreed to by the buyer and her
husband. Is the sale valid or void? Why?
Answer
The sale, although originally voidable under Articles 166 and 173 for having
been entered into without the wife's consent, was ratified or validated when the wife
gave her conformity to the extension of the period of redemption. (Lanuza v. de
Leon, 20 SCRA 369)
02; Family Code; property relations
Page 85 of 391
1975 No. IX
A husband and his wife, with conjugal funds, constructed a building on a lot
owned by the wife's parents. Subsequently, the parents donated said lot to the wife.
Who now owns the land? Explain.
Answer:
When the lot was donated to the wife by her parents, it became her
paraphernal property as it was acquired by lucrative title during marriage under Art.
148(2) and the donation transmitted to her the rights of a land-owner over a building
constructed on it.
Art. 158, paragraph 2 applies when the building was constructed on land
belonging to one of the spouses. Since the lot belongs to the wife's parents at the
time the building was constructed. Article 158, paragraph 2 of the Civil Code is not
applicable. (Caltex v. Felias, 108 Phil. 873)
02; Family Code; property relations
1976 No. III-a
After a whirlwind courtship, A and B decide to marry each other and consult
you on property rights. Will a Mercedez-Benz sedan that A bought while still single,
continue to be his exclusive property? Explain.
Answer
Yes, according to Article 148, paragraph 1, the following shall be the
exclusive property of each spouse:
1. That which is brought to the marriage as his or her own; "x x x
xx x"
02; Family Code; property relations
1976 No. III-b
A & B married. Will the prizes of Fremyo Savings bonds purchased during the
marriage, be considered conjugal property?
Answer
Yes, according to Article 160, properties acquired during marriage is
presumed to belong to the conjugal partnership unless it is proved that it was bought
by exclusive funds of either spouse. Hence prizes of the bond shall be considered
an increment of the conjugal property.
02; Family Code; property relations
1976 No. III-c
May gambling debts contracted by either spouse without the consent of the
other, be charged against the community? Explain.
Answer
No, according to Article 164, whatever may be lost during the marriage in any
kind of gambling, betting or game, whether permitted or prohibited by law, shall be
borne by the loser, and shall not be charged to the conjugal partnership.
02; Family Code; property relations
1977 No. II-c
There are cases where the husband as administrator of the conjugal
partnership need not secure the consent of the wife in order to dispose or
encumber conjugal property. Mention five (5) of said cases.
Page 86 of 391
Answer
The husband does not have to secure his wife's consent in the following
cases:
a. Alienations or encumbrances of personal property belonging to the
conjugal partnership, except donations which are not moderate donations for
charity (Art. 174, CC).
b. Alienations or encumbrances of real property belonging to the conjugal
partnership acquired before the effectivity of the new Civil Code (Art. 166, CC),
except donations which are not moderate donations for charity (Art, 174, CC).
c. Alienations or encumbrances of real property belonging to the conjugal
partnership acquired after the effectivity of the new Civil Code in the following
cases:
(1) When the wife has been declared a non composmentis;
(2) When the wife has been declared a spendthrift;
(3) When the wife is under civil interdiction;
(4) When the wife is confined in a leprosarium;
(5) Moderate donations for charity;
(6) Donations made or promised to the common children for securing
their future or the finishing of a career; and
(7) Those made in order to pay the obligations of the conjugal partnership.
(See Arts. 166, 162, 171, 174, CC.)
The wife may bind the conjugal partnership in the following cases;
(1) When the obligation is contracted for the daily expenses of the family
(Art. 115, CC).
(2) When the obligation is contracted in her business or profession, provided
that it has redounded to the benefit of the family. (Art. 117, CC).
(3) When she acts as the agent of her husband.
(4) When the administration of the conjugal partnership has been transferred
to her (Arts. 112, 168, 196, CC).
(5) Moderate donations for charity (Art. 174. CC).
02; Family Code; property relations
1978 No. III-a
A and B, a year after marriage, built a residential house on land belonging to
the latter as her paraphernal property, using conjugal funds for its construction. Their
marital life proving unhappy, they agreed to separate. Neither took the trouble to
obtain judicial separation. Sometime later, a big fire reduced the house to ashes.
Upon the death of B, the wife, there was a liquidation of the conjugal property. A, the
surviving spouse, contended that the lot should form part of the conjugal estate. The
heirs of B, the deceased wife, claimed that after the house was burned, having the
lot vacant once more, it reverted to its status of being paraphernal. Decide the case
with reasons.
Answer
A's contention that the lot should form part of the conjugal estate is not
correct. Under the Civil Code, B retains her right of ownership of the lot until she is
paid its value. It is now a well-settled doctrine that payment of the value of the lot
Page 87 of 391
can be made only once the conjugal partnership is dissolved and there is a
subsequent liquidation of the conjugal partnership properties. In other words, before
the lot can be converted or transformed into conjugal property, it is essential that the
condition that its value shall be reimbursed to B or her legal heirs must be complied
with. Such reimbursement can only take place during the liquidation proceedings.
Obviously, compliance with this condition presupposes that the building constructed
on the lot must still be in existence at the time of liquidation of the conjugal
partnership properties. In the instant case, the building was destroyed before the
condition could be complied with. The lot, therefore, never ceased to be
paraphernal.
(NOTE: The above answer is based on Art. 158, par. 2, Civil Code and on
Vda. de Padilla vs. Paterno, 113 Phil 656. See also Maramba vs. Lorenzo, 20 SCR
A 474.)
02; Family Code; property relations
1985 No. 3
At the time of the dissolution of the marriage by the death of (the husband) A,
he and his wife, B, were possessed of the following properties:
1) A house and lot 1/3 of the price of which A paid before his marriage, 1/3
during the marriage from his salary, and the balance also during the marriage from
money B received in payment of a loan obtained from her while still single; and
2) An apartment house constructed on a parcel of lot donated to 6 prior to
the marriage.
Who owns the foregoing properties and what obligations, if any, does the
owner have for the improvements introduced thereon? Discuss.
Answer:
I) 1. The house and lot is a separate property of A subject to reimbursement
of 1/3 of the price to the conjugal partnership and another 1/3 to his wife B.
2. If the sale was made before the marriage and there is no reservation of the
ownership then the property is capital of the husband but he must reimburse the
conjugal partnership for 1/3 of the price and wife another 1/3 of the price.
3. If there is a reservation of ownership and the ownership will transfer
after the full payment of the price, then the property is 1/3 capital, 1/3 conjugal
and 1/3 paraphernal.
4. If it was acquired during the marriage, then the property is 1/3 separate,
1/3 conjugal and 1/3 paraphernal.
5. Inasmuch as the property was acquired and 1/3 of the price was paid out
of exclusive property, 1/3 from the conjugal property, and 1/3 from the exclusive
property of B, then each will own the property proportionately1/3 to A, 1/3 to the
conjugal partnership and 1/3 to B.
6. The house and lot is exclusive in nature, but A shall reimburse the
conjugal partnership of gains for the 1/3 price paid during the marriage which
came from conjugal funds {salary of A) and B for the 1/3 balance which was
sourced from her paraphernal (see Art. 148, in relation to Art. 153, Civil Code;
Lorenzo vs. Nicolas, 91 Phil 686).
2) 1. The apartment house is owned by the conjugal partnership subject to
reimbursement to B for the value of the land. Improvements made on separate
property from advancements of the conjugal partnership or the industry of either
spouse belong to the conjugal partnership.
Page 88 of 391
2. If the house was constructed at the expense of the conjugal funds, the
land become automatically conjugal property but the payment of the price could be
nude after the liquidation.
3. If the house was constructed before the donation, then the house and lot
are paraphernal.
4. The apartment house is conjugal if it was constructed during the marriage
utilizing conjugal funds. In this event, the lot shall also be considered conjugal and B
shall be considered a creditor of the partnership for the value of the lot payable upon
liquidation of the conjugal partnership (Art. 158, Civil Code; Calimlim-Canullas vs.
Judge Fortun) under other circumstances, or where the above conditions do not
concur, .said pieces of property may be or remain exclusive in nature.
02; Family Code; property relations
1994 No. 6;
Paulita left the conjugal home because of the excessive drinking of her
husband, Alberto. Paulita, out of her own endeavor, was able to buy a parcel of land
which she was able to register under her name with the addendum "widow." She
also acquired stocks in a listed corporation registered In her name. Paulita sold the
parcel of land to Rafael, who first examined the original of the transfer certificate of
title.
1) Has Alberto the right to share in the shares of stock acquired by Paulita?
2) Can Alberto recover the land from Rafael?
Alternative Answers:
1. a) Yes. The Family Code provides that all property acquired during the
marriage, whether the acquisition appears to have been made, contracted or
registered in the name of one or both spouses, is presumed to be absolute
community property unless the contrary is proved.
b) Yes. The shares are presumed to be absolute community property having
been acquired during the marriage despite the fact that those shares were
registered only in her name. Alberto's right to claim his share will only arise,
however, at dissolution.
c) The presumption is still that the shares of stock are owned in common.
Hence, they will form part of the absolute community or the conjugal partnership
depending on what the property regime is.
d) Since Paulita acquired the shares of stock by onerous title during the
marriage, these are part of the conjugal or absolute community property, as the case
maybe (depending on whether the marriage was celebrated prior to. or after, the
effectivity of the Family Code). Her physical separation from her husband did not
dissolve the community of property. Hence, the husband has a right to share in the
shares of stock.
2) a) Under a community of property, whether absolute or relative, the
disposition of property belonging to such community is void if done by just one
spouse without the consent of the other or authority of the proper court. However,
the land was registered in the name of Paulita as "widow". Hence, the buyer has the
right to rely upon what appears in the record of the Register of Deeds and should,
consequently, be protected. Alberto cannot recover the land from Rafael but would
have the right of recourse against his wife,
b) The parcel of land is absolute community property having been acquired
during the marriage and through Paulita's industry despite the registration being only
in the name of Paulita. The land being community property, its sale to Rafael without
Page 89 of 391
the consent of Alberto is void. However, since the land is registered in the name of
Paulita as widow, there is nothing in the title which would raise a suspicion for
Rafael to make inquiry. He, therefore, is an innocent purchaser for value from whom
the land may no longer be recovered.
c) No. Rafael is an innocent purchaser in good faith who, upon relying on the
correctness of the certificate of title, acquires rights which are to be protected by the
courts.
Under the established principles of land registration law, the presumption is
that the transferee of registered land is not aware of any defect in the title of the
property he purchased. (See Tojonera v. Court of Appeals, 103 SCRA 467).
Moreover, the person dealing with registered land may safely rely on the correctness
of its certificate of title and the law will in no way oblige him to go behind the
certificate to determine the condition of the property. [Director of Lands v. Abache, et
al. 73 Phil. 606). No strong considerations of public policy have been presented
which would lead the Court to reverse the established and sound doctrine that the
buyer in good faith of a registered parcel of land does not have to look beyond the
Torrens Title and search for any hidden defect or inchoate right which may later
Invalidate or diminish his right to what he purchased. (Lopez v. Court of Appeals.
189 SCRA 271)
d) The parcel of land is absolute community property having been acquired
during the marriage and through Paulita's industry despite registration only in the
name of Paulita. The land being community property, its sale to Rafael without the
consent of Alberto is void.
02; Family Code; property relations
1987 No. 12:
Spouses Martin and Tecla bought a parcel of land on installment. At the time
the total sale price was paid, Martin had left the conjugal abode and was cohabiting
with Tina. Notwithstanding such separation, Tecla religiously paid the installments
as they fell due out of her earnings from a small sari-sari store. After the total
purchase price had been paid, Martin had the property titled in the name of "Martin
married to Tina." Tecla died and her two children by Martin demanded partition of
the property and their mother's share. Mariin and Tina refused, claiming that the
property belonged to their "conjugal partnership." No proof was presented that
Martin married Tina during or after the death of Tecla.
To whom does the property titled in the name of "Martin married to Tina
belong? How would the property be divided among Martin, the two children of Martin
and
Tecla and Tina? Explain.
Answer:
The property is conjugal property, 1/2 belongs to Martin and the other
half of Tecla. However, 1/2 belonging to Tecla will be divided among Martin and the
two children, each of them getting 1/3 of that 1/2.
02; Family Code; property relations
1989 No, 3:
(1) What properties are excluded from the regime of absolute community of
property between spouses?
Answer:
The following shall be excluded from the community property:
Page 90 of 391
(1) Property acquired during the marriage by gratuitous title by
either spouse, and the fruits as well as the income thereof, if any, unless it is
expressly provided by the donor, testator or grantor that they shall form part of the
community property;
(2) Property for personal and exclusive use of either spouse; however,
jewelry shall form part of the community property;
(3) Property acquired before the marriage by either spouse who has
legitimate descendants by a former marriage, and the fruits as well as the
income, if any, of such property.
(2) When should the property relations of the spouses be mandatorily
governed by the regime of complete separation of property?
Answer:
Should the surviving spouse contract a subsequent marriage without
complying with the requirement that the community or conjugal property be
liquidated judicially or extra-judicially within one year from the death of the deceased
spouse, a mandatory regime of complete separation of property shall govern the
property relations of the subsequent marriage.
02; Family Code; property relations
1995 No. 9:
2. Suppose Tirso and Tessie were married on 2 August 1988 without
executing any ante nuptial agreement. One year after their marriage, Tirso while
supervising the clearing of Tessie's inherited land upon the latter's request,
accidentally found the treasure not in the new river bed but on the property of
Tessie. To whom shall the treasure belong? Explain.
Answer:
2. Since Tirso and Tessie were married before the effectivity of the Family
Code, their property relation is governed by conjugal partnership of gains. Under Art.
54 of the Civil Code, the share of the hidden treasure which the law awards to the
finder or the proprietor belongs to the conjugal partnership of gains. The one-half
share pertaining to Tessie as owner of the land, and the one-half share pertaining to
Tirso as finder of the treasure, belong to the conjugal partnership of gains.
02; Family Code; property relations
1980 No. I
(a) "A" and "B" were husband and wife. During their marriage, they built a
house on a lot exclusively owned by "B", the wife, with funds earned by "A", the
husband. They had no children. Upon the death of "B", her relatives claimed the
house and lot from "A", the husband. The husband refused.
Decide the controversy.
Answer
(a) "A", the husband, is correct when he refused to give the house and lot to
the relatives of "B".
The funds used in constructing the house are conjugal. This is so because
such funds were earned by "A". Therefore, the house is conjugal True, the lot upon
which the house is constructed is paraphernal But then, according to the Civil Code,
it will also become conjugal upon compliance with the condition that its value shall
be reimbursed by the conjugal partnership to the wife. This condition can be fulfilled
only during the liquidation of the conjugal partnership. This is well-settled. Assuming
then that this condition has been fulfilled, both house and lot are conjugal in
Page 91 of 391
character. One-half (1/2) thereof belongs to "A", while the other one-half (1/2)
belongs to the estate of "B".
To whom shall the estate of "B" pass? Assuming that "B" died intestate and
assuming further that the relatives of "B" are brothers and sisters and/or nephews
and nieces, one-half (1/2) thereof shall pass to "A" and the other one-half (1/2) shall
pass to such relatives in accordance with the rules of intestacy. However, if such
relatives are not brothers and sisters and/or nephews and nieces, the entire estate
shall pass to "A" alone in accordance with the rules of intestacy.
(NOTE: The above answer is based on Art. 158, par. 2, Civil Code, and on
the cases of Coingco vs. Flores, 84 Phil 284; Vda. de Padilla vs. Paterno, 113 Phil.
656; and Maramba vs. Lozano, 20 SCR A 474).
02; Family Code; property relations; charges
2000 No I
a) As finance officer of K and Co., Victorino arranged a loan of P5 Million
from PNB for the corporation. However, he was required by the bank to sign a
Continuing Surety Agreement to secure the repayment of the loan. The corporation
failed to pay the loan, and the bank obtained a judgment against it and Victorino,
jointly and severally. To enforce the judgment, the sheriff levied on a farm owned by
the conjugal partnership of Victorino and his wife Elsa. Is the levy proper or not?
(3%)
SUGGESTED ANSWER:
The levy is not proper there being no showing that the surety agreement
executed by the husband redounded to the benefit of the family. An obligation
contracted by the husband alone is chargeable against the conjugal partnership only
when it was contracted for the benefit of the family. When the obligation was
contracted on behalf of the family business the law presumes that such obligation
will redound to the benefit of the family. However, when the obligation was to
guarantee the debt of a third party, as in the problem, the obligation is presumed for
the benefit of the third party, not the family. Hence, for the obligation under the
surety agreement to be chargeable against the partnership it must be proven that
the family was benefited and that the benefit was a direct result of such agreement,
(Ayala Investment v. Ching, 286 SCRA 272)
02; Family Code; property relations; charges
1979 No. V
H was engaged in the business of buying and selling rice. In the course
thereof, he incurred a P20,000.00 indebtedness from his supplier of rice. Because of
his gambling losses, his business went bankrupt and soon thereafter, he and his
wife W quarreled and live separately ever since. A year after they had separated,
H's creditor sued him and obtained a favorable judgment which however cannot be
enforced against H because he had no property at all. H's creditor now seeks to
enforce the judgment against W's brand new car which she had bought out of her
salary from the company where she works. The car is registered in W's maiden
name which she had resumed using. May the car be levied upon to answer for the
judgment? Why?
Answer
Yes, W's car may be levied upon to answer for the judgment. It must be
observed that the obligation of H is an obligation which had redounded to the benefit
of his family. Under the Civil Code, the conjugal partnership is liable for the payment
of such obligation. The fact that H's business went bankrupt because of his gambling
losses does not affect the liability of the partnerships for the obligation. Neither does
Page 92 of 391
the separation de facto of H and W have any effect. Under the law, such separation
does not terminate the conjugal partnership of grains existing between them. But
how about the fact that the car was bought to W out of her salary? Under the Civil
Code, W's salary is conjugal. Subsequently, heir car which she bought with her
salary would then be classified as properly acquired during the marriage with
conjugal funds by onerous title. As to the same Code, the car is also conjugal. Being
conjugal, it may be levied upon to answer for the judgment.
02; Family Code; property relations; charges and expenses
1985 No. 4
A) Aside from the foregoing assets, A and B in the preceding question had
these obligations outstanding at the time of A's death
1) A debt contracted by B for the repair of the house and apartment
building above referred to; and
2) A shortage in A's account as cashier of the firm where he was employed:
Against whom are these obligations chargeable? Score your reasons.
Answers:
A) 1. With respect to the house since it is a separate property of the husband,
minor repairs are to be shouldered by the conjugal partnership but major repairs are
for the account of the owner (husband A). With respect to the apartment building,
which is conjugal, then the major and minor repairs are for the account of the
conjugal partnership.
2) The debt is chargeable against the conjugal partnership since it redounds
to the benefit of the partnership.
3) The debt contracted by B for the repair of the house is an exclusive
obligation of the husband A, but the repair cost on the apartment house being
conjugal in nature, although contracted by B, the debt shall be borne by the conjugal
partnership (Art. 161, Civil Code).
2) 1. If this shortage is a civil liability and the obligation is contracted by
A in his employment as cashier from which he derives the support of the family then
the shortage is chargeable against the conjugal partnership. But if A was convicted
and ordered to pay the shortage, it is chargeable against his separate property,
2. If the shortage wag incurred due to fraud or crime, it is chargeable against
the separate property of A. But if A acted in good faith, the shortage is chargeable
against the conjugal partnership since the obligation was incurred in the exercise of
his profession, which redounds to the benefit of the family.
3) Fines and pecuniary indemnities imposed upon the spouses shall not
be charged to the conjugal partnership; if, however, the spouse liable therefore has
no sufficient exclusive property, said obligation may be enforced against the
partnership assets after the responsibilities mentioned in Art. 151 would have been
covered, but at the time of .the liquidation of the partnership, the said spouse shall
be charged for such payment (Art. 161, Civil Code).
02; Family Code; property relations; conjugal partnership pf gains
1998 No VI.
In 1970, Bob and Issa got married without executing a marriage settlement.
In 1975, Bob inherited from his father a residential lot upon which, in 1981, he
constructed a two-room bungalow with savings from his own earnings. At that time,
the lot was worth P800.000.00 while the house, when finished cost P600,000.00. In
Page 93 of 391
1989t Bob died, survived only by his wife, Issa and his mother, Sofia. Assuming that
the relative values of both assets remained at the same proportion:
1. State whether Sofia can rightfully claim that the house and lot are not
conjugal but exclusive property of her deceased son. [3%]
2. Will your answer be the same if Bob died before August 3, 1988? [2%]
Answer;
1. Since Bob and Sofia got married In 1970, then the law that governs is the
New Civil Code (Persons), in which case, the property relations that should be
applied as regards the property of the spouses is the system of relative community
or conjugal partnership of gains (Article 119, Civil Code). By conjugal partnership of
gains, the husband and the wife place in a common fund the fruits of their separate
property and the income from their work or Industry (Article 142, Civil Code). In this
instance, the lot inherited by Bob in 1975 is his own separate property, he having
acquired the same by lucrative title (par. 2, Art. 148, Civil Code). However, the
house constructed from his own savings in 1981 during the subsistence of his
marriage with Issa is conjugal property and not exclusive property in accordance
with the principle of "reverse accession" provided for in Art. 158, Civil Code.
Another Answer:
1. Sofia, being her deceased son's legal heir concurring with his surviving
spouse (Arts. 985, 986 and 997, Civil Code), may rightfully claim that the house and
lot are not conjugal but belong to the hereditary estate of Bob. the value of the land
being more than the cost of the improvement (Art. 120, Family Code).
Answer:
2. Yes, the answer would still be the same. Since Bob and Issa contracted
their marriage way back in 1970, then the property relations that will govern is still
the relative community or conjugal partnership of gains (Article 119, Civil Code). It
will not matter if Bob died before or after August 3. 1988 (effectivity date of the
Family Code], what matters is the date when the marriage was contracted. As Bob
and Issa contracted their marriage way back in 1970. the property relation that
governs them is still the conjugal partnership of gains. (Art. 158, Civil Code)
Another Answer:
2. If Bob died be fore August 3, 1988. which is the date the Family Code
took effect, the answer will not be the same. Art. 158. Civil Code, would then apply.
The land would then be deemed conjugal, along with the house, since conjugal
funds were used in constructing it. The husband's estate would be entitled to a
reimbursement of the value of the land from conjugal partnership funds.
02; Family Code; property relations; dissolution of partnership; custody of
children
1978 No I-b
Ester Santos and Pedro Reyes were married on September 7, 1962 and had
two (2) children, a girl of four (4) years and a boy of eleven (11) years of age. In
1976, Ester left the conjugal home in Zamboanga and resided in Manila. Ester and
Pedro reached an amicable settlement respecting custody of children, support,
separation of property and dissolution of the conjugal partnership. It was agreed that
the custody of the girl be given to the mother and that of the boy to the father.
Subsequently, the mother questioned in court the validity of the agreement with
respect to the custody of her son.
1. Is the amicable settlement valid with respect to the separation of property
and dissolution of the conjugal partnership? Reasons,
Page 94 of 391
2. Is the amicable settlement valid with respect to the custody of the
children? Reasons.
Answer
1. The amicable settlement with respect to the separation of property and
dissolution of the conjugal partnership is not valid unless judicially approved. The
Civil Code is explicit. According to one provision, separation of property between the
spouses during the marriage shall not take place save by virtue of a judicial order.
According to another provision, every extrajudicial agreement, during marriage, for
the dissolution of the conjugal partnership between husband and wife shall be void
and of no effect.
(NOTE: The above answer is based on Articles 190 and 221 of the Civil
Code.)
2. The amicable settlement with respect to the custody of the children is valid
provided that it is for their best interest and welfare. It is a well-settled rule in this
jurisdiction that in questions involving- custody of children, their interest and welfare
are always paramount. As a matter of fact, the law now declares it. Here, the
custody of the girl, who is four years aid, is given to the mother. There is no violation
of the law either under the Civil Code or under the Child and Youth Welfare Code.
The custody of the boy, who is eleven years old, is given to the father. There is also
no violation of the law either under the Civil Code or under the Child and Youth
Welfare Code.
(NOTE: The above answer is based on decided cases as well as pertinent
provisions of the Welfare Code, such as Articles 8 and 17.)
02; Family Code; property relations; donations between spouses
1977 No. III-b
H donated a parcel of land to W, his common-law wife. Later, they married,
but soon afterwards H died, survived only by a sister, S and W. S sued to recover
the property donated, but W resisted. Decide with reasons.
Answer
The action will prosper but only with respect to one-half of the property. As
held by the Court of Appeals in a 1954 decision (Buenaventura vs. Bautista, 60 Off
Gaz, 3679), the prohibition of donations between spouses during the marriage is
applicable to common-law relationship for the following reasons: The reason behind
the law that the donation was probably due to undue and improper influence is also
true with greater force in extra-marital relations. Besides, so long as marriage
remains the cornerstone of our family law, reason and morality demand that the
disabilities which are attached to marriage should likewise be attached to extra-
marital relation.
The lack of validity of the donation made by H to W, however, does not mean
that the entire property shall be given to S Because of her marriage to H, W is
entitled to one-half of the property and the plaintiff, as the surviving sister, to the
other half (Art. 1001, Civil Code).
02; Family Code; property relations; marriage settlements
1995 No, 6;
On 10 September 1988 Kevin, a 26-year old businessman, married Karla. a
winsome lass of 18. Without the knowledge of their parents or legal guardians,
Kevin and Karla entered into an ante-nuptial contract the day before their marriage
stipulating that conjugal partnership of gains shall govern their marriage. At the time
Page 95 of 391
of their marriage Kevin's estate was worth 50 Million while Karla's was valued at 2
Million.
A month after their marriage Kevin died in a freak helicopter accident. He left
no will, no debts, no obligations. Surviving Kevin, aside from Karla, are his only
relatives: his brother Luis and first cousin Lilia.
1. What property regime governed the marriage of Kevin and Karla?
Explain.
2. Determine the value of the estate of Kevin,
3. Who are Kevin's heirs?
4. How much is each of Kevin's heirs entitled to inherit?
Answer;
1. Since the marriage settlement was entered into without the consent and
without the participation of the parents (they did not sign the document), the
marriage settlement is Invalid applying Art. 78, F.C. which provides that a minor
who according to law may contract marriage may also enter into marriage
settlements but they shall be valid only if the person who may give consent to the
marriage are made parties to the agreement. (Karla was still a minor at the time the
marriage settlement was executed In September 1988 because the law, R.A. 6809,
reducing the age of majority to 18 years took effect on 18 December 1989). The
marriage settlement being void, the property regime governing the marriage Is,
therefore, absolute community of property, under Art. 75 of the FC.
2. All the properties which Kevin and Karla owned at the time of marriage
became community property which shall be divided equally between them at
dissolution, Since Kevin owned 50 Million and Karla. 2 Million, at the time of the
marriage, 52 Million constituted their community property. Upon the death of Kevin,
the community was dissolved and half of the 52 Million or 26 Million is his share in
the community. This 26 Million therefore is his estate.
3. Karla and Luis are the Intestate heirs of Kevin.
4. They are entitled to share the estate equally under Article 1001 of the
NCC. Therefore. Karla gets 13 Million and Luis gets 13 Million.
02; Family Code; property relations; marriage settlements
1991 No 3:
Bar Candidates Patricio Mahigugmaon and Rowena Amor decided to marry
each other before the last day of the 1991 Bar Examinations. They agreed to
execute a Marriage Settlement. Rowena herself prepared the document in her own
handwriting. They agreed on the following: (1) a conjugal partnership of gains; (2)
each donates to the other fifty percent (50%) of his/her present property, (3) Rowena
shall administer the conjugal partnership property; and (4) neither may bring an
action for the annulment or declaration of nullity of their marriage. Both signed the
agreement in the presence of two (2) witnesses. They did not, however,
acknowledge it before a notary public.
(a) As to form, is the Marriage Settlement valid? May it be registered in the
registry of property? If not, what steps must be taken to make it registerable?
(b) Are the stipulations valid?
(c) If the Marriage Settlement is valid as to form and the above stipulations
are likewise valid, does it now follow that said Marriage Settlement Is valid and
enforceable?
Answer:
Page 96 of 391
A. Yes, it is valid as to form because it is in writing.
No, it cannot be registered in the registry of property because it is not a public
document. To make it registerable, it must be reformed and has to be notarized.
B. Stipulations (1) and (3) are valid because they are not contrary to law.
Stipulation (4) is void because it is contrary to law. Stipulation (2) is valid up to 1/5
of their respective present properties but void as to the excess (Art 84, Family
Code).
C. No. on September 15,1991. the marriage settlement is not yet valid and
enforceable until the celebration of the marriage, to take place before the last day of
the 1991 bar Examinations.
Alternative Answers:
A. Yes, it is valid as between the parties but not as against third persons.
No, because it is not a public document. To make it registerable, it must be reformed
and has to be notarized.
B. It depends. As between the parties, stipulations (1) and (3) are valid
because they are not contrary to law. Stipulation (2) is void because it is contrary to
law. Stipulation (2) is valid up to 1/5 of their respective present properties but void
as to the excess (Art. 84, Family Code).
02; Family code; property relations; separation of property
1984 No. 1
Spouses Pedro and Maria decided to separate, and to voluntarily dissolve
their conjugal partnership. Hence, they executed a public document wherein they
declared that they had no debts, that they were voluntarily dissolving their conjugal
partnership, and that each of them would thereafter be free to acquire or dispose of
any property independently of the other. Thereafter, they lived apart.
Pedro engaged in business which unfortunately failed. On the other hand,
Maria continued to be gainfully employed and was able to acquire properties through
her own efforts.
The creditors of Pedro obtained a judgment against the latter which they
could not satisfy because Pedro was insolvent-
Could the creditors of Pedro obtain satisfaction of the judgment out of the
properties of Maria? Explain.
Answer:
A. Furnished by the Office of Justice Palma
Yes, Under Act 190, the separation of properties between spouses during the
marriage shall not take place save in virtue of a judicial order. Not having been
submitted to the court for approval, the agreement to dissolve the conjugal
partnership is void and cannot have any legal effects. The properties acquired by
Maria, being conjugal in character, would therefore be answerable for the debts
incurred by Pedro in business.
B. Comments and Suggested Answer
We agree with the answer of the Bar Examiner. However, we suggest that
the following should also be accepted as a correct answer:
Yes, the creditors can obtain satisfaction of the judgment out of the properties
of Maria.
It is obvious that the properties of Maria are conjugal because they were
acquired through her own effort or industry (Art. 153, No. (2), Civil Code). It is also
Page 97 of 391
obvious that the obligations of Pedro are conjugal obligations because they have
benefited his family (Art. 1G1, No. (1), Civil Code). Therefore, creditors of Pedro can
proceed after the properties acquired by Maria.
But how about the agreement between Pedro and Maria to separate and
dissolve their conjugal partnership voluntarily? This agreement is void because it
was never approved by a competent court. Consequently, it cannot produce any
affect.
02; Family Code; requisites for valid marriages
1999 No III.
What is the status of the following marriages and why?
(a) A marriage between two 19-year olds without parental consent, (2%)
(b) A marriage between two 21-year olds without parental advice. (2%)
(c) A marriage between two Filipino first cousins in Spain where such
marriage is valid. (2%)
(d) A marriage between two Filipinos in Hongkong before a notary public.
(2%)
(e) A marriage solemnized by a town mayor three towns away from his
jurisdiction, (2%)
ANSWER:
(a) The marriage is voidable. The consent of the parties to the marriage
was defective. Being below 21 years old, the consent of the parties is not full without
the consent of their parents. The consent of the parents of the parties to the
marriage is indispensable for its validity.
(b) Between 21-year olds, the marriage is valid de-spite the absence of
parental advice, because such absence is merely an irregularity affecting a formal
requisitei.e., the marriage license-and does not affect the validity of the marriage
itself. This is without prejudice to the civil, criminal, or administrative liability of the
party responsible therefor.
(c) By reason of public policy, the marriage between Filipino first cousins is
void [Art. 38, par. (1), Family Code], and the fact that it is considered a valid
marriage in a foreign country in this case, Spain does not validate it, being an
exception to the general rule in Art. 96 of said Code which accords validity to all
marriage solemnized outside the Philippine x x x and valid there as such.
ALTERNATIVE ANSWER
The marriage it void. Under Article 96 of the Family Code, a marriage valid
where celebrated is valid in the Philippines except those marriages enumerated in
said Article which marriages will remain void even though valid where solemnized.
The marriage between first cousins is one of those marriages enumerated therein,
hence, it is void even though valid in Spain where it was celebrated.
By reason of Art. 15 in relation to Article 38 of the Civil Code, which applies to
Filipinos wherever they are, the marriage is void.
(d) It depends. If the marriage before the notary public is valid under
Hongkong Law, the marriage is valid in the Philippines. Otherwise, the marriage
that is invalid in Hongkong will be invalid in the Philippine!.
ALTERNATIVE ANSWER:
If the two Filipinos believed in good faith that the Notary Public is authorized
to solemnize marriage, then the marriage is valid.
Page 98 of 391
(e) Under the Local Government Code, a town mayor may validly solemnize
a marriage but said law is silent as to the territorial limits for the exercise by a town
mayor of such authority. However, by analogy, with the authority of members of the
Judiciary to solemnize a marriage, it would seem that the mayor did not have the
requisite authority to solemnize a marriage outside of his territorial jurisdiction.
Hence, the marriage is void, unless it was contracted with either or both parties
believing in good faith that the mayor had the legal authority to solemnize this
particular marriage (Art 35, par 2 Family Code).
ALTERNATIVE ANSWER:
The marriage is valid. Under the Local Government Code, the authority of a
mayor to solemnize marriages is not restricted within his municipality implying that
he has the authority even outside the territory thereof. Hence, the marriage he
solemnized outside his municipality is valid. And even assuming that his authority is
restricted within his municipality, such marriage will nevertheless, be valid because
solemnizing the marriage outside said municipality is a mere irregularity applying by
analogy the case of Navarro v Domagtoy, 259 Scra 129. In this case, the Supreme
Court held that the celebration by a judge of a marriage outside the jurisdiction of his
court is a mere irregularity that did not affect the validity of the marriage
notwithstanding Article 7 of the Family Code which provides that an incumbent
member of the judiciary is authorized to solemnize marriages only within the courts
jurisdiction.
ANOTHER ALTERNATIVE ANSWER:
The marriage is void because the mayor has no authority to solemnize
marriage outside his jurisdiction.
02; Family Code; requisites of marriage
1989 No. 2:
(1) Paul, a 17-year old Filipino and a permanent resident in the United
States, married Jean, a 16-year old American in Las Vegas, Nevada. The parents
of both gave their consent to the marriage. The marriage is valid in Nevada. Is its
also valid in the Philippines? Give your reasons.
Answer:
No, the marriage is not valid. Under the Family Code, the law requires that
the contracting parties are at least eighteen (18) years of age.
Alternative Answer:
If the marriage took place before the effectivity of the Family Code, the
marriage will be valid since under the provisions of the Civil Code a marriage valid in
the place of celebration is valid in the Philippines except bigamous, polygamous,
and incestuous marriages as determined by Philippine law. The minimum age under
the old law was sixteen (16) for the male and fourteen (14) for the female.
02; Family Code; requisites of marriage
1995 No. 18:
Isidro and Irma, Filipinos, both 18 years of age, were passengers of Flight
No. 317 of Oriental Airlines. The plane they boarded was of Philippine registry.
While en route from Manila to Greece some passengers hijacked the plane, held the
chief pilot hostage at the cockpit and ordered him to fly Instead to Libya. During the
hijacking Isidro suffered a heart attack and was on the verge of death. Since Irma
was already eight months pregnant by Isidro, she pleaded to the hijackers to allow
the assistant pilot to solemnize her marriage with Isidro. Soon after the marriage,
Page 99 of 391
Isidro expired. As the plane landed in Libya Irma gave birth. However, the baby died
a few minutes after complete delivery.
Back in the Philippines Irma Immediately filed a claim for Inheritance. The
parents of Isidro opposed her claim contending that the marriage between her and
Isidro was void ab initio on the following grounds: (a) they had not given their
consent to the marriage of their son; (b) there was no marriage license; (c) the
solemnizing officer had no authority to perform the marriage; and, (d) the
solemnizing officer did not file an affidavit of marriage with the proper civil registrar.
1. Resolve each of the contentions ([a] to [d]) raised by the parents of Isidro.
Discuss fully.
Answer;
1. (a) The fact that the parents of Isidro and of Irma did not give their
consent to the marriage did not make the marriage void ab initio. The marriage is
merely voidable under Art 45 of the FC.
(b) Absence of marriage license did not make the marriage void ab initio.
Since the marriage was solemnized in articulo mortis, it was exempt from the license
requirement under Art. 31 of the FC.
(c) On the assumption that the assistant pilot was acting for and In behalf of
the airplane chief who was under disability, and by reason of the extraordinary and
exceptional circumstances of the case [ie. hostage situation), the marriage was
solemnized by an authorized officer under Art. 7 (3) and Art. 31. of the FC.
(d) Failure of the solemnizing officer to file the affidavit of marriage did not
affect the validity of the marriage. It is merely an irregularity which may subject the
solemnizing officer to sanctions.
Alternative Answer:
Considering that the solemnizing officer has no authority to perform the
marriage because under Art. 7 the law authorizes only the airplane chief, the
marriage is void, hence, a, c, and d are immaterial.
02; Family Code; requisites of marriage
1976 No. II-a
A, 19, male and B, 17, female contract marriage. May the marriage be
annulled an the ground of underage? Explain.
Answer
It depends on whether the parental consent was given to the marriage. If the
parents did not give their consent, the marriage may be annulled; otherwise it cannot
be annulled.
02; Family Code; requisites of marriage
1976 No. II-c
If B is only 13 but her parents give their consent, is the marriage valid?
Explain.
Answer
No, The marriage is void.
02; Family Code; requisites of marriage
1984 No. 2
Page 100 of 391
In 1974, Alfredo married Alicia, then only 16 years old, without the consent of
the latter's parents. After the birth of their only child, Alice, they separated.
In 197G, Alfredo married Benita with whom he begot one child, Benedicta.
Again, the union was not successful and the couple eventually separated.
In 1978, Alfredo married for the third time, His new bride, Consuelo, bore him
one child, Connie.
Unknown to Alfredo, his first wife, Alicia, died in 1977.
Discuss the validity of each of the three marriages of Alfredo and the status of
each of his children.
Answer:
A, Furnished by Office of Justice Palma
The marriage of Alfredo to Alicia is voidable, but at the instance only of Alicia
who was only 16 years old at the time (Art. 85, par. 1). Alfredo cannot ask for the
annulment of this marriage. The child Alice is legitimate (Art. 89).
The marriage of Alfredo to Benita is void, for being bigamous (Art. 80, par. 4).
The child Benedicta is a natural child by legal fiction (Art. 89).
The marriage of Alfredo to Consuelo is valid, since there was no pre-existing
marriage, in view of the dissolution of the marriage to Alicia upon the latter's death,
coupled with the fact that the marriage to Benita is void. The child Connie is
legitimate.
B, Comments and Suggested Answer
We agree with the answer of the Bar Examiner. However, we suggest that
the following should also be accepted as a correct answer;
The marriage of Alfredo to Alicia is voidable because of lack of parental
consent (Art. 85, No. (1), Civil Code), Hence, it is valid and binding until it is annulled
by a competent court. The child Alice is, therefore, legitimate since the marriage of
Alfredo and Alicia was never annulled.
The marriage of Alfredo to Benita is void because Alfredo married Benita
when his marriage to Alicia was still subsisting in the sense that it has not yet been
annulled. Consequently, the marriage is bigamous, and therefore, void (Art. 80, No.
(4), Civil Code . Hence, the child Benedicta is a natural child by legal fiction (Art. 89,
Civil Code).
The marriage of Alfredo to Consuelo is valid because there was no longer
any legal impediment to said marriage when it was celebrated. Alicia was already
dead and, of course, Alfredo's marriage to Benita is void. The child Connie is,
therefore, legitimate.
02; Family Code; requisites of marriage & void marriage
1993 No. 1:
A and B, both 18 years old, were sweethearts studying in Manila. On August
3, 1988, while in first year college, they eloped. They stayed in the house of a
mutual friend in town X, where they were able to obtain a marriage license. On
August 30, 1988, their marriage was solemnized by the town mayor of X in his
office. Thereafter, they returned to Manila and continued to live separately in their
respective boarding houses, concealing from their parents, who were living in the
province what they had done. In 1992, after graduation from college, A and B
decided to break their relation and parted ways. Both went home to their respective
towns to live and work.
Page 101 of 391
1) Was the marriage of A and B solemnized on August 30, 1988 by the town
mayor of X in his office a valid marriage? Explain your answer.
2) Can either or both of them contract marriage with another person without
committing bigamy? Explain your answer.
Answer;
1) The marriage of A and B is void because the solemnizing officer had no
legal authority to solemnize the marriage. But if either or both parties believed in
good faith that the solemnizing officer had the legal authority to do so, the marriage
is voidable because the marriage between the parties, both below 21 years of age,
was solemnized without the consent of the parents. (Art. 35, par. (2) and Art. 45 par.
(1), Family Code)
2) Either or both of the parties cannot contract marriage In the Philippines
with another person without committing bigamy, unless there is compliance with the
requirements of Article 52 Family Code, namely: there must be a judgment of
annulment or absolute nullity of the marriage, partition and distribution of the
properties of the spouses and the delivery of their children's presumptive legitimes.
which shall be recorded in the appropriate Civil Registry and Registry of Property,
otherwise the same shall not affect third persons and the subsequent marriage shall
be null and void. (Arts. 52 and 53. Family Code)
Alternative Answer:
2) Yes, they can. The subsequent marriage contracted by one of the parties
will not give rise to bigamy even in the absence of a court declaration of nullity of the
first marriage. The subsistence of a prior valid marriage is an indispensable element
of the crime of bigamy. The prior court declaration of nullity of the first marriage is
required by the Family Code only for the purpose of the validity of the subsequent
marriage, not as an element of the crime of bigamy.
02; Family Code; requisites of marriage; marriage license
1996 No. 4:
On Valentine's Day. 1996. Ellas and Fely, both single and 25 years of age,
went to the city hall where they sought out a fixer to help them obtain a quickie
marriage. For a fee, the fixer produced an ante-dated marriage license for them,
Issued by the Civil Registrar of a small remote municipality. He then brought them to
a licensed minister in a restaurant behind the city hall, and the latter solemnized
their marriage right there and then.
1) Is their marriage valid, void or voidable? Explain. Answer:
The marriage is valid. The irregularity in the issuance of a valid license does
not adversely affect the validity of the marriage. The marriage license is valid
because it was in fact issued by a Civil Registrar (Arts. 3 and 4. FC).
Alternative Answer:
It depends. If both or one of the parties was a member of the religious sect of
the solemnizing officer, the marriage Is valid. If none of the parties is a member of
the sect and both of them were aware of the fact, the marriage is void. They cannot
claim good faith In believing that the solemnizing officer was authorized because the
scope of the authority of the solemnizing officer is a matter of law. If, however, one
of the parties believed in good faith that the other was a member of the sect, then
the marriage Is valid under Article 35 (2), FC. In that case, the party in good faith is
acting under a mistake of fact, not a mistake of law,
2) Would your answer be the same if it should turn out that the marriage
license was spurious? Explain.
Page 102 of 391
Answer;
No, the answer would not be the same. The marriage would be void because
of the absence of a formal requisite. In such a case, there was actually no valid
marriage license.
02; Family Code; requisites of marriage; marriage license
2002 No I.
On May 1, 1978 Facundo married Petra, by whom he had a son Sotero.
Petra died on July 1, 1996, while Facundo died on January 1, 2002. Before his
demise, Facundo had married, on July 1, 2002, Quercia. Having lived together as
husband and wife since July 1, 1990, Facundo and Quercia did not secure a
marriage license but executed the requisite affidavit for the purpose.
To ensure that his inheritance rights are not adversely affected by his father
second marriage, Sotero now brings a suit to seek a declaration of the nullity of the
marriage of Facundo and Quercia, grounded on the absence of a valid marriage
license. Quercia contends that there was no need for a marriage license in view for
her having lived continuously with Facundo for five years before their marriage and
that has Sotero has no legal personality to seek a declaration of nullity of the
marriage since Facundo is now deceased.
A. Is the marriage of Facundo and Quercia valid, despite the absence of a
marriage license? Explain. (2%)
SUGGESTED ANSWER:
A. The marriage with Quercia is void. The exemption from the requirement
of a marriage license under Art, 34, Family Code, requires that the man and woman
must have lived together as husband and wife for at least five years and without any
legal impediment to marry each other during those five years. The cohabitation of
Facundo and Quercia for six years from 1990 to July 1, 1996 when Petra died was
one with a legal impediment hence, not in compliance with the requirement of law.
On other hand, the cohabitation thereafter until the marriage on July 1, 2000,
although free from legal impediment, did not meet the 5-year cohabitation
requirement.
ALTERNATIVE ANSWER:
A. The marriage of Facundo and Quercia is VALID. The second marriage
was solemnized on July 1, 2000, when the Family code was already affective. The
family code took effect on August 3, 1988. Under the Family Code, no marriage
license is required if the parties have been cohabiting for the period of five years and
there is no legal impediment. There must no legal impediment ONLY AT THE TIME
OF THE SOLEMNIZATION OF THE MARRIAGE, and not the whole five years
period. This is clearly the intent of the code framers (see Minutes of the 150th joint
Civil Code of the Family Law Committees held on August 9, 1986). Also, in Manzano
V. Sanchez, AM NO. MT 00-129, March 8, 2001, the Supreme Court said that, as
one of the requisites for the exception to apply, there must be no legal impediment
at the time of the marriage. The Supreme Court did not say that the legal
impediment must exist all throughout the five-year period.
This is different from the case of Ninl V. Bayadog, (328 SCRA 122 [2000]).
In the said case, the situation occurred during the regime of the new Civil Code
where Article 76 thereof clearly provides that during the five-year cohabitation, the
parties must be unmarried. This is not so anymore in the Family Code. The Change
in the Family Code is significant. If the second marriage occurred before the
effectivity of the Family Code, the answer would that be that the marriage is void.
02; Family Code; requisites of marriage; solemnizing officers
Page 103 of 391
1994 No. 9:
1} The complete publication of the Family Code was made on August 4,
1987. On September 4. 1987. Junior Cruz and Gemma Reyes were married before
a municipal mayor. Was the marriage valid?
2) Suppose the couple got married on September 1, 1994 at the Manila Hotel
before the Philippine Consul General to Hongkong, who was on vacation in Manila.
The couple executed an affidavit consenting to the celebration of the marriage at the
Manila Hotel. Is the marriage valid?
Answer;
1) a) Yes, the marriage is valid. The Family Code took effect on August 3,
1988. At the time of the marriage on September 4, 1987, municipal mayors were
empowered to solemnize marriage under the Civil Code of 1950.
2) a) The marriage is not valid. Consuls and vice-consuls are empowered
to solemnize marriages between Philippine citizens abroad in the consular office of
the foreign country to which they were assigned and have no power to solemnize
marriage on Philippine soil.
b) A Philippine consul is authorized by law to solemnize marriages abroad
between Filipino citizens. He has no authority to solemnize a marriage in the
Philippines. Consequently, the marriage in question is void, unless either or both of
the contracting parties believed in good faith that the consul general had authority to
solemnize their marriage in which case the marriage is valid.
02; Family Code; retroactive application; vested rights
2000 No I
b) On April 15, 1980, Rene and Angelina were married to each other
without a marriage settlement. In 1985, they acquired a parcel of land in Quezon
City. On June 1, 1990, when Angelina was away in Baguio, Rene sold the said lot to
Marcelo. Is the sale void or voidable? (2%)
SUGGESTED ANSWER:
The sale is void. Since the sale was executed in 1990, the Family Code is the
law applicable. Under Article 124 of the FC, the sale of a conjugal property by a
spouse without the consent of the other is void.
Alternative Answer;
The sale is voidable. The provisions of the Family Code may apply
retroactively but only if such application will not impair vested rights. When Rene
and Angelina got married in 1980, the law that governed their property relations was
the New Civil Code. Under the NCC. as interpreted by the Supreme Court in Heirs of
Felipe v. Aldon, 100 SCRA 628 and reiterated in Heirs of Ayuste v. Malabonga, G.R
No, 118784, 2 September 1999, the sale executed by the husband without the
consent of the wife is voidable. The husband has already acquired a vested right on
the voidable nature of dispositions made without the consent of the wife. Hence,
Article 124 of the Family Code which makes the sale void does not apply.
02; Family Code; rights and obligations of husband and wife
1983 No. 2
The wife, A, left her husband, B, taking along their 2-year old son, and
demanded separate maintenance for herself and their child. B refused, alleging that
A abandoned him without cause (a claim she denies), that his income is small and
inadequate for his own support and that of his wife and child, if they lived apart from
him, and prayed that his wife and child return to the conjugal home where he would
Page 104 of 391
support them and that, if she is not willing to do so, to surrender to him the custody
of their son, she being- unfit to take care of him.
(a) May A be compelled to live with B again? Why? Is the existence of a just
cause far her abandonment of the conjugal home decisive of this question? Why?
(b) Are A and the child entitled to separate maintenance even if B's income
is small? Why?
Answer
(Examiner's Answer)
(a) A cannot be compelled, irrespective of whether she left the conjugal
home with or without a just cause. The law has not prescribed a remedy by which
the wife may be forced to live with her husband again.
(b) If A abandoned B for a just cause, she may demand separate
maintenance; otherwise, she cannot. The child must be supported regardless of
where he is and of the amount of the father's income. If A is entitled to support, she
too has a right to it irrespective of how much the husband earns. Whether big or
small, he must share it with his wife and child.
(Committee's Answer)
(a) A cannot be compelled to live with B again. The reason is that the
obligation of cohabitation is a purely personal obligation, an obligation to do. In this
type of obligation, there can be no such thing as an action for specific performance.
To compel the wife to return to the conjugal home would constitute an
infringement of her liberty. This is well-settled.
The existence of a just cause for A's abandonment of the conjugal home is
not decisive of the question of whether or not A can be compelled to live again with
B. The existence of a just cause for A's abandonment is material only when it comes
to the question of whether or not a wife can be compelled to comply with his
obligation of cohabitation.
(Note: The above answer is based on Arts. 109, 178 no. 1, Civil Code and-
on decided cases.)
02; Family Code; rights and obligations of husband and wife; family
residence; support; custody
1984 No. 3
Spouses Mario and Lorna and their five-year old child Max, were living with
Lornas mother in Manila, Mario was offered an attractive job in Baguio City and
wanted to relocate his family. Lorna, however, refused to go with him. Accepting the
offer, Mario left alone for Baguio City and started working, where he did well.
After six months, Lorna filed a case against Mario for support for Max and
herself. Mario answered that he was willing to support them in his house in Baguio
City. He asked the court to award to him the custody of his son should Lorna refuse
to live in Baguio City.
What are the rights and obligations of Mario and Lorna under the
circumstances? Explain.
Answer:
A. Furnished by Office of Justice Palma
Mario has the right to fix the residence of the family (Art. 110). Having chosen
to live and work in Baguio, Mario is deemed to have selected that place as the
location of the family residence. The husband and wife are obliged to live together
Page 105 of 391
and render mutual help and support (Art. 109). However, Lorna cannot be forced to
live with Mario by court order (Arroyo vs. Vasquez de Arroyo 42 Phil. 54). The
person obliged to give support may, at his option, fulfill his obligation either by
paying the allowance fixed, or by receiving and maintaining in his house the person
who has a right to receive support (Art. 299). Hence, should Lorna refuse to live in
Baguio, Mario's obligation to give support will be discharged.
In all questions regarding the care and custody of Max, his welfare shall be
the paramount consideration (Art 8, Child and Youth Welfare Code). However, even
if Mario is not awarded the custody of Max, he win still be required to give support.
B. Comments and Suggested Answer
We agree with the answer of the Bar Examiner.
02; Family Code; special parental authority; liability of teachers
2003 No VIII.
If during class hours, while the teacher was chatting with other teachers in the
school corridor, a 7 year old male pupil stabs the eye of another boy with a ball pen
during a fight, causing permanent blindness to the victim, who could be liable for
damages for the boys injury: the teacher, the school authorities, or the guilty boys
parents? Explain.
SUGGESTED ANSWER:
The school, its administrators, and teachers have special parental authority
and responsibility over the minor child while under their supervision, instruction or
custody (Article 218, FC). They are principally and solidarily liable for the damages
caused by the acts or omissions of the unemancipated minor unless they exercised
the proper diligence required under the circumstances (Article 219, FC). In the
problem, the teacher and the school authorities are liable for the blindness of the
victim, because the student who cause it was under their special parental authority
and they were negligent. They were negligent because they were chatting in the
corridor during the class period when the stabbing incident occurred. The incident
could have been prevented had the teacher been inside the classroom at that time.
The guilty boys parents are subsidiarily liable under Article 219 of the Family Code.
02; Family Code; support
1980 No. I
(b) A wife filed against her husband a complaint for legal separation on the
ground that the latter had attempted on her life, with a prayer for support pendente
lite. The husband opposed the grant of support on the ground of adultery on the part
of the wife. Over the husband's opposition, the Juvenile and Domestic Relations
Court granted support pendente lite. The husband instituted certiorari proceedings
before the Court of Appeals seeking to annul the award.
Is the wife entitled to support pendente lite?
Answer
(b) Yes, the wife is entitled to support pendente lite. It is true that adultery of
the wife is a valid defense against an action for support commenced by a wife
against her husband. This is well-settled. However, the alleged adultery must be
established by competent evidence. The mere allegation that the wife has
committed adultery will not bar her from the right to receive support pendente lite.
(NOTE: The above answer is based on Reyes vs. Ines-Luciano, 88 SCRA
803. A contrary answer may also be correct provided that the bar candidate qualifies
his answer by assuming that the husband was able to prove adultery of his wife. See
Page 106 of 391
Quintana vs. Lerma, 24 Phil. 285; Sanchez vs. Zulueta, 68 Phil. ,110; Mangoma vs.
Macadaeg, 90 Phil. 508).
02; Family Code; support
1985 No. 4
B) His fortune having been wiped out by business reverses, A demands
support from C, the descendant of A's deceased illegitimate son
Is C bound to give support to A? Discuss.
Answers:
B) 1. Yes, C is bound to give support to A since the descendants of
acknowledged natural children and natural children by legal fiction are obliged to
support their grandparents. The exception arises in case of descendants of
illegitimate children who are not natural who do not have to support their
grandparents.
2. No, C is not bound to give support to A since descendants of illegitimate
children who are not natural do not have to support their grandparents.
3. No, because in the case of illegitimate children the support is only
between the twobetween the parent and child. In case however, the illegitimate is
an acknowledged natural child or a natural child by legal fiction, then, C is bound to
support A.
4. We must distinguish whether the illegitimate is an acknowledged natural
son or a natural son by legal fiction or a spurious son. C is not bound to support A if
the illegitimate son is spurious. In the other two cases, C is bound to support A.
5. A parent may demand support from his acknowledged natural children and
the legitimate or illegitimate of the latter. But if the illegitimate child is not natural, the
descendants of the latter may not be obligated to sup-port said parent. (Art 291, in
relation to Art, 287, Civil Code). Accordingly, the right of A to demand support from
C would depend on whether or not B is a natural or non-natural child of A.
02; Family Code; void marriages
2004 No. X
A. BONI and ANNE met while working overseas. They became sweethearts
and got engaged to be married on New Years Eve aboard a cruise ship in the
Caribbean. They took the proper license to marry in New York City, where there is a
Filipino consulate. But as planned the wedding ceremony was officiated by the
captain of the Norwegian-registered vessel in a private suite among selected friends.
Back in Manila, Anne discovered that Boni had been married in Bacolod City
5 years earlier but divorced in Oslo only last year. His first wife was also a Filipina
but now based in Sweden. Boni himself is a resident of Norway where he and Anne
plan to live permanently.
Anne retains your services to advise her on whether her marriage to Boni is
valid under Philippine law? Is there anything else she should do under the
circumstances? (5%)
02; Family Code; void marriages
1986 No. 20:
Makisig and Marikit, who are first cousins, are Filipino citizens. They had a
romantic affair and begot a child Maganda, in the Philippines. They decided to
migrate to the Republic of Helios where marriages between first cousins are
allowed. They got married there. Subsequently, they drew up a joint affidavit
Page 107 of 391
recognizing and legitimizing Maganda. Years later, they returned to the Philippines.
They never lost their Philippine citizenship. What is the status of the marriage
between Makisig and Marikit? What is Maganda's status? Explain.
Answer:
The marriage is void. Filipino citizens are bound by the laws of the Philippines
with regard to their capacity to marry, regardless of where the marriage is
celebrated. This is an incestuous marriage which under Philippine laws is void from
the very beginning.
The child is an illegitimate child other than natural, or spurious.
Answer The marriage between first cousins is void even if valid in the
place where celebrated. Philippine law follows the citizen even if abroad, and such
marriage is considered incestuous under Philippine law.
The child cannot be legitimated because she is not a natural child as her
parents suffered from a legal impediment to marry each other at the time of her
conception. She is an illegitimate child, who is not a natural child.
02; Family Code; void marriages
1987 No, 14:
Lina married Hugo in a church ceremony, Hugo discovered that five years
before, Lina married Sixto in a civil ceremony. Lina however, did not know at the
time she married Sixto that the latter was already married, Upon learning that Sixto
was already married Lina immediately left Sixto and since then had not seen nor
heard from him. Lina, however, did not take any step to have her marriage with Sixto
annuled before she married Hugo.
Could Hugo successfully sue for a declaration of nullity of his marriage with
Lina? Explain,
Answer:
No, Hugo cannot sue for a declaration of nullity of his marriage with Lina. The
marriage of Lina with Sixto was void, so the marriage of Hugo with Lina is valid.
There was no need for a prior court action to declare the marriage with Sixto as void
(Yap vs. Court of Appeals, 145 SCRA 229).
Answer:
Yes, because the marriage of Hugo with Lina is not valid, for the reason that
there was no prior judicial declaration that the marriage with Sixto is void. Such
judicial declaration is required by law (Wiegel vs. Sempio Dy 143 SCR A 49 9 \
02; Family Code; void marriages
1989 No. 2:
(2) Cesar and Baby contracted marriage on June 15, 1983. A year later,
Baby bore a child, "X" The following year, the couple acquired a car and a
residential lot in Metro Manila. On September 1, 1988, the marriage was declared
void from the beginning by a competent court because Cesar was below 16 years of
age at the time of the marriage. Sometime in December, 1988, Cesar met Rosa with
whom he fell in love, Cesar married Rosa on January 15, 1989. On September 1,
1989, Rosa gave birth prematurely to a child, "Y". Is the marriage of Cesar and Rosa
valid? What is the status of the child "Y?" Give your reasons.
Answer:
If there was a liquidation of the properties of the first marriage and the
presumptive legitime of "X" was duly delivered, the second marriage is valid. If there
Page 108 of 391
was no such compliance, then the marriage is void. The child is legitimate since "Y"
was born a full year after the termination of the first marriage and during the second
marriage.
Alternative Answers:
A. The Family Code requires the registration of the judgment of nullity, the
partition of the properties and the delivery of the legitimes to be made with the
appropriate civil registry and registries of property. It further provides that failure to
comply with the said requirement shall render the marriage null and void. If there
was such a recording, the marriage is valid. Otherwise, the marriage is void.
Nevertheless, child "Y" is a legitimate child because it was born during the marriage
of Cesar and Rosa,
B. Since the problem does not state that there was compliance with the
requirements as to recording of judgment of nullity and the liquidation and delivery of
the presumptive legitime of the child "X", the marriage of Cesar and Rosa is void.
However, the child "Y" is legitimate because it was born during the marriage of
Cesar and Rosa.
02; Family Code; void marriages
1982 No. 1
"S", the son of "A", married "D", the daughter of "X". "X" is a sister of "A". "S"
and "D" begot a daughter "C". In 1979, while "D" was still living, "S" married "M", and
during their covertures, a child "E" was born.
What is the legal status
(a) of the marriage of "S" to "M" ?
(b) of "C", the child of "S" and "D"?
(c) of "E", the child of "S" and "M"? Reasons.
Answer
(a) The marriage of "S" to "M" is valid. The reason is obvious. The previous
marriage of "S" to "D" is void from the very beginning. Consequently, there is no
legal impediment to the marriage of "S" to "M".
It must be observed that "X", the mother of "D", is a sister of "A", the father of
"S". Hence, "S" and "D" are first cousins. They are, therefore, collateral relatives
within the fourth degree. Under the Civil Code, their marriage is incestuous, and as a
consequence, void from the very beginning. No judicial declaration of nullity is
necessary in order that it is void. Hence, "S" was free to marry "M" despite his
previous marriage to his first cousin "D".
(b) "C", the child of "S" and "D", is a natural child by legal fiction. According to
the Civil Code, children conceived or born of marriage which are void from the
beginning shall have the same status, rights and obligations as acknowledged
natural children and are called natural children by legal fiction. Since as stated
above, the marriage of "S" and "D" is void, it is clear that "C" is a natural child by
legal fiction,
(c) "E" is a legitimate child. As we have stated above, the marriage of "S" and
"M" is valid. As a consequence, their child "E" is a legitimate child.
(Note; The answer to (a) is based on Art. 81, Civil Code; the answer to (b) is
based on Art. 89, Civil Code, while the answer to (c) necessarily follows from the
answer to (a).)
02; Family Code; void marriages
Page 109 of 391
1982 No. 6
Nora, the 13-year old daughter of Mr. and Mrs. Garcia eloped with her first
cousin, Rene, to Hongkong where they got married. Upon their return to the
Philippines a month later, they set up residence in Quezon City. Mr. and Mrs. Garcia
lost no time in filing with the proper court a petition for a writ of habeas corpus to
regain custody of Nora. Nora opposes said petition contending that her marriage to
Rene has emancipated her from the authority of her parents-petitioners. Is the
contention of Nora legally tenable? Reasons.
Answer
The contention of Nora is legally untenable. The reason is crystal clear. Her
marriage to Rene in Hongkong has not emancipated her from the parental authority
of her parents for the simple reason that said marriage is not valid in the Philippines.
According to the Civil Code, marriages performed outside the Philippines in
accordance with the laws in force in the country where they were performed, and
valid there as such, shall also be valid in this country, except bigamous,
polygamous, or incestuous marriages as determined by Philippine law. The instant
case falls within the purview of the general rule. The marriage of Nora and Rene is
incestuous as determined by Philippine law. According to the Civil Code, marriages
between collateral relatives within the fourth civil degree are incestuous, and
therefore, void ab initio. Nora and Rene are first cousins. Therefore, they are
collateral relatives within the fourth civil degree. Hence, their marriage is incestuous,
and therefore, void ab initio.
(Note; The above answer is based on Art, 71 of the Civil Code. The
committee respectfully recommends that an answer based on Art. 15 of the Code,
arriving at the same conclusions, should also be considered correct.)
02; Family Code; void marriages; legal personality to seek declaration of
nullity
2002 No I.
On May 1, 1978 Facundo married Petra, by whom he had a son Sotero. Petra
died on July 1, 1996, while Facundo died on January 1, 2002. Before his demise,
Facundo had married, on July 1, 2002, Quercia. Having lived together as husband
and wife since July 1, 1990, Facundo and Quercia did not secure a marriage license
but executed the requisite affidavit for the purpose.
To ensure that his inheritance rights are not adversely affected by his father
second marriage, Sotero now brings a suit to seek a declaration of the nullity of the
marriage of Facundo and Quercia, grounded on the absence of a valid marriage
license. Quercia contends that there was no need for a marriage license in view for
her having lived continuously with Facundo for five years before their marriage and
that has Sotero has no legal personality to seek a declaration of nullity of the
marriage since Facundo is now deceased.
A. Is the marriage of Facundo and Quercia valid, despite the absence of a
marriage license? Explain. (2%)
B. Does Sotero have the personality to seek the declaration of nullity of
the marriage, especially now that Facundo is already deceased? Explain. (3%)
SUGGESTED ANSWER:
A. It is void (abs of marriage license)
B. A void marriage may be questioned by any interested party in any
proceeding where the resolution of the issue is material. Being a compulsory heir,
Soterro has the personality to question the validity of the marriage of Facundo and
Page 110 of 391
Quercia. Otherwise, his participation in the estate on Facundo would be affected.
(Ninl V. Bayadog, 328 SCRA 122 [2000] ).
02; Family Code; void marriages; property regime
1991 No 3;
In June 1985, James married Mary. In September 1988, he also married
Ophelia with whom he begot two (2) children, A and B, In July 1989, Mary died. In
July 1990, he married Shirley and abandoned Ophelia, During their union. James
and Ophelia acquired a residential lot worth P300,000.00.
Ophelia sues James for bigamy and prays that his marriage with Shirley be
declared null and void James, on the other hand, claims that since his marriage to
Ophelia was contracted during the existence of his marriage with Mary, the former is
not binding upon him, the same being void ab initio he further claims that his
marriage to Shirley is valid and binding as he was already legally capacitated at the
time he married her.
(a) Is the contention of James correct?
(b) What property regime governed the union of James and Ophelia?
(c) Is the estate of Mary entitled to a share in the residential lot acquired by
James and Ophelia?
Answer;
A. Yes. His marriage to Ophelia is void ab initio because of his subsisting
prior marriage to Mary, His marriage to Shirley, after Mary's death, is valid and
binding.
Alternative Answer;
A. No. The contention of James is not correct.
Art. 40, Family Code, provides that the "absolute nullity of a previous
marriage may be invoked for purposes of remarriage on the basis solely of a final
judgment declaring such previous marriage void." It can be said, therefore, that the
marriage of James to Shirley is void since his previous marriage to Ophelia,
although itself void, had not yet been judicially declared void,
Alternative Answer:
A. No. The contention of James is not correct. He cannot set up as a
defense his own criminal act or wrongdoing-
Answer;
B. The provisions of Art 148 of the Family Code, shall govern:
Art. 148. In cases of cohabitation not falling under the preceding Article, only
the properties acquired by both of the parties through their actual joint contribution of
money, property, or industry shall be owned by them in common in proportion to
their respective contributions. In the absence, of proof to the contrary, their
contributions and corresponding shares are presumed to be equal. The same rule
and presumption shall apply to joint deposits of money and evidences of credit.
C. It should be distinguished when the property was acquired.
If it was acquired before Mary's death, the estate of Mary is entitled to 1/2 of
the share of James.
If it was acquired after Mary's death, there will be no share at all for the estate
of Mary.
02; Family Code; void marriages; psychological incpacity
Page 111 of 391
2002 No II.
A. Give a brief definition or explanation of the term psychological
incapacity as a ground for the declaration of nullity of a marriage. (2%)
B. If existing at the inception of marriage, would the state of being of
unsound mind or the concealment of drug addiction, habitual alcoholism,
homosexuality or lesbianism be considered indicia of psychological incapacity?
Explain. (2%).
SUGGESTED ANSWER:
A. Psychological incapacity is a mental disorder of the most serious type
showing the incapability of one or both spouses to comply the essential marital
obligations of love, respect, cohabitation, mutual help and support, trust and
commitment. It must be characterized by Juridical antecedence, gravity and
incurability and its root causes must be clinically identified or examined. (Santos v.
CA, 240 SCRA 20 [1995]).
B. In the case of Santos v. Court of Appeals, 240 SCRA 20 (1995), The
Supreme Court held that being of unsound mind, drug addiction, habitual
alcoholism, lesbianism or homosexuality may be indicia of psychological incapacity,
depending on the degree of severity of the disorder. However, the concealment of
drug addiction, habitual alcoholism, lesbianism or homosexuality is a ground of
annulment of marriage.
Page 112 of 391
03; Succession; acceptance, repudiation, collation
1977 No. XIII-a
Distinguish acceptance and repudiation of inheritance from collation.
Answer
Acceptance is the act of an heir, legatee or devisee in manifesting his desire
in accordance with the formalities prescribed by law to succeed to the inheritance,
legacy or devise, while repudiation is the act of an heir, legatee or devisee in
manifesting his desire in accordance with the formalities prescribed by law not to
succeed to such inheritance, legacy or devise.
Collation, on the other hand, refers to the act of restoring to the common
mass of the hereditary estate, either actually or fictitiously, any property or right,
which a compulsory heir, who succeeds with other compulsory heirs, may have
received by way of donation or any other gratuitous title from the decedent during
the lifetime of the latter, but which is understood for legal purposes as an advance of
his legitime.
(NOTE: The above answer may be stated substantially).
03; Succession; amount of successional rights
2004 No. VIII
B. Mr. XT and Mrs. YT have been married for 20 years. Suppose the wife,
YT, died childless, survived only by her husband, XT. What would be the share of
XT from her estate as inheritance? Why? Explain. (5%)
03; Succession; barrier between illegitimate and legitimate relatives
1984 No. 9
A had two sons, one legitimate (B) and the other illegitimate (C), who both
died in a car accident. At the time of the accident, B was not married but had an
illegitimate son, D. C also had an illegitimate son, E. Upon learning of the death of
his sons, A suffered a heart attack and died.
Can D and E inherit from A? Explain Answer:
A. Furnished by Office of Justice Palma,
D, cannot inherit. The illegitimate child cannot inherit from the legitimate
relatives of his father or mother, {Art. 992).
E can inherit. The rights of illegitimate children are transmitted upon their
death to their descendants, legitimate or illegitimate. (Art 990).
B. Comments and Suggested Answer
We agree with the answer of the Bar Examiner.
03; Succession; barrier between illegitimate and legitimate relatives
1996 No. 11:
Cristina. the illegitimate daughter of Jose and Maria, died Intestate, without
any descendant or ascendant. Her valuable estate is being claimed by Ana, the
legitimate daughter of Jose, and Eduardo, the legitimate son of Maria.
Is either, both, or neither of them entitled to inherit? Explain.
Answer;
Neither Ana nor Eduardo is entitled to inherit of ab intestato from Cristina.
Both are legitimate relatives of Cristina's illegitimate parents and therefore they fall
Page 113 of 391
under the prohibition prescribed by Art. 992, NCC (Manuel v. Ferrer, 242 SCRA 477;
Diaz v. Court of Appeals, 182 SCRA 427).
03; Succession; barrier between illegitimate and legitimate relatives
1993 No. 2;
A is the acknowledged natural child of B who died when A was already 22
years old. When B's full blood brother, C, died he (C) was survived by his widow and
four children of his other brother. D. Claiming that he is entitled to inherit from his
father's brother. C, A brought suit to obtain his share in the estate of C.
Will his action prosper? Answer:
No, the action of A will not prosper. On the premise that B, C and D are
legitimate brothers, as an illegitimate child of B, A cannot inherit in intestacy from C
who is a legitimate brother of B. Only the wife of C in her own right and the
legitimate relatives of C (i.e. the children of D as C's legitimate nephews inheriting as
collateral relatives) can inherit in intestacy. (Arts. 992, 1001, 1OO5 and 975, Civil
Code)
Alternative Answer:
The action of A will not prosper. Being an illegitimate, he is barred by Article
992 of the Civil Code from inheriting ab intestato from the legitimate relatives of his
father.
03; Succession; barrier between legitimate and illegitimate relatives
1983 No. 9
A, a spurious child, died intestate survived by B, the brother of his deceased
mother, and C, his mother's legitimate granddaughter.
May B and C inherit from A? Reasons.
Answer
B cannot because uncles have no right to inherit from their illegitimate
nephews. C cannot succeed either because legitimate relatives have no right to
inherit from an illegitimate child and vice versa.
03; Succession; capacity to inherit; conditional devise
1980 No. VII
(a) In his will, Reverend Father "R' devised a parcel of riceland in favor of "his
nearest male relative who would study for the priesthood." The Will was duly
probated. No nephew of the testator claimed the devise and the testate proceeding
remained pending. In the interim, the riceland was to be administered by the Parish
Priest of the locality pursuant to a project of partition approved by the Probate Court.
Twenty-one years after the testator's death, the Parish Priest filed a petition
before the Court for delivery of the riceland to the Church as trustee. The legal heirs
of Father "R" objected and prayed instead that the bequest be declared inoperative
and that they be adjudged entitled to the riceland. It also turned out that the testator
had a grandnephew (a grandson of his first cousin) who was taking the holy orders
in a Seminary.
Would you construe the testamentary provision liberally so as to render the
trust operative and to prevent intestacy, or would you declare the bequest
inoperative and the legal heirs entitled to the riceland?
Answer
(a) It depends.
Page 114 of 391
If the Seminarian, who is presently studying for the priesthood, was born
before the death of Father "R", it is submitted that the testamentary provision should
be liberally construed so as to prevent intestacy. The land should be delivered to the
Parish Priest as trustee or administrator. The reason is obvious. There is always the
possibility that the seminarian might not become a priest. True, Father "K" devised
the land to his nearest nephew male relative who would study for the priesthood.
Apparently, the condition has already been fulfilled. It is however, submitted that the
testatorial intention is clear. The devisee must not only study for the priesthood; he
must become a priest. Once he becomes a priest, the land should then be delivered
to him.
If the seminarian was born after the death of Father "R", bequest is certainly
inoperative and the legal heirs of the testator shall, therefore, be entitled to the rice-
land. In other -words, the land shall be merged in the mass of the hereditary estate,
and from there, it shall pass to the legal heirs in accordance with the rules of
intestacy (Art. 956, Civil Code). The reason is clear. The seminarian cannot inherit
from Father "R", Under our law, in order to be capacitated to inherit, the heir, legatee
or devisee must be living at the moment the succession opens, except in case of
representation when it is proper. (Art. 1025, Civil Code).
(NOTE: The above problem is obviously a modification of the problem
resolved in Parish Priest of Victoria vs. Rigor, 89 SCRA 493. Because of the fact
that only one out of several conditions imposed by the testator was retained by the
Honorable Bar Examiner in the above hypothetical problem, the Committee
respectfully prays that either one of the above assumptions should be considered as
a correct answer. The Committee further prays that if a bar candidate assumes that
the seminarian was born before the death of Father "R" and then advances the
opinion that the land should be delivered to him because the condition has already
been fulfilled, such an answer should be considered as a correct answer.)
03; Succession; collation
1993 No. 17;
Joaquin Reyes bought from Julio Cruz a residential lot of 300 square meters
in Quezon City for which Joaquin paid Julio the amount of P300,000.00, When the
deed was about to be prepared Joaquin told Julio that it be drawn in the name of
Joaquina Roxas. his acknowledged natural child. Thus, the deed was so prepared
and executed by Julio. Joaquina then built a house on the lot where she, her
husband and children resided. Upon Joaquin's death, his legitimate children sought
to recover possession and ownership of the lot. claiming that Joaquina Roxas was
but a trustee of their father.
Will the action against Joaquina Roxas prosper? Answer:
Yes, because there is a presumed donation in favor of Joaquina under Art.
1448 of the Civil Code (De los Santos v. Reyes, 27 January 1992, 206 SCRA 437).
However, the donation should be collated to the hereditary estate and the legitime of
the other heirs should be preserved.
Alternative Answer;
Yes, the action against Joaquina Roxas will prosper, but only to the extent of
the aliquot hereditary rights of the legitimate children as heirs. Joaquina will be
entitled to retain her own share as an illegitimate child, (Arts. 1440 and 1453. Civil
Code; Art. 176, F. C.)
03; Succession; collation
1978 No. VII-a
Are the following subject to collation? Explain fully your answers.
Page 115 of 391
1. Gifts bestowed by the deceased father during his lifetime for the debts of a
son,
2. Money paid by the deceased parent during his lifetime for the debts of a
son.
Answer
1. Gifts bestowed by the deceased father during his lifetime to the spouse of
his son should not be subject to collation. This means that the law will not consider
such gifts as advances made by the decedent of the legitime of his son, and
therefore, chargeable against such legitime during the partition of the hereditary
estate. Instead, the law considers such gifts as ordinary donations inter vivos made
to a stranger, and therefore, chargeable against the disposable free portion of the
estate. It would be different if the gifts are bestowed to the spouses jointly. In such
case, one-half (1/2) of the value of such gifts would then be charged against the
legitime of the son and the other one-half (1/2) against the disposable free portion.
2. Money paid by the deceased parent during his lifetime for the debts of a
son should be brought to collation. In reality, what we have here is a donation inter
vivos made to a compulsory heir. From the point of view of the law, the money is
considered as an advance of the legitime. Consequently, in the portion of the
hereditary estate, the amount should be charged against the legitime of the son.
(NOTE: The above answers are based on Arts. 1066 and 1069 of the Civil
Code and on the view sustained by practically all commentators on the real meaning
of collation under Arts. 1061, et seq., of the Civil Code.)
03; Succession; disinheritance
1999 No VIII,
(a.) Mr. Palma, widower, has three daughters D, D-l and D-3. He executes a
Will disinheriting D because she married a man he did not like, and instituting
daughters D-1 and D-2 as his heirs to his entire estate of P 1,000,000.00, Upon Mr,
Raima's death, how should his estate be divided? Explain. (5%)
ANSWER:
(a) This is a case of ineffective disinheritance because marrying a man that
the father did not approve of is not a ground for disinheriting D. Therefore, the
institution of D-l and D-2 shall be annulled insofar as it prejudices the legitime of D,
and the institution of D-l and D-2 shall only apply on the free portion in the amount of
P500,000.00. Therefore, D, D-l and D-2 will get their legitimes of P500.000.00
divided into three equal parts and D-l and D-2 will get a reduced testamentary
disposition of P250,OOO.OO each. Hence, the shares will be:
D - P166,666.66
D-l P166,666.66 + P25O.OOO.OO
D-2 P166,666.66 + P250,000.00
03; Succession; disinheritance vs preterition
2000 No IV
In his last will and testament, Lamberto 1) disinherits his daughter Wilma
because "she is disrespectful towards me and raises her voice talking to me", 2)
omits entirely his spouse Elvira, 3) leaves a legacy of P100,000.00 to his mistress
Rosa and P50,000.00 to his driver Ernie and 4) institutes his son Baldo as his sole
heir. How will you distribute his estate of P1,000,000.00? (5%)
SUGGESTED ANSWER:
Page 116 of 391
The disinheritance of Wilma was ineffective because the ground relied upon
by the testator does not constitute maltreatment under Article 919(6) of the New Civil
Code. Hence, the testamentary provisions In the will shall be annulled but only to the
extent that her legitime was impaired. The total omission of Elvira does not
constitute preterition because she is not a compulsory heir in the direct line. Only
compulsory heirs in the direct line may be the subject of preterition. Not having been
preterited, she will be entitled only to her legitime. The legacy in favor of Rosa is
void under Article 1028 for being in consideration of her adulterous relation with the
testator. She is, therefore, disqualified to receive the legacy of 100,000 pesos. The
legacy of 50,000 pesos in favor of Ernie is not inofficious not having exceeded the
free portion. Hence, he shall be entitled to receive It. The institution of Baldo, which
applies only to the free portion, shall be respected. In sum, the estate of Lamberto
will be distributed as follows:

Baldo----------------- 450,000
Wilma--------------- 250,000
Elvira----------------- 250,000
Ernie----------------- 50,000

1,000,000

ALTERNATIVE ANSWER;
The disinheritance of Wilma was effective because disrespect of, and raising
of voice to, her father constitute maltreatment under Article 919(6) of the New Civil
Code. She is, therefore, not entitled to inherit anything. Her inheritance will go to the
other legal heirs. The total omission of Elvira Is not preterition because she is not a
compulsory heir in the direct line. She will receive only her legitime. The legacy in
favor of Rosa is void under Article 1028 for being in consideration of her adulterous
relation with the testator. She is, therefore, disqualified to receive the legacy. Ernie
will receive the legacy In his favor because it is not inofficious. The institution of
Baldo, which applies only to the free portion, will be respected. In sum, the estate of
Lamberto shall be distributed as follows:

Heir Legitime Legacy Institution TOTAL

Baldo 500,000 200.000 700,000
Elvira 250,000 250,000
Ernie 50,000 50,000
TOTAL 750,000 50,000 200,000 1,000,000

ANOTHER ALTERNATIVE ANSWER:
Same answer as the first Alternative Answer except as to distribution. Justice
Jurado solved this problem differently. In his opinion, the legitime of the heir who
was disinherited Is distributed among the other compulsory heirs in proportion to
their respective legitimes, while his share in the intestate portion. If any, is
distributed among the other legal heirs by accretion under Article 1018 of the NCC in
Page 117 of 391
proportion to their respective intestate shares. In sum the distribution shall be as
follows:

Heir Legitime Distribution
of Wilmas Legacy Institution TOTAL
Legitime

Baldo 250,0000 125,000 200,000 575,000
Wilma (250.000)
Elvira 250,000 125.000 375.000
Ernie 50,000 50.000

TOTAL 500,000 250,000 50,000 200,000 1,000,000

03; Succession; disinheritance vs preterition
1993 No. 7:
Maria, to spite her husband Jorge, whom she suspected was having an affair
with another woman, executed a will, unknown to him, bequeathing all the properties
she inherited from her parents, to her sister Miguela. Upon her death, the will was
presented for probate. Jorge opposed probate of the will on the ground that the will
was executed by his wife without his knowledge, much less consent, and that it
deprived him of his legitime. After all, he had given her no cause for disinheritance,
added Jorge in his opposition.
How will you rule on Jorge's opposition to the probate of Maria's will. If you
were the Judge?
Answer;
As Judge, I shall rule as follows: Jorge's opposition should be sustained in
part and denied in part. Jorge's omission as spouse of Maria is not preterition of a
compulsory heir in the direct line. Hence, Art. 854 of the Civil Code does not apply,
and the institution of Miguela as heir is valid, but only to the extent of the free portion
of one-half. Jorge is still entitled to one-half of the estate as his legitime. (Art. 1001,
Civil Code)
Alternative Answers;
a) As Judge, I shall rule as follows: Jorge's opposition should be sustained in
part and denied in part. This is a case of ineffective disinheritance under Art, 918 of
the Civil Code, because the omission of the compulsory heir Jorge by Maria was
intentional. Consequently, the institution of Miguela as heir is void only insofar as the
legitime of Jorge is prejudiced. Accordingly, Jorge is entitled to his legitime of one-
half of the estate, and Miguela gets the other half.
b) As Judge. I shall rule as follows: Jorge's opposition should be sustained.
This is a case of preterition under Article 854 Civil Code, The result of the omission
of Jorge as compulsory heir having the same right equivalent to a legitimate child "in
the direct line" is that total intestacy will arise, and Jorge will inherit the entire estate.
Page 118 of 391
c) As Judge, I shall rule as follows: the opposition should be denied since it
is predicated upon causes not recognized by law as grounds for disallowance of a
wll, to wit:
1) that the will was made without his knowledge;
2) that the will was made without his consent; and
3) that it has the effect of depriving him of his legitime. which is a ground
that goes into the intrinsic validity of the will and need not be resolved during the
probate proceedings. However, the opposition may be entertained for, the purpose
of securing to the husband his right to the legitime on the theory that the will
constitutes an ineffective disinheritance under Art. 918 of the Civil Code,
d) As Judge. I shall rule as follows: Jorge is entitled to receive his legitime
from the estate of his wife. He was not disinherited in the will even assuming that he
gave ground for disinheritance, hence, he is still entitled to his legitime. Jorge,
however, cannot receive anything from the free portion. He cannot claim
preterition as he is not a compulsory heir in the direct line. There being no
preterition, the institution of the sister was valid and the only right of Jorge is to claim
his legitime.
03; Succession; disinheritance, ineffective
1982 No. 13
"X"' s only living relatives are his brothers "A" and "B". "X" executed a will
providing as follows: "I institute my brother "A" as my sole and universal heir; and I
am disinheriting my brother "B" because he refused to support me when I had
nothing."
After "X"' s demise, is "B" entitled to share in the inheritance on the ground
that the disinheritance was ineffective because "X" had not proved that he in fact
refused to support the testator? Reason.
Answer
"B" is not entitled to share in the inheritance not on the ground that the
disinheritance was ineffective because "X" had not proved that he in fact refused to
support the testator. The reason is evident, "B" is not a compulsory heir. The law on
disinheritance applied only to compulsory heirs, never to voluntary heirs or to
legatees or devisees. Consequently, even assuming that indeed "X" had not proved
that "B" refused to support him, such fact would not have only effect whatsoever.
The act of "X" in disinheriting "B" is clearly a surplusage.
(Note: The above answer is based on Arts, 915, et seq. of the Civil Code.)
03; Succession; disinheritance, ineffective
1984 No, 10
A had two legitimate children, namely, B and C. He made a will, instituting G
and a friend, D, as his heirs and giving a P10,000 legacy to E, his former driver. He,
however, expressly disinherited B without specifying the reason therefore.
Assuming that A's net estate is worth P100,000 upon his death, how will it be
distributed?
Answer.
A. Furnished by Office of Justice Palma
The disinheritance of B is invalid, because there is no specification of the
cause therefore. However, the institution of the heirs will only be partially annulled
insofar as it may prejudice his legitime (Art. 918). The legacies and other
Page 119 of 391
testamentary dispositions remain valid insofar as it will not impair his legitime. B
therefore gets his legitime which is 1/4 of the estate, or P25,000.00. The legacy of
P10,000 to E will be paid. The balance of the estate of P65,000 will, be divided
equally between the instituted heirs, C and D.
B. Comments and Suggested Answer
We suggest that the following should be accepted as a correct answer:
The disinheritance of B is defective or imperfect because there is no
specification of the cause in the will as required by law. However, the institution of
heirs will only be partially annulled insofar as it may prejudice his legitime (Art 918,
Civil Code). Therefore B will still be entitled to his legitime which is 1/2 of 1/2 of
P100,000, or P25,000. The legacies, however, are valid so long as they are not
inofficious (Ibid.) It is obvious that the legacy of P10,000 given to E is not inofficious
because it can easily be contained in the free portion of P50,000. Therefore, E will
be entitled to such legacy. Since A had instituted as heirs his child C and his friend
D as heirs without designation of shares, therefore, applying the view of Manresa,
which has been adopted by commentators in this country, Tolentino among them (6
Manresa 98-99; 3 Tolentino 161; Art. 846, Civil Code), the legitime of C, which is 1/2
of 1/2 of P100,000 or P25,000, must first be separated and allotted to him because
the testator cannot deprive him of not. Then, the remainder of P40,000 which is the
disposable free portion, will be divided equally between C and D, the two instituted
heirs.
Consequently, the estate of P100,000.00 will be distributed as follows:
B P25,000 as compulsory heir; CP25,000 as compulsory heir;
P20,000 as voluntary heir; DP20,000 as voluntary heir; E P10,000
as legatee.
03; Succession; disinheritance; compulsory heirs
1977 No. XII-c
Who are compulsory heirs? Give five (5) instances which shall be sufficient
causes for the disinheritance of children and descendants, legitimate as well as
illegitimate.
Answer
In general, compulsory heirs are those for whom the law has reserved a
portion of the testator's estate which is known as the legitime.
In particular, the following are compulsory heirs:
(1) Legitimate children and descendants, with respect to their legitimate
parents and ascendants;
(2) In default of the foregoing, legitimate parents and ascendants, with
respect to their legitimate children and descendants;
(3) The widow or widower;
(4) Acknowledged natural children and natural children by legal fiction;
(5) Other illegitimate children referred to in Art. 287.
Compulsory heirs mentioned in Nos. 3, 4, and 5 are not excluded by those in
Nos, 1 and 2; neither do they exclude one another.
In all cases of illegitimate children, their filiation must be duly proved.
Page 120 of 391
The father or mother of illegitimate children of the classes mentioned shall
inherit from them in the manner and to the extent established by the Civil Code. (Art.
887, Civil Code).
The following shall be sufficient causes for the disinheritance of children and
descendants, legitimate as well as illegitimate:
(1) When a child or descendant has been found guilty of an attempt against
the life of the testator, his or her spouse, descendants, or ascendants;
(2) When a child or descendant has accused the testator of a crime for
which the law prescribes imprisonment for six years or more if the accusation has
been found groundless;
(3) When a child or descendant has been convicted of adultery or
concubinage with the spouse of the testator;
(4) When a child or descendant by fraud, violence, intimidation, or undue
influence causes the testator to make a will or to change one already made;
(5) A refusal without justifiable cause to support the parent or ascendant
who disinherits such child or descendant;
(6) Maltreatment of the testator by word or deed, by the child or descendant;
(7) When a child or descendant leads a dishonorable or disgraceful life;
(8) Conviction of a crime which carries with it the penalty of civil interdiction.
{Art. 919, Civil Code).
03; Succession; disinheritance; grounds
1989 No. 11:
(2) Jose and Ana are husband and wife. On January 10, 1980, Jose learned
that Ana was having illicit relations with Juan, In fact, Jose personally saw his wife
and Juan leaving a motel on one occasion. Despite all the evidence he had at hand,
Jose did not bring any action for legal separation against Ana. Instead, Jose simply
prepared a will wherein he disinherited Ana for her acts of infidelity. The validity of
the disinheritance was questioned by Ana upon Jose's death. If you were the judge,
how would you resolve this question? Give your reasons.
Answer:
The disinheritance is valid. Under the Civil Code, the legal ground for
disinheriting a spouse is that the spouse has given cause for legal separation.
Therefore, a final judgment is not needed.
Alternative Answer:
The disinheritance is not valid. The facts indicate that there was condonation
by Jose of Ana's illicit relationship with Juan since they appear to have continued to
live together,
03; Succession; donation of a spouses share
1982 No. 10
"H" and "W" are husband and wife. They have neither descendants or
ascendants. "H" died and while the conjugal partnership was under judicial
administration and pending liquidation, "W" donated all her share in her husband's
estate to a friend "F". "W" died while the proceeding for the settlement of the
conjugal partnership was pending. The collateral heirs of "W" and the administrator
of the estate brought an action against the donee, "F", to set aside the donation on
the ground that it is void, as it is a donation of future property. Decide with reasons.
Page 121 of 391
Answer:
The contention of the collateral heirs of "W" and the administrator of the
estate that the donation made by "W" to her friend "F" is void because the donation
is a donation of future property is untenable. The reason is crystal clear. According
to the Civil Code, by future property is understood anything which the donor cannot
dispose of at the time of the donation. Obviously, "W's" share in her husband's
estate does not fall within the purview of the definition. Because of the principle that
successional rights are transmitted at the very moment of the death of the decedent,
it is evident that "W" had a perfect right to donate her share in her husband's estate
to her friend "F".
(Note: The above answer is based on Arts 777 and 751 of the Civil Code.
The Committee recommends most respectfully that if the bar candidate arrives at
the same conclusion by invoking the provisions of Art. 493 of the Civil Code, he or
she should be properly credited.)
03; Succession; incapacity
1988 No. 7:
(b) Suppose that the beneficiary in a will is the wife of the minister of the
gospel who rendered aid to the testator during the letter's last illness, would she be
disqualified from inheriting from the testator? Explain.
Answer:
(b) We believe that the wife of the minister would not be disqualified from
inheriting from the testator. Under No. 2 of Art. 1027 of the Civil Code, the law
extends the disqualification of priests and ministers of the gospel to their relatives
within the fourth degree as well as to the church, order, chapter, community,
organization or institution to which they may belong. The spouse is not included.
Consequently, such spouse is not disqualified. Otherwise, we would be reading into
the law what is not found there. Besides, capacity to succeed is the general rule,
while incapacity to succeed is the exception. Hence, the rules on incapacity must
always be strictly construed.
Suggested Alternative Answer To: No. 7(b):
(b) If the testamentary disposition was actually intended to favor the Minister
as a disqualified person and was ostensibly made thru an intermediary, namely, the
wife, then the Minister is considered disqualified as the real and intended heir.
03; Succession; incapacity; effect of legal separation
1976 No. VI-c
In case of a legal separation between A and the widow, will the surviving
widow inherit? Explain.
Answer
It depends. If the widow is the guilty spouse, she cannot inherit. If she is the
innocent spouse, she may inherit. (Article 106, paragraph 4 1002)
03; Succession; intestate heirs
1995 No. 18:
Isidro and Irma, Filipinos, both 18 years of age, were passengers of Flight
No. 317 of Oriental Airlines. The plane they boarded was of Philippine registry.
While en route from Manila to Greece some passengers hijacked the plane, held the
chief pilot hostage at the cockpit and ordered him to fly Instead to Libya. During the
hijacking Isidro suffered a heart attack and was on the verge of death. Since Irma
was already eight months pregnant by Isidro, she pleaded to the hijackers to allow
Page 122 of 391
the assistant pilot to solemnize her marriage with Isidro. Soon after the marriage,
Isidro expired. As the plane landed in Libya Irma gave birth. However, the baby died
a few minutes after complete delivery.
Back in the Philippines Irma Immediately filed a claim for Inheritance. The
parents of Isidro opposed her claim contending that the marriage between her and
Isidro was void ab initio on the following grounds: (a) they had not given their
consent to the marriage of their son; (b) there was no marriage license; (c) the
solemnizing officer had no authority to perform the marriage; and, (d) the
solemnizing officer did not file an affidavit of marriage with the proper civil registrar.
2. Does Irma have any successional rights at all? Discuss fully.
Answer;
2. Irma succeeded to the estate of Isidro as his surviving spouse to the
estate of her legitimate child. When Isidro died, he was succeeded by his surviving
wife Irma, and his legitimate unborn child. They divided the estate equally between
them, the child excluding the parents of Isidro. An unborn child is considered born
for all purposes favorable to it provided it is born later. The child was considered
born because, having an intra-uterine life of more than seven months, it lived for a
few minutes after its complete delivery. It was legitimate because it was born within
the valid marriage of the parents. Succession is favorable to it. When the child died,
Irma inherited the share of the child. However, the share of the child in the hands of
Irma is subject to reserva troncal for the benefit of the relatives of the child within the
third degree of consanguinity and who belong to the line of Isidro.
Alternative Answer:
If the marriage is void. Irma has no successional rights with respect to Isidro
but she would have successional rights with respect to the child.
03; Succession; intestate heirs and sharing
1977 No. XIII-b
Should brothers and sisters of the full blood survive together with brothers
and sisters of the half blood, how much is the former entitled compared to that of the
latter?
Answer
The former shall be entitled to a share double that of the latter (Art. 1006,
Civil Code).
03; Succession; intestate heirs; shares
2003 No XII.
(b) Luis was survived by two legitimate children, two illegitimate children,
his parents, and two brothers. He left an estate of P1 million. Luis died intestate.
Who are his intestate heirs, and how much is the share of each in his estate?
SUGGESTED ANSWER:
(b) The intestate heirs are the two (2) legitimate children and the two (2)
illegitimate children. In intestacy the estate of the decedent is divided among the
legitimate and illegitimate children such that the share of each illegitimate child is
one - half the share of each legitimate child.
Their share are :
For each legitimate child P333,333.33
For each illegitimate child P166,666.66
Page 123 of 391
(Article 983, New Civil Code; Article 176, Family Code)
03; Succession; intestate proceedings; jurisdiction
2004 No. X
B. In his lifetime, a Pakistani citizen, ADIL, married three times under
Pakistani law. When he died an old widower, he left behind six children, two sisters,
three homes, and an estate worth at least 30 million pesos in the Philippines. He
was born in Lahore but last resided in Cebu City, where he had a mansion and
where two of his youngest children now live and work. Two of his oldest children are
farmers in Sulu, while the two middle-aged children are employees in Zamboanga
City. Finding that the deceased left no will, the youngest son wanted to file intestate
proceedings before the Regional Trial Court of Cebu City. Two other siblings
objected, arguing that it should be in Jolo before a Sharia court since his lands are
in Sulu. But Adils sisters in Pakistan want the proceedings held in Lahore before a
Pakistani court.
Which court has jurisdiction and is the proper venue for the intestate
proceedings? The law of which country shall govern succession to his estate? (5%)
03; Succession; intestate succession
1992 No 5:
F had three (3) legitimate children: A, B, and C. B has one (1) legitimate child
X. C has two (2) legitimate children: Y and Z.
F and A rode together in a car and perished together at the same time in a
vehicular accident, F and A died, each of them leaving substantial estates in
intestacy.
a) Who are the intestate heirs of F? What are their respective fractional
shares?
b) Who are the intestate heirs of A? What are their respective fractional
shares?
c) If B and C both predeceased F, who are Fs intestate heirs? What are
their respective fractional shares? Do they inherit in their own right or by
representation? Explain your answer.
d) If B and C both repudiated their shares in the estate of Ft who are F's
intestate heirs? What are their respective fractional shares? Do they inherit in their
own right or by representation? Explain your answer,
Answer:
(a) B = 1/2 (c) X = 1/2 by representation of B C=l/2 Y
= 1/4 by representation of C
(b) B = 1/2 Z = 1/4 by representation of C C= 1/2
Article 982 of the Civil Code provides that grandchildren inherit by right of
representation.
(d) X - 1/3 in his own right Y- 1/3 in his own right 2 - 1/3 in his own right
Article 977 of the Civil Code provides that heirs who repudiate their share
cannot be represented.
03; Succession; intestate succession
1976 No. VI-a
Page 124 of 391
A dies without a will, leaving a modern hotel, a fleet of air-conditioned buses
and three helicopters. If his widow and brothers survive, how will they inherit the
estate?
Answer
One-half (1/2) to the widow and 1/2 to the brothers and sisters, regardless of
their number. (Article 1001)
03; Succession; intestate succession
1976 No. VI-b
If the widow and three legitimate children are left, what will be the share of
the widow?
Answer
Under Article 996, each shall receive 1/4 as the Civil Code provides that the
widow shall have the same share as that of each legitimate child.
Alternative Answer
Under the theory of exclusion advocated by some authors, 1/6 shall go to the
widow and 5/6 shall be received by the three children, shared equally by each child.
Or, stated otherwise:
Each child receives his legitime; the children getting 1/2 of the estate and the
widow getting a share equivalent to the share of one child, and the remaining free
portion shall be divided equally among the three legitimate children.
03; Succession; intestate succession
2000 No XI
Eugenio died without issue, leaving several parcels of land in Bataan. He was
survived by Antonio, his legitimate brother; Martina, the only daughter of his
predeceased sister Mercedes; and five legitimate children of Joaquin, another
predeceased brother. Shortly after Eugenio's death, Antonio also died, leaving three
legitimate children. Subsequently, Martina, the children of Joaquin and the children
of Antonio executed an extrajudicial settlement of the estate of Eugenio, dividing it
among themselves. The succeeding year, a petition to annul the extrajudicial
settlement was filed by Antero, an illegitimate son of Antonio, who claims he is
entitled to share in the estate of Eugenio. The defendants filed a motion to dismiss
on the ground that Antero is barred by Article 992 of the Civil Code from inheriting
from the legitimate brother of his father. How will you resolve the motion? (5%)
SUGGESTED ANSWER:
The motion to dismiss should be granted. Article 992 does not apply. Antero
is not claiming any inheritance from Eugenio. He is claiming his share in the
inheritance of his father consisting of his father's share in the inheritance of Eugenio
(Dela Merced v, Dela Merced, Gr No. 126707, 25 February 1999).
ALTERNATIVE ANSWER:
It depends. If Antero was not acknowledged by Antonio, the motion to dismiss
should be granted because Antero is not a legal heir of Antonio, If Antero was
acknowledged , the motion should be denied because Article 992 is not applicable.
This is because Antero is claiming his inheritance from his illegitimate father, not
from Eugenio.
03; Succession; intestate succession
1978 No. VI-b
Page 125 of 391
A, deceased, is survived by a half-sister B on his father's aide and an aunt C
his mother's sister. He left as his only property that which was inherited from his
mother. He died intestate. Who shall succeed to A's estate? Reasons for your
answer.
Answer
B shall succeed to A's estate. The law of intestate succession is explicit.
Since both B and C are collateral relatives of the decedent A, therefore, the rule of
proximity is applicable. Relatives nearest in degree exclude the more distant ones, B
is a second degree relative of A, while C is a third degree relative. Besides, under
the general order of intestate succession, brothers and sisters, whether of the full or
half blood, are always preferred to uncles or aunts.
(NOTE: The above answer is based on Art. 967, Civil Code, and on Arts.
1004 to 1009, Civil Code. It is suggested that if a bar candidate should also discuss
the question of whether or not there is a reserva troncal, this should not prejudice
him, provided that he will arrive at the correct conclusion that A's estate is not
reservable.)
03; Succession; intestate succession
1997 No. 11:
"T" died intestate on 1 September 1997.He was survived by M (his mother),
W (his widow), A and B (his legitimate children), C (his grandson, being the
legitimate son of B), D (his other grandson, being the son of E who was a legitimate
son of, and who predeceased, "T"), and F (his grandson, being the son of G, a
legitimate son who repudiated the inheritance from "T"). His distributable net estate
Is P120.000.00.
How should this amount be shared in intestacy among the surviving heirs?
Answer:
The legal heirs are A, B, D, and W. C is excluded by B who is still alive. D
Inherits in representation of E who predeceased. F is excluded because of the
repudiation of G, the predecessor. M is excluded by the legitimate children of T. The
answer may be premised on two theories: the Theory of Exclusion and the Theory of
Concurrence.
Under the Theory of Exclusion the legitimes of the heirs are accorded them
and the free portion will be given exclusively to the legitimate descendants. Hence
under the Exclusion Theory:
A will get P20.000.00. and P 13.333.33 (1/3 of the free portion)
B will get P 20,000.00. and P13. 333.33 (1/3 of the free portion)
D will get P20.000.00. and P13. 333.33 (1/3 of the free portion)
W, the widow is limited to the legitime of P20.000.00
Under the Theory of Concurrence. In addition to their legitimes. the heirs of A,
B, D and W will be given equal shares in the free portions:
A: P20.000.00 plus P10.000.00 (1 /4 of the free portion)
B: P20,000.00 plus P10.000.00 (l/4 of the free portlon)
C: P20,000.00 plus P10.000.00 (1/4 of the free portion)
W: P20,000.00 plus P10,000.00 (l/4 of the free portion)
Alternative Answer: Shares in Intestacy
T - decedent Estate: P120.000.00
Page 126 of 391
Survived by:
M - Mother............................None
W - Widow.............................P 30,000.00
A - Son.................................P 30,000.00
B - Son.................................P3O.OOO.OO
C - Grandson (son of B).............None
D - Grandson (son of E who predeceased T)................P 30,000.00
F - Grandson (son of G who repudiated the Inheritance
from"T").......................None
Explanation:
1) The mother (M) cannot inherit from T because under Art. 985 the
ascendants shall inherit in default of legitimate children and descendants of the
deceased.
2) The widow's share is P30.000.00 because under Art, 996 it states that if
the widow or widower and legitimate children or descendants are left, the surviving
spouse has in the succession the same share as that of each of the children,
3) C has no share because his father is still alive hence succession by
representation shall not apply (Art. 975).
4) D inherits P30.000 which is the share of his father E who predeceased T
by virtue of Art. 981 on the right of representation.
5) F has no share because his father G repudiated the inheritance. Under
Article 977 heirs who repudiate their share may not be represented.
03; Succession; intestate succession
1998 No XII
Enrique died, leaving a net hereditary estate of P1.2 million. He is survived by
his widow, three legitimate children, two legitimate grandchildren sired by a
legitimate child who predeceased him, and two recognized illegitimate children.
Distribute the estate in intestacy. [5%]
Answer:
Under the theory of Concurrence, the shares are as
follows:
A (legitimate child) = P200.OOO
B (legitimate child) = P2OO.OOO
C (legitimate child) = P2OO,OOO
D (legitimate child) = O (predeceased]
E (legitimate child of D) = P100.0OO - by right of representation
F (legitimate child of D) = P100.OOO - by right of representation
G (illegitimate child) = P1OO.OOO - 1/2 share of ft legitimate child
H (illegitimate child) = P100.OOO - 1/2 share of a legitimate child
W (Widow) = P200.0OO - same share as legitimate child
Another Answer:
Page 127 of 391
Under the theory of Exclusion the free portion (P300.OOO) is distributed only
among the legitimate children and is given to them in addition to their legitime. All
other Intestate heirs are entitled only to their respective legitimes. The distribution is
as follows:
Legitime Free Portion Total
A [legitimate child) P15O.OOO + P 75,OOO - P225.OOO
B {legitimate child) P15O.OOO + F15O.OOO - P225.OOO
C (legitimate child) P15O.OOO + P 75.OOO - P225.OOO
D (legitimate child) 0 0
0
E (legitimate child of D) P 75,OOO + P35.5OO - P112.5OO
F (legitimate child of D) P 75.OOO + P 37.5OO - P112,5OO
G (illegitimate child) P 75.OOO 0 -P 75.5OO
H (illegitimate child) P 75.OOO O - P 75.5OO
W (Widow) P15O.OOO 0 -
P15O.OOO
03; Succession; intestate succession
1984 No, 8
Spouses Guillermo and Pacita had three sons, namely, A, B and C. Beset by
quarrels, their marriage broke up. Guillermo left for and obtained a divorce in the
United States, where he subsequently married Juana, by whom he had a son, D.
Guillermo later died in the United States without even knowing that C had died
earlier, leaving a wife, E, and a legitimate son, F,
State the shares, if any, of the following: in the estate of Guillermo: A, B, D, E,
F, Pacita and Juana.
Answers:
A. Furnished by Office of Justice Palma
The legitimate sons are entitled to one (1) share each, together with the
surviving spouse, Pacita. Since divorce is not recognized, Juana, the second wife, is
not an heir and gets nothing. D is a spurious child and will get 2/5 of the share of a
legitimate son, C having predeceased Guillermo, F, the legitimate son of C, will
inherit by right of representation. E the wife of C, has no right of representation and
will get nothing. Hence, the estate will be divided as follows:
A 5/22
B 5/22
F 5/22
Pacita 5/22
D 2/22
B. Comments and Suggested Answer
We suggest that the following should be accepted as a correct answer:
A shall be entitled to the share of a legitimate child. B shall also be entitled to
the share of a legitimate child.
D, being a natural child by legal fiction, shall be entitled to one-half (1/2) of
the share of A or B. It must be observed that Guillermo's marriage to Juana id void
Page 128 of 391
from the point of view of Philippine Law since the decree of absolute divorce
obtained by him against Pacita is not recognized as a valid decree (see Arts. 15, 71,
80, No. (4).
E shall not participate in the inheritance because she is not a legal heir of
Guillermo.
Pacita however, shall be entitled to the same share as A or B, being the
surviving spouse of Guillermo (Art. 999, Civil Code). F, the legitimate son of C, will
inherit by right of representation. Juana shall not participate in the inheritance
because she is not a legal heir of Guillermo. Hence, the proportionate shares of A,
B, F, Pacita, and D in the inheritance will be: (2 for A, 2 for B; 2 for F, 2 for Pacita,
and 1 for F or (2:2:2:2:1).
A's share will be 2/9 of the estate; D's share will be 2/9 of the estate; F's
share will be 2/9 of the estate; Pacita's share will be 2/9 of the estate; and D's share
will be 1/9 of the estate.
03; Succession; intestate succession
1985 No. 9
A) Among the properties in the estate of A, who died intestate and
without issue, were a farm, which came from his father, B, and a house, which he
acquired from C, B's father. In the partition of A's inheritance, the house was
allotted to B and the farm to D, A's mother. Upon the death of B and D, who were
simultaneously killed in a car accident, the farm was claimed by C and X, a child of
B And D born after A's death, while the house was claimed also by C and X and Y,
D's child by a prior marriage.
Decide the conflicting claims over the farm and the house in controversy with
reasons.
B) By a letter written before his death, the deceased distributed and
partitioned among his three (3) legitimate sons. A, B, and C, his property in such
manner that A received 17/24 thereof, B, 1/6 and C, 1/8. The letter not having been
made in accordance with the formalities required for the execution of wills, B and C
claimed that their father died intestate and his inheritance should be divided equally
among his children.
Decide their claims and distribute the estate among A, B and C stating the
reasons in support of your disposition.
C) A, a bachelor, named his brother, B as heir if his sister, S, dies within 10
years after A's death. B died 2 years after A's death while S died 1 year later, A's
estate" is claimed by B's only child and S's 6 children.
Who are entitled to it and how much will each receive? Discuss,
Answers:
A) 1. As regards the house, this property was acquired by A from his
grandfather C and was transmitted by A to B, his father. There is no reserva troncal
because there is no change of line. Hence, X alone is entitled to inherit the house.
With respect to the farm, the farm originally came from B, the father of A, and
from A it went to his mother D, There is a change of line from the paternal to the
maternal line. The farm is reservable property and must be acquired by relatives
within the third degree of the propositus and belonging to the paternal line.
Regarding the sharing, there are two theories. In the "delayed intestacy
doctrine," the preferences in the rules of intestate succession must be observed.
The second theory is to the effect that relatives in the same degree inherit in equal
Page 129 of 391
shares without distinction as to the direct or collateral line. Under the first theory, the
"delayed intestacy theory," C alone will inherit the farm because in intestacy, the
direct line excludes the collateral line. Hence, C, the grandfather, should exclude X,
the brother of A. Under the second theory; which allows no distinction as to direct or
collateral line, C and X will inherit the farm in equal shares since they are both 2nd
degree relatives of A, both belonging to the paternal line. In any case Y does not
inherit, since Y is not a reservatario.
2. With respect to the house, the property was inherited by B who belongs
to the paternal line. There will be no reason for making the property reservable
because there is no danger of the property going to another line. Hence, when B
died, C and X belonging to the same line from which the property came will inherit
it equally. With respect to the house, not being reservable, the heir of B will be X
alone as the descendant excludes the ascendant. With respect to the farm being
reservable in character, C and X belonging to the same line from which the property
came will inherit equally.
3. With respect to the farm it will go to X and Y. With respect to the house, it
will go to X as the lone child and legal heir of B.
4. There is no reserva and the properties will go by intestate succession,
the farm going to X and the house going to C, since there is no showing that the
properties were received by B and D by operation of law as the question merely
says that they were "allotted" in the partition, and by the propositus A by gratuitous
title as the question merely says that the farm "came" from his father B and a
house which he "acquired" from C, B's father without stating whether it "came" or
was "acquired" by gratuitous title.
6. In the event that both farm and house were acquired by gratuitous title and
were inherited by B and D by operation of law, there is in both cases reserva
because reserva can exist although the properties come from the same line. Hence,
the farm will go to C because the direct line excludes the collateral line. As regards
the house, there is also reserva although it came from the same line and will also go
to C for the same reason.
8. The farm should be awarded to X, the legitimate child of B, who is
preferred over C, the surviving parent of B. The farm is not reservable property
having origin-ally come from a line to which B likewise belongs (B in fact, was the
donor of the property). The house shall be awarded to C as the preferred
reservatario. The requisites of reserva troncal concurred in the case of the house
since the property was acquired by gratuitous title by B from C, a paternal
ascendant, and upon the death of A (propositus) the same property went by
operation of Law to D (reservista), his mother. Both X and C are reservatarios since
they belong to the line where the property originally came from and related within
three degrees from the propositus but since C belongs to the direct line of A and X
being only a collateral relative, C would be preferred over X.
B) 1. Under Art, 1080 of the Civil Code, a person may partition his property by
an act inter vivos or by will. Under our present law, there is no need for the owner of
the property to make a valid will. However, the partition inter vivos made by him
must not prejudice the legitimes. In this particular case, the partition prejudices the
legitime of C because actually each one should get 1/6, or 1/3 of 1/2, of his estate or
4/24. The partition made here is 17/24 which is 1/24 in excess of what he ought to
get, therefore, the final distribution should be:
A = 16/24 which includes the whole
1/2 plus 1/3 of the other half B = is entitled only to 4/24 and C = 4/24
Page 130 of 391
2. Since the letter was not made in accordance with the formalities required
for the execution of wills, the father died intestate. Hence, A, B and C will divide the
inheritance equally.
3. Despite the fact that a will is no longer required for the execution of a
partition inter vivos within the meaning of the law, nevertheless, the different
formalities which are necessary in order to convey property must still be complied
with. Hence, A, B and C will divide the inheritance equally.
4. B and C are correct. While it is very true that the letter of the deceased did
not comply with the formalities required for the, execution of the wills, nevertheless,
under the law, a partition inter vivos may be effected provided, of course, that there
will be compliance with all of the formalities required for ordinary conveyance of
properties such as when real properties are involved. In other words, the partition
that was effected by means of a letter does not state that all of the formalities
prescribed for ordinary conveyances of properties are complied with.
5. This is not a partition because partition presupposes a division/separation
of a property. This is merely assigning an aliquot portion of the property. Therefore,
it is not a real partition contemplated under Article 1080. It should follow the
requirements of a will, and because it does not comply with the requirements of the
will, intestate succession will follow.
6. A shall be entitled to 4/6, B to 1/6 and C to 1/6 of the estate. Under Art,
1080 of the Civil Code, a person may partition his estate during his lifetime. Unlike
that which obtained during the regime of the old Civil Code, the present Code would
appear to permit a person to distribute his estate during his lifetime without having to
execute a will. A conflict of views among Civil Law commentators arises only where
the participants in the partition, or some of them, are voluntary heirs. Since those
who were given entitlement under the partition were all legitimate children, and there
being no one apparently preterited the partition can take effect but without prejudice
to their respective legitimes. The share given to C is less than his legitime for which
reason that share must be increased to 1/6 of the estate.
C) 1. The estate of A will be distributed in intestacy among the 6 children of S
and the child of B. The condition embodied by A was ineffective because B died
ahead of S. Hence, the condition was not fulfilled while B was alive. Intestate
succession for B and S will be determined as to the rights existing on A's death,
hence, one-half will go to the children of S and the other half to the child of B.
2. Only the 6 children of 3 will get the property because the heir died before
the fulfillment of the condition.
3. From the wording of the facts in the case, it would appear that S is the heir
and that if 3 died within 10 years after A's death, then B gets it, not the children of S.
If S died after B, the children of S get it
4. In a conditional institution, such as what has arisen in the problem, the
instituted heir must survive not only the testator but likewise the fulfillment of the
condition in the will (Art. 1034). Since B did not survive that condition, the institution
in his favor could not be operative. The estate, therefore, assuming that there are no
other relatives other than those named in the problem, would be the child of B and
the six children of S, who would get it in equal shares (per capita), each receiving
1/7 of the estate (Art. 975).
03; Succession; intestate succession
1998 No XI.
Tessie died survived by her husband Mario, and two nieces, Michelle and
Jorelle, who are the legitimate children of an elder sister who had predeceased her.
Page 131 of 391
The only property she left behind was a house and lot worth two million pesos,
which Tessie and her husband had acquired with the use of Mario's savings from his
income as a doctor. How much of the property or its value, if any, may Michelle and
Jorelle claim as their hereditary shares? [5%]
Answer:
Article 1001 of the Civil Code provides, "Should brothers and sisters or their
children survive with the widow or widower, the latter shall be entitled to one-half of
the inheritance and the brothers and sisters or their children to the other half."
Tessie's gross estate consists of a house and lot acquired during her
marriage, making it part of the community property. Thus, one-half of the said
property would have to be set aside as Mario's conjugal share from the community
property. The other half, amounting to one million pesos, is her conjugal share (net
estate), and should be distributed to her intestate heirs. Applying the above
provision of law, Michelle and Jorelle, Tessie's nieces, are entitled to one-half of her
conjugal share worth one million pesos, or 500,000 pesos, while the other one-half
amounting to P5OO,OOO will go to Mario, Tessie's surviving spouse. Michelle and
Jorelle are then entitled to P250,000 pesos each as their hereditary share.
03; Succession; intestate succession
1977 No. XII-a
X is the adulterous son of A and B and when he died in 1970 without a will,
he was survived only by his father A and his widow W: How would you divide his
estate valued at P100,000.00?
Answer
A shall be entitled to 1/2 of the estate, while W shall be entitled also to 1/2.
True, there is no express provision of the New Civil Code which directly governs this
situation, but this solution is the most equitable. Besides, in testamentary
succession, the legitime of A is 1/2 of X's estate, while the legitime of W is also 1/2
(Art. 903, Civil Code); and in intestate succession, had A been a legitimate parent,
his share would have been only 1/2, while the share of W would also be 1/2 (Art.
997, Civil code). These rules should be applied by analogy.
03; Succession; intestate succession
1977 No. XI-c
State the order of intestate succession. Answer
We must distinguish between the order of intestate succession if the
decedent is a legitimate person and the order if said decedent is an illegitimate
person,
If the decedent is a legitimate person, the order is:
(1) Legitimate children or descendants.
(2) Legitimate parents or ascendants,
(3) Illegitimate children or descendants.
(4) The surviving spouse subject to the concurrent right of brothers and
sisters, nephews and nieces.
(5) Brothers and sisters, nephews and nieces.
(6) Other collateral relatives within the fifth degree.
(7) The state.
Page 132 of 391
If the decedent is an illegitimate person, the order is: (1) Legitimate children
or descendants.
(2) Illegitimate children or descendants.
(3) Parents by nature,
(4) The surviving spouse subject to the concurrent right of brothers and
sisters, nephews and nieces.
(5) Brothers and sisters, nephews and nieces.
(6) The State.
If the decedent is an adopted person, and his natural parents are already
dead, then the adopter shall take the place of such parents in the above orders of
succession (Art. 39, No. 4, P.D. No. 603),
03; Succession; intestate succession
1999 No II.
Mr. and Mrs. Cruz, who are childless, met with a serious motor vehicle
accident with Mr. Cruz at the wheel and Mrs. Cruz seated beside him, resulting in
the instant death of Mr. Cruz. Mrs. Cruz was still alive when help came but she also
died on the way to the hospital. The couple acquired properties worth One Million (PI
,000,000.00) Pesos during their marriage, which are being claimed by the parents of
both spouses in equal shares. Is the claim of both sets of parents valid and why?
(3%)
ANSWER:
(a) No, the claim of both parents is not valid. When Mr. Cruz died, he was
succeeded by his wife and his parents as his intestate heirs who will share his estate
equally. His estate was 0.5 Million pesos which is his half share in the absolute
community amounting to 1 Million Pesos. His wife, will, therefore, inherit O.25
Million Pesos and his parents will inherit 0.25 Million Pesos.
When Mrs. Cruz died, she was succeeded by her parents as her intestate
heirs. They will inherit all of her estate consisting of her 0.5 Million half share in the
ab-solute community and her 0.25 Million inheritance from her husband, or a total of
0.750 Million Pesos.
In sum, the parents of Mr. Cruz will inherit 250,000 Pesos while the parents of
Mrs. Cruz will inherit 750,000 Pesos.
03; Succession; intestate succession
1999 No VIII,
(b.) Mr. Luna died, leaving an estate of Ten Million (PI 0,000,000.00) Pesos,
His widow gave birth to a child four months after Mr, Luna's death, but the child died
five hours after birth. Two days after the child's death, the widow of Mr. Luna also
died because she had suffered from difficult childbirth. The estate of Mr. Luna is now
being claimed by his parents, and the parents of his widow. Who is entitled to Mr.
Luna'a estate and why? (5%)
ANSWER:
(b.) Half of the estate of Mr. Luna will go to the parents of Mrs. Luna as their
inheritance from Mrs. Luna, while the other half will be inherited by the parents of
Mr. Luna as the reservatarios of the reserved property inherited by Mrs. Luna from
her child.
When Mr. Luna died, his heirs were his wife and the unborn child. The unborn
child inherited because the inheritance was favorable to it and it was born alive later
Page 133 of 391
though it lived only for five hours. Mrs. Luna inherited half of the 10 Million estate
while the unborn child inherited the other half. When the child died, it was survived
by its mother, Mrs. Luna. As the only heir, Mrs. Luna inherited, by operation of law,
the estate of the child consisting of its 5 Million inheritance from Mr. Luna. In the
hands of Mrs. Luna, what she inherited from her child was subject to reserva troncal
for the benefit of the relatives of the child within the third degree of consanguinity
and who belong to the family of Mr. Luna, the line where the property came from.
When Mrs. Luna died, she was survived by her parents as her only heirs. Her
parents will inherit her estate consisting of the 5 Million she inherited from Mr. Luna.
The other 5 Million she inherited from her child will be delivered to the parents of Mr.
Luna as beneficiaries of the reserved property.
In sum, 5 Million Pesos of Mr. Luna's estate will go to the parents of Mrs.
Luna, while the other 5 Million Pesos will go to the parents of Mr. Luna as
reservatarios.
ALTERNATIVE ANSWER:
If the child had an intra-uterine life of not less than 7 months, it inherited from
the father. In which case, the estate of 10M will be divided equally between the child
and the widow as legal heirs. Upon the death of the child, its share of 5M shall go by
operation of law to the mother, which shall be subject to reserva troncal. Under Art.
891, the reserva is in favor of relatives belonging to the paternal line and who are
within 3 degrees from the child. The parents of Mr, Luna are entitled to the reserved
portion which is 5M as they are 2 degrees related from child. The 5M inherited by
Mrs. Luna from Mr. Luna will be inherited from her by her parents.
However, if the child had intra-uterine life of less than 7 months, half of the
estate of Mr. Luna, or 5M, will be inherited by the widow (Mrs. Luna), while the other
half, or 5M, will be inherited by the parents of Mr. Luna. Upon the death of Mrs.
Luna, her estate of 5M will be inherited by her own parents.
03; Succession; intestate succession
1986 No. 12:
Carlos, legitimate son of Jaime and Maria, was legally adopted by Josefa.
Both Jaime and Maria died soon after the adoption. Carlos, himself died in 1986. His
survivors are Josefa, his legitimate maternal grandparents Daniel and Rosa, his wife
Fe and his acknowledged natural son, Gerardo.
How should the estate of Carlos worth P800,000.00 be apportioned among
the above survivors? Explain.
Answer:
The estate of Carlos worth P800,000 should be apportioned as follows:
1. Josefa - one-half, or P400.000.00;
2. Fe - one-fourth, or P200.000.00, and
3. Gerardo - one-fourth, or P200.000.00.
As a rule, the adopter cannot inherit from the adopted child by intestate
succession. If the adopted child dies intestate, leaving no child or descendant, his
parents and relatives by consanguinity and not by adoption shall be his legal heirs.
There is, however, an exception to has rule According to the law, if the parents by
nature of the adopted child are both dead, the adopter takes place of such parents in
the line of succession, whether testate or intestate. Therefore, in the instant
problem, Josefa shall take place of Jaime and Maria. The grandparents Daniel and
Rosa are therefore- excluded. Consequently, applying the rules of intestacy, Josefa
Page 134 of 391
shall be entitled to one-half (1/2) of the estate by substitution; Fe shall be entitled to
one-fourth (1/4) as surviving spouse; and Gerardo shall be entitled to one-fourth
(1/4) as illegitimate child,
(Note: The above answer is based upon Arts, 984 and 100 Civil Code and
upon Art. 39, No. (4), P.D. 603).
Answer - Under P.D. 603, the adopter takes place of the parents by nature if
the latter are dead, both as a compulsory and a legal heir. Therefore, as the adopted
is survived by his wife, an acknowledged son and his maternal grandparents,
adopter inherits in the same way as a legitimate parent, and they will share as
follows:
Josefa-----------------------------1/2
Fe(wife)---------------------- 1/4
Gerardo-------------------------- 1/4
(acknowledged natural child)
03; Succession; intestate succession
1987 No. 4:
Angel died intestate leaving considerable properties accumulated during 25
years of marriage. He is survived by his widow, a legally adopted son, the child of a
deceased legitimate daughter, two illegitimate children duly recognized by Angel
before his death and his ailing 93-year old mother who has wholly dependent on
him.
How would you distribute the estate indicating by fractions the portions of the
following who claim to be entitled to inherit:
(a) the widow?
(b) the adopted son?
(c) the child of the deceased legitimate daughter?
(d) the two recognized illegitimate children?
(e) the mother? Answer:
a. the widow - 1/4
b. the adopted son - 1/4
c. the child of the deceased legitimate daughter - 1/4
d. the two "recognized illegitimate children" we must distinguish:
Assuming that the two recognized illegitimate children are natural children,
then each of them will get 1/8.
Upon the other hand, if they are recognized spurious children then each of
them will get 2/5 of 1/4 of the estate. The remaining 1/5 of 1/4 will be distributed as
follows:
Under the theory of concurrence, that 1/5 of 1/4 will be divided equally among
the widow, the adopted son and the child of the deceased legitimate daughter.
Under the theory of exclusion that 1/5 of 1/4 will be divided equally between
the adopted son and the child of the deceased legitimate daughter.
e. the mother - will get nothing,
03; Succession; intestate succession
1979 No. II
Page 135 of 391
RD and BG, both Filipinos were married and lived in Manila. They begot 2
children and after some years of marriage, RD, being a physician, went to the United
States. After staying there for two years, RD got attached to a Filipina nurse. He got
a quick divorce on the ground of desertion and then married the Filipina nurse with
whom he also begot 2 children. RD died intestate in an automobile accident in the
United States leaving valuable properties in the Philippines both inherited by him
from his parents as well as acquired during his marriage to BG. How would BG and
her two children and the Filipina nurse and her two children share in the estate of
RD. Give reasons for your answer.
Answer
Before we can determine the shares of the claimants to the estate of RD, let
us first determine what is the estate of RD and what is the status of the claimants in
relation to RD,
Estate of RD: As far as the properties acquired by RD during his marriage to
BG are concerned, 1/2 thereof should be included in his estate and 1/2 should be
given to BG since they are conjugal in character. As far as the properties inherited
by him from his parents are concerned, since they are exclusive or separate in
character, they must also be included in his estate,
Status of the claimants: BG is the surviving spouse of RD. The decree of
absolute divorce secured by RD in the United States is not valid. In the first place,
we adhere to the nationality theory. Philippine laws shall be binding upon Filipino
citizens wherever they are with respect to family rights and duties as well as status,
condition and legal capacity. And in the second place, there is a declaration of public
policy in this country against absolute divorce. Such a declaration of public policy
cannot be rendered nugatory by the decree of absolute divorce secured RD in a
foreign country. Therefore, the marriage of RD to the Filipina nurse is not valid. It is
bigamous under the Philippine law. Hence, the nurse is not related to RD under our
law of succession. It is different in the case of the two children. Being born of a void
marriage, they are classified as natural children by legal fiction, and are, therefore,
entitled to the same rights as acknowledged natural children,
Division of the estate of RD: It is clear that only BG, as surviving spouse, the
two legitimate children of RD and BG, and the two natural children by legal fiction of
RD will be able to inherit. The Filipina nurse cannot. Since RD died intestate, the
proportions established under our law on legitime is applicable. In the instant case,
the proportions will be 10 for BG; 10 for each of the legitimate children; and 5 for
each of the natural children. Stated in another way, the two legitimate children shall
be entitled to 1/2, or 1/4, each, of the entire estate of RD; BG shall be entitled to the
same share as each of the legitimate children, or 1/4 of the entire estate; and the
two natural children by legal fiction shall be entitled to the balance of 1/4 or 1/8 each
of the entire estate.
03; Succession; intestate succession; order of succession and sharing
1982 No. 14
"X" died intestate, leaving two sons "A" and "B"; two grandchildren "C" and
"D", the children of the deceased daughter of "B"; and another grandchild "F", the
daughter of "A". Who will succeed to the estate of "X" and how will they divide the
inheritance?
Answer
Among the survivors, only the following shall participate in the division of the
inheritance: "A", in his own right; "B", in his own right; and "C" and "D", by right of
representation, "F", the daughter of "A", cannot participate because she is excluded
Page 136 of 391
by the latter applying the rule of proximity. Consequently, the inheritance shall be
divided as follows:
"A" one-third (1/3) of the inheritance;
"B" one-third (1/3) of the inheritance; "C" one-half (1/2) of one-third
(1/3) of the inheritance by right of representation;
"D" one-half (1/2) of one-third (1/3) of the inheritance by right of
representation.
03; Succession; joint wills
2000 No III.
a} Manuel, a Filipino, and his American wife Eleanor, executed a Joint Will in
Boston, Massachusetts when they were residing in said city. The law of
Massachusetts allows the execution of joint wills. Shortly thereafter, Eleanor died.
Can the said Will be probated in the Philippines for the settlement of her estate?
(3%)
SUGGESTED ANSWER;
Yes, the will may be probated in the Philippines insofar as the estate of
Eleanor is concerned. While the Civil Code prohibits the execution of Joint wills here
and abroad, such prohibition applies only to Filipinos. Hence, the joint will which is
valid where executed is valid in the Philippines but only with respect to Eleanor.
Under Article 819, it is void with respect to Manuel whose joint will remains void in
the Philippines despite being valid where executed.
ALTERNATIVE ANSWER:
The will cannot be probated in the Philippines, even though valid where
executed, because it is prohibited under Article 818 of the Civil Code and declared
void under Article 819, The prohibition should apply even to the American wife
because the Joint will is offensive to public policy. Moreover, it is a single juridical
act which cannot be valid as to one testator and void as to the other.
03; Succession; legal separation; effect
1982 No. 5
The husband was granted a decree of legal separation on the ground of
adultery on the part of the wife. May the wife inherit from the husband
(a) By intestate succession?
(b) By will? Reasons.
Answer
(a) The wife in the instant case cannot inherit from her husband by intestate
succession. According to the Civil Code, the offending spouse shall be disqualified
from inheriting from the innocent spouse by intestate succession.
(b) It depends. If the will was executed prior to the legal separation, it is clear
that in effect the wife cannot inherit from her husband. According to the Civil Code,
provisions in favor of the offending spouse made in the will of the innocent one shall
be revoked by operation of law. However, if the will was executed subsequent to the
legal separation, undoubtedly, the wife will then be able to inherit from her husband.
The reason is obvious. There is a tacit or implied pardon.
(Note: The above answers are based on No. 4 of Art. 106 of the Civil. The
last part regarding tacit pardon is based on Art. 1033 by analogy.)
03; Succession; legitime
Page 137 of 391
2003 No XII.
(a) Luis was survived by two legitimate children, two illegitimate children,
his parents, and two brothers. He left an estate of P1 million. Who are the
compulsory heirs of Luis, how much is the legitimate of each, and how much is the
free portion of his estate, if any?
SUGGESTED ANSWER:
(a) The compulsory heirs are the two legitimate children and the two
illegitimate children. The parents are excluded by the legitimate children, while the
brothers are not compulsory heirs at all.
Their respective legitimate are:
(1) The legitimate of the two (2) legitimate children is one half (1/2) of the
estate (P500,000.00) to be divided between them equally, or P250,000.00 each.
(2) The legitimate of each illegitimate child is one-half (1/2) the legitimate
of each legitimate child or P125,000.00.
Since the total legitimate of the compulsory heirs is P750,000.00, the balance
of P250,000.00 is the free portion.
03; Succession; legitime
1997 No. 12:
"X", the decedent, was survived by W (his widow). A (his son), B (a
granddaughter, being the daughter of A) and C and D (the two acknowledged
illegitimate children of the decedent). "X" died this year (1997) leaving a net estate of
PI 80.000.00. All were willing to succeed, except A who repudiated the inheritance
from his father, and they seek your legal advice on how much each can expect to
receive as their respective shares in the distribution of the estate.
Give your answer. Answer:
The heirs are B, W, C and D. A inherits nothing because of his renunciation.
B inherits a legitime of P90.000.00 as the nearest and only legitimate descendant,
inheriting in his own right not by representation because of A's renunciation. W gets
a legitime equivalent to one-half (1 / 2) that of B amounting to P45.000. C and D
each gets a legitime equivalent to one-half (1/2) that of B amounting to P45.000.00
each. But since the total exceeds the entire estate, their legitimes would have to be
reduced corresponding to P22.500.00 each (Art. 895. CC). The total of all of these
amounts to P180.000.00.
Alternative Answer:
INTESTATE SUCCESSION
ESTATE: P180,000.00
W- (widow gets 1/2 share) P90.000.00 (Art. 998)
A- (son who repudiated his inheritance) None (Art. 977)
B - (Granddaughter) None
C - (Acknowledged illegitimate child) P45.000.00 (Art.998)
D - (Acknowledged illegitimate child) P45,000.00 (Art. 998)
The acknowledged illegitimate child gets 1/2 of the share of each legitimate
child.
03; Succession; legitime
1982 No. 12
Page 138 of 391
The testator has three children "A", "B", and "C"; a wife "W"; a father "F"; an
acknowledged natural child "N"; and an adulterous child "T". "A" is a handicapped
child, and the testator wants to leave to him as much of his estate as he can legally
do under the law. State the specific aliquot parts of the estate that the testator can
leave "A", "B", and "C", as well as to his other aforementioned relatives. State how
you arrive at the result. (Assume a net estate of P1,200,000.00 and that all of the
above named relatives survived the testator.)
Answer
Under the law on legitime, the survivors shall be entitled to the following
legitime:
1. "A", "B" and "C"-one-half of the estate which they shall divide in equal
shares. Since the net value of the estate is Pl,200,000.00 each of them shall,
therefore, be entitled to P200,000.00.
2. "W" the same as each of the legitimate children, or P200,000.
3. "F" none.- "F" cannot participate in the succession because he is
excluded by the legitimate children of the testator.
4. "N" one-half of the legitime of each of the legitimate children, or
P100,000.00.
5. "T" _ four-fifths of the legitimate of "N" or two-fifths of the legitime of either
"A" or "B" or "C", or P80,000.00
Thus, the disposable free portion is P220,000. If the testator so desires, he
can leave this disposable portion to his son "A".
(Note: The above answer is based on Art. 888, 892, 895, 897 and 898 of the
Civil Code,)
03; Succession; order of succession and sharing; right of representation,
institution, accretion
1985 No. 8
In a will executed in 1970, A instituted his two (2) legitimate brothers, B and
C, as sole heirs to all the properties he then owned. B died in 1975, survived by his
legitimate daughter, D, while A died last year, leaving an estate, 1/2 of which was
acquired after the execution of his will.
Who will succeed A, how much and by what right will the heir or each of the
heirs, if more than one, inherit? Reason out your answer.
Answer:
1. Regarding 1/2 acquired after the execution of the will it will be inherited by
both D and C( C in his own right and D by right of representation because this 1/2 is
inherited by intestate succession. With regard to the 1/2 already owned at the time
of the execution of the will, C alone will get the property by right of institution and
accretion.
2. The half of the property existing at the time of the execution of the will
should go to C, the portion pertaining to him in his own right and the portion
pertaining to B by right of accretion.
The other half acquired after the execution of the will passes by intestacy,
equally to C in his own right and to D in representation of B.
3. The whole estate will go to the second brother C by right of accretion and
B gets nothing.
Page 139 of 391
4. With respect to the will as made by the testator B is a voluntary heir. He
transmits no right to his heir D, therefore his share in the will 'goes to the other heir
by right of accretion. As to the properties which are not covered by the will, intestate
succession will follow and therefore the legal heirs will be the brother and the niece
to inherit equally.
5. The 1970 will appears to nave only covered the property which the testator
had at the time of its execution. Accordingly, the half which wag acquired by him
after the execution of the will would be governed by the law on intestacy. As regards
the other half, disposed under the will, the property should go to C by right of
accretion considering that the institution in favor of B and C was pro-indiviso (Art.
1015). The other half, acquired after the execution of the will, will be distributed in
intestacy and assuming that the only legal heirs are those named in the problem,
such portion shall be equally divided between C (legitimate brother of the deceased)
and D (niece of the deceased) by right of representation (Art. 1005).
03; Succession; partition
1977 No. XI-b
Discuss briefly the right of a testator to partition his estate among his heirs in
the last will.
Answer
(NOTE: Either of the following should constitute a sufficient answer.)
First Answer
If the testator has no compulsory heirs, he may partition his estate in favor of
any person having capacity to succeed. If he has compulsory heirs, he may partition
his estate provided that he does not contravene the provisions of the Civil Code with
regard to the legitime of said heirs. (See Art. 842, Civil Code,)
Second Answer
Under the Civil Code, the testator may partition his estate either by an act
inter vivos or by a will. In either case, such partition shall be respected, insofar as it
does not prejudice the legitime of his compulsory heirs (Art. 1080). He may even
entrust the mere power to make the partition to a third person (Art. 1081); and if he
so desires, he may even prohibit the partition, in which case the period of indivision
shall not exceed -twenty years (Art. 1083).
03; Succession; preterition
1999 No VII.
(a) Mr, Cruz, widower, has three legitimate children, A, B and C. He
executed a Will instituting as his heirs to his estate of One Million (P1,QOO,000.00)
Pesos his two children A and B, and his friend F. Upon his death, how should Mr.
Cruz's estate be divided? Explain. (3%)
(b) In the preceding question, suppose Mr. Cruz instituted his two children
A and B as his heirs in his Will, but gave a legacy of P 100,000.00 to his friend F.
How should the estate of Mr, Cruz be divided upon his death? Explain, (2%)
ANSWER:
(a) Assuming that the institution of A, B and F were to the entire estate, there
was preterition of C since C is a compulsory heir in the direct line. The preterition will
result in the total annulment of the institution of heirs. Therefore, the institution of A,
B and F will be set aside and Mr. Cuz's estate will be divided, as in intestacy, equally
among A, B and C as follows: A P333,333.33; B - P333.333.33; and C -
P333,333.33.
Page 140 of 391
(b| On the same assumption as letter (a), there was preterition of C.
Therefore, the institution of A and B is annulled but the legacy of P100.000.00 to F
shall be respected for not being inofficious. Therefore, the remainder of P900.000.00
will be divided equally among A, B and C.
03; Succession; preterition
2001 No VI
Because her eldest son Juan had been pestering her for capital to start a
business, Josefa gave him P100,000. Five years later, Josefa died, leaving a last
will and testament In which she instituted only her four younger children as her sole
heirs. At the time of her death, her only properly left was P900,000.00 in a bank.
Juan opposed the will on the ground of preterition. How should Josefa's estate be
divided among her heirs? State briefly the reason(s) for your answer. (5%)
SUGGESTED ANSWER
There was no preterition of the oldest son because the testatrix donated
100,000 pesos to him. This donation is considered an advance on the son's
inheritance. There being no preterition, the institutions in the will shall be respected
but the legitime of the oldest son has to be completed if he received less.
After collating the donation of P100.000 to the remaining property of
P900,000, the estate of the testatrix is P1,000,000. Of this amount, one-half or
P500,000, is the legitime of the legitimate children and it follows that the legitime of
one legitimate child is P100,000. The legitime, therefore, of the oldest son is
P100,000. However, since the donation given him was P100,000, he has already
received in full his legitime and he will not receive anything anymore from the
decedent. The remaining P900,000, therefore, shall go to the four younger children
by institution in the will, to be divided equally among them. Each will receive
P225,000.
ALTERNATIVE ANSWER
Assuming that the donation is valid as to form and substance, Juan cannot
invoke preterition because he actually had received a donation inter vivos from the
testatrix (III Tolentino 188,1992 ed.). He would only have a right to a completion of
his legitime under Art. 906 of the Civil Code. The estate should be divided equally
among the five children who will each receive P225,000.00 because the total
hereditary estate, after collating the donation to Juan (Art. 1061, CC), would be P1
million. In the actual distribution of the net estate, Juan gets nothing while his
siblings will get P225,000.00 each.
03; Succession; preterition; substitutions; compulsory heirs
1988 No. 6:
(a) What is preterition? What are its requisites? What is its effect?
(b) What are the different limitations imposed by law upon fideicommissary
substitutions?
(c) Who are compulsory heirs?
Answer:
Preterition or pretermission, as it is sometimes called may be defined as the
omission in the testator's will of one, some, or all of the compulsory heirs in the
direct line, whether living at the time of the execution of the will or born after the
death of the testator (Art. 854, CC). Stated in another way, it consists in the
omission in the testator's will of the compulsory heirs in the direct line, or of anyone
of them, either because they are not mentioned therein, or, though mentioned, they
Page 141 of 391
are neither instituted as heir nor expressly disinherited (Neri vs. Akutin, 74 Phil 185;
Nuguid vs. Nuguid, 17 SCRA449). Its requisites are:
(1) The heir omitted must be a compulsory heir in the direct line;
(2) The omission must be total and complete; and
(3) The omitted heir must survive the testator.
The effect is to annul entirely the institution of heirs but legacies and devises
shall be valid insofar as they are not inofficious, (Art, 854, CC.)
(b) There are four limitations. They are:
(1) The substitution must not go beyond one degree from the heir originally
instituted (Art. 863, CC).
(2) The fiduciary and the fideicommissary must be living at the time of the
death of the testator (Ibid).
(3) The substitution must not burden the legitime of compulsory heirs (Art.
864, CC).
(4) The substitution must be made expressly (Art. 865, par. l. CC.)
(c) In general, compulsory heirs are those for whom the law has reserved a
portion of the testator's estate which is known as the legitime.
In particular, the following are compulsory heirs:
{1} Legitimate children and descendants, with respect to their legitimate
parents and ascendants;
(2) In default of the foregoing, legitimate parents and ascendants, with
respects to their legitimate children and descendants;
(3) The widow or widower;
(4) Acknowledged natural children and natural children by legal fiction;
(5) Other illegitimate children referred to in article 287.
Compulsory heirs mentioned in numbers 3, 4, and 5 are not excluded by
those in numbers 1 and 2; neither do they exclude one another.
Compulsory heirs mentioned in numbers 3, 4, and 5 are not excluded by
those in numbers 1 and 2; neither do they exclude one another.
In all cases of illegitimate children, their filiation must be duly proved.
The father or mother of illegitimate children of the classes mentioned shall
inherit from them in the manner and to the extent established by the Civil Code. (Art.
887, CC.)
Committee's Recommendation Re: No. 6 (c):
(c) It is recommended that an enumeration of numbers one to five without the
mention of the additional last three paragraphs should merit full credit for this
question.
03; Succession; probate
1988 No. 5:
(a) In probate proceedings, what are the only questions which a probate
court can determine?
(b) A presented for probate a will purporting to be the last will and testament
of his deceased wife. The will was admitted to probate without any opposition.
Sixteen months later, the brothers and sisters of the deceased discovered that the
Page 142 of 391
will was a forgery. Can A now be prosecuted for the criminal offense of forgery?
Give your reasons.
Answer:
(a) Under our law, there are only three possible questions which can be
determined by the probate court. They are;
(1) Whether or not the instrument which is offered for probate is the last will
and testament of the decedent; in other words, the question is one of identity.
(2) Whether or not the will has been executed in accordance with the
formalities prescribed by law; in other words, the question is one of due execution.
(3) Whether or not the testator had the necessary testamentary capacity at
the time of the execution of the will; in other words, the question is one of capacity.
Consequently, the probate court cannot inquire into the intrinsic validity of
testamentary dispositions.
(b) A can no longer be prosecuted for the criminal offense of forgery. This is
so because, according to the last paragraph of Art. 838 of the Civil Code, subject to
the right of appeal the allowance of the will, either during the lifetime of the testator
or after his death, shall be conclusive as to its due execution. Since sixteen months
have already elapsed from the allowance of the will to the time when the forgery was
discovered, there is now no possible remedy of impugning the validity of the will.
Even a petition to set aside a judgment or order of a Court of First Instance on the
ground of fraud in accordance with Secs. 2 and 3 of Rule 38 of the Rules of Court is
no longer possible because more than six months from the time of the promulgation
of the judgment or order have already elapsed. (Mercado vs. Santos, 66 Phil 215.)
Committee's Recommendation Re: No. 5 (a)
(a) It is recommended that a mention of numbers (2) or (3) should merit a full
credit for this question.
Suggested Alternative Answer To: No, 5 (b):
The criminal action can still prosper because the question of probate is a civil
law matter while the question of forgery is a penal matter. That does not preclude
the determination of guilt of the forger because the two are founded on different
legal bases. Besides, in a criminal case, proof beyond reasonable doubt is required
while in civil cases, only preponderance of evidence is required.
03; Succession; probate of notarial and holographic wills
1997 No. 10:
Johnny, with no known living relatives, executed a notarial will giving all his
estate to his sweetheart. One day, he had a serious altercation with his sweetheart.
A few days later, he was introduced to a charming lady who later became a dear
friend. Soon after, he executed a holographic will expressly revoking the notarial will
and so designating his new friend as sole heir. One day when he was clearing up his
desk, Johnny mistakenly burned, along with other papers, the only copy of his
holographic will. His business associate, Eduardo. knew well the contents of the will
which was shown to him by Johnny the day it was executed. A few days after the
burning Incident, Johnny died. Both wills were sought to be probated in two separate
petitions.
Will either or both petitions prosper? Answer:
The probate of the notarial will will prosper. The holographic will cannot be
admitted to probate because a holographic will can only be probated upon evidence
of the will Itself unless there Is a photographic copy. But since the holographic will
Page 143 of 391
was lost and there was no other copy, it cannot be probated and therefore the
notarial will will be admitted to probate because there is no revoking will.
Additional Answers;
1. In the case of Gan vs. Yap (104 Phil 509), the execution and the contents
of a lost or destroyed holographic will may not be proved by the bare testimony of
witnesses who have seen or read such will. The will itself must be presented
otherwise it shall produce no effect. The law regards the document itself as material
proof of authenticity. Moreover, in order that a will may be revoked by a subsequent
will, it is necessary that the latter will be valid and executed with the formalities
required for the making of a will. The latter should possess all the requisites of a
valid will whether it be ordinary or a holographic will, and should be probated in
order that the revocatory clause thereof may produce effect. In the case at bar, since
the holographic will itself cannot be presented, it cannot therefore be probated.
Since it cannot be probated, it cannot revoke the notarial will previously written by
the decedent.
2. On the basis of the Rules of Court, Rule 76, Sec. 6, provides that no will
shall be proved as a lost or destroyed will *** unless its provisions are clearly and
distinctly proved by at least two (2) credible witnesses. Hence, if we abide strictly by
the two-witness rule to prove a lost or destroyed will, the holographic will which
Johnny allegedly mistakenly burned, cannot be probated, since there is only one
witness, Eduardo, who can be called to testify as to the existence of the will. If the
holographic will, which purportedly, revoked the earlier notarial will cannot be proved
because of the absence of the required witness, then the petition for the probate of
the notarial will should prosper.
03; Succession; probate of wills of aliens
1989 No. 10:
(2) "X", a Spanish citizen and a resident of Los Angeles, California,
executed a will in Tokyo, Japan. May such will be probated in the Philippines? May
his estate located in the Philippines be distributed in conformity with the provisions
of the said will? Give your reasons.
Answer:
A. Yes, it may be made according to the formalities of Spanish law,
California law, Japanese law, or Philippine law.
B. Yes, provided that the provisions conform to the order of succession and
the amount of successional rights as regulated by Spanish law.
03; Succession; probate; intrinsic validity
1990 No 9:
H died leaving a last will and testament wherein it is stated that he was legally
married to W by whom he had two legitimate children A and B. H devised to his said
forced heirs the entire estate except the free portion which he gave to X who was
living with him at the time of his death.
In said will he explained that he had been estranged from his wife W for more
than 20 years and he has been living with X as man and wife since his separation
from his legitimate family.
In the probate proceedings, X asked for the issuance of letters testamentary
in accordance with the will wherein she is named sole executor. This was opposed
by W and her children.
(a) Should the will be admitted in said probate proceedings?
Page 144 of 391
(b) Is the said devise to X valid?
(c) Was it proper for the trial court to consider the intrinsic validity of the
provisions of said will? Explain your answers,
Answer:
(a) Yes. the will may be probated if executed according to the formalities
prescribed by law.
(b) The institution giving X the free portion is not valid, because the
prohibitions under Art. 739 of the Civil Code on donations also apply to testamentary
dispositions (Article 1028, Civil Code), Among donations which are considered void
are those made between persons who were guilty of adultery or concubinage at the
time of the donation.
(c) As a general rule, the will should be admitted in probate proceedings if
all the necessary requirements for its extrinsic validity have been met. and the court
should not consider the intrinsic validity of the provisions of said will. However, the
exception arises when the will in effect contains only one testamentary disposition.
In effect, the only testamentary disposition under the will is the giving of the free
portion to X, since legitimes are provided by law. Hence, the trial court may consider
the intrinsic validity of the provisions of said will. (Nuguid v. Nuguid, etal.. No. L-
23445, June 23, 1966, 17 SCRA; Nepomuceno v. CA, L-62952, 9 October 1985.
139 SCRA 206).
03; Succession; renunciation; compromise
1979 No. XIII
MN, a wealthy haciendero died leaving to his four legitimate children and his
widow an estate worth about P2 million. When the proceedings for the settlement of
his estate were pending, Rosie, a child he begot with his lavandera, filed a claim for
a share in the estate. The widow and four children contested the claim on the
ground that in a previous action for support filed by the lavandera when Rosie was
still a minor, the lavandera agreed to dismiss the case and signed an agreement
acknowledging that the sum of P50,000.00 paid thereunder included payment for
whatever inheritance Rosie was to have. Should Rosie's claim be granted? Why?
Answer
Rosie's claim should be granted but subject to the condition that the portion
of the P50,000 paid to her mother as her inheritance shall be brought to collation. It
must be observed that the agreement is actually a renunciation or compromise as
regards a future legitimate or inheritance between the person owing it and a
compulsory heir. According to the Civil Code, such a renunciation or compromise is
void, and the latter may claim the same upon the death of the former, but he must
bring to collation whatever he may have received by virtue of the renunciation or
compromise. (Art. 905, Civil Code).
(NOTE: If the bar candidate invokes either Art. 1347, par. 2, or Art. 2035, No.
6 of the Civil Code, instead of Art. 905, his answer should be considered correct
because the result would be the same).
03; Succession; representation
1988 No. 7:
(a) When does the right of representation take place?
Answer:
The right of representation shall take place in the following cases:
(1) In testamentary succession:
Page 145 of 391
(a) In case a compulsory heir in the direct descending line dies before the
testator survived by his children or descendants (Art. 856, CC).
(b) In case a compulsory heir in the direct descending line is incapacitated to
succeed from the testator and he has children or descendants. (Arts. 856,1035, CC).
(c) In case a compulsory heir in the direct descending line is disinherited
and he has children or descendants. (Art. 923, CC).
(2) In intestate succession:
(a) In case a legal heir in the direct descending line dies before the
decedent survived by his children or descendant (Arts. 981, 982, CC), or in the
absence of other heirs who can exclude them from the succession, a brother or
sister dies before the decedent survived by his or her own children. (Arts. 972,975,
CC).
(b) In case a legal heir in the direct descending line is incapacitated to
succeed from the decedent and he has children or descendants (Art. 1035, CC), or
in the absence of other heirs who can exclude them from the succession, a brother
or sister is incapacitated to succeed from the decedent and he or she has children,
(Arts, 972, 975,1035, CC),
03; Succession; reserva troncal
1987 No. 13:
Lilia and Nelia are relatives, Ulia being the grand niece of Nelia. They had a
common ancestor, Bonong, father of Nelia and great-grandfather of Lilia. Bonong
had a sister, Rosa, who donated gratuitously a parcel of land to her niece Mely,
sister of Nelia and grandmother of Lilia. Mely died intestate, leaving aforementioned
parcel of land, survived by her husband Jose and their two children, Rico and Nina.
Bonong died intestate survived by his legitimate grandchildren, Rico and Nina. In the
adjudication of his estate, the portion pertaining to Mely, who had predeceased her
father, went to her two legitimate children, Rico and Nina. Rico died intestate, single,
and without any issue, leaving his share in the inheritance to his father, Jose,
subject to a reserva troncal duly annotated on the tide. Thereafter Nina died
intestate and her rights and interests were inherited by her only legitimate child, Lilia.
Thereafter, Jose died intestate survived by his only descendant, Lilia. Nelia, aunt of
Rico, would like to lay claim as reservatario to a portion of the one-half pro indiviso
share of the property inherited by Jose from his son Rico,
How should the estate of Jose, including the property subject to reserva
troncal be adjudicated? Explain.
Answer:
This is a proper case of reserva troncal. The propositus is Rico, the reservista
is Jose and the reservatarios are-Lilia (a niece) and Nelia (an aunt), both of them
being relatives within the 3rd degree of Rico (the propositus)and belonging to the
maternal line represented by Mely. Accordingly, Nelia as reservatario cannot claim
any portion of the pro-indiviso share of the property inherited by Jose from Rico. Lilia
alone should inherit because in reserva troncal, the successional rights of relatives
who are reservatarios are determined by the rules of intestate succession. In
intestacy, nephews and nieces exclude uncles and aunts. Hence, Lilia the niece,
excludes Nelia, the aunt, from the reservable property (De Papa vs. Camacho 144
SCRA 281),
The rest of Jose's estate, not subject to reserva, will be inherited by his
granddaughter Lilia as sole intestate heir.
Answer;
Page 146 of 391
This is. a proper case of reserva troncal. The propositus is Rico, the
reservista and the reservatarios are Lilia and Nelia, both of them being relatives
within the 3rd degree computed from Rico and belonging to the maternal line
represented by Mely. Under the doctrine of "reserva integral'" all the reservatarios in
the nearest degree will inherit in equal shares the reservable portion of the pro-
indiviso share of the property inherited by Jose from Rico. The properties
transmitted to Jose by Rico are-the following:
Firstly, the property which Rico obtained from Mely consisted of his share in
Mely's interest as donee of Rosa's land. The interest acquired by Rico was 1/3,
because 1/3 thereof was inherited by Jose and 1/3 by Nina. So the property that was
obtained by Jose from Rico is the latter's 1/3 interest of the land.
In the case of Bonong's estate, the share of Mely was 1/2 and Nelia's was the
other half. Out of Mely's share, 1/2 belonged to Rico and the other half belonged to
Nina, both Inheriting by right of representation.
Summarizing, the reservable estate is the 1/3 share of Rico in Rosa's land
which was donated to Mely, and the 1/2 interest of Rico in Mely's share of the estate
of Bonong. These reservable properties should be divided equally between Nelia
and Lilia (Article 891).
03; Succession; reserva troncal
1979 No. XIV
A married B in 1950 bringing into the marriage a 10-hectare piece of
unregistered land in Antipolo which he inherited from his father. Of the marriage two
daughters were born. On February 10, 1956 A and his two daughters went to
Baguio. On the way they met an accident and A died instantly on the spot while the
two daughters died two days later in the hospital where they were brought. In 1960
B sold the land .to C. In 1977 B died so D, the only brother of A, asked C to
reconvey the land to him. Upon C's refusal, D filed a complaint for recovery of the
land. C raised the defense of prescription. Should the defense be sustained? Why?
Answer
The defense should be sustained but only with respect to one-third of the
subject property; however, with respect to the other two-thirds, it should not be
sustained.
It must be observed that when A died the subject property passed by
intestate succession to his wife B and his two daughters in the proportion of one-
third for each. When the two daughters died two hours later, their one-third shares
passed by intestate succession to their mother B. These shares which B acquired by
operation of law from her two daughters became reservable. In other words, by
mandate of the law, upon acquiring the two-thirds share of her daughters she was
obliged to reserve such share for the benefit of relatives of her two deceased
daughters who are within the third degree and who belong to the live from whence
the reservable property came. All of the requisites of reserva troncal are, therefore,
present. In the first place, the property was acquired by a descendant from an
ascendant or from a brother or sister by gratuitous title; in the second place, said
descendant died without any legitimate issue in the direct descending line who can
inherit from him; in the third place, the property is inherited by another ascendant by
operation of law; and in the fourth place, there are relatives of the descendant who
are within the third degree and who belong to the line from which said property
came. Consequently, when C bought the subject property from B in I960, he
acquired only that which B had and nothing more. In other words, when B, the
ascendant reservista sold the property to C in 1960, the latter acquired the one-third
share which B had inherited from A without any condition whatsoever. However, with
Page 147 of 391
respect to the other two-thirds share which is reservable, C acquired a limited and
revocable title only. Therefore, when B, the ascendant-reservista vendor finally died
in 1977, automatically, by operation of law, the two-thirds share which is reservable
passed to D, who is the reservee or reservatario.
Premises considered, the defense of prescription can only be sustained with
respect to the one-third share of B which she had inherited from A in 1955. The
computation of the 10-year period of prescription must commence from 1960. In the
case of the two-thirds share which is reservable, the computation must commence
from 1977 when B, the ascendant-reservista, died. When D, the reservatario,
therefore, filed his action after the death of B, he was very much in time to do so.
(Chua vs. CFI, 78 SCRA 412).
03; Succession; reserva troncal
1982 No. 4
(A) What is the reason or rationale for reserva troncal?
(B) May the reservor/reservista dispose of the reservable property
(1) By acts inter vivos?
(2) By acts mortis causa.? Reasons.
Answer
(A) The reason or rationale for reserva troncal is evident: it is to reserve
certain property in favor of certain relatives. Hence, its name reserva lineal or
troncal. It seeks to prevent persons outside a family from securing, by some special
accident of life, property that would otherwise remained therein. Its principal aim is to
maintain as absolutely as is possible, with respect to the property to which it refers,
a separation between the paternal and maternal lines, so that property of one line
may not pass to the other, or through them to strangers.
(Note: The above answer is lifted from 6 Sanchez Roman 1015 and 14
Scaevola 213,)
(B) (1) The reservista may dispose of the reservable property by acts inter
vivos. This is logical because he acquires the ownership of the reservable property
upon the death of the descendant-propositus subject to the resolutory condition that
there must exist at the time of his death relatives of the descendant who are within
the third degree and who belong to the line from which the property came. He can,
therefore, alienate or encumber the property if he so desires, but he will only
alienate or encumber what he has and nothing more. As a consequence, the
acquirer will only receive a limited and revocable title. Therefore, after the death of
the reservista, the reservatarios may then rescind the alienation or encumbrance,
because the resolutory condition to which the reserva is subject has already been
fulfilled.
(Note: The above answer is based on Edroso vs. Sablan, 25 Phil. 295 and
Lunsod vs. Ortega, 46 Phil. 664. It is also based on comments of recognized
commentators.)
(2) The reservista cannot dispose of the reservable property by acts mortis
causa. The reason is crystal clear. Upon the death of said ascendant-reservist,
reservable property does not belong to his or her estate. Because the resolutory
condition to which the reserva is subject has already been fulfilled, therefore, the
reservatarios or reservees nearest the descendant- propositus have already become
automatically and by operation of law owners of the reservable property.
(Note: The above answer is based on Cano vs. Director of Lands (105 Phil 1)
and on Gonzales vs. Legarda (May 19, 1981). The Committee respectfully
Page 148 of 391
recommends that if the bar candidate will support his answer by saying that the
reservista is merely a usufructuary as stated by Mr. Justice Aquino in Gonzales vs.
Legarda, the answer should be properly credited.)
03; Succession; right of representation
1977 No. XIII-c
What is meant by the right of representation in succession? In what line does
it take place?
Representation is a right created by fiction of law, by virtue of which the
representative is raised to the place and the degree of the person represented, and
acquires the rights which the latter would have if he were living or if he could have
inherited (Art. 970, Civil Code).
The right of representation takes place in the direct descending line, but
never in the ascending.
In the collateral line, it takes place only in favor of the children of brothers or
sisters, whether they be of the full or half blood. (Art. 972, Civil Code).
When children of one or more brothers or sisters of the deceased survive,
they shall inherit from the latter by representation, if they survive with their uncles or
aunts. But if they alone survive, they shall inherit in equal portions. (Art, 975, Civil
Code).
03; Succession; testate succession; institution of heirs; substitution of heirs
2002 No VIII.
By virtue of a Codicil appended to his will, Theodore devised to Divino a tract
of sugar land, with the obligation on the part of Divino or his heirs to deliver to Betina
a specified volume of sugar per harvest during Betinas lifetime. It is also stated in
the Codicil that in the event the obligation is not fulfilled, Betina should immediately
seize the property from Divino or latters heirs and turn it over to Theodores
compulsory heirs. Divino failed to fulfill the obligation under the Codicil. Betina brings
suit against Divino for the reversion of the tract of land.
A. Distinguish between modal institution and substation of heirs. (3%)
B. Distinguish between simple and fideicommissary substitution of heirs.
(2%)
C. Does Betina have a cause of action against Divino? Explain (5%)
SUGGESTED ANSWER:
A. A modal institution is the institution of an heir made for a certain
purpose or cause (Arts. 871 and 882, NCC). Substitution is the appointment of
another heir so that he may enter into the inheritance in default of the heir originality
instituted. (Art. 857, NCC).
B. In a simple substitution of heirs, the testator designates one or more
persons to substitute the heirs instituted in case such heir or heirs should die before
him, or should not wish or should be incapacitated to accept the inheritance. In a
fideicommissary substitution, the testator institutes a first heir and charges him to
preserve and transmit the whole or part of the inheritance to a second heir. In a
simple substitution, only one heir inherits. In a fideicommissary substitution, both the
first and second heirs inherit. (Art. 859 and 869, NCC)
C. Betina has a cause of action against Divino. This is a case of a
testamentary disposition subject to a mode and the will itself provides for the
consequence if the mode is not complied with. To enforce the mode, the will itself
Page 149 of 391
gives Betina the right to compel the return of the property to the heirs of Theodore.
(Rabadilla v. Conscoluella, 334 SCRA 522 [2000] GR 113725, 29 June 2000).
03; Succession; testate succession; order of succession and sharing
1983 No. 7
The deceased, A left a gross estate worth P360,000 and debts amounting to
P60,000. He was survived by his widow, three legitimate children, an acknowledged
natural child and an adulterous child. In his will, he bequeathed P6,000 to a friend,
leaving the remainder of his estate to his widow and children, legitimate as well as
illegitimate.
Divide A's estate among the persons entitled thereto. Give reasons for your
division.
Answer
The net estate is worth P300,000.00 [P360.000.00 (gross estate) --
P60,000.00 (debts)]
Each of the legitimate children will receive P50,000 as legitime.
The widow will receive the same amount as legitime.
The legitime of the acknowledged natural child is 1/2 of that of each
legitimate child or P25.000.00
The share of the spurious child is 4/5 of that of the acknowledged natural
child or P20,000.00,
In addition, the legitimate children, the widow, the acknowledged natural
child, and the adulterous child will each receive P8,166.66 2/3 as their share of the
remainder of the free portion, after deducting therefrom the legitimes of the surviving
spouse, the illegitimate children and the legacy.
The legatee, will get P6,000.00-Explanation:
Since the legitime of the legitimate children, which is 1/2 of the estate, cannot
be impaired, only the free portion, the other half of A's property, is available for the
satisfaction of the shares of the other distributees. From that part must first be taken
the legitimes of the surviving spouse and of the illegitimate children which total P95,-
000,00. To the remainder of the free portion, or P55,000 is likewise chargeable the
legacy. The remainder of the free portion will then be P49,000.00 which shall be
divided equally among the children, legitimate as well as illegitimate and the widow,
it having been left to them without any designation of the shares,
03; Succession; transmission of rights to succession
1983 No. 8
On A's death last year, his nearest of kin were a legitimate daughter born in
1945 and a spurious son born and recognized by A in 1949.
May the daughter oppose her brother's claim to their father's estate on the
ground that it would impair her right under the old Civil Code to succeed him to the
exclusion of spurious children? Why?
Answer
No. The rights to the succession are transmitted only from the moment of the
death of the decedent. Since A died last year when the old Civil Code was no longer
in force, she did not acquire the right granted by it to exclude her brother from A's
inheritance. Her right thereto is governed by the new Civil Code, the statute in force
at the time of the opening of the succession of A, under which spurious children
inherit together with legitimate descendants.
Page 150 of 391
03; Succession; when death takes place; pesumptive legitime
1991 No 6:
(a) For purposes of succession, when is death deemed to occur or take
place?
(b) May succession be conferred by contracts or acts inter vivos? Illustrate.
(c) Is there any law which allows the delivery to compulsory heirs of their
presumptive legitimes during the lifetime of their parents? If so, in what instances?
Answer:
A. Death as a fact is deemed to occur when it actually takes place. Death is
presumed to take place in the circumstances under Arts. 390-391 of the Civil Code.
The time of death is presumed to be at the expiration of the 10-year period as
prescribed by Article 390 and at the moment of disappearance under Article 391
B. Under Art. 84 of the Family Code amending Art 130 of the Civil Code,
contractual succession is no longer possible since the law now requires that
donations of future property be governed by the provisions on the testamentary
succession and formalities of wills,
Alternative Answer:
B. In the case of Coronado vs.CA(l91 SCRA81), it was ruled that no property
passes under a will without its being probated, but may under Article 1O58 of the
Civil Code of 1898, be sustained as a partition by an act inter vivos [Many-Oy vs. CA
144SCRA33).
And in the case of Chavez vs, IAC 1191 SCRA211), it was ruled that while
the law prohibits contracts upon future inheritance, the partition by the parent, as
provided in Art. 1080. is a case expressly authorized by law. A person has two
options in making a partition of his estate: either by an act inter vivos or by will. If the
partition is by will, it is imperative that such partition must be executed in accordance
with the provisions of the law on wills; if by an act inter vivos, such partition may
even be oral or written, and need not be in the form of a will, provided the legitime is
not prejudiced,
"Where several sisters execute deeds of sale over their 1 /6 undivided share
of the paraphernal property of their mother, in favor of another sister, with their
mother not only giving her authority thereto but even signing said deeds, there is a
valid partition inter vivos between the mother and her children which cannot be
revoked by the mother. Said deeds of sale are not contracts entered into with
respect to future inheritance.
"It would be unjust for the mother to revoke the sales to a son and to execute
a simulated sale in favor of a daughter who already benefited by the partition."
Answer:
C. Yes, under Arts. 51 and 52 of the New Family Code. In case of legal
separation, annulment of marriage, declaration of nullity of marriage and the
automatic termination of a subsequent marriage by the reappearance of the absent
spouse, the common or community property of the spouses shall be dissolved and
liquidated.
Art, 51. In said partition, the value of the presumptive legitimes of all common
children, computed as of the date of the final judgment of the trial court, shall be
delivered in cash, property or sound securities, unless the parties, by mutual
agreement, judicially approved, had already provided for such matters.
Page 151 of 391
The children of their guardian, or the trustee of their property, may ask for the
enforcement of the judgment.
The delivery of the presumptive legitimes herein prescribed shall In no way
prejudice the ultimate successional rights of the children accruing upon the death of
either or both of the parents; but the value of the properties already received under
the decree of annulment or absolute nullity shall be considered as advances on their
legitime.
Art. 52. The judgment of annulment or of absolute nullity of the marriage, the
partition and distribution of the properties of the spouses, and the delivery of the
children's presumptive legitimes shall be recorded in the appropriate civil registry
and registries of property; otherwise, the same shall not affect third persons.
03; Succession; wills
1989 No. 10:
(1) What are the characteristics of a will?
Answer:
A will is
1. Personal
2. Unilateral
3. Formal or Solemn
4. Ambulatory or revocable
5. Individual, not joint 6. Free and voluntary 7. Mortis causa
RECOMMENDATION OF THE COMMITTEE:
Since this is not a codal provision, it is recommended that an answer of three
(3) be given full credit.
03; Succession; wills; codicil
1977 No. XII-b
What is a codicil and how shall it be executed in order that it may be
effective?
Aimer
A codicil is a supplement or addition to a will, made after the execution of a
will and annexed to be taken as a part thereof, by which any disposition made in the
original will is explained, added to, or altered (Art. 825, Civil Code). The formalities
which are required in the execution of a codicil are the same as those required in the
execution of a will (Art. 826, Civil Code).
03; Succession; wills; formalities
1975 No. XII
The attestation clause of the will omits to state that testator signed in the
presence of the witnesses and that the latter signed in the presence of the testator
and of one another. May evidence aliunde be admitted to prove these facts to allow
the probate of the will? Explain.
Answer
No, evidence aliunde may not be admitted to prove that the testator and the
witnesses signed in one another's presence because such fact cannot be
determined from an examination of the will itself. The reason for this is that oral
evidence does not possess the reliability of an express statement in the attestation
Page 152 of 391
clause. It is for this reason that Article 805 of the Civil Code requires the attestation
clause to state these facts, (Uy Coque v. Sioca, 43 Phil. 405; Tenefrancia v. Abaya,
47 O.CT. No. 12 Suppl. p. 327)
03; Succession; wills; formalities
1986 No. 14:
Busalsal executed a will in his handwriting, signed by him at the end of each
page on the left marginal space of every page except the last page. The document
bore no date. However, below Busalsal's every signature, were the signatures of two
witnesses, who later testified that the will was executed in their presence on January
1, 1985, New Year's Day, and that Busalsal was in full possession of his faculties at
that time and even explained to them details of the will he was writing down.
Is the will formally valid? Explain. Answer:
The will is not valid either as a notarial will or a holographic will. It is not valid
as a notarial will because this requires 3 attesting witnesses. Neither is it a valid
holographic will because the will must be entirely written, dated and signed in the
handwriting of the testator. The fact that the witnesses testified as the date of
execution did not cure the defect.
Answer - The will is not formally valid. Whether we consider the will in the
instant problem as a notarial will or as a holographic will, it cannot be considered as
a valid will. If we consider it as a notarial will, it is not be valid because there are only
two instrumental witnesses. Under the law on notarial or ordinary wills, the will
should have been subscribed to and attested by three or more credible witnesses.
Therefore, it is void. If we consider it as a holographic will, it is not also valid
because it is not dated. Under the law on holographic wills, the will should have
been entirely written, dated and signed by the hand of the testator himself.
Therefore, it is void.
(Note: The above answer is based upon Arts. 805 and 810 of the Civil Code
and upon decided cases.)
Answer - The C.C. provides that a holographic will must be entirely in the
handwriting of the testator, signed and dated by him. It need not be witnessed.
Hence lacking the date, it cannot be allowed to probate. The law does not allow
extrinsic evidence to supply the omission.
03; Succession; wills; formalities
1990 No 11;
(1) If a will is executed by a testator who is a Filipino citizen, what law will
govern if the will is executed in the Philippines? What law will govern if the will is
executed in another country? Explain your answers.
(2) If a will is executed by a foreigner, for instance, a Japanese, residing In
the Philippines, what law will govern if the will is executed in the Philippines? And
what law will govern if the will is executed in Japan, or some other country, for
instance, the U.S.A.? Explain your answers.
Answer:
(1) a. If the testator who is a Filipino citizen executes his will in the
Philippines, Philippine law will govern the formalities.
b. If said Filipino testator executes his will in another country, the law of the
country where he maybe or Philippine law will govern the formalities. (Article 815,
Civil Code}
Page 153 of 391
(2) a. If the testator is a foreigner residing in the Philippines and he executes
his will in the Philippines, the law of the country of which he is a citizen or Philippine
law will govern the formalities.
b. If the testator is a foreigner and executes his will in a foreign country, the
law of his place of residence or the law of the country of which he is a citizen or the
law of the place of execution, or Philippine law will govern the formalities (Articles
17. 816. 817. Civil Code).
Possible Additional Answers:
a. In the case of a Filipino citizen, Philippine law shall govern substantive
validity whether he executes his will in the Philippines or in a foreign country.
b. In the case of a foreigner, his national law shall govern substantive validity
whether he executes his will in the Philippines or in a foreign country.
03; Succession; wills; formalities
1975 No. XI
Through negligence, one of the three witnesses to a will forgot to sign on the
third page of the original of a five-page will, but was able to sign on all the pages of
the duplicate. All other requisites were complied with. Can the will be admitted to
probate? Explain.
Answer
The will may be admitted to probate. Although the requirements of Article 805
of the Civil Code were not strictly complied with, the purposes of the law have been
satisfied. Impossibility of substitution is assured by the fact that the testator and the
two other witnesses signed the defective page. The law should not be strictly and
literally interpreted as to penalize the testator on account of the inadvertence of a
single witness over whose conduct he had no control, where the purpose of the law
to guarantee the identity of the testament and its component pages is sufficiently
attained and no intentional or deliberate deviation exists. (Icasiano v. Icasiano, 11
SCRA 422)
Besides, the signed duplicate copy has the same effect as the original.
03; Succession; wills; formalities; Holographic wills; effect of unauthenticated
insertions and cancellations
1996 No. 10:
1) Vanessa died on April 14. 1980. leaving behind a holographic will which
is entirely written, dated and signed in her own handwriting. However, it contains
Insertions and cancellations which are not authenticated by her signature. For this
reason, the probate of Vanessa's will was opposed by her relatives who stood to
inherit by her intestacy.
May Vanessa's holographic will be probated? Explain. Answer;
Yes, the will as originally written may be probated. The insertions and
alterations were void since they were not authenticated by the full signature of
Vanessa, under Art. 814, NCC. The original will, however, remains valid because a
holographic will is not invalidated by the unauthenticated insertions or alterations
(Ajero v. CA, 236 SCRA 468].
Alternative Answer:
It depends. As a rule, a holographic will is not adversely affected by
Insertions or cancellations which were not authenticated by the full signature of the
testator (Ajero v. CA, 236 SCRA 468). However, when the insertion or cancellation
amounts to revocation of the will, Art.814 of the NCC does not apply but Art. 830.
Page 154 of 391
NCC. Art. 830 of the NCC does not require the testator to authenticate his
cancellation for the effectivity of a revocation effected through such cancellation
(Kalaw v. Relova, 132 SCRA 237). In the Kalaw case, the original holographic will
designated only one heir as the only substantial provision which was altered by
substituting the original heir with another heir. Hence, if the unauthenticated
cancellation amounted to a revocation of the will, the will may not be probated
because it had already been revoked.
03; Succession; wills; formalities; revocation of wills
2003 No XI.
Mr. Reyes executed a will completely valid as to form. A week later, however,
he executed another will which expressly revoked his first will, which he tore his first
will to pieces. Upon the death of Mr. Reyes, his second will was presented for
probate by his heirs, but it was denied probate due to formal defects. Assuming that
a copy of the first will is available, may it now be admitted to probate and given
effect? Why?
SUGGESTED ANSWER:
Yes, the first will may be admitted to probate and given effect. When the
testator tore first will, he was under the mistaken belief that the second will was
perfectly valid and he would not have destroyed the first will had he known that the
second will is not valid. The revocation by destruction therefore is dependent on the
validity of the second will. Since it turned out that the second will was invalid, the
tearing of the first will did not produce the effect of revocation. This is known as the
doctrine of dependent relative revocation (Molo v. Molo, 90 Phil 37.)
ALTERNATIVE ANSWERS:
No, the first will cannot be admitted to probate. While it is true that the first will
was successfully revoked by the second will because the second will was later
denied probate, the first will was, nevertheless, revoked when the testator
destroyed it after executing the second invalid will. (Diaz v. De Leon, 43 Phil 413
[1922]).
03; Succession; wills; revocation
1981 No. 7
A testator, a bachelor of 60, executed a Will bequeathing a ricefield to the
Church worth P100,000.00. The Will further provided that "all other assets owned by
me after death shall be equally divided among my two brothers "A" and "B". The
testator subsequently married a young woman, begot a son, and left another Will
designating his wife and son as his heirs in equal shares. The second Will did not
expressly revoke the first Will. He left an estate worth P300,000.00 (including the
ricefield).
a) Who is entitled to the ricefield? Reasons. b| Who acquires the rest of the
assets? Explain. Answer
(a) It must be observed that the testator left two wills. In his first will, he
bequeathed the ricefield to the Church and instituted as heirs in equal shares his two
brothers "A" and "B" with respect to the rest of his estate. In his second will, he
instituted his wife and his son as heirs in equal shares. Under our law on revocation
of wills, a will may be revoked by another will- The revocation may be effected either
expressly or impliedly. Since there is no express revocation, is there an implied
revocation in the instant case? It is undeniable that there is an implied revocation if
the testamentary dispositions found in the first will are totally or partially incompatible
with those found in the second will. It is also undeniable that the incompatibility must
be absolute in character in the sense that the testamentary dispositions cannot
Page 155 of 391
stand together. The real issue, therefore, is whether the two testamentary
dispositions found in the first will can stand together with the single testamentary
disposition found in the second will.
There are two views.
According to one view, reading the two wills together it is clear that the
testatorial intention is that only the testator's wife and son shall inherit. They are
instituted as universal heirs with respect to the hereditary estate in its totality.
Therefore, the second will in its totality cannot stand together with the first will in its
totality. Consequently, the incompatibility between the testamentary dispositions
found in the, first will and those found in the second will is both total and absolute in
character. Hence, the first will is impliedly revoked by the second will. The testator's
widow and his son are, therefore, entitled to the ricefield
According to a second view, only the institution of "A" and " B'' in the firs t will
as heirs and that portion or part of the bequest given to the Church which will impair
the legitime of the testator's son and widow are revoked by the second will. The
reason is that it is only to that extent that there is absolute incompatibility between
the testamentary dispositions found in the first will and those found in the second
will. Consequently, the Church shall be entitled to the ricefield but only to the extent
that it does not encroach upon the legitime of the testator's son and widow.
(Nme: The above answers are based on the law on revocation of wills, such
as Arts. 830. et. seq.. Civil Code and on well settled principles in American
jurisprudence. The Committee respectfully recommends that whether the bar
candidate will solve the problem either in accordance with the first view or in
accordance with the second view, it should be considered as a correct answer.)
(b) Suggested answer for those who adhere to the first view stated above:
There are two views with regard to the distribution of the entire estate,
including the ricefield.
According to one view, one-half (1/2) shall be given to the testator's widow
and the other one-half (1/2) shall be given to the testator's son. This division would
be more in conformity with the testatorial intention.
According to another view, first satisfy the legitime of the two heirs. The
testator's widow shall be entitled to one-fourth (1/4) of P300,000, or P75,000, while
the testator's son shall be entitled to one-half, or P150,000. The disposable free
portion shall then be divided equally between the two. This would be more in
conformity with the testatorial intention.
(Note: The Committee recommends that either answer should be considered
correct.)
Suggested answer for those who adhere the second view stated above:
The testator's son shall be entitled to a legitime of one-half |l/2) of the entire
estate, or P150.000; the widow shall be entitled to a legitime of one-fourth on the
entire estate, or P75,000. That means that the bequest in favor of the Church is
inofficious to the extent of P25.000, considering that the value of the ricefield is
P100,000. Consequently, said bequest or devise should be reduced by one-fourth
(1/4). Therefore, the Church shall be entitled only to an undivided share of three-
fourth (3/4) of the ricefield.
03; Succession; wills; testamentary intent
1996 No. 10:
2) Alfonso, a bachelor without any descendant or ascendant, wrote a last
will and testament in which he devised." all the properties of which I may be
Page 156 of 391
possessed at the time of my death" to his favorite brother Manuel. At the time he
wrote the will, he owned only one parcel of land. But by the time he died, he owned
twenty parcels of land. His other brothers and sisters insist that his will should pass
only the parcel of land he owned at the time it was written, and did not cover his
properties acquired, which should be by intestate succession. Manuel claims
otherwise.
Who is correct? Explain. Answer:
Manuel is correct because under Art. 793, NCC, property acquired after the
making of a will shall only pass thereby, as if the testator had possessed it at the
time of making the will, should it expressly appear by the will that such was his
intention. Since Alfonso's intention to devise all properties he owned at the time of
his death expressly appears on the will, then all the 20 parcels of land are included
in the devise.
03; Succession; wills; witnesses to holographic wills
1989 No. 11:
(1) The probate of the will of Nicandro is contested on the ground that the
notary public before whom the will, was acknowledged was also one of the three
instrumental witnesses. If you were the probate judge, how would you decide the
contest? Give your reasons.
Answer:
The will is void. The acknowledging officer cannot serve as attesting witness
at the same time. In effect there are only two witnesses since the notary cannot
swear before himself.
03; Succession; wills; witnesses to holographic wills
1994 No. 10:
On his deathbed, Vicente was executing a will. In the room were Carissa,
Carmela, Comelio and Atty. Cimpo, a notary public. Suddenly, there was a street
brawl which caught Comelio's attention, prompting him to look out the window.
Cornelio did not see Vicente sign a will. Is the will valid?
Alternative Answers:
a) Yes, The will is valid. The law does not require a witness to actually see
the testator sign the will. It is sufficient if the witness could have seen the act of
signing had he chosen to do so by casting his eyes to the proper direction.
b) Yes, the will is valid. Applying the "test of position", although Comelio did
not actually see Vicente sign the will, Cornelio was in the proper position to see
Vicente sign if Cornelio so wished.
Page 157 of 391
04; Donation & Sales ownership of the thing donated
2003 No XV.
(a) May a person sell something that does not belong to him? Explain.
(b) May a person donate something that does not belong to him? Explain.
5%
SUGGESTED ANSWERS:
(a) Yes, a person may sell something which does not belong to him. For
the sale to be valid, the law does not require the seller to be the owner of the
property at the time of the sale. (Article 1434, NCC). If the seller cannot transfer
ownership over the thing sold at the time of delivery because he was not the owner
thereof, he shall be liable for breach of contact.
(b) As a general rule, a person cannot donate something which he cannot
dispose of at the time of the donation (Article 751, New Civil Code).
04; Donation; annulment
1982 No. 3
"H", the husband, during the marriage to his second wife, donated a parcel of
land to "A", a son of the second wife by her first marriage. (Assume that the
formalities required by law had been complied with). After the death of the husband
donor, his brother, "B", his nearest living relative, brought an action to annul the
donation. Will the action prosper? Reason.
Answer:
Yes, the action will prosper. According to the Civil Code, a donation during
the marriage by one of the spouses to the children whom the other spouse had by
another marriage, or to persons of whom the other spouse is a presumptive heir at
the time of the donation is voidable at the instance of the donor's heirs after his
death. Since the problem says that "B", a brother of the decedent donor, is the
nearest living relative of said donor, it is clear that he can now institute the action for
annulment.
(Note: The above answer is based on Art. 134 of the Civil Code.)
04; Donations
1999 No I.
Elated that her sister who had been married for five years was pregnant for
the first time, Alma donated P100,000.00 to the unborn child. Unfortunately, the
baby died one hour after delivery. May Alma recover the P100.000.00 that she had
donated to said baby before it was born considering that the baby died? Stated
otherwise, is the donation valid and binding? Explain. (5%)
ANSWER:
The donation is valid and binding, being an act favorable to the unborn child,
but only if the baby had an intra-uterine life of not less than seven months and pro-
vided there was due acceptance of the donation by the proper person representing
said child. If the child had less than seven months of intra-uterine life, it is not
deemed born since it died less than 24 hours following its delivery, in which ease the
donation never became effective since the donee never became a person, birth
being determinative of personality.
ALTERNATIVE ANSWER:
Even if the baby had an intra-uterine life of more than seven months and the
donation was properly accepted, it would be void for not having conformed with the
Page 158 of 391
proper form. In order to be valid, the donation and acceptance of personal property
exceeding five thousand pesos should be in writing. (Article 748, par. 3)
04; Donations
2003 No X.
In 1950, Dr. Alba donated a parcel of land to Central University on condition
that the latter must establish a medical college on the land to be named after him. In
the year 2000, the heirs of Dr. Alba filed an action to annul the donation and for the
reconveyance of the property donated to them for the failure, after 50 years, of the
University to established on the property a medical school named after their father.
The University opposed the action on the ground of prescription and also because it
had not used the property for some purpose other than that stated in the donation.
Should the opposition of the University to the action of Dr. Albas heirs be
sustained? Explain.
SUGGESTED ANSWER:
The donation may be revoked. The non-established of the medical college on
the donated property was a resolutory condition imposed on the donation by the
donor. Although the Deed of Donation did not fix the time for the established of the
medical college, the failure of the donee to establish the medical college after fifty
(50) years from the making of the donation should be considered as occurrence of
the resolutory condition, and the donation may now be revoked. While the general
rule is that in case the period is not fixed in the agreement of the parties, the period
must be fixed first by the court before the obligation may be demanded, the period of
fifty (50) years was more than enough time for the donee to comply with the
condition. Hence, in this case, there is no more need for the court to fix the period
because such procedure with the condition. (Central Philippine University v. CA. 246
SCRA 511).
ANOTHER SUGGESTED ANSWER:
The donation may not as yet revoked. The establishment of a medical
college is not a resolutory or suspensive condition but a charge, obligation, or a
mode. The non- compliance with the charge or mode will give the donor the right
to revoke the donation within four (4) years from the time the charge was supposed
to have been complied with, or to enforce the charge by specific performance within
ten (10) years from the time the cause of action accrued. Inasmuch as the time to
established the medical college has not been fixed in the Deed of Donation, the
donee is not yet default in his obligation until the period is fixed by order of the court
under Article 1197 of the New Civil Code. Since the period has not been fixed as
yet, the donee is not yet default, and therefore the donor has no cause of action to
revoke the donation. (Dissenting opinion of Davide, CJ, Central Philippine University
v. Court of Appeals, 246 SCRA 511 [1995])
04; Donations
1988 No. 4:
(b) A donated to X a parcel of land in 1975. The donation was made in a
public instrument, while the acceptance made by X was embodied in the same
public instrument. The Deed of Donation was entitled "Donation Inter Vivos." There
is however a provision in the deed to the effect that, although the land donated shall
be delivered immediately to X upon the perfection of the donation with full right to
enjoy all of the fruits thereof, "title shall pass to the donee only upon the donor's
death." Upon the death of A, his widow and only heir, B, brought an action for the
recovery of the property on the ground that the donation is a donation mortis causa
and not a donation inter vivos. Will the action prosper? Give your reasons.
Page 159 of 391
Answer:
(b) Yes, the action will prosper. In Bonsato vs. Court of Appeals, and Howard
vs. Court of Appeals, the Supreme Court declared that in order that a donation will
be considered a disposition post mortem, it should reveal any or all of the following
characteristics:
(1) Convey no title or ownership to the transferee before the death of the
transferor; or, what amounts to the same thing, that the transferor should retain the
ownership, full or naked, and control the property while alive;
(2) That before his death the transfer should be revocable by the transferor
at will, ad nutum; but revocability may be provided for indirectly by means of a
reserved power in the donor to dispose of the property conveyed:
(3) That the transfer should be void if the transferor should survive the
transferee.
It is clear from the facts stated in the problem that the donation reveals the
first characteristic. Hence, it is a disposition, post mortem. Therefore, in order that
the donation can take effect it is essential that it must be made in a will executed in
accordance with all of the formalities prescribed by law (Art. 728. CC}. Since this
requisite has not been complied with, the donation in the instant case is void or
inexistent.
Committee's Recommendations Re: No. 4 (b)
(b) It is recommended that the mention of the first characteristic of the three
mentioned above, should merit a full credit for this question.
04; Donations
1982 No. 8
"A", residing in Manila, and having the capacity to enter into a contract and
dispose of his properties, donated a parcel of land to "B", a resident of Davao. The
deed of donation was sent to "B" in Davao. One year later, "B" accepted and notified
the donor. In the meantime, the donor became insane, and was still insane at the
time he received the notice of acceptance. (Assume that all other formalities of the
donation and acceptance had been complied with). The donor died a few days later,
without having recovered his sanity. The heirs refused to deliver the land to "B" on
the ground that the donation had not been perfected for lack of capacity of the donor
at the time he received the notice of acceptance. Decide with reasons.
Answer
There are two views.
According to one view, the donation is binding. The exponents of this view
maintain that there is only one moment which must be considered in order to
determine the donor's capacity to make the donation and that is "the time of the
making of the donation" in accordance with the literal tenor of Art. 737 of the Civil
Code. According to them, when the law speaks of the making of the donation, it can
only refer to that precise moment when the donor manifests his offer to make the
donation to the donee. And besides, even if the donor becomes insane
subsequently, the acceptance can always be conveyed to his legal representative.
According to another view, the donation is not binding for the following
reasons:
(1) Art. 737 of the Civil Code declares that the donor's capacity shall be
determined as of the time of the making of the donation. Correlating this with Art.
734 which declares that a donation is perfected from the moment the donor knows
Page 160 of 391
of the acceptance by the donee, it is clear that when the law speaks of the making of
a donation, it refers not only to that moment when the donor manifest to the donee
his offer to make the donation but also to that more decisive moment the moment
of perfection of the contract the moment the donor knows of the acceptance by
the donee.
(2) Furthermore, if the donation in the instant case is binding, by parity of
reasoning, a donation made by one who dies subsequent to his offer to make a
donation would also be binding. Such a situation would, of course, be juridically
absurd. This is clear from Art, 1323 of the Civil Code which declares that an offer
becomes ineffective upon the death, civil interdiction, insanity, or insolvency of either
party before acceptance is conveyed.
We believe that the latter view is more logical. Consequently, the contention
of the heirs of "A" is correct
(Note: The above answer is based on the opinions of commentators on the
New Civil Code.)
04; Donations
1993 No. 6;
On January 2t 1986, A executed a deed of donation inter vivos of a parcel of
land to Dr. B who had earlier constructed thereon a building in which researches on
the dreaded disease AIDS were being conducted. The deed, acknowledged before a
notary public, was handed over by A to Dr. B who received it. A few days after, A
flew to Davao City. Unfortunately, the airplane he was riding crashed on landing
killing him. Two days after the unfortunate accident. Dr. B, upon advice of a lawyer,
executed a deed acknowledged before a notary public accepting the donation.
Is the donation effective? Explain your answer.
Answer:
No, the donation is not effective. The law requires that the separate
acceptance of the donee of an immovable must be done in a public document during
the lifetime of the donor (Art. 746 & 749, Civil Code) In this case, B executed the
deed of acceptance before a notary public after the donor had already died
04; Donations; donations mortis causa
1998 No IX.
Ernesto donated in a public instrument a parcel of land to Demetrio, who
accepted it in the same document. It is there declared that the donation shall take
effect immediately, with the donee having the right to take possession of the land
and receive its fruits but not to dispose of the land while Ernesto is alive as well as
for ten years following his death. Moreover, Ernesto also reserved in the same deed
his right to sell the property should he decide to dispose of it at any time - a right
which he did not exercise at all. After his death, Ernesto's heirs seasonably brought
an action to recover the property, alleging that the donation was void as it did not
comply with the formalities of a will. Will the suit prosper? [5%]
Answer:
Yes, the suit will prosper as the donation did not comply with the formalities of
a will. In this instance, the fact that the donor did not intend to transfer ownership or
possession of the donated property to the donee until the donor's death, would result
in a donation mortis causa and in this kind of disposition, the formalities of a will
should be complied with, otherwise, the donation is void. In this Instance, donation
mortis causa embodied only in a public instrument without the formalities of a will
could not have transferred ownership of disputed prop-city to another.
Page 161 of 391
Alternative Answer
One of the essential distinctions between a donation inter vivos and a
donation mortis causa is that while the former is irrevocable, the latter is revocable.
In the problem given, all the clauses or conditions mentioned in the deed of
donation, except one, are consistent with the rule of irrevocability and would have
sustained the view that the donation is inter vivos and therefore valid. The lone
exception is the clause which reserves the donor's right to sell the property at any
time before his death. Such a reservation has been held to render the donation
revocable and, therefore, becomes a donation mortis causa (Puig vs. Penqflorida,
15 SCRA 276, at p. 286). That the right was not exercised is immaterial; its
reservation was an implied recognition of the donor's power to nullify the donation
anytime he wished to do so. Consequently, it should have been embodied in a last
will and testament. The suit for nullity will thus prosper.
04; Donations; donees civil personality
1981 No. 2
"D" donated P100,000.00 to the unborn child of his pregnant girlfriend, which
she accepted. After six months of pregnancy, the fetus was born and baptized
Angel. Angel died twenty (20) hours after birth "D" sought to recover the
P100,000.
Is "D" entitled to recover? Explain.
Answer
"D" is entitled to recover the P100,000. The reason is that there is no donee.
The supposed donee never acquired any civil personality. Consequently, the
donation is void or in-existent.
According to the Civil Code, for civil purposes, the fetus is considered born if
it is alive at the time it is completely delivered from the mother's womb. However, if
the fetus had an intra-uterine life of less than seven months, it is not deemed born if
it dies within twenty-four hours after its complete delivery from the maternal womb.
The facts show that the fetus in the instant case had an intra-uterine life of less than
seven months and that it died twenty hours after birth. Therefore, the provisional or
conditional civil personality which is accorded to a conceived child under both the
Civil Code and the Child and Youth Welfare Code is not present here. In other
words, Angel has not acquired any civil personality. Therefore, the donation by "D"
never produced any legal effect. It is in-existent.
{Note: The above answer is based on Arts. 40 and 41 of the Civil Code and
on Art. 5 of PD 603.)
04; Donations; effect of illegal and immoral conditions
1997 No. 9:
(b) Are the effects of illegal and immoral conditions on simple donations the
same as those effects that would follow when such conditions are imposed on
donations con causa onerosa?
Answer:
(b) No, they don't have the same effect. Illegal or impossible conditions In
simple and remuneratory donations shall be considered as not imposed. Hence the
donation is valid. The donation will be considered as simple or pure. The condition
or mode is merely an accessory disposition, and its nullity does not affect the
donation, unless it clearly appears that the donor would not have made the donation
without the mode or condition.
Page 162 of 391
Donations con causa onerosa is governed by law on obligations and
contracts, under which an impossible or Illicit condition annuls the obligation
dependent upon the condition where the condition is positive and suspensive. If the
impossible or illicit condition Is negative, it is simply considered as not written, and
the obligation is converted into a pure and simple one. However, in order that an
illegal condition may annul a contract, the Impossibility must exist at the time of the
creation of the obligation; a supervening impossibility does not affect the existence
of the obligation.
Additional Answer;
No. In simple or pure donation, only the illegal or Impossible condition is
considered not written but the donation remains valid and becomes free from
conditions. The condition or mode being a mere accessory disposition. Its nullity
does not affect the donation unless it clearly appears that the donor would not have
made the donation without the mode or condition. On the other hand, onerous
donation is governed by the rules on contracts. Under Article 1183, Impossible or
illegal conditions shall annul the obligation which depends upon them. In these
cases. both the obligation and the condition are void.
04; Donations; kinds
1982 No. 9
Classify the following donations, stating your reasons for the classification:
(a) "1 hereby donate to 'A' mortis causa a parcel of land (here follows the
description) on the condition that this donation shall be deemed revoked if he fails to
build a house on the land worth at least P50,000.00 within two (2) years from date
hereof."
(b) "In consideration of the services rendered to me for which he refused to
accept my remuneration, I hereby donate to "A" the following parcel of land
(description follows)".
(c) "I hereby donate to "A" the following parcels of land (description follows)
with the obligation on his part to defray the expenses for my subsistence during my
lifetime, and the burial expenses after my death."
Answer
(a) This donation is a conditional donation inter vivos. It is clear that the
donor in the instant case has imposed upon the donee a burden or charge whose
value is less than the value of the thing given. According- to the Civil Code, such a
donation inter vivos is conditional. True, the donor designated the donation as a
donation mortis causa, but this is not controlling. It merely indicates when the
delivery to the donee shall be effected. The condition imposed by the donor, on the
other hand, indicates that the donation is immediately operative. Hence, it is inter-
vivos in character. The same is also true with the specification that the donation will
be deemed revoked if the donee does not comply with the condition. This indicates
that the donation is inter vivos in character.
(b) This donation is clearly a remuneratory donation inter vivos because it is
given by a person to another on account of the services rendered by the latter to the
former, provided that they do not constitute a demandable debt. According to the
Civil Code, such a donation is a remuneratory donation inter vivos.
(c) First Answer: This donation is an onerous donation inter vivos. The
obligation of the donee to defray the expenses for the donor is subsistence during
his entire lifetime and the burial expenses indicates that such obligation is the
consideration for the donation and vice versa. The properties donated are the
consideration for the obligation.
Page 163 of 391
Second Answer: This donation is a conditional donation inter vivos. It is clear
that the obligation imposed upon the donee is merely a charge or burden whose
value is less than the value of the thing given. According to the Civil Code, such a
donation is a conditional donation inter vivos.
(Note: The answer to (a) is based on Art. 726 of the Civil Code and on
decision of the Supreme Court such as Laureta vs. Mata, 46 Phil. 668, Concepcion
vs. Concepcion, 91 Phil. 823; Cuevas vs, Cuevas, 98 Phil. 68, Zapanta vs. Posadas,
62 Phil. 557, and Puig vs. Penaflorida, 15 SCRA 876.
The answer to (b) is based on Art. 726 of the Civil Code.
The two answers to (c) are based on Arts. 7S3 and 726 of the Civil Code
respectively. The Committee respectfully recommends that both answers should be
considered correct.)
04; Donations; mortis causa; formalities
1990 No 3:
B donated to M a parcel of land in 1980. B made the deed of donation,
entitled Donation Inter Vivos, in a public instrument and M accepted the donation in
the same document. It was provided in the deed that the land donated shall be
immediately delivered to M and that M shall have the right to enjoy the fruits fully.
The deed also provided that B was reserving the right to dispose of said land during
his (Bs) lifetime, and that M shall not register the deed of donation until after Bs
death. Upon Bs death, W, Bs widow and sole heir, filed an action for the recovery
of the donated land, contending that the donation made by B is a donation mortis
causa and not a donation inter vivos. Will said action prosper? Explain your answer.
Answer:
Yes, the action will prosper. The donation is a donation mortis causa because
the reservation is to dispose of all the property donated and, therefore, the donation
is revocable at will. Accordingly, the donation requires the execution of a valid will,
either notarial or holographic (Arts 755, 728 NCC).
04; Donations; requisited for perfection
1998 No VIII.
On July 27, 1997, Pedro mailed in Manila a letter to his brother, Jose, a
resident of Hollo City, offering to donate a vintage sports car which the latter had
long been wanting to buy from the former. On August 5, 1997, Jose called Pedro by
cellular phone to thank him for his generosity and to inform him that he was sending
by mail his letter of acceptance. Pedro never received that letter because it was
never mailed. On August 14,1997, Pedro received a telegram from Iloilo informing
him that Jose had been killed in a road accident the day before (August 13, 1997)
1. Is there a perfected donation? [2%]
2. Will your answer be the same if Jose did mail his acceptance letter but it
was received by Pedro in Manila days after Jose's death? [3%]
Answer:
1. None. There is no perfected donation. Under Article 748 of the Civil Code,
the donation of a movable may be made orally or in writing. If the value of the
personal property donated exceeds five thousand pesos, the donation and the
acceptance shall be made in writing. Assuming that the value of the thing donated, a
vintage sports car, exceeds P5.OOO.OO, then the donation and the acceptance
must be in writing. In this instance, the acceptance of Jose was not in writing,
therefore, the donation is void. Upon the other hand, assuming that the sports car
Page 164 of 391
costs less than P5.00O.OO. then the donation maybe oral, but still, the simultaneous
delivery of the car is needed and there being none, the donation was never
perfected.
Answer:
2. Yes, the answer is the same. If Jose's mail containing his acceptance of
the donation was received by Pedro after the former's death, then the donation is
still void because under Article 734 of the Civil Code, the donation is perfected the
moment the donor knows of the acceptance by the donee. The death of Jose before
Pedro could receive the acceptance Indicates that the donation was never
perfected. Under Article 746 acceptance must be made during the lifetime of both
the donor and the donee.
04; Donations; requisites
b) Anastacia purchased a house and lot on installments at a housing project
in Quezon City. Subsequently, she was employed in California and a year later, she
executed a deed of donation, duly authenticated by the Philippine Consulate in Los
Angeles, California, donating the house and lot to her friend Amanda. The latter
brought the deed of donation to the owner of the project and discovered that
Anastacia left unpaid installments and real estate taxes. Amanda paid these so that
the donation in her favor can be registered in the project owner's office. Two months
later, Anastacia died, leaving her mother Rosa as her sole heir. Rosa filed an action
to annul the donation on the ground that Amanda did not give her consent in the
deed of donation or in a separate public instrument. Amanda replied that the
donation was an onerous one because she had to pay unpaid installments and
taxes; hence her acceptance may be implied. Who is correct? (2%)
SUGGESTED ANSWER:
Rosa is correct because the donation is void. The property donated was an
immovable. For such donation to be valid, Article 749 of the New Civil Code requires
both the donation and the acceptance to be in a public Instrument. There being no
showing that Amanda's acceptance was made in a public instrument, the donation is
void. The contention that the donation is onerous and, therefore, need not comply
with Article 749 for validity is without merit. The donation is not onerous because it
did not impose on Amanda the obligation to pay the balance on the purchase price
or the arrears in real estate taxes. Amanda took it upon herself to pay those
amounts voluntarily. For a donation to be onerous, the burden must be imposed by
the donor on the donee. In the problem, there is no such burden imposed by the
donor on the donee. The donation not being onerous, it must comply with the
formalities of Article 749.
ALTERNATIVE ANSWER:
Neither Rosa nor Amanda is correct. The donation is onerous only as to the
portion of the property corresponding to the value of the installments and taxes paid
by Amanda.
The portion in excess thereof is not onerous. The onerous portion is governed
by the rules on contracts which do not require the acceptance by the donee to be in
any form. The onerous part, therefore, is valid. The portion which is not onerous
must comply with Article 749 of the New Civil Code which requires the donation and
the acceptance thereof to be in a public instrument in order to be valid. The
acceptance not being in a public instrument, the part which is not onerous is void
and Rosa may recover it from Amanda.
04; Donations; revocation
1991 No 14;
Page 165 of 391
Spouses Michael and Linda donated a 3-hectare residential land to the City
of Baguio on the condition that the city government would build thereon a public park
with a boxing arena, the construction of which shall commence within six (6) months
from the date the parties ratify the donation. The donee accepted the donation and
the title to the property was transferred in its name. Five years elapsed but the public
park with the boxing arena was never started. Considering the failure of the donee to
comply with the condition of the donation, the donor-spouses sold the property to
Ferdinand who then sued to recover the land from the city government.
Will the suit prosper?
Answer:
Ferdinand has no right to recover the land. It is true that the donation was
revocable because of breach of the conditions. But until and unless the donation
was revoked, it remained valid. Hence, Spouses Michael and Linda had no right to
sell the land to Ferdinand. One cannot give what he does not have. What the donors
should have done first was to have the donation annulled or revoked. And after that
was done, they could validly have disposed of the land in favor of Ferdinand.
Alternative Answer:
A. Until the contract of donation has been resolved or rescinded under
Article 1191 of the Civil Code or revoked under Art. 764 of the Civil Code, the
donation stands effective and valid. Accordingly, the sale made by the donor to
Ferdinand cannot be said to have conveyed title to Ferdinand, who, thereby, has no
cause of action for recovery of the land acting for and in his behalf.
B. The donation is onerous, And being onerous, what applies is the law on
contracts, and not the law on donation (De Luna us. Abrigo, 81 SCRA 150).
Accordingly, the prescriptive period for the filing of such an action would be the
ordinary prescriptive period for contacts which may either be six or ten depending
upon whether it is verbal or written. The filing of the case five years later is within the
prescriptive period and, therefore, the action can prosper,
Alternative Answer;
The law on donation lays down a special prescriptive period in the case of
breach of condition, which is four years from non-compliance thereof (Article 764
Civil Code). Since the action has prescribed, the suit will not prosper,
04; Donations; valid acceptance
1985 No. 6
As a token of affection and esteem for his friend, B, A donated to him by
means of a public document his lot at No. 2 Dart, Paco, Manila. In the same
instrument, he also donated to B an apartment of the Towers Condominium, Makati,
in consideration of his services as manager of Aps business during his long illness.
B accepted the two donations in a separate public instrument executed on the same
day A died but sent to him a day later. B thereafter demanded the delivery of the lot
and apartment donated to him but A's heirs objected on the ground that the
donations were void because the donor did not come to know of the acceptance
prior to his death.
Discuss the validity of the questioned donations.
Answer:
1. The donation of the lot is out of pure liberality and therefore governed by
the provisions of donation wherein it is required that the donor must be notified of
the acceptance during his lifetime. There is, here, no valid acceptance, hence the
donation of the lot is not valid.
Page 166 of 391
The donation of the apartment to B is an onerous or remuneratory donation
governed by general provisions On contract. There is no need of knowledge of the
acceptance by the other party because the services have already been, rendered.
The donation of the apartment being in consideration of services is an onerous
donation governed by the rules on Contracts which requires knowledge of the
acceptance.
2. The donation of the lot is not valid because it was not properly accepted
since the donor died before knowing of the acceptance.
3. The donation of the lot (Art. 726) cannot be said to have been validly
perfected. The donation is perfected from the moment the donor knows of the
acceptance (Art. 734) which the law requires must be made during the lifetime of the
donor and of the donee (Art. 746). Since the donor never came to know of the
acceptance, the contention of the heirs of A that the donation did not become
operative is well-taken.
The same is true as regards the apartment unit
4. The donation of the lot (Art. 726) cannot be said to have validly perfected.
The donation is perfected from the moment the donor knows of the acceptance (Art.
734) which the law requires must be made during the lifetime of the donor and of the
donee (Art. 746). Since the donor never came to know of the acceptance, the
contention of the heirs of A that the donation did not become operative is well-taken.
As regards the apartment unit, the "donation" therefore may be interpreted as
dacion en pago (Art. 1245) on the assumption that the services rendered to the
"donor" constituted a demandable debt (Art. 726). Hence, the donation is valid.
5. Considering the management of A's business during his long illness, the
donations are in contemplation of death, and therefore void.
Page 167 of 391
04; Intellectual creation
2004 No. III
B. Dr. ALX is a scientist honored for work related to the human genome
project. Among his pioneering efforts concern stem cell research for the cure of
Alzheimers disease. Under corporate sponsorship, he helped develop a microbe
that ate and digested oil spills in the sea.
Now he leads a college team for cancer research in MSS State. The team
has experimented on a mouse whose body cells replicate and bear cancerous
tumor. Called oncomouse, it is a life-form useful for medical research and it is a
novel creation. Its body cells do not naturally occur in nature but are the product of
mans intellect, industry and ingenuity. However, there is a doubt whether local
property laws and ethics would allow rights of exclusive ownership on any life-form.
Dr. ALX needs your advice: (1) whether the reciprocity principle in private
international law could be applied in our jurisdiction; and (2) whether there are legal
and ethical reasons that could frustrate his claim of exclusive ownership over the
life-form called oncomouse in Manila? What will be your advice to him? (5%)
05; Law as a mode of acquiring ownership
1988 No. 4:
(a) What is meant by "law" as a mode of acquiring ownership? What are the
different instances under the Civil Code whereby there is an acquisition of
ownership by operation of law? State at least three.
Answer:
(A) When the Civil Code speaks of law as a distinct mode of acquiring
ownership, it refers to those instances where the law, independently of the other
modes of acquiring ownership, automatically and directly vests the ownership of the
thing in a certain individual once the prescribed requisites or conditions are present
or complied with. Examples of this are:
(1) Land which belongs exclusively to either of the spouses where a
building is constructed with conjugal funds. Here, the ownership of the land is vested
automatically in the conjugal partnership once the condition that its value has been
reimbursed to the owner has been complied with (Art. 158, par. 2, CC.)
(2) Hidden treasure which a stranger discovers by chance on another's
property. Here, one-half of the treasure belongs by right of occupation to the
stranger, while the other half belongs by operation of law to the proprietor. (Art. 438,
par. 2, CC.)
(3) Abandoned beds, when a river or stream suddenly changes its course to
traverse private lands. The former owners of the new bed shall be the owners of the
abandoned bed in proportion to the area lost by each. (Art. 58, P.D. No. 1067.)
(4) Fruits naturally falling from a tree upon adjacent land. Here, the
ownership of the fruits is vested automatically in the owner of the adjacent land. (Art.
681, CC.)
Committee's Recommendations Re: No. 4 (a)
(a) It is recommended that the following be likewise considered as instances
whereby there is acquisition of ownership by operation of law:
(1) The acquisition of property in co-ownership under a marriage governed by
the absolute community regime.
(2) Estoppel under article 1434 of the Civil Code which provides that:
Page 168 of 391
"When a person who is not the owner of a thing sells or alienates and
delivers it, and later the seller or grantor acquires title thereto, such title passes by
operation of law to the buyer or grantee".; and
(3) Registration of land under Act 496 where the applicant is not the real
owner.
05; Modes of acquiring ownership
1977 No. VI-a
What are the modes of acquiring ownership and other real rights under the
New Civil Code of the Philippines?
Answer
Under our Civil Code, the modes of acquiring ownership and other real rights
are the following:
(1) Occupation;
(2) Intellectual creation;
(3) Prescription;
(4) Law;
(5) Donation;
(6) Testate and intestate succession; and
(7) In consequence of certain contracts, tradition. (Art. 712, Civil Code).
Page 169 of 391
05; Property; accession
1977 No. III-c
Distinguish the following concepts: Accession, accretion, alluvion, avulsion.
Answer
Accession is the right pertaining to the owner of a thing over everything which
is produced thereby, or which is incorporated or attached thereto, either naturally or
artificially (Art. 440, Civil Code).
As applied to accession continua with respect to immovable property
{accession natural), accretion refers to that which is added or attached to the
principal thing due to natural causes.
Alluvion refers to the accretion which land adjoining the banks of rivers, lakes,
creeks or torrents gradually receive from the effects of the current of the water (Art.
467, Civil Code).
Avulsion refers to the accretion which takes place whenever the current of a
river, lake, creek or torrent segregates from an estate on its bank a known portion of
land and transfers it to another estate (Art. 459, Civil Code),
05; Property; accession
1981 No. 18
(d) True or false? -- To the owners of lands adjoining the sea belong the
accretion which they gradually receive from effects of the current on the waters.
Answer
(d) False. The principle of alluvium cannot be applied to accretions due to
the action of the current of the sea. It can be applied only to accretions due to the
action of the current of a river, lake, creeks or torrent. (Art. 457, Civil Code)
05; Property; accession; accretion
1989 No. 6:
(2) Spouses "A" and "B" are registered owners of lot "1" consisting of 20,000
square meters while spouses "C" and "D" are owners of lot "2". These lots arc
separated by a river. For a period of more than 40 years, the river overflowed its
banks yearly and the property of the spouses "C" and "D* gradually received
deposits of soil from the effects of the current of the river so that an alluvial deposit
of 29,000 square meters was added to their lot, 11,000 square meters of which used
to be part of lot "1." Spouses "A" and "B" contend that accretion should not extend to
registered land because to allow the spouses "C" and "D" to acquire title over the
accretion will be in derogation of the indefeasibility of the Torrens Title of spouses
"A" and "B". Is this contention correct? Explain.
Answer:
No, the contention of A and B is not correct because the registration under
the Torrens Law does not protect the owner against the diminution of his land
through gradual changes due to the effects of the current of the river. The accretion
will benefit C and D.
05; Property; accession; accretion
1979 No. VI
In 1951, PE occupied the bank of the river in San Mateo which wag called
Libis. By force of the current of the river, silt was deposited on this river bank so that
it gradually became wider and wider until it measured about 2 hectares. In 1976, VA,
Page 170 of 391
the registered owner of the land adjoining the old river bank, demanded that PE
vacate the land but the latter refused claiming he had acquired the alluvial deposits
by prescription. VA then filed an action to quiet the title and recover possession. Will
the action prosper? Why?
Answer
The action will prosper. According to the Civil Code, to the owners of lands
adjoining the banks of rivers belong the accretion which they gradually receive from
the effects of the current of the waters. The silt or alluvial deposit occupied by
defendant PE, therefore, belongs to VA, the riparian owner. However, since the
latter never sought registration of the accretion in accordance with the Land
Registration Law, said accretion never became registered property. It was therefore
possible for PE to become absolute owner thereof by extraordinary acquisitive
prescription. Since obviously, he was in bad faith, thirty years of continuous, public
and peaceful possession in concept of owner would be necessary in order that he
shall become the absolute owner of the accretion by prescription. He took
possession of the property in 1951; the action of VA against him was filed in 1976,
That means that he has been in continuous possession for only twenty five years.
The defense of acquisitive prescription, therefore, is untenable. (See Grande vs. CA
L-17652, June 30,1962).
05; Property; accession; avulsion
1985 No. 7
After one week of torrential rains, a portion of A's plantation, with an area of
one (1) hectare and planted to 100 coconut trees, was eroded, while to B's farm, on
the other bank of the same river, a tract of land, also one (1) hectare in area, on
which stood 50 coconut trees, was added. An equal number of trees, their roots
exposed, were found lying on the ground in B's property. Seven months later, A,
alleging that the one-hectare lot and 100 coconut trees were his, demanded their
return but B, who had previously token possession of them, refused, claiming that
the land was formed by alluvion and, therefore, belongs to him and that A has lost
his right to the coconut trees because he did not lay claim to them in due time. A
thereupon sued B for the recovery of the land and the coconuts.
Will the action prosper? State the legal basis of your answer.
Answers:
1. The one hectare land and the fifty coconut trees still standing thereon, if
identifiable as the same property detached from A's plantation can be recovered by
A since the two-year period of prescription of the action has not yet lapsed.
The 50 uprooted coconut trees and the coconuts can no longer be claimed by
A since the claim has already prescribed, such claim having been made beyond the
period of six months.
2. A can recover the land and the standing coconut trees provided he prove
three things:
a. a known portion of land
b. (that it was detached by the current of the river
c. that he recover the land within 2 years, This is a case of avulsion and not
alluvion.
3. There is no alluvion because the addition to B's land was not gradual and
imperceptible. There is no avulsion because the detachment of the land was not due
to the action of the current of the river but due to torrential rains.
Page 171 of 391
However, A may recover land and standing trees if he can identify the same
due to the principle of unjust enrichment because he was damaged and B was
enriched without just cause due to fortuitous event. As regards to uprooted trees, A's
action is lost because he did not claim them within 6 months.
4. The provisions on avulsion, rather than alluvion apply. Accordingly, the
action for the recovery of the land will still prosper. The law allows the owner of the
segregated land to remove the same within two years from the time avulsion takes
place (Art. 459). A, however, had lost his right to recover the coconut trees which
can only be claimed within six months (Art. 460).
05; Property; accession; commixtion
1986 No. 7:
Mr. Magabun and Mr. Ortelano each delivered 1,000 kilos of palay to Mr.
Kono for milling. Magabun's rice was Milagrosa quality and was worth three times
more per kilo than the rice of Ortelano. Before Mr, Kono could mill the rice, an
accidental fire broke out in the mill. Kono was able to save one half of the rice of
both Magabun and Ortelano but in the confusion, the rice ended up mixed and
commingled. What are the respective rights of Magabun and Ortelano over the
mixture? Explain.
Answer:
Magabun, owner of the Milagrosa rice, shall own 3/4 interest of the mixture,
while Ortelano, the owner of the regular rice shall own 1/4 interest in the mixture.
This is because the Milagrosa rice was worth three times more than the ordinary
rice.
Answer - This is a case of commixtion under C.C, Both Magabun and
Ortelano will be owners of the rice saved, in the proportion of 3:1.
Answer - Magabun and Ortelano shall each acquire a right proportional to the
part belonging to him, bearing in mind the value of the things mixed.
We have here a perfect example of what the law calls commixtion effected by
chance. That means that each shall be considered as having acted in good faith.
Since the rice belonging to Magabun is worth three times more- per kilo than the rice
belonging to Ortelano, therefore, the mixture will be divided between the two in the
proportion of three is to one. Magabun shall be entitled to three-fourths (3/4) of the
mixture, while Ortelano shall be entitled to one-fourth (1/4).
(Note - The above answer is based upon Art. 472 of the Civil Code.)
05; Property; accession; rivers
1977 No. IV-b
A's land is bounded on the South by the sea and on the East by a river. Both
sides have grown by accretion. What should A do to obtain a certificate of title to the
enlarged areas?
Answer
A cannot do anything with respect to the accretion on the south by the sea.
He has no right over it. Such accretion is property of public dominion (Art. 420, Civil
Code). As such, it is outside the commerce of man. Therefore, it cannot be
appropriated; neither can it be acquired by prescription. (Ignacio vs. Dir. of Lands,
108 Phil. 335.)
However, as far as the accretion on the east by the river is concerned,
assuming that all of the requisites of alluvion under the Civil Code (Art. 457) are
present, A is now ipso jure the owner of such accretion. In order that he can acquire
Page 172 of 391
a certificate of title (or an amendment thereof), he should now comply with all of the
different steps dictated by the Land Registration Law (Act No. 496) in order that
there will be a judicial confirmation of his title over the accretion.
05; Property; accretion
2003 No IX.
Andres is a riparian owner of a parcel of registered land. His land, however,
has gradually diminished in area due to the current of the river, while the registered
land of Mario on the opposite bank has gradually increased in area by 200-square
meters.
(a) Who has the better right over the 200-square meter area that has been
added to Marios registered land, Mario or Andres?
(b) May a third person acquire said 200-square meter land by prescription?
SUGGESTED ANSWER:
a. Mario has a better right over the 200 square meters increase in area by
reason of accretion, applying Article 457 of the New Civil Code, which provides that
to the owners of lands adjoining the banks of rivers belong the accretion which they
gradually received from the effects of the current of the waters.
Andres cannot claim that the increase in Marios land is his own, because
such is an accretion and not result of the sudden detachment of a known portion of
his land and its attachment to Marios land, a process called avulsion. He can no
longer claim ownership of the portion of his registered land which was gradually and
naturally eroded due to the current of the river, because he had lost it by operation
of law. That portion of the land has become part of the public domain.
b. Yes, a third party may acquire by prescription the 200 square meters,
increase in area, because it is not included in the Torrens Title of the riparian owner.
Hence, this does not involve the Imprescriptibility conferred by Section 47, P.D. No.
1529. The fact that the riparian land is registered does not automatically make the
accretion thereto a registered land. (Grande v. CA, 115 521 (1962); Jagualing v. CA,
194 SCRA 607 (1991).
05; Property; accretion; alluvion
2001 No IV
For many years, the Rio Grande river deposited soil along its bank, beside the
titled land of Jose. In time, such deposit reached an area of one thousand square
meters. With the permission of Jose, Vicente cultivated the said area. Ten years
later, a big flood occurred In the river and transferred the 1000 square meters to the
opposite bank, beside the land of Agustin. The land transferred is now contested by
Jose and Agustin as riparian owners and by Vicente who claims ownership by
prescription. Who should prevail,? Why? (5%)
SUGGESTED ANSWER
Jose should prevail. The disputed area, which is an alluvion, belongs by right
of accretion to Jose, the riparian owner (Art. 457 CC). When, as given in the
problem, the very same area" was "transferred" by flood waters to the opposite
bank, it became an avulsion and ownership thereof is retained by Jose who has two
years to remove it (Art. 459, CC). Vicente's claim based on prescription is baseless
since his possession was by mere tolerance of Jose and, therefore, did not
adversely affect Jose's possession and ownership (Art. 537, CC). Inasmuch as his
possession is merely that of a holder, he cannot acquire the disputed area by
prescription.
Page 173 of 391
05; Property; chattel mortgage over immovables
1994 No. 15:
Vini constructed a building on a parcel of land he leased from Andrea. He
chattel mortgaged the land to Felicia. When he could not pay Felicia. Felicia initiated
foreclosure proceedings. Vini claimed that the building he had constructed on the
leased land cannot be validly foreclosed because the building was, by law, an
immovable.
Is Vini correct? Alternative Answers':
a) The Chattel Mortgage is void and cannot be foreclosed because the
building is an immovable and cannot be an object of a chattel mortgage.
b) It depends. If the building was intended and is built of light materials, the
chattel mortgage may be considered as valid as between the parties and it may be
considered in respect to them as movable property, since it can be removed from
one place to another. But if the building is of strong material and is not capable of
being removed or transferred without being destroyed, the chattel mortgage is void
and cannot be foreclosed.
c) If it was the land which Vini chattel mortgaged, such mortgage would be
void, or at least unenforceable, since he was not the owner of the land.
If what was mortgaged as a chattel is the building, the chattel mortgage is
valid as between the parties only, on grounds of estoppel which would preclude the
mortgagor from assailing the contract on the ground that its subject-matter Is an
immovable. Therefore Vini's defense is untenable, and Felicia can foreclose the
mortgage over the building, observing, however, the procedure prescribed for the
execution of sale of a judgment debtor's immovable under Rule 39, Rules of Court,
specifically, that the notice of auction sale should be published in a newspaper of
general circulation.
d) The problem that Vini mortgaged the land by way of a chattel mortgage is
untenable. Land can only be the subject matter of a real estate mortgage and only
an absolute owner of real property may mortgage a parcel of land. (Article 2085 (2)
Civil Code). Hence, there can be no foreclosure.
But on the assumption that what was mortgaged by way of chattel mortgage
was the building on leased land, then the parties are treating the building as chattel.
A building that uis not merely superimposed on the ground is an immovable property
and a chattel mortgage on said building is legally void but the parties cannot be
allowed to disavow their contract on account of estoppel by deed. However, if third
parties are involved such chattel mortgage Is void and has no effect.
05; Property; co-ownership
2000 No XVII.
In 1955, Ramon and his sister Rosario inherited a parcel of land in Albay from
their parents. Since Rosario was gainfully employed in Manila, she left Ramon alone
to possess and cultivate the land. However, Ramon never shared the harvest with
Rosario and was even able to sell one-half of the land in 1985 by claiming to be the
sole heir of his parents. Having reached retirement age in 1990 Rosario returned to
the province and upon learning what had transpired, demanded that the remaining
half of the land be given to her as her share. Ramon opposed, asserting that he has
already acquired ownership of the land by prescription, and that Rosario is barred by
laches from demanding partition and reconveyance. Decide the conflicting claims.
(5%)
SUGGESTED ANSWER:
Page 174 of 391
Ramon is wrong on both counts: prescription and laches. His possession as
co-owner did not give rise to acquisitive prescription. Possession by a co-owner is
deemed not adverse to the other co-owners but is, on the contrary, deemed
beneficial to them (Pongon v. GA, 166 SCRA 375). Ramon's possession will
become adverse only when he has repudiated the co-ownership and such
repudiation was made known to Rosario. Assuming that the sale in 1985 where
Ramon claimed he was the sole heir of his parents amounted to a repudiation of the
co-ownership, the prescriptive period began to run only from that time. Not more
than 30 years having lapsed since then, the claim of Rosario has not as yet
prescribed. The claim of laches is not also meritorious. Until the repudiation of the
co-ownership was made known to the other co-owners, no right has been violated
for the said co-owners to vindicate. Mere delay in vindicating the right, standing
alone, does not constitute laches.
ALTERNATIVE ANSWER:
Ramon has acquired the land by acquisitive prescription, and because of
laches on the part of Rosario. Ramon's possession of the land was adverse because
he asserted sole ownership thereof and never shared the harvest therefrom. His
adverse possession having been continuous and uninterrupted for more than 30
years, Ramon has acquired the land by prescription. Rosario is also guilty of laches
not having asserted her right to the harvest for more than 40 years.
05; Property; co-ownership
2002 No IV.
Antonio, Bart, and Carlos are brothers. They purchased from their parents
specific portions of a parcel of land as evidenced by three separates deeds of sale,
each deed referring to a particular lot in meter and bounds. When the deeds were
presented for registration, the Register of Deeds could not issue separate
certificates of Title had to be issued, therefore, in the names of three brothers as co-
owners of the entire property. The situation has not changed up to now, but each of
the brothers has been receiving rentals exclusively from the lot actually purchased
by him. Antonio sells his lot to a third person, with notice to his brothers. To enable
the buyer to secure a new title in his name, The deed of sale was made to refer to
undivided interest in the property of the seller (Antonio), with the metes and bounds
of the lot sold being stated. Bart and Carlos reacted by signifying their exercise of
their right of redemption as co owners. Antonio in his behalf and in behalf of his
buyer, contends that they are no longer co-owners, although the title covering the
property has remained in their names as such.
May Bart and Carlos still redeem the lot sold by Antonio? Explain. (5%)
SUGGESTED ANSWER:
No, they may not redeem because there was no Co-ownership among
Antonio, Bart, and Carlos to start with. Their parents already partitioned the land in
selling separate portions to them. The situation is the same as in the case Si v.
Court of Appeals, (342 SCRA 653 [2000]).
05; Property; co-ownership
1986 No. 6:
Magaling, Malugod and Masanting are co-owners in equal shares, pro
indiviso, of a 9,000 square meter residential lot in Quezon City. Magaling needs
money badly and sold a specified 3,000 square meter portion of the lot, describing in
the deed the metes and bounds of the part sold. When the buyer demanded the
portion sold to him, Malugod and Masanting argued that under no circumstances
Page 175 of 391
whatsoever may any part of the lot be sold without the consent of the two other co-
owners. Is their contention correct? Explain.
Answer:
A co-owner can always sell his share in the co-ownership without the consent
of the other co-owners. However, the sale is limited to the ideal share or pro-indiviso
share of the vendor, subject to partition later on. The sale of Magaling would,
therefore, only pertain to 1/3 share of the property and the buyer cannot demand a
specific portion of the lot.
Answer - Under Art. 493, a co-owner may sell his undivided interest in the
thing owned in common, without the consent of her co-owners. However, what was
sold here was a determinate portion of the land, which cannot be sold by a co-owner
until the property is partitioned.
Answer - I would like to qualify my answer. If by "part of the lot," Malugod and
Masanting refer to a specific portion of the property owned in common, then they are
correct. The buyer cannot now demand for the delivery of the property sold by
Magaling to him. However, if by, "part of the lot", Malugod and Masanting refer to the
undivided or pro indiviso share of Magaling, then they are not correct. Under the law,
each co-owner may alienate his "part," but the effect with respect to the co-owners
shall be limited to the portion which may be allotted to him in the division upon the
termination of the co-ownership.
(Note - The above answer is based upon Art. 493 of the Civil Code. We
recommend that an answer declaring that
Malugod and Masanting are correct using Art. 493 as reason should also be
considered correct.)
Answer - The sale is valid (Clarin vs. Rulona 127 SCRA 512). The Supreme
Court has ruled that sale of a definite portion is valid (Per J. Gutierrez).
05; Property; co-ownership
1992 No 6;
A, B and C are the co-owners in equal shares of a residential house and lot.
During their co-ownership, the following acts were respectively done by the co-
owners:
1. A undertook the repair of the foundation of the house, then tilting to one
side, to prevent the house from collapsing.
2. B and C mortgaged the house and lot to secure a loan.
3. B engaged a contractor to build a concrete fence all around the lot.
4. C built a beautiful grotto In the garden.
5. A and C sold the land to X for a very good price.
a) Is A's sole decision to repair the foundation of the house binding on B and
C? May A require B and C to contribute their 2/3 share of the expense? Reasons.
b) What is the legal effect of the mortgage contract executed by B and C?
Reasons.
c) Is B's sole decision to build the fence binding upon A and C? May B
require A and C to contribute their 2/ 3 share of the expense? Reasons.
d) Is C's sole decision to build the grotto binding upon A and B? May C
require A and B to contribute their 2/ 3 share of the expense? Reasons.
Page 176 of 391
e) What are the legal effects of the contract of sale executed by A. C and X?
Reasons.
Answer:
(a) Yes. A's sole decision to repair the foundation is binding upon B and C.
B and C must contribute 2/3 of the expense. Each co-owner has the right to compel
the other co-owners to contribute to the expense of preservation of the thing (the
house) owned in common in proportion to their respective interests (Arts. 485 and
488, Civil Code).
(b) The mortgage shall not bind the 1/3 right and Interest of A and shall be
deemed to cover only the rights and Interests of B and C in the house and lot. The
mortgage shall be limited to the portion (2/3) which may be allotted to B and C in the
partition (Art. 493, Civil Code). (c) B's sole decision to build the concrete fence is
not binding upon A and C. Expenses to improve the thing owned in common must
be decided upon by a majority of the co-owners who represent the controlling
interest (Arts. 489 and 492. Civil Code).
(d) C's sole decision to build the grotto is not binding upon A and B who
cannot be required to contribute to the expenses for the embellishment of the thing
owned in common if not decided upon by the majority of the co-owners who
represent the controlling interest (Arts. 489 and 492, Civil Code).
(e) The sale to X shall not bind the 1 /3 share of B and shall be deemed to
cover only the 2/3 share of A and C in the land (Art. 493, Civil Code). B shall have
the right to redeem the 2/3 share sold to X by A and C since X is a third person (Art.
1620, Civil Code).
05; Property; co-ownership
1975 No. X
A lot containing an area of 1,561 square meters located in Plaza Sta. Cruz
and Escolta in Manila is owned in common by six (6) persons. One of the co-owners
asked for a physical segregation of his one-sixth (1/6) share. The five (5) co-owners
objected on the ground that the lot being commercial, its value would be greatly
impaired should there be a physical partition. Decide.
Answer
The co-owner who asked for a partition is entitled to the segregation of his
share. The general rule enunciated in Article 494 of the Civil Code is applicable,
namely that each co-owner may demand at any time the partition of the thing owned
in common, insofar as his share is concerned.
The objection of the five other co-owners is obviously based on Article 495
which provides that notwithstanding the provisions of Article 494, the co-owners can
not demand a physical division of the thing owned in common, when to do so would
render it unserviceable for the use for which it is intended. Article 495, however, can
not apply to this case for if the one-sixth share of the co-owner is segregated, there
will still remain over 1,300 square meters for the remaining five co-owners and a real
estate of this size in the heart of Manila is not inconsequential. (Ramirez v. Ramirez,
20 SCRA 384)
05; Property; co-ownership
1981 No. 5
In a partition between brothers of inherited property, brother "A" got the
property north of the river. Brother "B" received the property south of the river.
Without the knowledge of the brothers, squatters had moved into the property north
of the river allotted to "A". "A" also learned that "B" was about to sell his property.
Page 177 of 391
a) "A" wanted "B" to join him in the ejectment case against the squatters and
share in litigation expenses. "B" refused. Was "B's" refusal valid? Why?
b) Could "A" prevent the sale by "B" of his portion of the property? Reasons.
Answer
(a) "B's" refusal is valid. The reason is obvious. There was already a partition.
In the partition, brother "A" was awarded the property north of the river, while brother
"B" was awarded the property south of the river. Consequently, the tie of co-
ownership or indivision was dissolved completely- Thus, the problem of squatters in
the portion allotted to "A" has become his exclusive problem.
(Note: The above answer is based on general principles of co-ownership and
partition. The Committee, however, respectfully recommends that if the bar
candidate invokes Art. 501 of the Civil Code or the principle of warranty among
partitioners, it should be properly credited. |.
(b) " A'' cannot prevent the sale of "B'' of the portion allotted to him. That
would constitute a violation of the jus disponendi or right of disposition of "B". As a
matter of fact, "A" cannot even avail of the right of legal redemption of adjacent
owners. Such right is not available because the two lands are separated by a river.
(Note: The above answer is based on the general principles of ownership
{Art. 428, Civil Code) and on Art. 1621, Civil Code.)
05; Property; co-ownership
1983 No. 6
Three of four brothers, the sole heirs of their deceased parents, agreed to
convert a ricefield in the estate into a subdivision and spend the money, also left by
their parents, for developing the subdivision. The fourth son disagreed and brought a
suit to enjoin his brothers from proceeding with the subdivision and spending the
money they inherited for its development.
Will the action prosper? Why?
Answer
It will, in so far as it seeks to stop the conversion of the ricefield into a
subdivision, for this is an alteration which requires the consent of all the co-owners.
But if the refusal of the plaintiff is clearly prejudicial to the common interest, the
alteration may be allowed.
It will also prosper in so far as it prays that the money of the estate be not
diverted to the development of the subdivision. The expenditure is not a mere act of
administration but of dominion which requires the consent of all.
With respect to the lease, the action will succeed if It created a real right;
otherwise, it will fail.
05; Property; co-ownership vs partnership
1988 No. 2:
(a) Distinguish co-ownership from partnership.
(b) Is the lease of the entire community property in co-ownership an act of
administration or an act of ownership or alteration? Explain, in relation to the need of
consent of the co-owners.
Answer:
(a) Co-ownership is distinguished from an ordinary partnership in the
following ways:
Page 178 of 391
(1) As to creation; Whereas co-ownership may be created by law, contract,
succession, fortuitous event or occupancy, partnership is always created by
contract.
(2) As to purpose: Whereas the purpose of co-ownership is the common
enjoyment of the thing or right owned in common, the purpose of a partnership is to
obtain profits
(3) As to personality; Whereas a co-ownership has no juridical personality
which is separate and distinct from that of the owners, a partnership has.
(4) As to duration: Whereas an agreement not to divide the community
property for more than ten years is not allowed by law, such an agreement would be
perfectly valid in the case of partnerships. This is so, because under the law, there is
no limitation upon the duration of partnerships,
(5) As to power of members; Whereas a co-owner has no power to
represent the co-ownership, unless there is an agreement to that effect, a partner
has the power to represent the partnership, unless there is a stipulation to the
contrary.
(6) As to effect of disposition of shares; If a co-owner transfers his share to a
third person, the latter becomes automatically a co-owner, but if a partner transfers
his share to a third person, the latter does not become a partner, unless agreed
upon by all of the partners.
(7) As to division of profits: Whereas in co-ownership the division of the
benefits and charges is fixed by law, in a partnership the division of profits and
losses may be subject to the agreement of the partners.
(8) As to effect of death: Whereas the death of a co-owner has no effect
upon the existence of the co-ownership, the death of a partner shall result in the
dissolution of the partnership.
(b) Lease of personal property is a mere act of administration, and, therefore,
requires the resolution of the majority of the co-owners. However, Lease of real
property may be an act of administration or an act of alteration depending upon the
circumstances of each particular case. (1) If the lease is recorded in the Registry of
Property, whatever may be the duration thereof, it is an act of ownership, and
therefore, requires the unanimous consent of all the co-owners, since under the law,
a special power of attorney is required (See Art. 1647, CC). (2) If the lease is not
recorded in the Registry of Property, but the duration thereof is more than one year,
it is also an act of ownership, and therefore, requires the unanimous consent of all
the co-owners, since, again, under the law, a special power of attorney is required
(See Art. 1878 No. 8, CC). (3) If the lease, however, is not recorded in the Registry
of Property and the duration thereof is only one year or less, it is an act of
administration, and therefore, merely requires the resolution of the majority of the
co-owners.
Committee's Recommendation Re: No, 2 (a):
(a) It is recommended that a mention of three distinctions should merit a full
credit for this question.
05; Property; co-ownership; acts of administration
1984 No. 6
Brothers A, B and C are co-owners of a two-story building which they
inherited from their deceased parents A, the eldest, requested B and C to contribute
for the repair of the roof which was already leaking and for the renovation of the first
floor of the building so that the same could be rented out. B and C refused.
Page 179 of 391
Nevertheless, A had the repair and renovation dams at his own expenses.
Consequently, the brothers were able to lease out the first floor and generate rental
in-come.
What are the right of A, B and C with respect to the expenditures made by A
and the subsequent rental income? Explain.
Answer:
A. Furnished by Office of Justice Palma
The repairs of the building are a necessary expenses for its preservation.
Hence, A can proceed to incur the expenses for such repairs even without the
consent of the co-owners (Art. 489). He can recover the proportionate share of the
others in these expenses.
Since B and C, who constitute the majority in interest in the co-ownership,
refused to have the renovation of the ground floor made, A, had no authority to
undertake the renovation (Art. 492). Since be proceeded to do so anyway, he must
bear the costs thereof without right to reimbursement. The co-owners would be
entitled to the rentals in proportion to their interest in the co-ownership.
B. Comments and Suggested Answer:
We agree with the answer of the Bar Examiner.
05; Property; co-ownership; prescription
2002 No V.
Senen and Peter are brothers. Senen migrated to Canada early while still a
teenager. Peter stayed in Bulacan to take care of their widowed mother and
continued to work on the Family farm even after her death. Returning to the country
some thirty years after he had left, Senen seeks a partition of the farm to get his
share as the only co-heir of Peter. Peter interposes his opposition, contending that
acquisitive prescription has already set in and that estoppel lies to bar the action for
partition, citing his continuous possession of the property for at least 10 years, for
almost 30 years in fact. It is undisputed that Peter has never openly claimed sole
ownership of the property. If he ever had the intention to do so , Senen was
completely ignorant of it. Will Senens action prosper? Explain. (5%).
SUGGESTED ANSWER:
Senens action will prosper. Article 494 of the New Civil Code provides that
no prescription shall run in favor of a co-owner or co-heir against his co-owners or
co-heirs so long as he expressly or impliedly recognizes the co-ownership nor
notified Senen of his having repudiated the same.
ALTERNATIVE ANSWER:
Senens action will prosper. This is a case of implied trust. (Art 1441, NCC)
For purposes of prescription under the concept of an owner (Art. 540, NCC). There
is no such concept here. Peter was a co-owner, he never claimed sole ownership of
the property. He is therefore estopped under Art. 1431, NCC.
05; Property; co-ownership; redemption
2000 No X
a) Ambrosio died, leaving his three daughters, Belen, Rosario and Sylvia a
hacienda which was mortgaged to the Philippine National Bank. Due to the failure of
the daughters to pay the bank, the latter foreclosed the mortgage and the hacienda
was sold to it as the highest bidder. Six months later, Sylvia won the grand prize at
the lotto and used part of it to redeem the hacienda from the bank. Thereafter, she
took possession of the hacienda and refused to share its fruits with her sisters,
Page 180 of 391
contending that it was owned exclusively by her, having bought it from the bank with
her own money. Is she correct or not? (3%)
SUGGESTED ANSWER!
Sylvia is not correct. The 3 daughters are the co-owners of the hacienda
being the only heirs of Ambrosio. When the property was foreclosed, the right of
redemption belongs also to the 3 daughters. When Sylvia redeemed the entire
property before the lapse of the redemption period, she also exercised the right of
redemption of her co-owners on their behalf. As such she is holding the shares of
her two sisters in the property, and all the fruits corresponding thereto, in trust for
them. Redemption by one co-owner inures to the benefit of all (Adille v. CA.157
SCRA 455). Sylvia, however, is entitled to be reimbursed the shares of her two
sisters in the redemption price.
05; Property; co-ownership; redemption by co-owners
1993 No. 4;
In 1937, A obtained a loan of P20,000.00 from the National City Bank of New
York, an American-owned bank doing business in the Philippines. To guarantee
payment of his obligation, A constituted a real estate mortgage on his 30-hectare
parcel of agricultural land. In 1939, before he could pay his obligation. A died
intestate leaving three children. B, a son by a first marriage, and C and D, daughters
by a second marriage. In 1940. the bank foreclosed the mortgage for non-payment
of the principal obligation. As the only bidder at the extrajudicial foreclosure sale, the
bank bought the property and was later issued a certificate of sale. The war
supervened in 1941 without the bank having been able to obtain actual possession
of the property which remained with A's three children who appropriated for
themselves the income from it. In 1948, B bought the property from the bank using
the money he received as back pay from the U. S. Government, and utilized the
same in agribusiness. In 1960, as B's business flourished, C and D sued B for
partition and accounting of the income of the property, claiming that as heirs of their
father they were co-owners thereof and offering to reimburse B for whatever he had
paid in purchasing the property from the bank.
In brief, how will you answer the complaint of C and D, if you were engaged
by D as his counsel?
Answer;
As counsel of B, I shall answer the complaint as follows: When B bought the
property, it was not by a right of redemption since the period therefore had already
expired. Hence, B bought the property in an independent unconditional sale. C and
D are not co-owners with B of the property. Therefore, the suit of C and D cannot
prosper.
Alternative Answer:
As counsel of B, I shall answer the complaint as follows: From the facts
described, it would appear that the Certificate of sale has not been registered. The
one-year period of redemption begins to run from registration. In this case, it has not
yet even commenced. Under the Rules of Court, the property may be released by
the Judgment debtor or his successor in interest. (Sec. 29, Rule 27). It has been
held that this includes a joint owner. (Ref. Magno vs.Ciola, 61 Phil. 80).
05; Property; easement vs usufruct
1995 No. 1;
1. What is easement? Distinguish easement from usufruct.
Page 181 of 391
2. Can there be (a) an easement over a usufruct? (b) a usufruct over an
easement? (c) an easement over another easement? Explain.
Answer;
1. An easement or servitude is an encumbrance imposed upon an immovable
for the benefit of another Immovable belonging to a different owner. (Art. 613, NCC)
Usufruct gives a right to enjoy the property of another with the obligation of
preserving its form and substance, unless the title constituting it or the law otherwise
provides. (Art. 562, NCC). An easement or servitude is an encumbrance imposed
upon an immovable for the benefit of another immovable belonging to a different
owner (Art. 613, NCC).
Alternative Answer;
Easement is an encumbrance imposed upon an immovable for the benefit of
another immovable belonging to a different owner in which case it is called real or
predial easement, or for the benefit of a community or group of persons in which
case it is known as a personal easement.
The distinctions between usufruct and easement are:
a. Usufruct includes all uses of the property and for all purposes, including
jus fruendi. Easement is limited to a specific use.
b. Usufruct may be constituted on immovable or movable property.
Easement may be constituted only on an immovable property.
c. Easement is not extinguished by the death of the owner of the dominant
estate while usufruct is extinguished by the death of the usufructuary unless a
contrary intention appears.
d. An easement contemplates two (2) estates belonging to two (2) different
owners; a usufruct contemplates only one property (real or personal) whereby the
usufructuary uses and enjoys the property as well as its fruits, while another owns
the naked title during the period of the usufruct.
e. A usufruct may be alienated separately from the property to which it
attaches, while an easement cannot be alienated separately from the property to
which it attaches.
NOTE: It is recommended by the Committee that any two (2) distinctions
should be given full credit.
Answer:
2. (a) There can be no easement over a usufruct. Since an easement may be
constituted only on a corporeal immovable property, no easement may be
constituted on a usufruct which is not a corporeal right
(b) There can be no usufruct over an easement. While a usufruct maybe
created over a right, such right must have an existence of its own independent of the
property. A servitude cannot be the object of a usufruct because It has no existence
independent of the property to which It attaches.
Alternative Answers:
There cannot be a usufruct over an easement since an easement
presupposes two (2) tenements belonging to different persons and the right attaches
to the tenement and not to the owner. While a usufruct gives the usufructuary a right
to use, right to enjoy, right to the fruits, and right to possess, an easement gives only
a limited use of the servient estate.
Page 182 of 391
However, a usufruct can be constituted over a property that has in its favor an
easement or one burdened with a servitude. The usufructuary will exercise the
easement during the period of usufruct.
(c) There can be no easement over another easement for the same reason
as in (a). An easement, although it is a real right over an immovable, is not a
corporeal right. There is a Roman maxim which says that: There can be no servitude
over another servitude.
05; Property; easements; basic
1998 No XVI.
Distinguish between:
1. Continuous and discontinuous easements; |2%]
2. Apparent and non-apparent easements; and [2%]
3. Positive and negative easements. [1%] Answer:
1. Continuous easements are those the use of which is or may be
incessant, without the intervention of any act of man, while discontinuous easements
are those which are used at intervals and depend upon the acts of man. (Art. 615,
Civil Code)
Answer:
2. Apparent easements are those which are made known and are
continually kept in view by external signs that reveal the use and enjoyment of the
same, while non-apparent easements are those which show no external indication of
their existence. (Art. 615, Civil Code)
Answer:
3. Positive easements are those which impose upon the owner of the
servient estate the obligation of allowing something to be done or of doing it himself,
while negative easements are those which prohibit the owner of the servient estate
from doing something which he could lawfully do if the easement did not exist. (Art.
615. Civil Code)
05; Property; easements; basic
1977 No. IV-a
Discuss briefly five (5) instances of legal easements.
Answer
(NOTE: There are many legal easements regulated by both the Civil Code
and the Water Code. It is the consensus that in answering the above question it will
be sufficient for the bar candidate to give merely the essence of the easement or to
state the substance of the codal provisions regulating them.)
Among the legal easements which may be discussed are: (1) Easement of
drainage of waters (Art. 637); (2) easement of public use in the interest of
navigation, floatage, fishing and salvage (Art. 638) ; (3) easement of tow-path (Art.
638); (4) easement of abutment of a dam (Art. 639); (5) easement for drawing water
or watering of animals (Art. 640); (6) easement of aqueduct (Arts. 642, 643); (7)
easement of right of way (Arts 649, 650); (8) easement of a party wall (Arts. 659,
666); (9) easement of tight and view (Arts. 668, 670, 673); (10) easement of
drainage of buildings (Art. 676); (11) easement against nuisance (Art. 682); and (12)
easement of lateral and subjacent support (Art. 684).
05; Property; easements; light and view
Page 183 of 391
1979 No. VII
FS was the owner of a big lot in Dagupan Street, Tondo, Manila. On the
southern portion of the lot was a home with doors and windows overlooking the
northern portion of the lot on which a small house was standing. FS subdivided the
lot into two, and sold the southern portion to JB and the northern portion to TV. TY
demolished the small house and obtained a permit to construct a four story building
on his portion which would thus obstruct the view from the doors and windows of
JB's house. JB therefore filed an action to enjoin TY from constructing his building
unless it is at a distance of not less than 3 meters from the boundary lines of the two
portions. Will the action prosper? Why?
Answer
The action will prosper. According to the Civil Code, the existence of an
apparent sign of easement between two estates established or maintained by the
owner of both shall be considered, should either of them be alienated, as a title in
order that the easement shall continue actively and passively, unless at the time the
ownership of the two estates is divided, the contrary should be provided in the title of
conveyance of either of them or the sign should be removed before the execution of
the deed. All of the requisites prescribed by the law are present in the instant case.
There is an apparent sign of the existence of an easement of light and view involving
two estates originally owned by one and the same person. This is indicated by the
doors and windows of the house in the southern portion overlooking the northern
portion. The sign was established by the original owner. The southern portion was
subsequently alienated to JB and the northern portion was also alienated to TY.
furthermore, nothing contrary to the easement was stated in the deed of
conveyance; neither was the sign removed. Therefore, the easement of light and
view shall now be protected by the law. TY cannot construct his building unless he
complies with the three-meter rule as provided by law. (Gargantos vs. Tan Yanon,
108 Phil. 888).
05; Property; easements; light and view
1988 No. 3:
(a) How are easements acquired?
(b) In acquiring easement by prescription, how shall the period of
possession be computed?
(c) About fifteen years ago, Adelaida constructed a house on her lot at
Quezon City adjoining a lot owned by Bernie. She provided it with several windows
overlooking Bernie's lot half a meter away from the boundary line. A month ago,
Bernie brought an action against Adelaida for the closure of the windows alleging
that they violate the law on distances.
1. Has Adelaida acquired an easement of light and view by prescription?
2. Will the action of Bernie prosper?
3. If the action will not prosper, will that not be tantamount to saying that
Adelaida has already acquired an easement of light and view?
Answer:
(a) Continuous and apparent easements are acquired either by virtue of a
title or by prescription of ten years (Art. 620, CC), while continuous nonapparent
easements and discontinuous easements whether apparent or nonapparent, can
only be acquired by virtue of a title (Art. 622, CC).
(b) In order that an easement may be acquired by prescription, the time of
possession shall be computed thus: In positive easements, from the day on which
Page 184 of 391
the owner of the dominant estate, or. the person who may have made use of the
easement, commenced to exercise it upon the servient estate; and in negative
easements, from the day on which the owner of the dominant estate forbade, by an
instrument acknowledged before a notary public, the owner of the servient estate,
from executing an act which would be lawful without the easement. (Art. 621, CC.)
(c) (1) Adelaide has not acquired an easement of light and view by
prescription after ten years. There are two reasons for this, In the first place, there
was no formal prohibition as required by law. This should have been done by means
of an instrument acknowledged before a notary public wherein she should have
prohibited Bernie from obstructing his light and view. She did not. In the second
place, she did not observe the legal requirement that there should be a distance of
at least two meters between the windows and Bernie's lot, since the view is direct.
According to the Civil Code, non-observance of this distance does not give rise to
prescription.
(2) The action will not prosper because more than ten years has already
elapsed from the time of the opening of the windows. Bernie's right of action has
already prescribed.
(3) This is not tantamount to saying that Adelaida has already acquired an
easement of light and view. Under the Civil Code, nobody can prevent Bernie from
obstructing Adelaida's light and view by constructing a building on his lot or by
raising a wall thereon contiguous to the windows of Adelaida.
05; Property; easements; light and view
1985 No. 5
A is the owner of a four-story building which adjoins a three-story house
owned by B, A story of the two buildings has a height of 3 meters. In 1950, A
opened in the dividing wall of the edifices two windows each measuring one meter
square, the first in the fourth story of his building close to the ceiling, and the second
in the third story, the window directly overlooking a small open terrace in the third
floor of B's house In 1981, B demanded the closure of the two (2) windows.
As A's counsel, what legal advice will you extend to him and the reasons in
support thereof?
Answers:
1. We have to qualify as to which window, the first window or the second
window. With regard to the first window on the fourth floor, the action will prosper
because A has not acquired any legal easement of light and view. He did not make
any notarial demand and the window opens through a wall belonging exclusively to
A, not a party wall. On the other hand, with respect to the second window, the wall is
a party wall and it has been existing there for 30 years, therefore A has already
acquired a legal easement
2. As regards the easement of light and view, opening of windows which
violate the provisions of the Civil Code with respect to distances to the boundary line
do not give rise to prescription by express provision of law.
3. The dividing- wall appears to be owned by A exclusively. Accordingly, Art.
670 will apply. If it were not a party wall, Art, 669 would apply.
If it is a party wall, since the dividing wall is evidently the common party wall
of these two edifices, the easement of light and view has been acquired by
prescription after the lapse of 31 years from the opening through the party wall.
Page 185 of 391
4. As counsel for A, I would advise him to resist the demand of B on the
ground that A has an easement in regard to these openings, it appearing that A had
continuous use thereof that lasted for over 30 years.
5. The dividing wall appears to be owned by A. Art. 670 provides that
"No windows, apertures, balconies, or other similar projections which afford a direct
view upon or towards an adjoining land or tenement can be made, without leaving a
distance of two meters between the wall in which they are made and such
contiguous property. Neither can side or oblique views upon or towards such
conterminous property be had, unless there be a distance of sixty centimeters. The
non observance of these distances does not give rise to prescription".
Art. 669 states that "when the distances in Article 670 are not observed, the
owner of a wall which is not a party wall, adjoining a tenement or piece of land
belonging to another, can make in it openings to admit light at the height of the
ceiling joints or immediately under the ceiling, and of the size of thirty centimeters
square, and, in every case, with an iron grating imbedded in the wall and with a wire
screen."
6. Since the dividing wall is evidently the common wall {party wall) of the two
adjoining edifices, the easement of fight and view has been acquired by prescription
after the lapse of 31 years from the opening through the party wall (Art. 668, Civil
Code). Nevertheless, the owner of the tenement or property adjoining the wall in
which the openings are made can close them should he acquire part-ownership
thereof, if there is no stipulation to the contrary.
7. As A's legal counsel, I would advise him to resist B's demand as the action
to close the openings has already prescribed although the easement itself of light
and view has not been acquired by prescription.
05; Property; easements; prescriptive period
1982 No. 7
"A" built a house on his residential lot up to the boundary line. In the presence
of "B", the adjoining owner, "A" opened windows with a direct view over the lot of
"B". Twelve (12) years later, "B" built a house on his own lot also right up to the
boundary line "A" brought an action against "B" to enjoin the latter from building a
house up to the boundary line, alleging that "B" cannot build less than three (3)
meters from the boundary line (a) as he had acquired an easement of light and view
by prescription; and (b) the action of "B", if any, had already prescribed. Decide with
reasons.
Answer
(a) I shall decide against the contention of "A" that he has already acquired
an easement of light and view by prescription after ten years. There are two reasons
for this. In the first place, there was no formal prohibition as required by law. This
should have been done by means of an instrument acknowledged before a notary
public wherein he should have prohibited "B" from obstructing his light and view. He
did hot. In the second place, he did not observe by legal requirement that there
should be a distance of at least two meters between the windows, since the view is
direct, and "B's" lot. According to the Civil Code, non-observance of this distance
does not give rise to prescription.
(Note: The above answer is based on Arts 668 and 67Q of the Civil Code and
on Cortes vs. Yutivo, 2 Phil. 24; Fabie vs. Lichauco, 11 Phil. 14; Cid vs. Javier, IDS
Phil. 850).
(b) However, I agree with the contention of "A" that "B's" right of action to
compel "A" to close the windows has already prescribed. The period of prescription
Page 186 of 391
for such action is ten years to be counted from the time that the windows were
constructed. Nevertheless, this will not help "A" very much. Since he has not
acquired an easement of light and view, nobody can prevent "B" from obstructing
"A's" light and view either by constructing his own house up to the boundary line or
by constructing a wall at the boundary line contiguous to the windows of "A".
(Note: The above answer is based on Art, 670 of the Civil Code and on
Sternberg vs. Soriano, 41 Phil. 210.)
05; Property; easements; right of way
1993 No. 16:
Tomas Encarnacion's 3.000 square meter parcel of land, where he has a
plant nursery, is located just behind Aniceta Magsino's two hectare parcel land. To
enable Tomas to have access to the highway, Aniceta agreed to grant him a road
right of way a meter wide through which he could pass. Through the years Tomas'
business flourished which enabled him to buy another portion which enlarged the
area of his plant nursery. But he was still landlocked. He could not bring in and out
of his plant nursery a jeep or delivery panel much less a truck that he needed to
transport his seedlings. He now asked Aniceta to grant him a wider portion of her
property, the price of which he was willing to pay, to enable him to construct a road
to have access to his plant nursery. Aniceta refused claiming that she had already
allowed him a previous road right of way.
Is Tomas entitled to the easement he now demands from Aniceta?
Answer:
Art. 651 of the Civil Code provides that the width of the easement must be
sufficient to meet the needs of the dominant estate, and may accordingly change
from time to tune. It is the need of the dominant estate which determines the width of
the passage. These needs may vary from time to time. As Tomas' business grows,
the need for use of modern conveyances requires widening of the easement.
Alternative Answer:
The facts show that the need for a wider right of way arose from the
increased production owing to the acquisition by Tomas of an additional area. Under
Art. 626 of the Civil Code, the easement can be used only for the immovable
originally contemplated. Hence, the increase in width is Justified and should have
been granted.
05; Property; easements; right of way
2000 No VI
The coconut farm of Federico is surrounded by the lands of Romulo. Federico
seeks a right of way through a portion of the land of Romulo to bring his coconut
products to the market. He has chosen a point where he will pass through a housing
project of Romulo. The latter wants him to pass another way which is one kilometer
longer. Who should prevail? (5%)
SUGGESTED ANSWER:
Romulo will prevail. Under Article 650 of the New Civil Code, the easement of
right of way shall be established at the point least prejudicial to the servient estate
and where the distance from the dominant estate to a public highway is the shortest.
In case of conflict, the criterion of least prejudice prevails over the criterion of
shortest distance. Since the route chosen by Federico will prejudice the housing
project of Romulo, Romulo has the right to demand that Federico pass another way
even though it will be longer.
Page 187 of 391
05; Property; easements; right of way
1981 No. 6
"A", owner of an agricultural land, which had no connection with a public
road, has been passing through a pathway across the land of "B" with the latter's
tolerance for over twenty years.
"A" subdivided his property into 20 residential lots and sold them to different
persons. "B" blocked the pathway and refused to let the buyers pass.
a) Did "A" acquire an easement of right of way? Why?
b) Could "B" close the pathway and refuse to let the lot buyers pass?
Reason.
c) What are the rights of the lot buyers, if any? Explain.
Answer
(a) Answer No. 1 - "A" did not acquire an easement of right of way. According
to the Civil Code, continuous and apparent easements are acquired either by virtue
of a title or by prescription after ten years. Continuous non-apparent easements and
discontinuous easements, whether apparent or not, may be acquired only by virtue
of a title. It is obvious that an easement of right of way is discontinuous in character
because it is used only at intervals and its use requires the acts or intervention of
man. It is also obvious that use by tolerance is not equivalent to a title.
Consequently, since "A" never acquired any title from "B" and since the easement
cannot be acquired by prescription because of its discontinuous character, "A " did
not acquire any easement of right of way.
Answer No. 2 "A" did not acquire an easement of right of way. Obviously,
he does not possess any title to pass through the property of "B". So, the only
possible basis for the acquisition of the easement would be prescription after ten
years. In order that the easement can be acquired by prescription after ten years, it
is essential that it may be both continuous and apparent at the same time. An
easement of right of way is discontinuous. Therefore, it cannot be acquired by
prescription after ten years.
(Note: The above answers are based on Arts. 620 and S22 of the Civil Code
and on Cuayang vs. Benedicto vs. Benedicto, 37 Phil. 781, and Ronquillo vs. Roco,
103 Phil. 84. The Committee, however, respectfully recommends that if the bar
candidate answers the problem by invoking Tolentino's opinion that if the right of
way is permanent and has on apparent sign, there is no reason why it cannot be
acquired by prescription, it should be properly credited.)
(b) Yes. "B" could close the pathway and refuse to let the buyers pass. The
pathway belongs to him. Under the Civil Code, every owner may enclose or fence
his land or tenement by means of walls, ditches, live or dead hedges, or by other
means without detriment to servitudes constituted thereon.
(Note: The above answer is based on Art. 430 of the Civil Code. The
Committee again recommends that if the bar candidate comes out with an answer in
accordance with Tolentino's opinion (supra), it should be properly credited.)
(c) The lot buyers can now demand for the establishment of a legal or
compulsory easement of right of way. Since their lots are surrounded by other
immovables without adequate outlet to the public highway and this is not imputable
to their own acts, they now have a perfect right to do so. However, they must comply
with two other requisites. They must pay to "B" the proper indemnity. The right of
way must be at a point least prejudicial to the servient estate, and, insofar as
Page 188 of 391
consistent with the rule, where the distance to the public highway may be the
shortest.
(Note: The above answer is based on Arts. 649 and 650 of the Civil Code and
on Locsin vs. Climaco, 26 SCRA 816).
05; Property; easements; right of way; inseparability
2001 No V
Emma bought a parcel of land from Equitable-PCI Bank, which acquired the
same from Felisa, the original owner. Thereafter, Emma discovered that Felisa had
granted a right of way over the land in favor of the land of Georgina, which had no
outlet to a public highway, but the easement was not annotated when the servient
estate was registered under the Torrens system. Emma then filed a complaint for
cancellation of the right of way, on the ground that it had been extinguished by such
failure to annotate. How would you decide the controversy? (5%)
SUGGESTED ANSWER
The complaint for cancellation of easement of right of way must fall. The
failure to annotate the easement upon the title of the servient estate is not among
the grounds for extinguishing an easement under Art. 631 of the Civil Code. Under
Article 617, easements are inseparable from the estate to which they actively or
passively belong. Once it attaches, it can only be extinguished under Art. 631, and
they exist even if they are not stated or annotated as an encumbrance on the
Torrens title of the servient estate. (II Tolentino 326, 1987 ed.)
ALTERNATIVE ANSWER
Under Section 44, PD No. 1529, every registered owner receiving a certificate
of title pursuant to a decree of registration, and every subsequent innocent
purchaser for value, shall hold the same free from all encumbrances except those
noted on said certificate. This rule, however, admits of exceptions.
Under Act 496, as amended by Act No. 2011, and Section 4, Act 3621, an
easement if not registered shall remain and shall be held to pass with the land until
cutoff or extinguished by the registration of the servient estate. However, this
provision has been suppressed in Section 44, PD No. 1529. In other words, the
registration of the servient estate did not operate to cut-off or extinguish the right of
way. Therefore, the complaint for the cancellation of the right of way should be
dismissed.
05; Property; easements; right of way; requisites
1996 No. 8:
David is the owner of the subdivision in Sta. Rosa, Laguna, without an access
to the highway. When he applied for a license to establish the subdivision, David
represented that he will purchase a rice field located between his land and the
highway, and develop it into an access road. But. when the license was already
granted, he did not bother to buy the rice field, which remains unutilized until the
present. Instead, he chose to connect his subdivision with the neighboring
subdivision of Nestor, which has an access to the highway. Nestor allowed him to do
this, pending negotiations on the compensation to be paid. When they failed to
arrive at an agreement, Nestor built a wall across the road connecting with David's
subdivision. David filed a complaint In court, for the establishment of an easement of
right of way through the subdivision of Nestor which he claims to be the most
adequate and practical outlet to the highway.
1) What are the requisites for the establishment of a compulsory easement of
a right of way?
Page 189 of 391
Answer:
Art, 649, NCC. The owner, or any person who by virtue of a real right may
cultivate or .use any immovable which is surrounded by other immovables pertaining
to other persons and without adequate outlet to a public highway, is entitled to
demand a right of way through the neighboring estates, after payment of the
property indemnity.
Should this easement be established in such a manner that its use may be
continuous for all the needs of the dominant estate, establishing a permanent
passage, the indemnity shall consist of the value of the land occupied and the
amount of the damage caused to the servient estate.
In case the right of way is limited to the necessary passage for the cultivation
of the estate surrounded by others and for the gathering of its crops through the
servient estate without a permanent way, the indemnity shall consist In the payment
of the damage cause by such encumbrance.
This easement is not compulsory If the isolation of the Immovable is due to
the proprietor's own acts. (564a).
The easement of right of way shall be established at the point least prejudicial
to the servient estate, and insofar as consistent with this rule, where the distance
from the dominant estate to a public highway may be the shortest (Art. 650, NCC:
Vda. de Baltazar v. CA. 245 SCRA 333}
Alternative Answer:
The requisites for a compulsory easement of right of way are: (a) the
dominant estate is surrounded by other immovables and is without an adequate
outlet to a public street or highway; (b) proper indemnity must be paid; (c) the
isolation must not be due to the acts of the owner of the dominant estate; and (d) the
right of way claimed is at a point least prejudicial to the servient estate and, insofar
as is consistent with this rule, where the distance to the street or highway is shortest.
2) Is David entitled to a right of way in this case? Why or why not?
Answer;
No, David is not entitled to the right of way being claimed. The isolation of his
subdivision was due to his own act or omission because he did not develop into an
access road the rice field which he was supposed to purchase according to his own
representation when he applied for a license to establish the subdivision (Floro us.
Llenado, 244 SCRA713).
05; Property; good faith/ bad faith
1999 No IX.
(a) Because of confusion as to the boundaries of the adjoining lots that
they bought from the same subdivision company, X constructed a house on the
adjoining lot of Y in the honest belief that it is the land that he bought from the
subdivision company. What are the respective rights of X and Y with respect to X's
house? (3%)
(b) Suppose X was in good faith but Y knew that X was constructing on his
(Y's) land but simply kept quiet about it, thinking perhaps that he could get X's house
later. What are the respective rights of the parties over X's house in this case? (2%)
ANSWER:
(a) The rights of Y, as owner of the lot, and of X, as builder of a house
thereon, are governed by Art. 448 of the Civil Code which grants to Y the right to
choose between two remedies: (a) appropriate the house by indemnifying X for its
Page 190 of 391
value plus whatever necessary expenses the latter may have incurred for the
preservation of the land, or (b) compel X to buy the land if the price of the land is not
considerably more than the value of the house. If it is, then X cannot be obliged to
buy the land but he shall pay reasonable rent, and in case of disagreement, the
court shall fix the terms of the lease.
(b) Since the lot owner Y is deemed to be in bad faith (Art 453), X as the
party in good faith may (a) remove the house and demand indemnification for
damages suffered by him, or (b) demand payment of the value of the house plus
reparation for damages (Art 447, in relation to Art 454). Y continues as owner of the
lot and becomes, under the second option, owner of the house as well, after he pays
the sums demanded.
05; Property; good faith/ bad faith
2000 No IX
a) Demetrio knew that a piece of land bordering the beach belonged to
Ernesto. However, since the latter was studying in Europe and no one was taking
care of the land, Demetrio occupied the same and constructed thereon nipa sheds
with tables and benches which he rented out to people who want to have a picnic by
the beach. When Ernesto returned, he demanded the return of the land. Demetrio
agreed to do so after he has removed the nipa sheds. Ernesto refused to let
Demetrio remove the nipa sheds on the ground that these already belonged to him
by right of accession. Who is correct? (3%)
SUGGESTED ANSWER;
Ernesto is correct, Demetrio is a builder in bad faith because he knew
beforehand that the land belonged to Ernesto, Under Article 449 of the New Civil
Code, one who builds on the land of another loses what is built without right to
indemnity. Ernesto becomes the owner of the nipa sheds by right of accession.
Hence, Ernesto is well within his right in refusing to allow the removal of the nipa
sheds.
05; Property; good faith/ bad faith
2000 No IX
b) In good faith, Pedro constructed a five-door commercial building on the
land of Pablo who was also in good faith. When Pablo discovered the construction,
he opted to appropriate the building by paying Pedro the cost thereof. However,
Pedro insists that he should be paid the current market value of the building, which
was much higher because of inflation.
1) Who is correct Pedro or Pablo?(1%)
2) In the meantime that Pedro is not yet paid, who is entitled to the rentals of
the building, Pedro or Pablo? (1%)
SUGGESTED ANSWER:
Pablo is correct. Under Article 448 of the New Civil Code in relation to Article
546, the builder in good faith is entitled to a refund of the necessary and useful
expenses incurred by him, or the increase in value which the land may have
acquired by reason of the improvement, at the option of the landowner. The builder
is entitled to a refund of the expenses he incurred, and not to the market value of the
Improvement
The case of Pecson v. CA, 244 SCRA 407, is not applicable to the problem,
In the Pecson case, the builder was the owner of the land who later lost the property
at a public sale due to non-payment of taxes. The Court ruled that Article 448 does
not apply to the case where the owner of the land is the builder but who later lost the
Page 191 of 391
land; not being applicable, the indemnity that should be paid to the buyer must be
the fair market value of the building and not just the cost of construction thereof. The
Court opined in that case that to do otherwise would unjustly enrich the new owner
of the land.
ALTERNATIVE ANSWER:
Pedro is correct. In Pecson a CA, it was held that Article 546 of the New Civil
Code does not specifically state how the value of useful Improvements should be
determined in fixing the amount of Indemnity that the owner of the land should pay
to the builder in good faith. Since the objective of the law is to adjust the rights of the
parties in such manner as "to administer complete justice to both of them in such a
way as neither one nor the other may enrich himself of that which does not belong to
him", the Court ruled that the basis of reimbursement should be the fair market value
of the building.
SUGGESTED ANSWER:
2) Pablo is entitled to the rentals of the building. As the owner of the land,
Pablo is also the owner of the building being an accession thereto. However, Pedro
who is entitled to retain the building is also entitled to retain the rentals. He,
however, shall apply the rentals to the indemnity payable to him after deducting
reasonable cost of repair and maintenance.
ALTERNATIVE ANSWER:
Pablo is entitled to the rentals. Pedro became a possessor in bad faith from
the time he learned that the land belongs to Pablo. As such, he loses his right to the
building, including the fruits thereof, except the right of retention.
05; Property; good faith/ bad faith
1992 No 4:
A owns a parcel of residential land worth P50O,OOO.OO Unknown to A, a
residential house costing P 100,000.00 is built on the entire parcel by B who claims
ownership of the land. Answer all the following questions based on the premise that
B is a builder in good faith and A is a landowner in good faith.
a) May A acquire the house built by B? If so, how?
b) If the land increased in value to P5OO,OOO.OO by reason of the building
of the house thereon, what amount should be paid by A in order to acquire the
house from B?
c) Assuming that the cost of the house was P900,000.00 and not
P100,000.00, may A require B to buy the land?
d) If B voluntarily buys the land as desired by A, under what circumstances
may A nevertheless be entitled to have the house removed?
e) In what situation may a "forced lease" arise between A and B. and what
terms and conditions would govern the lease?
Give reasons for your answers.
Answer;
(a) Yes, A may acquire the house build by B by paying Indemnity to B. Article
448 of the Civil Code provides that the owner of the land on which anything has
been built, sown or planted in good faith, shall have the right to appropriate as his
own the works, sowing or planting, after payment of the indemnity provided for in
Articles 546 and 546 of the Civil Code.
Page 192 of 391
(b) A should pay B the sum of P50,000. Article 548 of the Civil Code
provides that useful expenses shall be refunded to the possessor In good faith with
the right of retention, the person who has defeated him in the possession having the
option of refunding the amount of the expenses or of paying the Increase in value
which the thing may have acquired by reason thereof. The increase in value
amounts to P50,000.00.
(c) Yes, A may require B to-buy the land. Article 448 of the Civil Code
provides that the owner of the land on which anything has been built in good faith
shall have the right to oblige the one who built to pay the price of the land if its value
is not considerably more than that of the building,
(d) If B agrees to buy land but fails to pay, A can have the house removed (
Depra vs. Dumlao, 136 SCRA 475).
(e) Article 448 of the Civil Code provides that the builder cannot be obliged to
buy the land if its value is considerably more than that of the building. In such case,
he shall pay reasonable rent, if the owner of the land does not choose to appropriate
the building after proper indemnity, The parties shall agree upon the terms of the
lease and in case of disagreement, the court fix the terms thereof.
05; Property; good faith/ bad faith
1984 No. 5
Believing that a piece of land belonged to him, A erected thereon a building,
using materials belonging to C. B, the owner of the land, was aware of the
construction being made by A, but did not do anything to stop it.
What are the rights of A, B and C with respect to the building and as against
each other?
Answer:
A. Furnished by Office of Justice Palma.
B, regardless of his good or bad faith, becomes the owner of the building (Art,
445 and 448). However, A, a builder in good faith, will be entitled to reimbursement
of his necessary and useful expenses, with a right to retain the same until paid. He
may also remove the construction, since B, the landowner, acted in bad faith in not
stopping the construction (Art. 454, 447). C, the owner of the materials shall have
the right to reimbursement C may also remove them but only if he can do so without
injury to the work (Art. 447).
B. Comments and Suggested Answer
We suggest that the following should be accepted as a correct answer:
A can choose between (1) reimbursement by B of the value of the building
plug damages, or (2) removal of the materials, with or without injury to the building,
plus damages. It must be observed that B is in bad faith. According to the law (Art.
454, Civil Code), when the landowner acted in bad faith and the builder acted in
good faith, the provision of Art. 447 of the Civil Code shall apply.
The rights of B, owner of the land, will depend upon the option selected by A.
If A decides to demand reimbursement of the building plus damages, of course, B
becomes the owner of the building. If A decides to remove the materials regardless
of whether or not there is injury to the .building, plus damages, B does not become
the owner of the building.
In the case of C, owner of the materials, assuming that he was in good faith,
obviously, he can proceed against A for the value of his materials and against E for
damages. If A cannot pay him the valve of his materials, he (C) can then proceed
Page 193 of 391
against B for the value of said materials (Art. 455, Civil Code). In other words, B will
then be liable not only for the value of C's materials but also for damages,
05; Property; good faith/ bad faith
1977 No. V-c
Discuss briefly the effects of possession in good faith and possession in bad
faith with respect to the fruits, the charges, the expenses, and deterioration or loss.
Answer
A. Fruits received:
1. Possessor in good faith is entitled to fruits received while his possession is
still in good faith (Art. 644, Civil Code).
2. Possessor in bad faith shall reimburse fruits received or which legitimate
possessor could have received, subject to Art. 443 (Art. 549, Civil Code).
B. Pending fruits:
1. Possessor in good faith and legitimate possessor shall be liable for
expenses of cultivation and shall share in net harvest in proportion to the time of
their possession (Art. 545, par. l, Civil Code).
2. Possessor in bad faith shall not have any right,
C. Charges:
Possessor, whether in good or bad faith, and legitimate possessor shall share
charges in proportion to the time of their possession (Art, 545, par. 1, Civil Code).
D. Expenses:
1. Necessary expenses
a. Rights of possessor in good faith:
(1) Right of reimbursement (Art. 546, par. 1, Civil Code).
(2) Right of retention (Art, 546, par. 1, Civil Code).
b. Rights of possessor in bad faith: Right of reimbursement only (Art. 546,
par. 1, Civil Code).
2. Useful expenses
a. Rights of possessor in good faith:
(1) Right of reimbursement.
(2) Right of retention (Art. 546, par. 2, Civil Code).
(3) Limited right of removal (Art 547, Civil Code).
b. Rights of possessor in bad faith: None.
3. Ornamental expenses
a. Rights of possessor in good faith: Limited right of removal (548, Civil
Code),
b. Rights of possessor in bad faith: Limited right of removal (549, Civil Code).
E. Deterioration or loss:
1. Possessor in good faith No liability, unless due to his fault or
negligence after he had become possessor in bad faith (Art. 552, Civil Code).
2. Possessor in bad faith Always liable, whether due to his fault or
negligence or due to a fortuitous event (Art. 552, Civil Code).
Page 194 of 391
(NOTE: One way of making the above answer a short one would be to look at
the question only from the point of view of the defendant possessor, and then,
determine whether he is entitled to the fruits or not, whether he can demand
reimbursement or not, and whether he may be held liable or not.)
05; Property; good faith/ bad faith; expenses
1983 No. 4
A, a squatter who is sought to be evicted by the landowner, B, seeks
reimbursement from the latter for the improvements he made on the property, while
B demands the value of all the fruits A gathered from the land during his occupancy
thereof.
Is A entitled to the indemnity he prays for? Is he bound to pay for the fruits he
received? Why?
Answer
(Examiner's Answer)
As a possessor in bad faith, A may recover only the necessary expenses he
may have incurred while in possession and reimbursement for useful improvements
introduced by him if the owner chooses to retain them and he must pay him the
value of all the fruits he received.
(Committee's Answer)
A is entitled to reimbursement for ail expenses incurred by him for necessary
improvements. Under the law, this is the only right to which a possessor in bad faith
(builder in bad faith) is entitled as against the legitimate owner or possessor.
(Note: The above answer is based on Arts. 546, par. 1 and 452, Civil Code.
We submit most respectfully that A is not entitled to reimbursement for useful
expenses. If B chooses to retain the useful improvements, Arts. 449 and 451, Civil
Code, are directly applicable. The second paragraph of Art. 546, Civil Code is
applicable only to a possessor in good faith.)
Yes, A is bound to pay for the fruits received and those which B could have
received minus necessary expenses and expenses for the production, gathering and
preservation of the fruits. This is expressly ordained by the law.
(Note: The above answer is based on Arts. 549, 546, par. 1 and 442, Civil
Code)
05; Property; good faith/ bad faith; presumption of good faith : builder in good
faith
2001 No III
Mike built a house on his lot in Pasay City. Two years later, a survey
disclosed that a portion of the building actually stood on the neighboring land of
Jose, to the extent of 40 square meters. Jose claims that Mike is a builder in bad
faith because he should know the boundaries of his lot, and demands that the
portion of the house which encroached on his land should be destroyed or removed.
Mike replies that he is a builder in good faith and offers to buy the land occupied by
the building instead.
1) Is Mike a builder in good faith or bad faith? Why? (3%)
2) Whose preference should be followed? Why? (2%)
SUGGESTED ANSWER
1) Yes, Mike is a builder in good faith. There is no showing that when he built
his house, he knew that a portion thereof encroached on Jose's lot. Unless one is
Page 195 of 391
versed in the science of surveying, he cannot determine the precise boundaries or
location of his property by merely examining his title. In the absence of contrary
proof, the law presumes that the encroachment was done in good faith [Technogas
Phils, v. CA, 268 SCRA 5, 15 (1997)].
2} None of the preferences shall be followed. The preference of Mike cannot
prevail because under Article 448 of the Civil Code, it is the owner of the land who
has the option or choice, not the builder. On the other hand, the option belongs to
Jose, he cannot demand that the portion of the house encroaching on his land be
destroyed or removed because this is not one of the options given by law to the
owner of the land. The owner may choose between the appropriation of what was
built after payment of indemnity, or to compel the builder to pay for the land if the
value of the land is not considerably more than that of the building. Otherwise, the
builder shall pay rent for the portion of the land encroached.
ALTERNATIVE ANSWER
1) Mike cannot be considered a builder in good faith because he built his
house without first determining the corners and boundaries of his lot to make sure
that his construction was within the perimeter of his property. He could have done
this with the help of a geodetic engineer as an ordinary prudent and reasonable man
would do under the circumstances.
2) Jose's preference should be followed. He may have the building
removed at the expense of Mike, appropriate the building as his own, oblige Mike to
buy the land and ask for damages in addition to any of the three options. (Articles
449, 450, 451, CC)
05; Property; good faith/ bad faith; rights of a possessor (gf/bf) vs
usufructuary
1996 No. 7:
Bartolome constructed a chapel on the land of Eric. What are Bartolome's
rights if he were:
1) a possessor of the land in good faith?
2) a possessor of the land in bad faith?
4) a lessee of the land?
3) a usufructuary of the land?
Answer:
1) possessor in good faith ---
A chapel is a useful improvement. Bartolome may remove the chapel if it can
be removed without damage to the land, unless Eric chooses to acquire the chapel.
In the latter case, Bartolome has the right to the reimbursement of the value of the
chapel with right of retention until he is reimbursed. [Art. 448 in relation to Art. 546 &
547. NCC).
Alternative Answer;
Assuming that Eric acted in good faith, Bartolome's rights will depend upon
what option Eric chooses. Eric, the owner of the land, may choose to acquire the
chapel, which Is a useful expense or to sell the land to the builder (Bartolome),
If Eric chooses to acquire the chapel, he has the right to reimbursement for
useful expenses, with a right of retention until paid.
Page 196 of 391
If Eric chooses to sell the land to Bartolome. Bartolome may refuse to buy the
land if the value of the land is considerably more than the value of the building, in
which case, there will be a forced leased between them,
Additional Answer:
If Eric acted in bad faith, then Bartolome has the right of absolute removal of
the chapel, plus damages. However, if Eric chooses to acquire the chapel, then
Bartolome has the right to reimbursement, plus payment of damages, with right of
retention (Art. 454 in relation of Art. 447. NCC)
2) Possessor in bad faith ---
Bartolome, under Art. 449 of the NCC, loses whatever he built, without any
right to indemnity.
Alternative Answer:
It is the owner of the land who has the right to acquire the chapel without
paying indemnity, plus damages, or to require Bartolome to remove the chapel, plus
damages or to require Bartolome to buy the land, without any option to refuse to buy
it. (Arts. 449 and 458, NCC)
If Eric acted in bad faith, then his bad faith cancels the bad faith of Bartolome,
and both will be taken to have acted in good faith. (Art. 453, NCC)
3) Lessee ---
Bartolome has the right to remove the improvement if it is possible to do so
without causing damage to the property (Art. 579, NCC). He may also set off the
improvement against any damages which the property held in usufruct suffered
because of his act or the acts of his assignee. (Art. 580, NCC).
4) Usufructuary ---
The owner of the land, as lessor, can acquire the improvement by paying for
one-half of its value. Should the lessor refuse to reimburse said amount, the lessee
may remove the Improvement, even though the principal thing may suffer damage
thereby (Art. 1678. NCC).
05; Property; good faith/ bad faith; sower in good faith
2000 No X
b) Felix cultivated a parcel of land and planted it to sugar cane, believing it to
be his own. When the crop was eight months old, and harvestable after two more
months, a resurvey of the land showed that it really belonged to Fred. What are the
options available to Fred? (2%)
SUGGESTED ANSWER:
As to the pending crops planted by Felix in good faith, Fred has the option of
allowing Felix to continue the cultivation and to harvest the crops, or to continue the
cultivation and harvest the crops himself. In the latter option, however, Felix shall
have the right to a part of the expenses of cultivation and to a part of the net harvest,
both in proportion to the time of possession. (Art. 545 NCC),
ALTERNATIVE ANSWER:
Since sugarcane Is not a perennial crop. Felix is considered a sower in good
faith. Being so, Art. 448 applies. The options available to Fred are: (a) to appropriate
the crop after paying Felix the indemnity under Art. 546, or (b) to require Felix to pay
rent.
05; Property; hidden treasure
Page 197 of 391
1995 No. 9:
Tim came into possession of an old map showing where a purported cache of
gold bullion was hidden. Without any authority from the government Tim conducted
a relentless search and finally found the treasure buried in a new river bed formerly
part of a parcel of land owned by spouses Tirso and Tessie. The old river which
used to cut through the land of spouses Ursula and Urbito changed its course
through natural causes.
1. To whom shall the treasure belong? Explain.
Answer:
1. The treasure was found in a property of public dominion, the new river
bed. Since Tim did not have authority from the government and, therefore, was a
trespasser, he is not entitled to the one-half share allotted to a finder of hidden
treasure. All of it will go to the State. In addition, under Art. 438 of the NCC. in order
that the finder be entitled to the 1/2 share, the treasure must be found by chance,
that is by sheer luck. In this case, since Tim found the treasure not by chance but
because he relentlessly searched for it, he is not entitled to any share in the hidden
treasure.
Alternative Answer:
The law grants a one-half share to a finder of hidden treasure provided he is
not a trespasser and the finding is by chance. It is submitted that Tim is not a
trespasser despite his not getting authority from the government, because the new
river bed where he found the treasure is property for public use (Art. 420 NCC), to
which the public has legitimate access. The question, therefore, boils down to
whether or not the finding was by chance in view of the fact that Tim "conducted a
relentless search" before finding the treasure. The strict or literal view holds that
deliberate or Intentional search precludes entitlement to the one-half share allotted
by law to the finder since the phrase "by chance" means "by accident", meaning an
unexpected discovery. The liberal view, however, would sustain Tim's right to the
allocated share Interpreting the phrase in question as meaning "by a stroke of good
fortune", which does not rule out deliberate or intentional search. It is submitted that
the liberal view should prevail since In practical reality, hidden treasure is hardly
ever found without conscious effort to find it, and the strict view would tend to render
the codal provision In question Illusory.
05; Property; hidden treasure
1976 No. X-b
Under a tip that hidden treasure is burned in the land of A, B leases the
property and conducts excavation thereon. If in the land valuable gold coins are
found, is B entitled to the same? Explain.
Answer
No, it was not found "by chance". (Article 438) B deliberately searched for it.
Moreover, treasure is defined as hidden and unknown deposit of precious objects,
the lawful ownership of which does not appear. There being a "tip", the deposit is
"known". (Article 439)
Alternative Answer
As a "tip" is not certain, the finding could still be considered as by chance or a
stroke of fortune. Hence, B is entitled to 1/2.
05; Property; hidden treasures
1997 No, 7:
Page 198 of 391
Marcelino, a treasure hunter as just a hobby, has found a map which appears
to indicate the location of hidden treasure. He has an idea of the land where the
treasure might possibly be found. Upon Inquiry, Marcelino learns that the owner of
the land, Leopoldo, is a permanent resident of Canada, Nobody, however, could
give him Leopoldo's exact address. Ultimately, anyway, he enters the land and
conducts a search. He succeeds.
Leopoldo. learning of Marcelino's "find", seeks to recover the treasure from
Marcelino but the latter Is not willing to part with it. Falling to reach an agreement,
Leopoldo sues Marcelino for the recovery of the property. Marcelino contests the
action.
How would you decide the case? Answer;
I would decide in favor of Marcelino since he is considered a finder by chance
of the hidden treasure, hence, he is entitled to one-half (1/2) of the hidden treasure.
While Marcelino may have had the intention to look for the hidden treasure, still he is
a finder by chance since it is enough that he tried to look for it. By chance in the law
does not mean sheer luck such that the finder should have no intention at all to look
for the treasure. By chance means good luck, implying that one who intentionally
looks for the treasure is embraced in the provision. The reason is that it is extremely
difficult to find hidden treasure without looking for it deliberately,
Marcelino is not a trespasser since there is no prohibition for him to enter the
premises, hence, he is entitled to half of the treasure.
Alternative Answers:
1. Marcelino did not find the treasure by chance because he had a map, he
knew the location of the hidden treasure and he intentionally looked for the treasure,
hence, he is not entitled to any part of the treasure.
2. Marcelino appears to be a trespasser and although there may be a
question of whether he found it by chance or not, as he has found the hidden
treasure by means of a treasure map, he will not be entitled to a finder's share. The
hidden treasure shall belong to the owner.
3. The main rule is that hidden treasure belongs to the owner of the land,
building or other property on which It is found. If it is found by chance by a third
person and he is not a trespasser, he is entitled to one-half (1/2). If he is a
trespasser, he loses everything.
05; Property; immovable properties; chattel mortgage over immovables
2003 No XIX.
X constructed a house on a lot which he was leasing from Y. Later, X
executed a chattel mortgage over said house in favor of Z as security for a loan
obtained from the latter. Still later, X acquired ownership of the land where his house
was constructed, after which he mortgaged both house and land in favor of a bank,
which mortgage was annotated on the Torrens Certificate of Title. When X failed to
pay his loan to the bank, the latter, being the highest bidder at the foreclosure sale,
foreclosed the mortgage and acquired Xs house and lot. Learning of the
proceedings conducted by the bank, Z is now demanding that the bank reconvey to
him Xs house or pay Xs loan to him plus interests. Is Zs demand against the bank
valid and sustainable? Why? 5%
SUGGESTED ANSWER:
No, Zs demand is not valid. A building is immovable or real property whether
it is erected by the owner of the land, by a usufructuary, or by a lessee. It may be
treated as a movable by the parties to chattel mortgage but such is binding only
Page 199 of 391
between them and not on third parties (Evangelista v. Alto Surety Col, inc. 103 Phil.
401 [1958]). In this case, since the bank is not a party to the chattel mortgage, it is
not bound by it. as far as the Bank is concerned, the chattel mortgage, does not
exist. Moreover, the chattel mortgage does not exist. Moreover, the chattel mortgage
is void because it was not registered. Assuming that it is valid, it does not bind the
Bank because it was not annotated on the title of the land mortgaged to the bank. Z
cannot demand that the Bank pay him the loan Z extended to X, because the Bank
was not privy to such loan transaction.
ANOTHER SUGGESTED ANSWER:
No, Zs demand against the bank is not valid. His demand that the bank
reconvey to him Xs house presupposes that he has a real right over the house. All
that Z has is a personal right against X for damages for breach of the contract of
loan.
The treatment of a house, even if built on rented land, as movable property is
void insofar as third persons, such as the bank, are concerned. On the other hand,
the Bank already had a real right over the house and lot when the mortgage was
annotated at the back of the Torrens title. The bank later became the owner in the
foreclosure sale.
Z cannot ask the bank to pay for Xs loan plus interest. There is no privity of
contract between Z and the bank.
ALTERNATIVE ANSWER:
The answer hinges on whether or not the bank is an innocent mortgagee in
good faith or a mortgagee in bad faith. In the former case, Zs demand is not valid. In
the latter case, Zls demand against the bank is valid and sustainable.
Under the Torrens system of land registration, every person dealing with
registered land may rely on the correctness of the certificate of title and the law will
not in any way oblige to him to look behind or beyond the certificate in order to
determine the condition of the title. He is not bound by anything not annotated or
reflected in the certificate. If he proceeds to buy the land or accept it as a collateral
relying on the certificate, he is considered a buyer or a mortgagee in good faith. On
this ground, the Bank acquires a clean title to the land and the house.
However, a bank is not an ordinary mortgagee. Unlike private individuals, a
bank is expected to exercise greater care and prudence in its dealings. The
ascertainment of the condition of a property offered as collateral for a loan must be a
standard and indispensable part of its operation. The bank should have conducted
further inquiry regarding the house standing on the land considering that it was
already standing there before X acquired the title to the land. The bank cannot be
considered as a mortgagee in good faith. On this ground, Zs demand against the
Bank is valid and sustainable.
05; Property; nuisance
2002 No VI.
Lauro owns an agricultural land planted mostly with fruit trees. Hernando
owns an adjacent land devoted to his piggery business, which is two (2) meters
higher in elevation. Although Hernando has constructed a waste disposal lagoon for
his piggery, it is inadequate to contain the waste water containing pig manure, and it
often overflows and inundates Lauros plantation. This has increased the acidity of
the soil in the plantation, causing the trees to wither and die. Lauro sues for
damages caused to his plantation. Hernando invokes his right to the benefit of a
natural easement in favor of his higher estate, which imposes upon the lower estate
Page 200 of 391
of Lauro the obligation to receive the waters descending from the higher estate. Is
Hernando correct? (5%)
SUGGESTED ANSWER:
Hernando is wrong. It is true that Lauros land is burdened with the natural
easement to accept or receive the water which naturally and without interruption of
man descends from a higher estate to a lower estate. However, Hernando has
constructed a waste disposal lagoon for his piggery and it is this waste water that
flows downward to Lauros land. Hernando has, thus, interrupted the flow of water
and has created and is maintaining a nuisance. Under Act. 697 NCC, abatement of
a nuisance does not preclude recovery of damages by Lauro even for the past
existence of a nuisance. The claim for damages may also be premised in Art. 2191
(4) NCC.
ANOTHER ANSWER:
Hernando is not correct. Article 637 of the New Civil Code provides that the
owner of the higher estate cannot make works which will increase the burden on the
servient estate. (Remman Enterprises, Inc. v. CA, 330 SCRA 145 [2000]). The
owner of the higher estate may be compelled to pay damages to the owner of the
lower estate.
05; Property; nuisance
1977 No. V-b
P built a house on a lot close to his place of work, an electric plant. Years
later, after his retirement, the plant bought additional equipment; P brought suit for
abatement of nuisance and damages, claiming that the new units were so powerful
that the noise and the vibrations they caused bothered the whole neighborhood. Is
the defendant liable?
Answer
The defendant electric plant is liable for damages. In a similar case, Velasco
vs; Manila. Electric Co., 40 SCRA 342, the Supreme Court held that the general rule
is that everyone is bound to bear the habitual or customary inconveniences that
result from the proximity of others, and so long as this level is not surpassed, he
may not complain against them. But if the prejudice exceeds the inconveniences
that such proximity habitually brings, the neighbor who causes such disturbance is
held responsible for the resulting damages, being guilty of causing nuisance. The
noise continuously emitted by a Meralco sub-station day and night, constitutes an
actionable nuisance for which the plaintiff is entitled to damages and other relief.
(NOTE: An opposite answer may be sustained by first recognizing the fact
that sound may now constitute an actionable nuisance, but plaintiff assumed all of
the risks by constructing his house close to the plant.)
05; Property; nuisance
1980 No. III
(b) "J" and his family lived in his house in Metro Manila. Adjoining "J's"
house was a vacant lot MERALCO purchased this lot and built thereon a sub-
station, which reduced high voltage electricity to a current suitable for distribution to
its customers. The sub-station transformers made a lot of noise during all hours of
the day and night and made life miserable for "J" and his family. "J" filed an action
against MERALCO to stop the operation of the sub-station and for damages. Decide
the case.
Answer
Page 201 of 391
(b) Meralco must take appropriate measures to reduce to normal sound
levels. At the same time, it is also liable to "J" and to the members of "J's" family for
damages plus attorney's fees. That sound may constitute an actionable nuisance is
not settled (Velasco vs. Manila Electric Co., 40 SCRA 342). However, it must be a
noise which affects injuriously the health and comfort of ordinary people to an
unreasonable extent. The test, therefore, is whether the health and comfort of "J"
and the members of his family are so injuriously affected by the noise in question so
that they are subjected to a loss which goes beyond reasonable limits. Applying this
test, it is clear that Meralco has not taken the necessary precaution to reduce the
intensity level of the noise to normal or reasonable level. Consequently, the noise
constitutes an actionable nuisance.
05; Property; possession vs occupation
1997 No. 9:
(a) Distinguish between "possession" and "occupation" as these terms are
commonly used in Book II and Book III of the Civil Code.
Answer:
(a) Possession is a real right, while occupation is one of the original modes
of acquiring ownership and other real rights. Possession, the holding of a thing or
the exercise of a right, does not in itself constitute ownership. Whereas, occupation
is a mode of acquiring ownership. There can be possession without ownership.
Additional Answer;
Possession is the holding of a thing or the enjoyment of a right (Art 532, CC).
It can refer to all kinds of property whether with or without an owner while occupation
can take place only with respect to property without an owner (Articles 531 & 713).
Occupation in itself, when proper, confers ownership but possession does not by
itself give rise to ownership.
05; Property; possession; 559
1987 No. 9
Rita owned a valuable painting which was stolen from her house. The theft
was duly reported to the authorities. A year after, Rita saw the painting hanging in
the office of Mario. When queried, Mario said that he bought the painting in a gallery
auction. The painting was positively identified as the one stolen from the house of
Rita.
(a) Could Rita recover the painting? If so, would Mario be entitled to
reimbursement of the amount he paid for the painting? Explain.
(b) Supposing Mario bought the painting from a friend, would your answer be
the same? Explain.
Answer:
a. Yes, Rita could recover the painting, but Mario is not entitled to
reimbursement because a gallery auction is a public sale (Article 559).
b. Yes. insofar as recovery of the painting is concerned. Rita can recover it.
No, as regards reimbursement, because, the painting was not bought at a public
sale as provided under Article 559 of the Civil Code. Mario is entitled to
reimbursement.
05; Property; possession; 559
1978 No. III-b
Page 202 of 391
A bought a white gold ring with a two-karat emerald from La Estrella del Norte
in 1964. In 1966, said ring was stolen from her house. She traced the ring to B
whom she saw was wearing the same at a party in 1967. B, on the other hand,
claimed that she bought the ring from her aunt. It was established, however, that it
was the very same ring. Can A recover the ring from B, without reimbursing the price
paid by B? Reasons for your answer.
Answer
Yes, A can recover the ring from B, without reimbursing the price paid by B to
his aunt. According to the law, one who has lost any movable or has been deprived
thereof, may recover it from any possessor without any obligation at all. The only
exception is when there is acquisition in good faith by the possessor at a public sale,
in which case the owner cannot obtain its return without reimbursing the price paid
therefore.
The right of the owner to recover personal property acquired in good faith by
another is based in his being dispossessed without his consent. The common law
principle that where one of two innocent persons must suffer by a fraud perpetrated
by another, the law imposes the loss upon the party who by misplaced confidence,
has enabled the fraud to be committed, cannot be applied in a case which is
covered by an express provision of the Civil Code. Between a common law principle
and a statutory provision, the latter must prevail.
(NOTE: The above answer is based on De Garcia vs. Court of Appeals, 37
SCR A 129.)
05; Property; possession; 559
1998 No IV
Using a falsified manager's check, Justine, as the buyer, was able to take
delivery of a second hand car which she had just bought from United Car Sales. Inc.
The sale was registered with the Land Transportation Office. A week later, the seller
learned that the check had been dishonored, but by that time, Justine was nowhere
to be seen. It turned out that Justine had sold the car to Jerico, the present
possessor who knew nothing about the falsified check. In a suit by United Car Sales,
Inc. against Jerico for recovery of the car, plaintiff alleges it had been unlawfully
deprived of its property through fraud and should, consequently, be allowed to
recover it without having to reimburse the defendant for the price the latter had paid.
Should the suit prosper? [5%]
Answer:
The suit should prosper as to the recovery of the car. However, since Jerico
was not guilty of any fraud and appears to be an innocent purchaser for value, he
should be reimbursed for the price he paid. This is without prejudice to United Car
Sales, Inc. right of action against Justine. As between two Innocent parties, the party
causing the Injury should suffer the loss. Therefore, United Car Sales, Inc. should
suffer the loss.
Alternative Answer:
Yes, the suit will prosper because the criminal act of estafa should be
deemed to come within the meaning of unlawful deprivation under Art. 559, Civil
Code, as without It plaintiff would not have parted with the possession of its car.
Another Answer:
No, the suit will not prosper. The sale is valid and Jerico is a buyer in good
faith.
Another Answer:
Page 203 of 391
Under the law on Sales, when the thing sold is delivered by the seller to the
buyer without reservation of ownership, the ownership is transferred to the buyer.
Therefore in the suit of United Car Sales, Inc. against Jerico for the recovery of the
car, the plaintiff should not be allowed to recover the car without reimbursing the
defendant for the price that the latter paid. (EDCA Publishing and Distributing Corp.
vs. Santos, 184 SCRA 614, April 26, 1990)
05; Property; possession; 559
1986 No. 5:
Mahinhin lost her diamond ring when the bus she was riding on was help up
by a band of brigands who divested the passengers of all their money and
valuables. The ring found its way to the Pasanglaan pawnshop, where one of the
robbers had pawned it. The pawnshop, in due time, foreclosed the pledge and sold
the ring at public auction to Maya-man, the highest bidder.
Three years after the loss, Mahinhin was able to trace the ring to Mayaman
and demanded that the latter give the ring back to her. Mayaman refused, saying
that he had acquired the ring in good faith.
Who was the better right to the ring? Explain, Answer:
Mahinhin, the owner, still has a better right to the ring. She had been
unlawfully deprived of the ring, therefore, not even an innocent purchaser in a
pawnshop can claim a better right. However, Mayaman has to be reimbursed.
Answer Art. 559 will apply as the owner was unlawfully deprived on her
right and may recover it even from an innocent purchaser. However, as the buyer
bought it in a public sale, Mahinhin is duty bound to reimburse the price paid before
she can recover the ring.
Answer - Mahinhin has the better right to the ring, but he must reimburse
Mayaman the price paid by the latter in acquiring said ring at the public auction sate.
True, Mayaman acquired the ring in good faith. His possession, therefore, is
equivalent to a title. But then, Mahinhin was unduly deprived of the ring. Under the
law, one who has lost any movable or who has been unduly deprived thereof can
recover the movable even from a possessor in good faith. The only exception the
law allows is when the possessor had acquired the movable in good faith at a public
sale. In such case, the owner cannot obtain its return without reimbursing the price
paid therefor. As held in so many notable decisions, even the common law principle
(that were one of two innocent persons must suffer by fraud perpetrated by another,
the law imposes the loss upon the party who, by misplaced confidence, has enabled
the fraud to be committed) cannot be applied in a case which is covered by an
express provision of the Civil Code. Between a common law principle and a
statutory provision, the latter must prevail.
(Note The above answer is based on Art, 559 of the Civil Code and on a
long line of decisions of the Supreme Court.)
05; Property; possession; 559
1993 No. 5;
A, about to leave the country on a foreign assignment, entrusted to B his
brand new car and its certificate of registration. Falsifying A's signature. B sold A's
car to C for P200,000.00. C then registered the car in his name. To complete the
needed amount, C borrowed P100.000.00 from the savings and loan association in
his office, constituting a chattel mortgage on the car. For failure of C to pay the
amount owed, the savings and loan association filed in the RTC a complaint for
collection with application for issuance of a writ of replevin to obtain possession of
Page 204 of 391
the vehicle so that the chattel mortgage could be foreclosed. The RTC issued the
writ of replevin. The car was then seized from C and sold by the sheriff at public
auction at which the savings and loan association was the lone bidder. Accordingly,
the car was sold to it. A few days later, A arrived from his foreign assignment.
Learning of what happened to his car, A sought to recover possession and
ownership of it from the savings and loan association.
Can A recover his car from the savings and loan association? Explain your
answer.
Answer:
Under the prevailing rulings of the Supreme Court, A can recover the car from
the Savings and Loan Association provided he pays the price at which the
Association bought the car at a public auction. Under that doctrine, there has been
an unlawful deprivation by B of A of his car and, therefore, A can recover it from any
person in possession thereof. But since it was bought at a public auction in good
faith by the Savings and Loan Association, he must reimburse the Association at the
price for which the car was bought.
Alternative Answer:
Yes, A can recover his car from the Savings and Loan Association. In a
Chattel Mortgage, the mortgagor must be the absolute owner of the thing
mortgaged. Furthermore, the person constituting the mortgage must have the free
disposal of the property, and in the absence thereof, must be legally authorized for
the purpose. In the case at bar, these essential requisites did not apply to the
mortgagor B, hence the Chattel Mortgage was not valid.
05; Property; possession; 559
1977 No. IX-a
A entrusted a diamond ring to S to sell. S pledged it instead in a pawnshop.
Can A recover the ring from the pawnshop? Can the pawnshop insist on payment
first of the loan from A?
Answer
Yes, A can recover the ring from the pawnshop. The Civil Code is explicit.
The legitimate owner who had lost the thing or who had been unduly deprived
thereof can recover it even from a possessor in good faith. (Art. 559.)
And the pawnshop owner cannot insist on reimbursement of the amount for
which the ring was pledged. In the first place, the contract of pledge is void because
the pledgor is not the owner of the thing pledged; neither has he been duly
authorized to pledge it. Consequently, an essential requisite is lacking (Art. 2085). In
the second place, because of enormous profits, pawnshop operators must
necessarily assume the corresponding risks. The things pledged might have been
robbed, stolen or embezzled. Hence, the pawnshop cannot insist, on reimbursement
of the amount for which the thing was pledged, (Dizon vs. Suntay, 47 SCRA 160.)
05; Property; possession; squatting
1980 No. III
(a) Several families had erected their houses on the river bank portion of a
lot owned by "X". The construction of the houses was with the knowledge and
consent of "X". The latter did not collect rentals. About five years later, "X" being in
need of the lot, demanded that the families vacate. The latter refused contending
that they were not squatters considering that "X" had allowed their occupation and
is, therefore, estopped from ejecting them.
Page 205 of 391
1) Are the families "squatters"? 2) Do they have a right to continue in the
occupation of the land?
Answer
(a) (1) Whether they are possessors by tolerance or squatters, will not
make any difference. Legally, they are squatters. A squatter is one who settles on
land of another without any legal authority. This definition contemplates a right that
owes its source from the law, and which accordingly may be protected by and under
the law. In the instant case, it is clear that the families settled down on the land
without any legal authority (Buonavente vs. Melchor, 89 SCR A 222),
(2) They do not have a right to continue in the occupation of the land. Their
occupation by mere tolerance, can not give rise to a right that the law should protect
in their favor as against the true legal owner. Besides, the owner "X" is certainly not
barred, under any known principle of law, either by estoppel or waiver, to demand
that the law should protect in their favor as against the true legal owner. (Ibid).
(Note: The Committee respectfully recommends that an answer based on the
provisions of Arts. 449, 450 and 451 of the Civil Code should also be considered as
correct. Objectively considered, the families in the above problem are builders or
possessors in bad faith. They cannot assert any right as against the legal owner.)
05; Property; real vs personal property
1995 No. 13:
Salvador, a timber concessionaire, built on his lot a warehouse where he
processes and stores his timber for shipment. Adjoining the warehouse is a furniture
factory owned by NARRAMIX of which Salvador is a majority stockholder.
NARRAMIX leased space In the warehouse where it placed its furniture-making
machinery.
1. How would you classify the furniture-making machinery as property under
the Civil Code? Explain.
2. Suppose the lease contract between Salvador and NARRAMIX stipulates
that at the end of the lease the machinery shall become the property of the lessor,
will your answer be the same? Explain.
Answer:
1. The furniture-making machinery is movable property because it was not
installed by the owner of the tenement. To become immovable under Art. 415 (5)
of the NCC, the machinery must be installed by the owner of the tenement.
Alternative Answer:
It depends on the circumstances of the case. If the machinery was attached
in a fixed manner, in such a way that it cannot be separated from the tenement
without breaking the material or causing deterioration thereof, It is Immovable
property [Art. 415 (3), NCC]. However, if the machinery can be transported from
place to place without Impairment of the tenement to which they were fixed, then it is
movable property. [Art. 416 (4), NCCJ
2. It is immovable property. When there is a provision in the lease contract
making the lessor, at the end of the lease, owner of the machinery installed by the
lessee, the said machinery is considered to have been installed by the lessor
through the lessee who acted merely as his agent. Having been installed by the
owner of the tenement, the machinery became immovable .under Art. 415 of the
NCC. (Davao Sawmill v. Castillo 61 Phil. 709)
05; Property; real vs personal property
Page 206 of 391
1997 No. 6:
Pedro is the registered owner of a parcel of land situated in Malolos. Bulacan.
In 1973, he mortgaged the land to the Philippine National Bank (PNB) to secure a
loan of PI00.000.00. For Pedro's failure to pay the loan, the PNB foreclosed on the
mortgage in 1980, and the land was sold at public auction to PNB for being the
highest bidder. PNB secured title thereto in 1987.
In the meanwhile, Pedro, who was still in possession of the land, constructed
a warehouse on the property. In 1988, the PNB sold the land to Pablo, The Deed of
Sale was amended in 1989 to include the warehouse.
Pedro, claiming ownership of the warehouse, files a complaint to annul the
amended Deed of Sale before the Regional Trial Court of Quezon City, where he
resides, against both the PNB and Pablo. The PNB filed a motion to dismiss the
complaint for improper venue contending that the warehouse is real property under
Article 415(1) of the Civil Code and therefore the action should have Instead been
filed in Malolos, Bulacan. Pedro claims otherwise. The question arose as to whether
the warehouse should be considered as real or as personal property.
If consulted, what would your legal advice be? Answer:
The warehouse which is a construction adhered to the soil is an immovable
by nature under Art. 415 (1). and the proper venue of any case to recover ownership
of the same, which is what the purpose of the complaint to annul the amended Deed
of Sale amounts to, should be the place where the property is located, or the RTC of
Bulacan.
Additional Answers;
1. Buildings are always immovable property, and even in the instances
where the parties to a contract seem to have dealt with it separate and apart from
the land on which it stood in no wise does it change its character as immovable
property. A building is an immovable even if not erected by the owner of the land.
The only criterion Is union or incorporation with the soil. (Ladera vs. Hodges (CA) 48
O.G. 4374) (Reyes and Puno, Outline of Philippine Civil Law, Vol. 2. p.7)
2. The warehouse built by Pedro on the mortgaged property Is real property
within the context of Article 415 of the New Civil Code, Although it was built by
Pedro after the foreclosure sale without the knowledge and consent of the new
owner which makes him a builder in bad faith, this does not alter the character of the
warehouse as a real property by incorporation. It is a structure which cannot be
removed without causing injury to the land. So, my advice to Pedro is to file the case
with the RTC of Bulacan, the situs of the property,
(Note: If the examinee does not mention that the structure was built by a
builder In bad faith, it should be given full credit).
05; Property; real vs personal property
1983 No. 5
To secure the payment to B of a loan, A, the owner of a lot, executed a
chattel mortgage on the building he erected thereon as well as on some newly
bought machinery stored therein. Thereafter, a judgment was rendered against A in
favor of C who had the building and machinery levied upon to satisfy the judgment,
Is the chattel mortgage binding on C? Explain,
Answer
It is, in so far as the machinery is concerned but void as to the building. The
machinery is movable property as it does not appear that A introduced it in the
Page 207 of 391
building in connection with any industry or works being carried on therein, while the
building is immovable property and consequently cannot be the subject of a chattel
mortgage,
05; Property; rights of a property owner; limitations
1977 No. III-a
The rights of a person over his property are:
a. The right to enjoy, which includes:
(1) Jus utendi, or the right to use;
(2) Jus fruendi, or the right to enjoy the fruits;
(3) Jut abutendi, or the right to consume the by its use.
b. The right to dispose (jus disponendi)t or the right to alienate, encumber,
transform, or even to destroy the property.
c. The right to vindicate (jus vindicandi), or the right of action available to the
owner to recover the property against the holder or possessor. (Art 428, Civil Code).
The limitations upon the right of ownership are:
(1) General limitations imposed by the State for its benefit, such as the
power of eminent domain, the police power, and the power of taxation;
(2) Specific limitations imposed by law, such as legal servitudes;
(3) Limitations imposed by the party transmitting the property either by
contract or by will;
(4) Limitations imposed by the owner himself, such as voluntary servitudes,
mortgages, pledges, and lease rights; and
(5) Inherent limitations arising from conflict with other rights, such as those
caused by contiguity of property.
We might add to the above enumeration the Constitutional prohibition
regarding acquisition of private land by aliens and other Constitutional
limitations.
05; Property; usufruct
1997 No. 8:
On 1 January 1980, Minerva, the owner of a building, granted Petronila a
usufruct over the property until 01 June 1998 when Manuel, a son of Petronila,
would have reached his 30th birthday. Manuel, however, died on 1 June 1990 when
he was only 26 years old.
Minerva notified Petronila that the usufruct had been extinguished by the
death of Manuel and demanded that the latter vacate the premises and deliver the
same to the former. Petronila refused to vacate the place on the ground that the
usufruct in her favor would expire only on 1 June 1998 when Manuel would have
reached his 30th birthday and that the death of Manuel before his 30th birthday did
not extinguish the usufruct.
Whose contention should be accepted? Answer:
Petronila's contention is correct. Under Article 606 of the Civil Code, a
usufruct granted for the time that may elapse before a third person reaches a certain
age shall subsist for the number of years specified even if the third person should
die unless there is an express stipulation in the contract that states otherwise. In the
case at bar, there is no express stipulation that the consideration for the usufruct is
Page 208 of 391
the existence of Petronila's son. Thus, the general rule and not the exception should
apply in this case.
Alternative Answer;
This is a usufruct which is clearly intended for the benefit of Manuel until he
reaches 30 yrs. of age. with Petronila serving only as a conduit, holding the property
in trust for his benefit. The death of Manuel at the age of 26. therefore, terminated
the usufruct.
05; Property; usufruct
1989 No. 6:
(1) What is USUFRUCT? How is usufruct extinguished?
Answer:
Usufruct gives a right to enjoy the property of another with the obligation of
preserving its form and substance, unless the title constituting it or the law otherwise
provides.
Usufruct is extinguished:
(1) By the death of the usufructuary, unless a contrary intention clearly
appears;
(2) By the expiration of the period for which it was constituted, or by the
fulfillment of any resolutory condition provided in the title creating the usufruct;
(3) By merger of the usufruct and ownership in the same person;
(4) By renunciation of the usufructuary;
(5) By the total loss of the thing in usufruct;
(6) By the termination of the right of the person constituting the usufruct;
(7) By prescription.
RECOMMENDATION OF THE COMMITTEE:
An enumeration of four (4) should be given full credit.
05; Property; usufruct
1977 No VI-c
What are the requisites of usufruct? How is it constituted and how do you
distinguish it from ownership and from lease? What are the modes of extinguishing
them?
Answer
There are two requisites of usufruct the essential and the accidental. The
essential requisite is the right to enjoy the property of another, while the accidental
requisite is the obligation of preserving the form and substance of such property.
The latter is accidental, because the title constituting the usufruct or the law may
otherwise provide as in the case of abnormal usufruct. (4 Manresa 322.)
A usufruct may be constituted: (1) by law, (2) by the will of private persons
expressed in acts inter vivos, (3) by the will of private persons expressed in a last
will and testament, and (4) by prescription (Art. 563, Civil Code).
While ownership has for its attributes (1) the right to enjoy (jus utendi, jus
fruendi, jus abutendi), (2) the right to dispose (jus disponendi), and (3) the right to
vindicate or recover the property (jus vindicandi), usufruct is limited merely to the
enjoyment of the property (jus utendi and jus fruendi),
Page 209 of 391
Usufruct is distinguished from lease as follows :
(1) As to nature of right Usufruct is always a real right, whereas lease
becomes a real right only when registered;
(2) As to constitution Usufruct is constituted by law, by the will of private
persons expressed in acts inter vivos or in a last will and testament, and by
prescription, whereas lease is as a rule constituted by contract;
(3) As to the -person constituting it - In usufruct the person constituting it is
the owner, whereas in lease the person constituting it need not be the owner;
(4) As to extent Usufruct includes the right to use and to enjoy the
fruits (jus utendi and jus fruendi) of the thing, whereas lease is more limited;
(5) As to duration There is no limitation to the duration of a usufructuary
right, whereas there is a limitation to the duration of a lease right;
(6) As to repairs The usufructuary is responsible for ordinary repairs,
whereas the lessee is not; and
(7) As to taxes The usufructuary is responsible for taxes on the fruits,
whereas in lease the lessee is not.
Usufruct is extinguished:
(1) By the death of the usufructuary, unless a contrary intention clearly
appears;
(2) By the expiration of the period for which it was constituted, or by the
fulfillment of any resolutory condition provided in the title creating the usufruct;
(3) By merger of the usufruct and ownership in the same person;
(4) By renunciation of the usufructuary;
(6) By the total loss of the thing in usufruct;
(6) By the termination of the right of the person constituting the usufruct; and
(7) By prescription, (Art. 603, Civil Code).
05; Property; usufruct
1978 No. IV-a
A. sold a parcel of land with two (2) buildings thereon to B for P50,000
subject to the condition that A shall receive from B by way of life pension one-third
(1/3) of the rents of the two (2) buildings. Without B's fault, the two (2) buildings were
totally destroyed by fire. B now alleges that the right to receive a life pension was
extinguished upon the loss of the buildings. Is such a contention valid? Reasons for
your answer.
Answer
The right to receive a life pension was not extinguished upon the loss of the
buildings.
It must be observed that under the condition agreed upon by and between A
and B that A shall receive from B by way of life pension one-third (1/3) of the rents of
the two buildings, in reality, what was created was a life usufruct, with A as
usufructuary, over the two buildings under an arrangement whereby A shall always
be entitled during his lifetime to one-third (1/3) of the rents of the two buildings. Now,
it is well-settled that rents constitute earnings of both buildings and the land on
which it is constructed. There can be land without building, but there can be no
building without land. Therefore, in the case before us, the things in usufruct are the
two buildings and the land on which the buildings are constructed. Under our law, in
Page 210 of 391
order that there will be an extinguishment of the usufruct, it is essential that the
things in usufruct must be totally lost or destroyed. Here, there was no total loss.
The land remains intact. Therefore, pursuant to the law on usufructs, A the
usufructuary, shall now have the right to make use of one-third of the land and the
materials. This is a temporary measure calculated to maintain the usufruct alive until
the very things destroyed are reconstructed or replaced.
(NOTES: The above answer is based on Arts. 603. No. 5, and 607, Civil
Code, and on Vda, de Albur vs. Fabie, 106 Phil. 855, which is almost identical to the
above problem.)
05; Property; usufruct
1979 No. XVII
On June 5, 1960, DP delivered possession of his house and lot in the
Poblacion of Polo, Bulacan to AB who in turn delivered to the former possession of
his 2-hectare rice land. Both properties were unregistered. They executed a
document entitled "Barter" which, among others provided that both parties shall
enjoy the material possession of their respective properties; that neither party shall
encumber, alienate or dispose of their respective properties as bartered without the
consent of the other; and that DP shall be obliged to return the property to AB when
the latter's son shall attain majority and decide to return DP's property. After AB's
death and his son S attained majority in 1977, the latter demanded for the return of
the 2 hectares of rice land which had then increased tremendously in value. DP
refused and 30 S filed an action for recovery of the land. Will the action prosper?
Why?
Answer
Yes, the action will prosper. The stipulations in the barter agreement are
clear. All that the parties intended was to transfer the material possession and use of
the subject properties to the other. There was, therefore, no conveyance of their
right of ownership. In fact the parties retained their right to, alienate their right of
ownership, a right which is an element of ownership. What was, therefore,
transferred was merely their right of usufruct. But then, the document also says that
DP shall be obliged to return the property to AB when the latter's son shall attain
majority and decide to return DP's property. The mutual agreement, therefore, was
subject to a resolutory condition the happening of which would extinguish or
terminate their right of usufruct over the subject properties. The facts are clear. Said
condition has already been fulfilled.
Page 211 of 391
06; LTD
1988 No. 14:
(a) 1) Is title to registered land subject to prescription? Explain your answer.
2) How about the right of the registered owner to recover possession, is it
equally imprescriptible? Why?
3) What effect has the equitable principle of laches on the imprescriptibility of
Torrens Title? Explain.
(b) In passing upon the registrability of a document sought to be registered,
what formal requisites is the Register of Deeds charged to determine, under his
responsibility, whether or not they have been complied with?
Answer:
(a) 1) No because under Section 47, P.D, 1529, no title to registered land in
derogation of that of the registered owner shall be acquired by prescription or
adverse possession. A similar provision is found in the Civil Code. The reason is that
once a piece of land is registered under the Torrens System, it operates as a notice
to the whole world. All persons are bound by it. No one can plead ignorance of the
registration.
(2) The right to recover the land from another person holding it is equally
imprescriptible, the reason being that possession is a mere consequence of
ownership.
(3) While a Torrens Title is imprescriptible, under certain exceptional
circumstances, it may yield to the equitable principle of laches. In other words,
certain circumstances such as inaction or utter neglect on the part of the owner and
the intervention of rights by third parties may, for reasons of equity, convert the claim
of imprescriptibility into a stale demand, (Mejia vs. Gamponia, 100 Phil. 277;
Miguel vs. Catalino, G.R. L-23072, Nov. 29, 1968; Heirs of Batiog Lacamen vs.
Heirs of Laruan, G.R. L-27058, July 31,1985).
(b) To be registerable, a voluntary document affecting registered land must
be sufficient in law. (Section 51, P.D. 1529) Sufficiency refers to both substance
and form. As to form, it is the ROD's responsibility to check such items as the full
name and signature of vendor or grantor, the marital consent of the wife if the land
sold is conjugal, the full name, nationality, the civil status, the name of spouse, if
married, the resident and postal address of the grantee. If the grantee is a
corporation, the deed must be accompanied with the Articles of Incorporation, a
board resolution authorizing the corporation to buy and another resolution of the
Board naming the corporate officer authorized to execute and sign the contract. This
is not to mention the proper observance of the requirements in the acknowledgment
portion of the deed.
06; LTD; acquisition of lands; citizenship requirement
2003 No XVIII.
In 1970, the spouses Juan and Juana de la Cruz, then Filipinos, bought the
parcel of unregistered land in the Philippines on which they built a house which
became their residence. In 1986, they migrated to Canada and became Canadian
citizens.
Thereafter, in 1990, they applied, opposed by the Republic, for the
registration of the aforesaid land in their names. Should the application of the
spouses de la Cruz be granted over the Republics opposition? Why? 5%
SUGGESTED ANSWER:
Page 212 of 391
Yes, the application should be granted. As a rule, the Constitution prohibits
aliens from owning private lands in the Philippines. This rule, however, does not
apply to the spouses Juan and Juana de la Cruz because at the time they acquired
ownership over the land, albeit imperfect, they were still Filipino citizens. The
application for registration is a mere confirmation of the imperfect title which the
spouses have already acquired before they became Canadian citizens. (Republic v.
CA, 235 SCRA 567 [1994]).
06; LTD; Act 3344
1994 No. 3;
3) How do you register now a deed of mortgage of a parcel of land
originally registered under the Spanish Mortgage Law?
Alternative Answers;
3) a) After the Spanish Mortgage Law was abrogated by P.D. 892 on
February 16.1976, all lands covered by Spanish titles that were not brought under
the Torrens system within six 16] months from the date thereof have been
considered as "unregistered private lands."
Thus, a deed of mortgage affecting land originally registered under the
Spanish Mortgage Law is now governed by the system of registration of transactions
or instruments affecting unregistered land under Section 194 of the Revised
Administrative Code as amended by Act No. 3344. Under this law, the instrument or
transaction affecting unregistered land is entered in a book provided for the purpose
but the registration thereof is purely voluntary and does not adversely affect third
persons who have a better right.
b) By recording and registering with the Register of Deeds of the place where
the land is located, in accordance with Act 3344. However, P.D. 892 required
holders of Spanish title to bring the same under the Torrens System within 6 months
from its effectivity on February 16, 1976.
06; LTD; amendment of entries in a certificate of title
1981 No. 17
In a verified petition filed before the Court of First Instance, sitting as a land
registration Court, and under the summary proceeding for amendment or alteration
outlined in section 112 of the Land Registration Act, husband "H", being the
registered owner of three parcels of land, sought to strike out the words "married to
W" appearing in the said titles, and to place in Lieu thereof the word "single" on the
ground that the phrase "married to W" was entered by reason of clerical error or
oversight. Opposition was filed by "W" who alleged that she is the legal wife of "H",
and that the insertion of the phrase "married to W" was not the result of clerical error
but was the voluntary act of "H".
May the Court of First Instance, sitting as a land registration Court, continue
to take cognizance of the case and resolve the issue posed? Explain.
Answer
The Court of First Instance, sitting as a Land Registration Court, cannot
continue to take cognizance of the case and resolve the issue posed.
It is apparent that the Court, sitting as a land registration court, cannot alter
the description of the civil status of the petitioner in the transfer certificates of title in
question. It will have to receive evidence and determine the civil status of said
petitioner. This requires a full-dressed trial, thus rendering the summary proceeding
envisaged in Sec. 112 of Act 496 inadequate.
Page 213 of 391
Therefore, the remedy of "W" against her husband "H" or of "H" against his
wife "W" would be to thresh out the question of their status in a separate and
independent action filed for that purpose.
(Note: The above answer is based on Martinez vs. Evangelista, L-26399,
Jan. 31,1981.)
However, the Committee respectfully recommends that a contrary answer
may also be considered as a correct answer. Under Section 2 of P.O. No. 1529, the
jurisdiction of a Court of First Instance acting as a Land Registration Court has been
broadened.
06; LTD; annotation of lis pendens
2001 No XX
Mario sold his house and lot to Carmen for P1 million payable in five (5) equal
annual installments. The sale was registered and title was issued in Carmen's name.
Carmen failed to pay the last three installments and Mario filed an. action for
collection, damages and attorneys fees against her. Upon filing of the complaint, he
caused a notice of lis pendens to be annotated on Carmen's title. Is the notice of lis
pendens proper or not? Why? (5%)
SUGGESTED ANSWER..
The notice of lis pendens is not proper for the reason that the case filed by
Mario against Carmen is only for collection, damages, and attorney's fees.
Annotation of a lis pendens can only be done in cases involving recovery of
possession of real property, or to quiet title or to remove cloud thereon, or for
partition or any other proceeding affecting title to the land or the use or occupation
thereof. The action filed by Mario does not fall on anyone of these.
06; LTD; annotations; classification
1985 No. 13
B) A bought a house and lot in a subdivision, subject to the condition,
annotated on the certificate of title, that they shall be used for residential purposes
only. Ten years later, A sold the property to B who converted it into a restaurant. The
owner demanded its closure but B refused alleging (1) that although he
subsequently came to know the title issued to him bears such an annotation, he was
unaware of it at the time of the sale as the seller did not tell him so and the deed of
sale in his favor makes no mention of it; (2) that his lot has been re-classified by
ordinance as commercial; and (3) that it has in fact become commercial because of
its proximity to some stores and a shopping center in an adjoining subdivision.
Rule on the validity of said defenses.
Answers:
B. 1. There are now 3 decisions of the Intermediate Appellate Court. The first
decision was incorporated in 1984 in the Silverio case. The facts are identical. There
was a legitimate exercise of the police power. The stipulation in the contract of sale
which was properly annotated at the back of the title will have to give way to that
legitimate exercise of the police power of the State.
Besides (under Art. 1266 of the New Civil Code), we have here an obligation
which cannot be complied with because of a legal impossibility by reason of that
ordinance. The classification supersedes the annotation.
2. Fulfillment of the obligation is prevented by a law and therefore no
longer tenable. Since continuing with the house as residential is. no longer
possible, the annotation is no longer valid.
Page 214 of 391
3. The owner has dominion over his property, but he is bound by fair
restrictions on the title since he is charged with constructive notice of such
restrictions.
4. The contention that B was unaware, at the time of the sale, of the
encumbrance in the certificate of title is not valid since the annotation on the
certificate is constructive notice to him.
5. The contention that the property became commercial because of its
proximity to some stores and a shopping center is not sufficient legal justification for
a breach of the agreement.
06; LTD; foreshore lands
2000 No VIII
b) Regina has been leasing foreshore land from the Bureau of Fisheries and
Aquatic Resources for the past 15 years. Recently, she learned that Jorge was able
to obtain a free patent from the Bureau of Agriculture, covering the same land, on
the basis of a certification by the District Forester that the same is already "alienable
and disposable". Moreover, Jorge had already registered the patent with the
Register of Deeds of the province, and he was issued an Original Certificate of Title
for the same. Regina filed an action for annulment of Jorge's title on the ground that
it was obtained fraudulently. Will the action prosper? (2%)
SUGGESTED ANSWER:
An action for the annulment of Jorge's Original Certificate of Title will prosper
on the following grounds:
(1) Under Chapter IX of C .A, No. 141, otherwise known as the Public Land
Act, foreshore lands are disposable for residential, commercial, industrial, or similar
productive purposes, and only by lease when not needed by the government for
public service.
(2) If the land is suited or actually used for fishpond or aquaculture
purposes, it comes under the Jurisdiction of the Bureau of Fisheries and Aquatic
Resources (BFAR) and can only be acquired by lease. (P.D. 705)
(3) Free Patent is a mode of concession under Section 41, Chapter VII of
the Public Land Act, which is applicable only for agricultural lands.
(4) The certificate of the district forester that the land is already "alienable
and disposable" simply means that the land Is no longer needed for forest purposes,
but the Bureau of Lands could no longer dispose of it by free patent because It Is
already covered by a lease contract between BFAR and Regina. That contract must
be respected.
(5) The free patent of Jorge is highly Irregular and void ab initio, not only
because the Bureau has no statutory authority to issue a free patent over a
foreshore area, but also because of the false statements made in his sworn
application that he has occupied and cultivated the land since Jury 4, 1945, as
required by the free patent law. Under Section 91 of the Public Land Act, any patent
concession or title obtained thru false representation is void ab initio. In cases of this
nature. It is the government that shall institute annulment proceedings considering
that the suit carries with it a prayer for the reversion of the land to the state.
However, Regina is a party in interest and the case will prosper because she has a
lease contract for the same land with the government.
06; LTD; homestead patents; void sale
1999 No XII.
Page 215 of 391
In 1950, the Bureau of Lands issued a Homestead patent to A, Three years
later, A sold the homestead to B. A died in 1990, and his heirs filed an action to
recover the homestead from B on the ground that its sale by their father to the latter
is void under Section 118 of the Public Land Law. B contends, however, that the
heirs of A cannot recover the homestead from him anymore because their action
has prescribed and that furthermore, A was in pari delicto. Decide. (5%)
ANSWER:
The sale of the land by A to B 3 years after issuance of the homestead
patent, being in violation of Section 118 of the Public Land Act, is void from its
inception.
The action filed by the heirs of B to declare the nullity or inexistence of the
contract and to recover the land should be given due course.
B's defense of prescription is untenable because an action which seeks to
declare the nullity or inexistence of A contract does not prescribe. (Article 1410;
Banaga vs. Soler, 2 8CRA 765)
On the other hand, B's defense of pari delicto is equally untenable. While as
a rule, parties who are in pari delicto have no recourse against each other on the
principle that a transgressor cannot profit from his own wrongdoing, such rule does
not apply to violations of Section 118 of the Public Land Act because of the
underlying public policy in the said Act "to conserve the land which a homesteader
has acquired by gratuitous grant from the government for himself and his family". In
keeping with this policy, it has been held that one who purchases a homestead
within the five-year prohibitory period can only recover the price which he has paid
by filing a claim against the estate of the deceased seller (Labrador vs. Delos
Santos 66 Phil. 579) under the principle that no one shall enrich himself at the
expense of another. Applying the pari delicto rule to violation of Section 118 of the
Public Land Act, the Court of Appeals has ruled that "the homesteader suffers the
loss of the fruits realized by the vendee who in turn forfeits the improvement that he
has introduced into the land." (Obot vs. SandadiUas, 69 OG, April 35, 1966}
FIRST ALTERNATIVE ANSWER:
The action to declare the nullity of the sale did not prescribe (Art. 1410}, such
sale being one expressly prohibited and declared void by the Public Lands Act [Art.
1409, par. (7)]. The prohibition of the law is clearly for the protection of the heirs of A
such that their recovering the property would enhance the public policy regarding
ownership of lands acquired by homestead patent (Art. 1416). The defense of pari
delicto is not applicable either, since the law itself allows the homesteader to
reacquire the land even if it has been sold.
SECOND ALTERNATIVE ANSWER:
Prescription does not arise with respect to actions to declare a void contract a
nullity (Article 1410). Neither is the doctrine of pari delicto applicable because of
public policy. The law is designed for the protection of the plaintiff so as to enhance
the public policy of the Public Land Act to give land to the landless.
If the heirs are not allowed to recover, it could be on the ground of laches
inasmuch as 40 years had elapsed and the owner had not brought any action
against B especially if the latter had improved the land. It would be detrimental to B if
the plaintiff is allowed to recover.
06; LTD; homestead; reversion
1978 No. IX-a
Page 216 of 391
Free Patent No. V-347 was issued in favor of A for the land in question.
Subsequently, Original Certificate of Title No. P-25G7 was issued in A's name on
June 15, 1976. On July 30, 1977, the Republic of the Philippines at the instance of
the Bureau of Forestry filed a complaint to declare the free patent and the Original
Certificate of Title null and void on the ground that the land covered thereby is forest
land. Decide the case, with reasons.
Answer
The annulment and cancellation of a homestead patent and the consequent
reversion of the property to the State are matters between the State and the grantee
or his heirs. Hence, the government may take appropriate steps to annul the grant
and assert title to the homestead. Assuming then that the action in the instant case
is for reversion of the land, obviously, it will still prosper. The period of prescription is
five years, not one year, to be counted from the time the right to such action has
accrued However, if the action merely seeks a review of the homestead award, the
period of prescription is one year.
(NOTE: The above answer is based on decided cases, more specifically,
Lopez vs. Padilla, L-27559, May 18, 1972, De la Cruz vs. Corpus, 62 Off. Gaz. 49, p
9271, Dec. 5, 1966, CA. The period of prescription of five years is based on Art.
1149, Civil Code.)
06; LTD; homestead; right to repurchase
1978 No. IX-b
A's father was the original homesteader of a parcel of land covered by a
homestead patent issued on June 3, 1932. On April 27, 1958, A's father sold said
property to B. On July 15, 1962, A's father died. On March 15, 1963, A sought to
repurchase the property from B. B refused. Can A repurchase the property? Give
reasons for your answer.
Answer:
Yes, A can repurchase the property. According to the Public Land Law, every
conveyance of land acquired under the free patent or homestead provisions, when
proper, shall be subject to repurchase by the applicant, his widow, or legal heirs,
within a period of five years from the date of conveyance. The homestead was sold
by the applicant on April 27, 1958. A, the legal heir of the applicant sought to
repurchase the land on March 15, 1963. It is evident that he is exercising his right of
legal redemption within the prescribed period of five years.
(NOTE: The above answer is based on Section 119 of the Public Land Law
and on decided cases.)
06; LTD; homestead; right to repurchase
1978 No. XI-b
A's father was the original homesteader of a parcel of land covered by a
homestead patent issued on June 3, 1932. On April 27, 1968, A's father sold said
property to B. On July 15, 1962, A's father died. On March 15, 1963, A sought to
repurchase the property from B. B refused. Can A repurchase the property? Give
reasons for your answer.
Answer
Sec. 119, CA 141, provides that every conveyance of land acquired under the
free patent or homestead provisions, when proper, shall be subject to repurchase by
the applicant, his widow, or legal heirs, within a period of five years from the date of
the conveyance.
Page 217 of 391
Reason: To preserve and keep in the family of the homesteader (or patentee)
that portion of public land which the State had gratuitously given to him. It is in
keeping with the fundamental idea to hold that the right to repurchase exists not only
when the original homesteader makes the conveyance, but also when it is made by
his widow or heirs (Pascua vs. Talens, 89 Phil. 792).
06; LTD; increase of area by accession
1989 No. 7:
(2) Subsequent to the original registration of a parcel of land bordering a
river, its area was increased by accession. This additional area was not included in
the technical description appearing on the Torrens Certificate of Title having been
acquired subsequent to the registration proceedings. May such additional area be
acquired by third persons thru prescription? Give your reasons.
Answer:
The Land Registration Law provides that no title in derogation of the
registered owner may be acquired by adverse possession or acquisitive possession.
Since the law refers to registered lands, the accession mentioned in this question
may be acquired by a third person through adverse possession or acquisitive
possession.
Alternative Answer:
If the accession is man made, then it cannot be considered as private
property. It belongs to the public domain, and, therefore, cannot be acquired by
adverse possession or acquisitive possession.
06; LTD; indefeasibility of title
1983 No. 18
A filed in the regional trial court an application for registration (confirmation of
title) of a parcel of land in his name. B filed a motion to dismiss A's application on the
ground that the court has no jurisdiction to entertain it, the land having already been
registered in his name after the grant to him of a homestead patent. Should the
motion to dismiss be granted? Why?
Answer:
The motion should be granted. Since a patent and a certificate of title to the
land have already been issued, it has become registered property, the title thereto is
indefeasible and is no longer subject to the jurisdiction of a Court of First Instance
acting as a land registration court.
06; LTD; IPV; adverse claims
1998 No XIX,
Section 70 of Presidential Decree No. 1529, concerning adverse claims on
registered land, provides a 30-day period of effectivity of an adverse claim, counted
from the date of its registration. Suppose a notice of adverse claim based upon a
contract to sell was registered on March 1, 1997 at the instance of the BUYER, but
on June 1, 1997, or after the lapse of the 30-day period, a notice of levy on
execution in favor of a JUDGMENT CREDITOR was also registered to enforce a
final judgment for money against the registered owner. Then, on June 15, 1997
there having been no formal cancellation of his notice of adverse claim, the BUYER
pays to the seller-owner the agreed purchase price in full and registers the
corresponding deed of sale. Because the annotation of the notice of levy is carried
over to the new title in his name, the BUYER brings an action against the
JUDGMENT CREDITOR to cancel such annotation, but the latter claims that his lien
Page 218 of 391
is superior because it was annotated after the adverse claim of the BUYER had ipso
facto ceased to be effective. Will the suit prosper? [5%]
Answer:
The suit will prosper. While an adverse claim duly annotated at the back of a
title under Section 7O of P.D. 1529 is good only for 30 days, cancellation thereof is
still necessary to render it ineffective, otherwise, the inscription thereof will remain
annotated as a lien on the property. While the life of adverse claim is 3O days under
P.D. 1529, it continuous to be effective until it Is canceled by formal petition filed with
the Register of Deeds.
The cancellation of the notice of levy is justified under Section 108 of P.D.
1529 considering that the levy on execution can not be enforced against the buyer
whose adverse claim against the registered owner was recorded ahead of the notice
of levy on execution.
06; LTD; IPV; buyer in good faith; laches
1983 No. 17
In 1930, A sold a piece of land to B and delivered his certificate. of title
thereto. B occupied the land but did not have A's title cancelled and a new one
issued in his name. Upon B's death in 1950, the land passed to his son, C, who
continued in possession thereof.
In 1970, A's two sons, X and Y, secured the cancellation of the title of their
father who had died, and the issuance of two titles, one in X's name, covering 1/2 of
the land, and the other in Y's name, embracing the other half. X thereafter sold his
part to Z, who was unaware of the antecedents.
Is C bound to deliver to Z the portion sold to the latter by X, and to Y the part
embraced in Y's title?
Answer
C must deliver the portion sold to Z, he being a buyer in good faith but not the
part embraced in Y's title, he having lost the right to recover it by laches.
06; LTD; IPV; collateral attack
1977 No. XVII-a
A owned a titled lot which he sold to B, who sold it to C, who obtained in
certificate in his own name, remained in possession, and paid taxes. Later, A
managed to obtain from the court an order for the cancellation of C's title and the
issuance of a title in her own name, on the false claim that B had resold the property
to her, but she lost the deed of sale. The Court acted as a cadastral court. 6 years
late, C sued to have this ownership recognized and to cancel the title in the name of
X, who had bought the lot from A. X resisted, invoking prescription and the in-
defeasibility of his title. Decide, stating reasons.
Answer
C should prevail. C's title to the land being evidenced by a Torrens title
issued to him who is legally presumed to have acquired it in good faith, nothing to
the contrary being shown, his title by law is indefeasible. The mere fact that his
action for recognition of his ownership was instituted after six years cannot militate
against him inasmuch as his title by provision of law is imprescriptible.
The Cadastral Court which ordered the cancellation of C's title and the
issuance of a title in the name of A based on a false claim that B had resold the
property to her but she lost the deed of sale, had no power to issue such an order. It
Page 219 of 391
had no power to determine such highly controversial issue which should have been
threshed out in an ordinary civil action.
On the other hand, X who bought the lot from A, could not have acquired any
better right than A under any standard of justice, and since C is the owner by prior
registration, the latter's title cannot be defeated by any subsequent title or by any
collateral attack.
06; LTD; IPV; constructive trust
1980 No. IX
(b) "HH", "II", and "JJ" inherited from their parents a large parcel of land.
"HH" and "II" went abroad to reside in Canada. In their absence, "JJ"' applied for the
registration of the whole land in his name only. In due time, "JJ" obtained a Torrens
Title for the land.
When "H" and "H" returned from Canada after seven years, they found out
what "JJ" did and sued him for their respective shares. "JJ" contended that the
decree of title can no longer be reviewed or changed because of the lapse of more
than one year from its issuance.
In whose favor would you decide?
Answer
(b) My decision is in favor of "HH" and "II".
In reality, the action commenced by plaintiffs against defendant is an action
for reconveyance of their respective shares in the subject property based on the
constructive trust recognized and sanctioned by the Civil Code which declares that if
the property is acquired through mistake or fraud, the person obtaining it is, by force
of law, considered a trustee of an implied trust for the benefit of the person from
whom the property comes. Since the obligation is created by law, the action
commenced by the beneficiaries against him shall prescribe and the period of
prescription is ten years which shall be counted from the time of the discovery of the
fraud. When did the plaintiff discover the fraud committed by defendant? Under the
constructive notice rule, they are deemed to have discovered the fraud as of the
date the trustee set up in himself a title adverse to the title of the beneficiaries.
Normally, this would be the date the trustee ("JJ") obtained his Torrens Title. Since
the instant action was commenced seven years after the issuance of said Title, it is
obvious that it was commenced in time.
(NOTE: See: Art. 1456, Civil Code; Gerona vs. De Guzman, 11 SCRA 163;
Fabian vs. Fabian, 22 SCRA 231; Cuaycong vs. Cuaycong, 21 SCRA 1192; De-la
Cerna vs. De la Cerna, 72 SCRA 515; Jaramil vs. CA, 78 SCRA 420; Duque vs.
Domingo, 80 SCRA 654; Nacalaban vs. CA 80 SCRA 428.)
06; LTD; IPV; effect of entry in day book; acquisitive prescription; laches
1998 No XX.
In 1965, Renren bought from Robyn a parcel of registered land evidenced by
a duty executed deed of sale. The owner presented the deed of sale and the
owner's certificate of title to the Register of Deeds. The entry was made in the
daybook and corresponding fees were paid as evidenced by official receipt.
However, no transfer of certificate of title was Issued to Renren because the original
certificate of title in Robyn's name was temporarily misplaced after fire partly gutted
the Office of the Register of Deeds. Meanwhile, the land had been possessed by
Robyn's distant cousin, Mikaelo, openly, adversely and continuously in the concept
of owner since 1960. It was only in April 1998 that Renren sued Mikaelo to recover
Page 220 of 391
possession. Mikaelo invoked a) acquisitive prescription and b) laches, asking that he
be declared owner of the land. Decide the case by evaluating these defenses, [5%]
Answer:
a) Renren's action to recover possession of the land will prosper. In 1965,
after buying the land from Robyn, he submitted the Deed of Sale to the Registry of
Deeds for registration together with the owner's duplicate copy of the title, and paid
the corresponding registration fees. Under Section 56 of PD No. 1529, the Deed of
Sale to Renren is considered registered from the time the sale was entered in the
Day Book (now called the Primary Entry Book).
For all legal intents and purposes, Renren is considered the registered owner
of the land. After all, it was not his fault that the Registry of Deeds could not issue
the corresponding transfer certificate of title.
Mikaelo's defense of prescription can not be sustained. A Torrens title is
imprescriptible. No title to registered land in derogation of the title of the registered
owner shall be acquired by prescription or adverse possession. (Section 47, P.D.
No, 1529)
The right to recover possession of registered land likewise does not prescribe
because possession is just a necessary incident of ownership.
b) Mikaelo's defense of laches, however, appears to be more sustainable.
Renren bought the land and had the sale registered way back in 1965. From the
facts, it appears that it was only in 1998 or after an inexplicable delay of 33 years
that he took the first step asserting his right to the land. It was not even an action to
recover ownership but only possession of the land. By ordinary standards, 33 years
of neglect or inaction is too long and maybe considered unreasonable. As often held
by the Supreme Court, the principle of imprescriptibility sometimes has to yield to
the equitable principle of laches which can convert even a registered land owner's
claim into a stale demand.
Mikaelo's claim of laches, however, is weak insofar as the element of equity
is concerned, there being no showing in the facts how he entered into the ownership
and possession of the land.
06; LTD; IPV; effect of laches
1977 No. XVII-b
A sold a titled land to L in 1928, but the sale was not approved by the
Provincial Governor as required by law, vendor and vendee being non-Christians.
L took immediate possession and introduced improvements, but the title
remained in A's name. After 30 years the heirs of L sued the heirs of A to have the
land registered in their name. What grounds can be invoked by the defendants to
prevail? How should the case be decided?
Answer
In order that they shall prevail, defendants can invoke the doctrine of "stale
demands" or laches.
The case should be decided in favor of defendants. As held by the Supreme
Court in a similar case, even granting plaintiffs' proposition that no prescription lies
against their predecessor's recorded title, their passivity and inaction for more than
30 years justify the defendants in setting up the defense of laches. All of the four
elements of laches are present. As a result, the plaintiffs' action must be considered
barred. (Miguel vs. Catalino, 26 SCRA 243; Lacamen vs. Laruan, 65 SCRA 605)
06; LTD; IPV; foreclosure of registered lands; prescription
Page 221 of 391
1989 No. 12:
(2) Does an action to foreclose a real estate mortgage affecting registered
land under the Torrens System prescribe? Give your reasons.
Answer:
Even if the property given as collateral is covered by a Torrens Title, the right
to foreclose a real estate mortgage thereon prescribes. This is really an action to
enforce collection of the loan.
06; LTD; IPV; forged deed
1991 No 15:
Bruce is the registered owner, of a parcel of land with a building thereon and
is in peaceful possession thereof. He pays the real estate taxes and collects the
rentals therefrom. Later, Catalino, the only brother of Bruce, filed a petition where
he, misrepresenting to be the attorney-in-fact of Bruce and falsely alleging that the
certificate of title was lost, succeeded in obtaining a second owner's duplicate copy
of the title and then had the same transferred in his name through a simulated deed
of sale in his favor. Catalino then mortgaged the property to Desiderio who had the
mortgage annotated on the title. Upon learning of the fraudulent transaction, Bruce
filed a complaint against Catalino and Desiderio to have the title of Catalino and the
mortgage in favor of Desiderio declared null and void.
Will the complaint prosper, or will the title of Catalino and the mortgage to
Desiderio be sustained?
Answer.
The complaint for the annulment of Catalino's Title will prosper. In the first
place, the second owner's copy of the title secured by him from the Land
Registration Court is void ab initio, the owner's copy thereof having never been lost,
let alone the fact that said second owner's copy of the title was fraudulently procured
and improvidently issued by the Court. In the second place, the Transfer Certificate
of Title procured. by Catalino is equally null and void, it having been issued on the
basis of a simulated or forged Deed of Sale. A forged deed is an absolute nullity and
conveys no title.
The mortgage in favor of Desiderio is likewise null and void because the
mortgagor is not the owner of the mortgaged property. While it may be true that
under the "Mirror Principle" of the Torrens System of Land Registration, a buyer or
mortgagee has the right to rely on what appears on the Certificate of Title, and in the
absence of anything to excite suspicion, is under no obligation to look beyond the
certificate and investigate the mortgagor's title, this rule does not find application in
the case at hand because here. Catalino's title suffers from two fatal infirmities,
namely:
1. The fact that it emanated from a forged deed of a simulated sale;
2. The fact that it was derived from a fraudulently procured or improvidently
issued second owner's copy, the real owner's copy being still intact and in the
possession of
the true owner, Bruce.
The mortgage to Desiderio should be cancelled without prejudice to his right
to go after Catalino and/or the government for compensation from the assurance
fund.
06; LTD; IPV; forged deed as a root of a title
1989 No. 16:
Page 222 of 391
(2) "A" is the owner of a registered land. The Torrens Title is entrusted to "B"
his clerk secretary, who forges "A's" signature on a deed of sale of said land in his
(B's) favor. A new title is issued in the name of "B," upon registration. Does "B" have
a valid title over the land? If "B" sells the property to "C", does the latter acquire a
valid title over it?
Answer:
A forged deed is an absolute nullity and conveys no title but it can be the root
of a title. If title to the land has been transferred to a party based upon a forged
deed, and later on after the issuance of such title the property is transferred to
another who is an innocent purchaser for value, then the latter acquires a valid title.
06; LTD; IPV; forged deed as a root of title
1985 No. 14
B) After finding on a bus an envelope containing two Torrens certificates of
title in A's name, B posing as A and forging his signature, sold the two parcels of
land described in the Titles to X who bought them in good faith and for value and to
whom transfer certificates were issued in his name. He then conveyed one parcel to
Y, a bonafide purchaser for value, while the other was levied upon to satisfy the
judgment against X.
Who has a better right to the aforementioned parcels of land, A, Y, or the
judgment creditor? Discuss.
Answer;
B) 1. I believe A retains ownership of both parcels of land. A forged deed of
sale is an absolute nullity and, therefore, conveys no title. A deed of sale executed
by an impostor has no legal force and effect. Registration of the deed and the
consequent registration and issuance of a transfer certificate of title, even to an
innocent purchaser, will not cure the infirmity.
It is also an established rule that a forged deed can be the root of a good title.
This can happen where title has been registered in the name of the forger and he
later transfers or mortgages the same to an innocent third party for value. In this
case, the third party who relied on what appears in the certificate of title should be
protected. (Duran vs. Caspar vs. I AC, Tangco, GR No. L-64159, Sept 10, 1985.
However, in the instant case, no title was ever transferred or registered in the
name of the forger. The rule discussed above does not apply. Therefore, A never
lost his right and can recover the land from Y and the judgment creditor.
2. Y has a better right than A over the parcel of land he bought from X in
good faith and for value, because Y bought the land from the registered owner X,
and therefore Y is a purchaser for value and in good faith, whose title is protected by
P.D. 1529.
3. A has a better right than the judgment creditor over the parcel of land still
registered in the name of X, because X is not a purchaser for value and in good
faith, as defined in P.D. 1529, having bought the land not from the registered owner
A but from forger B, such that X has no title thereto and the levy on execution
thereon to satisfy a judgment creditor of X has no force and effect against A.
4. Y should be protected because he is a purchaser in good faith as he
bought it from X who has a transfer certificate of title in his name. This is in
consonance with the "chain of title" doctrine.
With respect to the creditor who sought to levy the judgment against X who
was the buyer from the forger, the creditor stands in the same shoes as X who did
not acquire a better right than his vendor because he did not buy it from the
Page 223 of 391
registered owner but from a forger B. Therefore, A is entitled to the other parcel over
the judgment creditor.
06; LTD; IPV; forged document as a root of a certificate of title
1976 No. VII-a
A. is the owner of a registered parcel of land. The Torrens title is entrusted to
B, his clerk. B forges A's signature on the deed of sale of said land in his (B's) favor.
A new title is issued in the name of B. Does B have a valid title over the land?
Explain.
Answer
B does not have a valid title over the land. A, whose signature was forged,
cannot be deprived of his property inasmuch as by express provision of Section 55
of Act 496, any subsequent registration procured by a forged deed shall be null and
void While it may be true that the law gives an innocent purchaser for value some
protection, greater protection is given to the registered owner. B in this case is not
such an innocent purchaser inasmuch as he himself is the forger.
1976 No. VII-b
If B sells the property to C, does C acquire a valid title over it? Explain.
Answer
C, the subsequent buyer, acquires a valid title over the land in question. This
is a case of a void title ripening into a valid title inasmuch as C qualifies as an
innocent purchaser for value, assuming of course that he meets the three requisites,
namely:
1. That he did not know of any flaw or defect of the title of B;
2. That he paid a reasonable price; and
3. That he acquired the property from the person who stands in .the
records as the registered owner (Rivera vs. Tirona) or;
C may acquire a valid title if he is an innocent purchaser because C bought it
from the registered owner B, who has a certificate of title in his name. C need not
inquire into the validity of the title of B. The land registration certificate protects an
innocent purchaser for value who relies on a certificate of title by providing that he
takes it free from all liens and encumbrances not noted in the title.
1976 No. VII-c
Does the delivery of the title from A to B constitute an act of negligence?
Explain.
Answer
If the clerk is charged, as part of his duty, with the filing and keeping of
documents safely, then the entrusting of that document to him in the course of his
normal duties is not an act of negligence. However, for purposes of recovering from
the Assurance Fund, it is necessary that the owner must be free from any fault. Here
his loss was occasioned by a breach of trust, for which reason he would not be
entitled to compensation from the Assurance Fund,
06; LTD; IPV; forgery; innocent purchaser for value
1999 No X.
The spouses X and Y mortgaged a piece of registered land to A, delivering as
well the OCT to the latter, but they continued to possess and cultivate the land,
giving 1/2 of each harvest to A in partial payment of their loan to the latter, A,
Page 224 of 391
however, without the knowledge of X and Y, forged a deed of sale of the aforesaid
land in favor of himself, got a TCT in his name, and then told the land to B, who
bought the land relying on A's title, and who thereafter also got a TCT in his name, It
was only then that the spouses X and Y learned that their land had been titled in B's
name. May said spouses file an action for reconveyance of the land in question
against b? Reason. (5%)
ANSWER:
The action of X and Y against B for reconveyance of the land will not prosper
because B has acquired a clean title to the property being an innocent purchaser for
value.
A forged deed is an absolute nullity and conveys no title. The fact that the
forged dead was registered and a certificate of title was issued in his name, did not
operate to vest upon A ownership over the property of X and V. The registration of
the forged deed will not cure the infirmity. However, once the title to the land is
registered in the name of the forger and title to the land thereafter falls into the
hands of an innocent purchaser for value, the latter acquires a clean title thereto. A
buyer of a registered land is not required to explore beyond what the record in the
registry indicates on its face in quest for any hidden defect or inchoate right which
may subsequently defeat his right thereto. This is the "mirror principle'* of the
Torrens system which makes it possible for a forged deed to be the root of a good
title.
Besides, it appears that spouses X and Y are guilty of contributory negligence
when they delivered this OCT to the mortgagee without annotating the mortgage
thereon. Between them and the innocent purchaser for value, they should bear the
loss.
ALTERNATIVE ANSWER:
If the buyer B, who relied on the teller A's title, was not aware of the adverse
possession of the land by the spouses X and Y, then the latter cannot recover the
property from B. B has in his favor the presumption of good faith which can only be
overthrown by adequate proof of bad faith. However, nobody buys land without
seeing the property, hence, B could not have been unaware of such adverse
possession. If after learning of such possession, B simply closed his eyes and did
nothing about it, then the suit for reconveyance will prosper as the buyer's bad faith
will have become evident.
06; LTD; IPV; fraud in the procurement of patent
2000 No XII
In 1979, Nestor applied for and was granted a Free Patent over a parcel of
agricultural land with an area of 30 hectares, located in General Santos City. He
presented the Free Patent to the Register of Deeds, and he was issued a
corresponding Original Certificate of Title (OCT) No. 375, Subsequently, Nestor sold
the land to Eddie. The deed of sale was submitted to the Register of Deeds and on
the basis thereof, OCT No, 375 was cancelled and Transfer Certificate of Title (TCT)
No. 4576 was issued in the name of Eddie. In 1986, the Director of Lands filed a
complaint for annulment of OCT No, 375 and TCT No. 4576 on the ground that
Nestor obtained the Free Patent through fraud. Eddie filed a motion to dismiss on
the ground that he was an innocent purchaser for value and in good faith and as
such, he has acquired a title to the property which is valid, unassailable and
Indefeasible. Decide the motion. (5%)
SUGGESTED ANSWER:
Page 225 of 391
The motion of Nestor to dismiss the complaint for annulment of O.C.T. No.
375 and T.C.T. No. 4576 should be denied for the following reasons:
1) Eddie cannot claim protection as an innocent purchaser for value nor can
he interpose the defense of indefeasibility of his title, because his TCT is rooted on a
void title. Under Section 91 of CA No. 141, as amended, otherwise known as the
Public Land Act, statements of material facts in the applications for public land must
be under oath. Section 91 of the same act provides that such statements shall be
considered as essential conditions and parts of the concession, title, or permit
issued, any false statement therein, or omission of facts shall ipso facto produce the
cancellation of the concession. The patent issued to Nestor in this case is void ab
initio not only because it was obtained by fraud but also because it covers 30
hectares which is far beyond the maximum of 24 hectares provided by the free
patent law.
2) The government can seek annulment of the original and transfer
certificates of title and the reversion of the land to the state. Eddie's defense is
untenable. The protection afforded by the Torrens System to an innocent purchaser
for value can be availed of only if the land has been titled thru judicial proceedings
where the issue of fraud becomes academic after the lapse of one (1) year from the
issuance of the decree of registration. In public land grants, the action of the
government to annul a title fraudulently obtained does not prescribe such action and
will not be barred by the transfer of the title to an innocent purchaser for value.
06; LTD; IPV; free patents; jurisdiction of Director of Lands
1978 No. XI-a
Free Patent No. V-347 was issued in favor of A for the land in question.
Subsequently, Original Certificate of Title No. P-2507 was issued in A's name on
June 15, 1976. On July 30, 1977, the Republic of the Philippines at the instance of
the Bureau of Forestry filed a complaint to declare the free patent and the Original
Certificate of Title null and void on the ground that the land covered thereby is forest
land. Decide the case with reasons.
Answer
Any patent issued by the Director of lands to private parties is void ab initio,
for lack of jurisdiction. The indefeasibility of the Torrens title that may be issued
pursuant to such patent will not be against the State in an action for reservation of
land covered thereby when such land is a part of a public forest reservation, Neither
prescription will lie against the State in such cases in accordance with Art. 1108, par.
4 of the Civil Code. Even granting that Torrens Title can no longer be reopened
under the Land Registration Act, still the land covered thereby be reconveyed to the
State in action for reconveyance under Sec. 101 of CA 141. (RP vs. Samson
Animas, L-37682, May 29t 1974; 70 OG. 32)
Another reason. Act 496 only aim to confirm ownership. So where land
involved is not susceptible of private appropriation, no private ownership may be
confirmed.
06; LTD; IPV; government lands
1975 No. XX
If there is any error in a Torrens title in the sense that it included government
lands, who can properly question the error and how?
Answer
The State can question the error by filing an action to annul the certificate of
title, if the entire land covered by the title, is government property or, for reversion of
Page 226 of 391
property to the State (Republic v. Ananias, March 29, (1974), This remedy, of
course, is available only when the property involved is not the patrimonial property of
the state because the final decree of registration became incontrovertible, even as
against the government.
In one case where public navigable river was included in the Torrens title, the
court held that the title may be attacked collaterally or directly by the State (Martinez
v. Court of Appeals, April 29, 1976).
06; LTD; IPV; innocent purchaser for value
2001 No XVII
Cesar bought a residential condominium unit from High Rise Co. and paid the
price in full. He moved into the unit, but somehow he was not given the
Condominium Certificate of Title covering the property. Unknown to him, High Rise
Co. subsequently mortgaged the entire condominium building to Metrobank as
security for a loan of P500 million. High Rise Co. failed to pay the loan and the bank
foreclosed the mortgage. At the foreclosure sale, the bank acquired the building,
being the highest bidder. When Cesar learned about this, he filed an action to annul
the foreclosure sale insofar as his unit was concerned. The bank put up the defense
that it relied on the condominium certificates of title presented by High Rise Co.,
which were clean. Hence, it was a mortgagee and buyer in good faith. Is this
defense tenable or not? Why? (5%.)
SUGGESTED ANSWER:
Metrobank's defense is untenable. As a rule, an innocent purchaser for value
acquires a good and a clean title to the property. However, it is settled that one who
closes his eyes to facts that should put a reasonable man on guard is not an
innocent purchaser for value. In the present problem the bank is expected, as a
matter of standard operating procedure, to have conducted an ocular inspection, of
the promises before granting any loan. Apparently, Metrobank did not follow this
procedure. otherwise, it should have discovered that the condominium unit in
question was occupied by Cesar and that fact should have led it to make further
inquiry. Under the circumstances. Metrobank cannot be considered a mortgagee
and buyer in good faith.
06; LTD; IPV; innocent purchaser for value
1986 No. 19.
Through fraud, Manukso was able to obtain in his name a transfer certificate
of title over a piece of land belonging to his ward, Kamusmusan. Subsequently,
Manukso donated the property to Hinandugan, who, completely unaware of
Manukso's prior fraudulent conduct, obtained a Torrens Title in his name.
Five years after the registration in Hinandugan's name, Kamusmusan filed an
action for reconveyance. Will the action prosper? Explain,
Answer:
The Torrens system protects only an innocent purchaser for value who relies
on the certificate of title of the vendor. Hinandugan is not a purchase but a mere
donee. Hence the real owner Kamusmusan can recover the land from Hinandugan
on the ground of constructive trust, which the court ruled prescribes in 10 years.
Answer - The rule of Constructive Trust will not apply if the property is in the
hands of a third person who got the property for value and in good faith Since under
our taw, love and affection are sufficient consideration, it falls under the term "value"
because under our law "value" is not limited to material consideration. Hence,
Page 227 of 391
Hernandez is a third person within the meaning of the law. The action will not
prosper*
Answer - No. The conclusive presumption of ownership covered by a Torrens
title, cannot be defeated except if such registration is obtained by fraud. But the
fraud that was perpetrated by the guardian is not the fraud that can defeat the
collusiveness of ownership under a Torrens title. Furthermore, the fact that the
registration had subsisted for five years strengthens the indefeasibility of the title of
the registrant.
The ward can perhaps go after his guardian for the damage or loss caused to
him, but not the person who obtained the Torrens title,
06; LTD; IPV; lis pendens
1995 No, 4:
Rommel was issued a certificate of title over a parcel of land in Quezon City.
One year later Rachelle, the legitimate owner of the land, discovered the fraudulent
registration obtained by Rommel. She filed a complaint against Rommel for
reconveyance and caused the annotation of a notice of lis pendens on the certificate
of title issued to Rommel. Rommel now invokes the indefeasibility of his title
considering that one year has already elapsed from its issuance. He also seeks the
cancellation of the notice of Lis pendens.
1. Will Rachelle's suit for reconveyance prosper? Explain.
2. May the court cancel the notice of lis pendens even before final judgment
is rendered? Explain.
Answer:
1. Yes..
2. A notice of Lis pendens may be canceled even before final Judgment upon
proper showing that the notice is for the purpose of molesting or harassing the
adverse party or that the notice of lis pendens is not necessary to protect the right of
the party who caused it to be registered. (Section 77, P.D. No. 1529)
In this case, it is given that Rachelle is the legitimate owner of the land in
question. It can be said, therefore, that when she filed her notice of lis pendens her
purpose was to protect her interest in the land and not Just to molest Rommel. It is
necessary to record the Lis pendens to protect her interest because if she did not do
it, there is a possibility that the land will fall into the hands of an innocent purchaser
for value and in that event, the court loses control over the land making any
favorable judgment thereon moot and academic. For these reasons, the notice of lis
pendens may not be canceled.
06; LTD; IPV; mirror principle
1990 No 2:
In 1950's, the Government acquired a big landed estate in Central Luzon
from the registered owner for subdivision into small farms and redistribution of
bonafide occupants, F was a former lessee of a parcel of land, five hectares in area.
After completion of the resurvey and subdivision, F applied to buy the said land in
accordance with the guidelines of the implementing agency. Upon full payment of
the price in 1957, the corresponding deed of absolute sale was executed in his favor
and was registered, and in 1961, a new title was issued in his name. In 1963, F sold
the said land to X; and in 1965 X sold it to Y, New titles were successively issued in
the names of the said purchasers.
Page 228 of 391
In 1977, C filed an action to annul the deeds of sale to F,X and Y and their
titles, on the ground that he (C) had been in actual physical possession of the land,
and that the sale to F and the subsequent sales should be set aside on the ground
of fraud. Upon motion of defendants, the trial court dismissed the complaint,
upholding their defenses of their being innocent purchasers for value, prescription
and laches. Plaintiff appealed.
(a) Is the said appeal meritorious? Explain your answer
(b) Suppose the government agency concerned joined C in filing the said
action against the defendants, would that change the result of the litigation? Explain.
Answer;
(a) The appeal is not meritorious. The trial court ruled correctly in granting
defendant's motion to dismiss for the following reasons:
1. While there is the possibility that F, a former lessee of the land was aware
of the fact that C was the bona fide occupant thereof and for this reason his transfer
certificate of title may be vulnerable, the transfer of the same land and the issuance
of new TCTs to X and Y who are innocent purchasers for value render the latter's
titles indefeasible. A person dealing with registered land may safely rely on the
correctness of the certificate of title and the law will not in any way oblige him to go
behind the certificate to determine the condition of the property in search for any
hidden defect or inchoate right which may later invalidate or diminish the right to the
land. This is the mirror principle of the Torrens System of land registration.
2. The action to annul the sale was instituted in 1977 or more than (10) years
from the date of execution thereof in 1957, hence, it has long prescribed.
3. Under Sec 45 of Act 496, the entry of a certificate of title shall be regarded
as an agreement running with the land, and binding upon the applicant and all his
successors in title that the land shall be and always remain registered land. A title
under Act 496 is indefeasible and to preserve that character, the title is cleansed
anew with every transfer for value (De Jesus v City of Manila; 29 Phil. 73; Laperal v
City of Manila, 62 Phil 313; Penullar v PNB 120 S 111).
(b) Even if the government joins C, this will not alter the outcome of the case
so much because of estoppel as an express provision in Sec 45 of Act 496 and Sec
31 of PD 1529 that a decree of registration and the certificate of title issued in
pursuance thereof shall be conclusive upon and against all persons, including the
national government and all branches thereof, whether mentioned by name in the
application or not.
06; LTD; IPV; notice of lis pendens; transferee pendente lite
2002 No XI.
Sancho and Pacifico are co-owners of a parcel of land. Sancho sold the
property to Bart. Pacifico sued Sancho and Bart for annulment of the sale and
reconveyance of the property based on the fact that the sale included his one-half
pro-indiviso share. Pacifico had a notice of lis pendens annotated on the title
covering the property and ordered the cancellation of the notice of lis pendens. The
notice of lis pendens could not be cancelled immediately because the title over the
property was with a bank to which the property had been mortgaged by Bart.
Pacifico appealed the case. While the appeal was pending and with the notice of lis
pendens still uncancelled, Bart sold the property to Carlos, who immediately caused
the cancellation of the notice of lis pendens, as well as the issuance of a new title in
his name.
Page 229 of 391
Is Carlos (a) a purchaser in good faith, or (b) a transferee pendente life? If
your answer is (a), how can the right of Pacifico as co-owner be protected? Explain.
(5%)
SUGGESTED ANSWER:
A. Carlos is a buyer in bad faith. The notice of lis pendens was still
annotated at the back of the title at the time he bought the land from Bart. The
uncancelled notice of lis pendens operates as constructive notice of its contents as
well as interests, legal or equitable, included therein. All persons are charged with
the knowledge of what it contains.
In an earlier case, it was held that a notice of an adverse claim remains
effective and biding notwithstanding the lapse of the 30 days from its inscription in
the registry. This ruling is even more applicable in a lis pendens.
Carlos is a transferee pendente lite insofar as Sanchos share in the co-
ownership in the land is concerned because the land was transferred to him during
the pendency of the appeal.
A. Pacifico can protect his right as a co-owner by pursuing his appeal;
asking the Court of Appeals to order the re-annotation of the lis pendens on the title
of Carlos; and by invoking his right of redemption of Barts share under Articles 1620
of the New Civil Code.
ALTERNATIVE ANSWER:
A. Carlos is a purchaser in good faith.
A possessor in good faith has been defined as one who is unaware that
there exists a flaw which invalidates his acquisition of the thing (Art. 526, NCC).
Good faith consists in the possessors belief that the person from whom he received
the thing was the owner of the same and could convey his title. In the case [at bar],
in question, while Carlos bought the subject property from Bart while a notice of lis
pendens was still annotated thereon, there was also an existing court order
canceling the same. Hence, Carlos cannot be considered as being aware of a flaw
which invalidates [their] the acquisition of the thing since the alleged flaw, the notice
of lis pendens, was already being ordered cancelled at the time of the purchase. On
this ground alone, Carlos can already be considered a buyer in good faith. (Po Lam
v. Court of Appeals, 347 SCRA 86, [2000]).
B. To protect his right over the subject property, Pacifico should have
timely filed an action for reconveyance and reinstated the notice of lis pendens.
06; LTD; IPV; reconveyance
1995 No, 4:
Rommel was issued a certificate of title over a parcel of land in Quezon City.
One year later Rachelle, the legitimate owner of the land, discovered the fraudulent
registration obtained by Rommel. She filed a complaint against Rommel for
reconveyance and caused the annotation of a notice of lis pendens on the certificate
of title issued to Rommel. Rommel now invokes the indefeasibility of his title
considering that one year has already elapsed from its issuance. He also seeks the
cancellation of the notice of Lis pendens.
1. Will Rachelle's suit for reconveyance prosper? Explain.
Answer:
1. Yes, Rachelle's suit will prosper because all elements for an action for
reconveyance are present, namely:
a. Rachelle is claiming dominical rights over the same land.
Page 230 of 391
b. Rommel procured his title to the land by fraud.
c. The action was brought within the statutory period of four (4) years from
discovery of the fraud and not later than ten (10} years from the date of registration
of Rommel's title.
d. Title to the land has not passed into the hands of an innocent purchaser
for value.
Rommel can invoke the indefeasibility of his title if Rachelle had filed a
petition to reopen or review the decree of registration. But Rachelle instead filed an
ordinary action in personam for reconveyance. In the latter action, indefeasibility is
not a valid defense because, in filing such action, Rachelle is not seeking to nullify
nor to impugn the indefeasibility of Rommel's title. She is only asking the court to
compel Rommel to reconvey the title to her as the legitimate owner of the land.
Alternative Answer;
Yes. The property registered is deemed to be held in trust for the real owner
by the person in whose name it is registered. The Torrens system was not designed
to shield one who had committed fraud or misrepresentation and thus holds the title
in bad faith. (Walstrom v. Mapa Jr., (G .R 38387, 29 Jan. 1990) as cited in Martinez,
D., Summary of SC Decisions, January to June, 1990, p. 359],
06; LTD; IPV; registration as the operative act of conveying the land
1984 No. 19
A had a piece of land which was registered under the Torrens system. He
sold it to B, but the sale was not registered Subsequently, C, a creditor of A,
obtained an attachment on the property, which was duly registered with the Registry
of Deeds. B thereafter attempted to register the sale in his favor. The Register of
Deeds was willing to do so and issue a transfer certificate of title to B, subject
however, to the condition that the attachment in favor of C would be inscribed on the
new certificate of title. B objected, contending that at the time of the attachment of
the property, A was no longer the owner thereof and therefore had no more interest
therein which could be attached.
Is B's posture valid? Explain.
Answer: A. Furnished by Office of Justice Palma
No. Section 51 of the Property Registration Decree provides that the
execution of a deed shall not take effect as a conveyance, or bind the land, except
as between the parties. The act of registration shall be the operative act to convey or
affect the land as far as third parties are concerned. Hence, since C has secured a
lien by attachment prior to registration, the rights acquired by B under the deed shall
be subject to the rights of C and cannot be enforced until after the rights of C have
been fully satisfied.
B. Comments and Suggested Answer
We agree with the answer of the Bar Examiner,
06; LTD; IPV; remedies; prescriptive period
2003 No XX.
Louie, before leaving the country to train as a chef in a five-star hotel in New
York, U.S.A., entrusted to his first-degree cousin Dewey an application for
registration, under the Land Registration Act, of a parcel of land located in Bacolod
City. A year later, Louie returned to the Philippines and discovered that Dewey
registered the land and obtained an Original Certificate of Title over the property in
his Deweys name. Compounding the matter, Dewey sold the land to Huey, an
Page 231 of 391
innocent purchaser for value. Louie promptly filed an action for reconveyance of the
parcel of land against Huey.
(a) Is the action pursued by Louie the property remedy?
(b) Assuming that reconveyance is the proper remedy, will the action
prosper if the case was filed beyond one year, but within ten years, from the entry of
the decree of registration? 5%
SUGGESTED ANSWER:
(a) An action for reconveyance against Huey is not the proper remedy,
because Huey is an innocent purchaser for value. The proper recourse is for Louie
to go after Dewey for damages by reason of the fraudulent registration and
subsequent sale of the land. If Dewey is insolvent, Louie may file a claim against the
Assurance Fund (Heirs of Pedro Lopez v. De Castro 324 SCRA 591 [2000] citing
Sps. Eduarte v. CA, 323 Phil. 462, 467 [1996]).
(b) Yes, the remedy will prosper because the action prescribes in ten (10)
years, not within one (1) year when a petition for the reopening of the registration
decree may be filed. The action for reconveyance is distinct from the petition to
reopen the decree of registration (Grey Alba v. De la Cruz, 17 Phil. 49 [1910}).
There is no need to reopen the registration proceedings, but the property should just
be reconveyed to the real owner.
The action for reconveyance is based on implied or constructed trust, which
prescribes in ten (10) years from the date of issuance of the original certificate of
title. This rule assumes that the defendant is in possession of the land. Where it is
the plaintiff who is in possession of the land, the action for reconveyance would be in
the nature of a suit for quieting for the title which action is imprescriptible (David v.
Malay, 318 SCRA 711 [1999]).
06; LTD; IPV; remedies; reconveyance
1984 No. 18
More than one year had elapsed since the issuance of the final decree of
registration when A discovered that his land had been fraudulently registered in the
name of his caretaker B.
What right of action, if any, does A have and against whom? Explain.
Answer:
A. Furnished by Office of Justice Palma,
The only remedy of A is to bring an action for reconveyance against B, or for
damages if the property has passed into the hands of an innocent purchaser for
value. Such a complaint would not seek a review of the decree or the reopening of
the registration case which is already incontrovertible, but be for the enforcement of
a trust. Section 96 of PD No. 1529 provides that nothing in the Act shall be
construed to deprive any party of any action which he may have against any person
for loss or damage or deprivation of land or any interest therein. It may also be
stated that an action by A against B to compel him to convey the property held in B's
name to A as the beneficiary, does not prescribe. (Caladiao, et. al. vs. Blas, 119 Phil
969).
B. Comments and Suggested Answer
We suggest that the following should be accepted as correct answer:
Answer No. 1: The only remedy of A is to bring an action for reconveyance
against B, or for damages if the property has passed to an innocent purchaser for
value (Sec. 96, PD 1529). Such an action would not seek a review of the decree or
Page 232 of 391
the reopening of the registration case which is already incontrovertible. Instead, it
will be an action for the enforcement of a trust. However, A must institute the action
within ten years to be counted from the discovery of the fraud. The fraud is deemed
to have been discovered at the moment B set up a title in himself adverse to the title
of A.
(Note: The doctrine that the action shall prescribe and that the period of
prescription is ten years is supported by: Buencamino vs. Matias, 16 SCR A 849;
Araneta vs. Perez, 17 SCR A 643; Pascual vs. Meneses,. 20 SCRA 219; Julio vs.
Dalandan, 21 SCRA 643; Cuaycong vs. Cuaycong, 21 SCRA 1192; Fabian vs,
Fabian, 22 SCRA 231; Bueno vs. Reyes, 27 SCRA 1179; Dela Cerna vs. Dela
Cerna 72 SCRA 515; Jaramil vs. CA, 78 SCRA 420; Duque vs. Domingo, 80 SCRA
654; Nacalaban vs. CA, 80 SCRA 428).
Answer No. 2: The only remedy of A is to bring an action for reconveyance
based on the implied or constructive trust recognized in the Civil Code (Art. 1456,
Civil Code) According to the Civil Code, if property is acquired through mistake or
fraud, the person obtaining it is, by force of law, considered a trustee for the benefit
of the person from whom the property comes. It is, however, well-settled that the
action shall prescribe after ten years from the discovery of the fraud. It is also well-
settled that the fraud is deemed to have been discovered at the moment the trustee
set up a title in himself adverse to the title of the beneficiary.
06; LTD; IPV; reopening of a decree of registration
1992 No 11:
What are the essential requisites or elements for the allowance of the
reopening or review of a decree of registration?
Answer;
The essential elements are: (1) that the petitioner has a real or dominical
right; (2) that he has been deprived thereof through fraud; (3) that the petition is filed
within one (1) year from the issuance of the decree; and (4) that the property has not
yet been transferred to an innocent purchaser {Rublico vs. Orellana 30 SCRA 511;
Ubudan vs. Gil 45 SCRA 17).
Optional extended answer:
Petition for review of the Decree of Registration. A remedy expressly
provided in Section 32 of P. D. No. 1529 (formerly Section 38. Act 496), this remedy
has the following elements:
a. The petition must be filed by a person claiming dominical or other real
rights to the land registered in the name of respondent.
b. The registration of the land in the name of respondent was procured by
means of actual, (not just constructive) fraud, which must be extrinsic. Fraud is
actual if the registration was made through deceit or any other intentional act of
downright dishonesty to enrich oneself at the expense of another. It is extrinsic when
it Is something that was not raised, litigated and passed upon in the main
proceedings.
c. The petition must be filed within one (1) year from the date of the
issuance of the decree.
d. Title to the land has not passed to an Innocent purchaser for value
(Libudan vs. Gil, 45_ SCRA 27, 1972), Rublico vs. Orrelana. 30 SCRA 511, 1969);
RP vs. CA, 57 G. R No. 40402. March 16, 1987).
1. The buyer In good faith of a registered parcel of land does not have to look
beyond the Torrens title in search for any hidden defect or inchoate right which may
Page 233 of 391
later invalidate or diminish his right to what he purchased [Lopez vs. CA, G. R.
49739. January 20, 1989].
06; LTD; IPV; reopening of a decree of registration
1975 No. XIX
What are the grounds justifying the review of a decree of registration?
Answer
The only ground justifying a review of a decree of registration is fraud, which
must be actual and extrinsic.
To be considered actual fraud, there must have been an intentional
concealment or omission of a fact required by law to be stated in the application or a
willful statement of a claim against the truth, either of which is calculated to deceive
or deprive another of his legal rights.
Extrinsic fraud is employed to deprive a party of his day in court, thereby
preventing him from asserting his right to the property registered in the name of the
applicant (Cruz v. Navarro, 54 SCRA 109).
06; LTD; IPV; reopening of a decree of registration
1985 No. 14
A) Within one year from the issuance of the decree of registration and
certificate of title in A's name, B brought an action for their annulment in the
Regional Trial Court on the ground that A obtained them thru fraud. In his answer
to the complaint, A alleged that the court had no jurisdiction over the case and
averred, by way of counterclaim, that he has just discovered that B succeeded 3
years ago in registering in his name, by false and fraudulent representations,
another parcel of land owned and possessed by A. He, therefore, prayed that B's
action be dismissed and that he be ordered to transfer his title to him (A). The trial
court dismissed both the complaint and counterclaim.
Comment on the legality of the order of dismissal.
Answer;
A) 1. Under Section 32, P.B. 1529, a person who owns a piece of land or any
interest therein that was fraudulently registered in another's name is given the right
to file a petition to review or reopen the decree of registration not later than one year
from the date of entry thereof. This remedy will not lie, however, if title to the land
has been transferred to an innocent purchaser for value.
The case at hand is denominated as an action for annulment of both the
decree and the title, based on fraud. If we treat this case as an ordinary civil action
that was filed with the RTC in the latter's capacity as a court of general jurisdiction, I
believe that the Court acted correctly in dismissing it, because the proper remedy of
the aggrieved party would be a petition for review of the decree which must be filed
with the RTC, in its capacity as a land registration court. The reason for this is that
the case, in essence, petition for review, a mere continuation of the original
proceedings, over which the RTC sitting as a land registration court has exclusive
jurisdiction. So, the RTC acted correctly in dismissing the action for lack of
jurisdiction.
On the other hand, if the case was filed with the RTC in its capacity as a land
registration court, I believe the dismissal was in error, because in that capacity it has
I in fact exclusive jurisdiction to hear and decide what in essence is a petition for
review of the decree. I think mere error in nomenclature of the pleading should not
be a ground for dismissal.
Page 234 of 391
But as a land registration court, the RTC acted correctly in dismissing the
counterclaim of B because in that capacity it enjoys only special and limited
jurisdiction and, therefore, it can not take cognizance thereof. The counterclaim for
reconveyance of another parcel of land is an action in personam which falls properly
within the competence of ordinary civil courts.
2. If the action were brought under the same land registration proceedings
that issued the decree, in which event the Regional Trial Court would be acting as a
land registration court with special and limited jurisdiction__
a. the dismissal of the complaint is error, for the Regional Trial Court, as a
land registration court, has jurisdiction there over;
b. the dismissal of the counterclaim is proper, because the same is in the
nature of a permissive counterclaim, which is essentially an independent ordinary
civil action, over which the Regional Trial Court, acting as a land registration
court, has no jurisdiction.
3. If the action were merely an ordinary civil action
a. the dismissal of the complaint is proper, because jurisdiction to annul a
decree, brought within one (1) year from its issuance, properly belongs to the
Regional Trial Court which issued the decree, acting as a land registration court;
b. the dismissal of the counterclaim is error, because the Regional Trial
Court, as a court of general jurisdiction, has jurisdiction there over, and since the
counterclaim is in the nature of a permissive counterclaim, it can proceed
independently and regardless of the dismissal of the complaint.
4. As to the complaint, since it is brought for annulment of the certificate of
title on the ground of fraud, a petition for review should be filed in the same
registration proceeding within one year from the issuance of the decree on the
ground of extrinsic fraud.
Since it is brought within one year, dismissal of the action is not proper
provided the petitioner proves extrinsic fraud With regard to the counterclaim, this
should not be dismissed because it is a permissive counterclaim.
6. The counterclaim was validity dismissed, since it should have been
brought in a separate action. The action for annulment was not validly dismissed,
having been brought within the one-year period provided in P.D. 1529. It is, of
course, understood that the action was filed in the Regional Trial Court which
granted the decree of registration, since the facts in the question do not state
otherwise.
6. The Regional Trial Court has no jurisdiction over the action for annulment.
Within one year from the issuance of the decree of registration, the proper remedy
would be an action for a review of the decree by the Court in its capacity as a Land
Registration Court. The order, therefore, of .dismissal of the complaint was correctly
issued by the Court.
The counterclaim, however, should not have been dismissed, since it partook
of the nature of an action for reconveyance which can be considered by the RTC.
06; LTD; IPV; reopening of decree of registration
1977 No. XVIII-b
Under what circumstances may a decree of registration under Act No, 496 be
reviewed?
Answer
Page 235 of 391
The circumstances under which a decree of registration under Act 496 may
be reviewed are:
(1) That the petitioner is the owner of the land ordered registered in the
name of another, or that his lien or interest therein does not appear in the decree or
title issued;
(2) that the registration was procured through actual fraud, or that the
omission of the lien or interest was fraudulent;
(3) that the property has not yet been transferred to an innocent purchaser for
value; and
(4) that the action is filed within one year from the issuance and entry of the
decree of registration. (Cruz vs. Navarro, G.R. No. L-27644, Nov. 29, 1973.)
06; LTD; IPV; reopening of decree of registration
1979 No. XIX
X applied for the registration of a 20-hectare piece of land. During the
pendency of the registration proceedings, X sold the land to Y for P100,000.00, its
then fair market value, and presented the Deed of Sale to the court and testified
therein as to the due execution of the same. The registration court therefore
rendered a decision ordering the registration of the land in the name of Y. After said
decision became final, the land court, upon motion of X, issued a decree on the
basis of which an Original Certificate of Title was issued in the name of Y. Alleging
that the title should not have been issued in the name of Y and that there was fraud
because Y failed to pay him the price of the sale, X moved for reconsideration of the
decree ten (10) months after the issuance of the title. Should the motion be granted?
Why?
Answer
The motion should not be granted. It is well-settled that a decree of
registration may be set aside only on the ground of fraud in obtaining the same and
not on the ground of failure to pay the purchase price. X, therefore, cannot complain
of fraud in obtaining the decree of registration It was solely upon his testimony and
proof that the land was registered and solely upon his motion that the decree was
issued. (Mendoza vs. CA, 84 SCRA 67), The issue of breach of contract by Y
should, therefore, be threshed out in the ordinary courts.
Question No. XX
OP filed in the Office of the Register of Deeds an Affidavit of Adverse Claim
wherein he claims ownership of a parcel of land described in the Title issued in the
name of JG. The affidavit states that the sale of the land to JG was void because the
latter is a Chinese; that OP has been in actual continuous, public, exclusive and
uninterrupted possession of the lot for more than ten years and that as a matter of
fact he had constructed a house thereon; that no one has claimed from his
ownership nor possession nor rental for his occupancy and that therefore, he had
acquired the land by prescription; and that whatever right the Chinese registered
owner had over the property had prescribed. The Register of Deeds refused to
record the adverse claim on the title of JG. Is this refusal correct? Why?
Answer
The refusal of the Register of Deeds to register the adverse claim of OP on
the title of JG, is correct. It is, of course, true that generally, under our Torrens
system, the duties of a Register of Deeds are material in character, but then, an
adverse claim of ownership based upon prescription and adverse possession cannot
be registered because under the Land Registration Law, no title in derogation of that
Page 236 of 391
of the registered owner may be acquired by prescription or adverse possession.
Hence, the registration of such adverse possession. Hence, the registration of such
adverse claim will serve no purpose and cannot validly and legally affect the land in
question. (Estella vs. Register of Deeds, 106 Phil. 911).
06; LTD; IPV; sale of unregistered lands
1977 No. XVIII-a
A and B sold to C an unregistered lot; the deed was not registered, but C took
possession. Later, D obtained a judgment against A and B and the lot in C's
possession was levied upon and sold by the Sheriff to D. The Sheriff's sale was
registered. All the parties acted in good faith. Who has better rights to the land? Give
reasons.
Answer
C has better rights to the land. It is now well-settled that the rule on double
sales in Art. 1544 of the Civil Code applies only to lands covered by a Torrens title.
Where the land is not registered under Act No. 496, the rule is different. The reason
is that the purchased of unregistered land at a sheriff's sale only steps into the shoes
of the judgment debtor, and merely acquires the latter's interest in the property as of
the time the property was levied upon, as provided in Sec 35 of Rule 39 of the Rules
of Court. (Carumba vs. Court of Appeals. 31 SCRA 558.)
06; LTD; IPV; unregistered land
1991 No 12:
Maria Enriquez failed to pay the realty taxes on her unregistered agricultural
land located in Magdugo, Toledo City. In 1989, to satisfy the taxes due, the City sold
it at public auction to Juan Miranda, an employee at the Treasurer's Office of said
City, whose bid at P10,000.00 was the highest. In due time, a final bill of sale was
executed in his favor.
Maria refused to turn-over the possession of the property to Juan alleging
that (1) she had been, in the meantime, granted a free patent and on the basis
thereof an Original Certificate of Title was issued to her, and (2) the sale in favor of
Juan is void from the beginning in view of the provision in the Administrative Code of
1987 which prohibits officers and employees of the government from purchasing
directly or indirectly any property sold by the government for nonpayment of any tax.
fee or other public charge.
(a) Is the sale to Juan valid? If so, what is the effect of the Issuance of the
Certificate of Title to Maria?
(b) If the sale is void, may Juan recover the P10,000.00? If not, why not?
(c) If the sale is void, did it not nevertheless, operate to divert Maria of her
ownership? If it did, who then is the owner of the property?
Answer:
A. The sale of the land to Juan is not valid, being contrary to law. Therefore,
no transfer of ownership of the land was effected from the delinquent taxpayer to
him. The original certificates of title obtained by Maria thru a free patent grant from
the Bureau of Lands under Chapter VII, CA 141) is valid but in view of her
delinquency, the said title Is subject to the right of the City Government to sell the
land at public auction. The issuance of the OCT did not exempt the land from the tax
sales. Section 44 of P.O. No. 1529 provides that every registered owner receiving a
Certificate of Title shall hold the same free from an encumbrances, subject to certain
exemptions.
Page 237 of 391
B. Juan may recover because he was not a party to the violation of the law.
C. No, the sale did not divest Maria of her title precisely because the sale is
void. It is as good as if no sale ever took place.
In tax sales, the owner is divested of his land initially upon award and
issuance of a Certificate of Sale, and finally after the lapse of the 1 year period from
date of registration, to redeem, upon execution By the treasurer of an instrument
sufficient in form and effects to convey the property. Maria remained owner of the
land until another tax sale is to be performed in favor of a qualified buyer.
06; LTD; IPV; use of fraud in aplication of title; prescriptive period for
reconveyance
1997 No. 20:
On 10 September 1965, Melvin applied for a free patent covering two lots -
Lot A and Lot B - situated in Santiago, Isabela. Upon certification by the Public Land
Inspector that Melvin had been in actual, continuous, open, notorious, exclusive and
adverse possession of the lots since 1925. the Director of Land approved Melvin's
application on 04 June 1967. On 26 December 1967, Original Certificate of Title
(OCT) No. P-2277 was Issued in the name of Melvln.
On 7 September 1971, Percival filed a protest alleging that Lot B which he
had been occupying and cultivating since 1947 was included in the Free Patent
issued in the name of Melvin. The Director of Lands ordered the investigation of
Percival's protest. The Special Investigator who conducted the investigation found
that Percival had been in actual cultivation of Lot B since 1947.
On 28 November 1986, the Solicitor General filed in behalf of the Republic of
the Philippines a complaint for cancellation of the free patent and the OCT issued in
the name of Melvin and the reversion of the land to public domain on the ground of
fraud and misrepresentation In obtaining the free patent. On the same date, Percival
sued Martin for the reconveyance of Lot B.
Melvin filed his answers interposing the sole defense in both cases that the
Certificate of Title issued in his name became incontrovertible and indefeasible upon
the lapse of one year from the issuance of the free patent.
Given the circumstances, can the action of the Solicitor General and the case
for reconveyance filed by Percival possibly prosper?
Answer:
"If fraud be discovered in the application which led to the Issuance of the
patent and Certificate of Title, this Title becomes ipso facto null and void. Thus. In a
case where a person who obtained a free patent, knowingly made a false statement
of material and essential facts in his application for the same, by stating therein that
the lot in question was part of the public domain not occupied or claimed by any
other person, his title becomes ipso facto canceled and consequently rendered null
and void."
"It is to the public Interest that one who succeeds In fraudulently acquiring
title to public land should not be allowed to benefit therefrom and the State, through
the Solicitor General, may file the corresponding action for annulment of the patent
and the reversion of the land involved to the public domain" (Dinero us. Director of
Lands; Kayaban vs. Republic L-33307,8-20-73; Director of Lands us. Hon. Pedro
Samson Animas, L-37682, 3-29-74.)
This action does not prescribe.
With respect to Percival's action for reconveyance, it would have prescribed,
having been filed more than ten (10) years after registration and Issuance of an
Page 238 of 391
O.C.T. in the name of Melvin, were it not for the inherent infirmity of the latter's title.
Under the facts, the statute of limitations will not apply to Percival because Melvin
knew that a part of the land covered by his title actually belonged to Percival. So,
instead of nullifying in toto the title of Melvin, the court, in the exercise of equity and
jurisdiction, may grant prayer for the reconveyance of Lot B to Percival who has
actually possessed the land under a claim of ownership since 1947. After all, if
Melvin's title is declared void ab initio and the land is reverted to the public domain,
Percival would just the same be entitled to preference right to acquire the land from
the government. Besides, well settled is the rule that once public land has been in
open, continuous, exclusive and notorious possession under a bonafide claim of
acquisition of ownership for the period prescribed by Section 48 of the Public Land
Act, the same ipso jure ceases to be public and in contemplation of law acquired the
character of private land. Thus, reconveyance of the land from Melvin to Percival
would be the better procedure, (Vitale vs. Anore, 90 Phil. 855; Pena, Land Titles and
Deeds, 1982, Page 427)
Alternative Answer;
The action of the Solicitor General should prosper, considering that the
doctrine of indefeasibility of title does not apply to free patent secured through fraud.
A certificate of title cannot be used as shield to perpetuate fraud. The State is not
bound by the period of prescription stated in Sec. 38 of Act 496. (Director of Lands
vs. Abanilla, 124SCRA358)
The action for reconveyance filed by Percival may still prosper provided that
the property has not passed to an innocent third party for value (Dablo us. Court of
Appeals. 226 SCRA 618), and provided that the action is filed within the prescriptive
period of ten years (Tale vs. Court of Appeals. 208 SCRA 266). Since the action was
filed by Percival 19 years after the issuance of Melvin's title, it is submitted that the
same is already barred by prescription.
Alternative Answer (to second part of question)
The action for reconveyance filed by Percival will prosper, because the land
has ceased to be public land and has become private land by open, continuous,
public, exclusive possession under a bona fide claim of ownership for more than
thirty years, and Percival is still in possession of the property at present. His action
for reconveyance can be considered as an action to quiet title, which does not
prescribe if the plaintiff is in possession of the property. (Olviga v. CA. GR 1048013.
October 21, 1993)
06; LTD; judicial confirmation of imperfect title
1993 No. 20:
On June 30, 1986. A filed in the RTC of Abra an application for registration of
title to a parcel of land under P. D. No. 1529, claiming that since June 12. 1945. he
has been in open, continuous, exclusive and notorious possession and occupation
of said parcel of land of the public domain which was alienable and disposable,
under a bona fide claim of ownership. After issuance of the notice of initial hearing
and publication, as required by law. the petition was heard on July 29, 1987. On the
day of the hearing nobody but the applicant appeared. Neither was there anyone
who opposed the application. Thereupon, on motion of the applicant, the RTC
issued an order of general default and allowed the applicant to present his evidence.
That he did. On September 30. 1989. the RTC dismissed A's application for lack of
sufficient evidence. A appealed to the Court of Appeals.
The appellant urged that the RTC erred in dismissing his application for
registration and in not ordering registration of his title to the parcel of land in
Page 239 of 391
question despite the fact that there was no opposition filed by anybody to his
application.
Did the RTC commit the error attributed to it?
Answer;
No, the RTC did not commit the error attributed to it. In an application for
Judicial confirmation of imperfect or incomplete title to public agricultural land under
Section 48 of the Public Land Act, the lack of opposition and the consequent order
of default against those who did not answer or show up on the date of initial hearing,
does not guarantee the success of the application. It Is still incumbent upon the
applicant to prove with well nigh incontrovertible evidence that he has acquired a
title to the land that is fit for registration. Absent such registrable title, it is the clear
duty of the Land Registration Court to dismiss the application and declare the land
as public land.
An application for land registration is a proceeding in rem. Its main objective
is to establish the status of the res whether ft is still part of our public domain as
presumed under the Regalian doctrine or has acquired the character of a private
property. It is the duty of the applicant to overcome that presumption with sufficient
evidence.
06; LTD; judicial reconstitution of title
1996 No. 17;
In 1989, the heirs of Gavino, who died on August 10, 1987, filed a petition for
reconstitution of his lost or destroyed Torrens Title to a parcel of land in Ermita,
Manila. This was opposed by Marilou. who claimed ownership of the said land by a
series of sales. She claimed that Gavino had sold the property to Bernardo way
back in 1941. and as evidence thereof, she presented a Tax Declaration in 1948 in
the name of Bernardo, which cancelled the previous Tax Declaration in the name of
Gavino. Then she presented two deeds of sale duly registered with the Register of
Deeds, the first one executed by Bernardo in 1954. selling the same property to
Carlos, and the second one executed by Carlos in 1963, selling the same property
to her. She also claimed that she and her predecessors in Interest have been in
possession of the property since 1948.
If you were the judge, how will you decide the petition? Explain.
Answer;
If I were the judge. I will give due course to the petition of the heirs of Gavino
despite the opposition of Marilou for the following reasons:
Judicial reconstitution of a certificate of title under RA. No. 26 partakes of a
land registration proceeding and is perforce a proceeding in rem. It denotes
restoration of an existing Instrument which has been lost or destroyed in its original
form and condition. The purpose of reconstitution of title or any document is to have
the same reproduced, after proceedings. In the same form they were when the loss
or destruction occurred.
If the Court goes beyond that purpose, It acts without or in excess of
jurisdiction. Thus, where the Torrens Title sought to be reconstituted is in the name
of Gavino. the court cannot receive evidence proving that Marilou is the owner of the
land. Marilou's dominical claim to the land should be ventilated in a separate civil
action before the Regional Trial Court in its capacity as a court of general
jurisdiction.
References: Heirs of Pedro Pinate vs. Dulay. 187 SCRA 12-20 (1990);
Bunagan vs. CF1 Cebu Branch VI. 97 SCRA 72 (1980); Republic vs. IAC. 157
Page 240 of 391
SCRA 62,66 (1988); Margolles vs. CA, 230 SCRA 709; Republic us, Feliciano, 148
SCRA 924.
06; LTD; ministerial duty of the Register of deeds to register
1985 No. 14
C) The register of deeds refused to record a deed of sale executed in favor
of a Filipino woman on the ground that she is an alien because her husband is an
alien and although she secured an absolute divorce from him abroad, the divorce
is void since our law, which governs her status, does not recognize absolute
divorce.
Rule on the legality of the register of deeds' refusal to register.
Answer;
C) 1. I believe the Register of Deeds erred in refusing to register the same for
the reason stated in the question.
When all the formal requisites for registration are presented, it is the duty of
the RD to effect the registration. And this duty is clearly ministerial and mandatory in
character. The main purpose of registration is merely to give notice to the public,
either actually or constructively. In one case, the Supreme Court ruled that
suspected invalidity of the contract is not a valid ground to refuse registration.
In the instant case, unlike in Krivenko where vendee was admittedly an alien,
the vendee is a Filipino. The only reason the RD refused to register is because she
was married to an alien. Aside from the fact that they have divorced, it is not even
certain or clear that she lost her Filipino citizenship by reason alone of her marriage.
This depends upon the laws of her husband's country. The question whether she
lost Filipino citizenship, under these facts, should be decided after registration.
2. The refusal to register by the Register of Deeds is improper, because
under the 1973 Constitution, a Filipino woman who marries an alien remains a
Filipino citizen; however, the certificate of title that would be issued in favor of this
Filipino woman should expressly state that the subject land is her exclusive
paraphernal property, for without such statement the land would be considered
conjugal property, over which the alien husband is entitled to a share of 1/2 pro
indiviso, in which event there would be a violation of the Constitution as 1/2 of the
subject land would in effect be owned by an alien.
3. It depends when the Filipino woman acquired the property.
Under the 1935 Constitution, a woman, follows the citizenship of the husband
provided she is qualified. Therefore, she followed the citizenship of her husband.
She is an alien and therefore the sale to her is not valid.
But if it was under the 1973 Constitution, the Filipino wife does not follow the
nationality of the husband. Therefore, the sale is perfectly valid.
4. It is ministerial if the defect does not appear on the face of the document,
but if it appears on the face of the document, then he can refuse,
5. It is not within the authority of the Register of Deeds to question
citizenship. As long as the documents are in order, it is his ministerial duly to record
the deed. What the Register of Deeds could have done was to refer the question to
the Land Registration Commissioner "en consulta",
6. The refusal by the Register of Deeds to record the sale is not warranted.
The marriage of the Filipino woman to a foreigner does not result in her loss of
Philippine citizenship. The matter of her subsequent divorce, whether valid or
invalid, is immaterial.
Page 241 of 391
06; LTD; procedure; consulta
1994 No. 3;
1) What is the procedure of consulta when an instrument is denied
registration?
Alternative Answers;
1) a) (1) The Register of Deeds shall notify the interested party in writing,
setting forth the defects of the Instrument or the legal ground relied upon for denying
the registration, and advising that if he Is not agreeable to such ruling, he may,
without with drawing the documents from the Registry, elevate the matter by
Consulta to the Administrator of the Land Registration Authority (LRA).
(2) Within five {5) days from receipt of notice of denial, the party in Interest
shall file his Consulta with the Register of Deeds concerned and pay the consulta
fee.
(3) After receipt of the Consulta and payment of the corresponding fee. the
Register of Deeds makes an annotation of the pending consulta at the back of the
certificate of title.
(4) The Register of Deeds then elevates the case to the LRA Administrator
with certified records thereof and a summary of the facts and issues involved.
(5) The LRA" Administrator then conducts hearings after due notice or may
just require parties to submit their memoranda.
(6) After hearing, the LRA Administrator issues an order prescribing the step
to be taken or the memorandum to be made. His resolution in consulta shall be
conclusive and binding upon all Registers of Deeds unless reversed on appeal by
the Court of Appeals or by the Supreme Court. (Section 117, P.D. 1529).
b) The procedure of consulta is a mode of appeal from denial by the Register
of Deeds of the registration of the instrument to the Commissioner of Land
Registration.
c) Within five days from receipt of the notice of denial, the interested party
may elevate the matter by consulta to the Commissioner of Land Registration who
shall enter an order prescribing the step to be taken or memorandum to be made.
Resolution in consulta shall be binding upon all Registers of Deeds provided that the
party in interest may appeal to the Court of Appeals within the period prescribed
(Sec. 117, P.D. 1529).
06; LTD; reclamation of foreshore lands
2000 No VIII
a) Republic Act 1899 authorizes municipalities and chartered cities to reclaim
foreshore lands bordering them and to construct thereon adequate docking and
harbor facilities. Pursuant thereto, the City of Cavite entered into an agreement with
the Fil-Estate Realty Company, authorizing the latter to reclaim 300 hectares of land
from the sea bordering the city, with 30% of the land to be reclaimed to be owned by
Fil-Estate as compensation for its services. The Solicitor General questioned the
validity of the agreement on the ground that it will mean reclaiming land under the
sea which is beyond the commerce of man. The City replies that this is authorized
by RA. 1899 because it authorizes the construction of docks and harbors. Who is
correct? (3%)
SUGGESTED ANSWER:
The Solicitor General is correct. The authority of the City of Cavite under RA
1899 to reclaim land is limited to foreshore lands. The Act did not authorize it to
Page 242 of 391
reclaim land from the sea. "The reclamation being unauthorized, the City of Cavite
did not acquire ownership over the reclaimed land. Not being the owner, it could not
have conveyed any portion thereof to the contractor.
Alternative Answer;
It depends. If the reclamation of the land from the sea is necessary in the
construction of the docks and the harbors, the City of Cavite is correct. Otherwise, It
is not. Since RA 1899 authorized the city to construct docks and harbors, all works
that are necessary for such construction are deemed authorized. Including the
reclamation of land from the sea. The reclamation being authorized, the city is the
owner of the reclaimed land and it may convey a portion thereof as payment for the
services of the contractor.
ANOTHER ALTERNATIVE ANSWER:
On the assumption that the reclamation contract was entered into before RA
1899 was repealed by PD 3-A, the City of Cavite is correct. Lands under the sea are
"beyond the commerce of man" in the sense that they are not susceptible of private
appropriation, ownership or alienation. The contract In question merely calls for the
reclamation of 300 hectares of land within the coastal waters of the city. Per se, it
does not vest, alienate or transfer ownership of land under the sea. The city merely
engaged the services of Fil-Estate to reclaim the land for the city.
06; LTD; redemption
1989 No. 7:
(1) "X" mortgaged his land to the Philippine National Bank (PNB) to secure a
promissory note. He defaulted in the payment of the loan so that the land was sold
at public auction on January 20, 1960, for P3,500 with the PNB as the highest
bidder. On January 20, 1970, "X" offered to redeem the property in the amount of
P3,500. He enclosed a postal money order for P1,000 as partial payment and stated
that the balance is to be paid in 12 monthly installments. The PNB then discovered
that the sheriffs certificate of sale prepared after the public auction of the land was
not registered so that it cause the same to be registered on January 30, 1970. The
PNB refused the offer of "X" contending that the offer to redeem was beyond the
one-year period provided under Act No. 3135 and that it was not accompanied by an
actual and simultaneous tender of the entire repurchase price. In view of the refusal
of the PNB, "X" filed an action to repurchase on February 20, 1970, Will the action
prosper? Give your reasons.
Answer:
Yes, the action should prosper. The one (1) year period of redemption is
counted from the registration of the sheriffs certificate of sale hence the action has
not yet prescribed. However, there need not be a tender of the redemption price
because the filing of the judicial action to enforce the right of redemption within the
redemption period suffices.
06; LTD; scope of registration
1989 No. 16:
(1) May the owner of a building constructed on an unregistered land
belonging to another apply for the registration of such building under the Land
Registration Act and P.D. 1529? What should he do to protect his rights in case the
owner of the land applied for registration thereof? Give your reasons.
Answer:
The Land Registration Act and PD 1529 apply to registration of land only. It
may include the building as an accessory but the building cannot be registered
Page 243 of 391
independently of the land because registration contemplated under this Act refers
only to ownership of land.
The owner of the building should file an opposition or answer to the
application for registration and ask the court that his right to the building be
annotated in the decree and later in the certificate of title.
06; LTD; torrens vs recording
1994 No. 3;
2) Distinguish the Torrens system of land registration from the system of
recording of evidence of title.
Alternative Answers;
2) a) The Torrens system of land registration is a system for the registration
of title to the land. Thus, under this system what is entered in the Registry of Deeds,
is a record of the owner's estate or interest in the land, unlike the system under the
Spanish Mortgage Law or the system under Section 194 of the Revised
Administrative Code as amended by Act 3344 where only the evidence of such title
is recorded. In the latter system, what is recorded is the deed of conveyance from
hence the owner's title emanatedand not the title itself.
b) Torrens system of land registration is that which is prescribed in Act 496
(now PD 1529), which is either Judicial or quasi-judicial. System or recording of
evidence of title is merely the registration of evidence of acquisitions of land with the
Register of Deeds, who annotates the same on the existing title, cancels the old one
and Issues a new title based on the document presented for registration.
06; LTD; writ of possession
1981 No. 16
In a cadastral case. Lot No. 123 was claimed and applied for by spouses "S-
T", The cadastral Court adjudicated the lot in their favor, the claim having been
uncontested. Three months thereafter, the heirs of "Z " filed a petition for the review
of the registration decree alleging that they were the true owners and were in actual
legal possession. After hearing, the Court denied the petition for review. No appeal
was taken.
Two years later, the spouses "S-T" filed a petition for the issuance of a Writ of
Possession in the cadastral proceedings. Opposed by the heirs of "Z", the Court
refused to issue the Writ on the ground that the heirs of "Z" were not specifically
named as parties in the cadastral case so that said Writ cannot issue as against
them.
Are the spouses "S-T" entitled to a Writ of Possession?
Explain.
Answer
The spouses "S-T" are entitled to a Writ of Possession. The heirs of "Z"
cannot be said to be strangers to the registration proceedings. A cadastral
proceeding is a proceeding in rem and against everybody, including the heirs of "Z",
who are deemed included in the general order of default entered in the case.
Besides, said heirs filed a petition for the review of the decree of registration,
thereby becoming a direct party in the registration proceedings by their voluntary
appearance. The fact that the spouses "S-T" filed the instant petition two years later
is of no moment. The right of the applicants or of a subsequent purchaser to ask for
the issuance of a writ of possession never prescribes.
Page 244 of 391
(Note: The above answer is based on Rodil vs. Benedicto, L-28616,
Jan.22,1980.)
Page 245 of 391
06; Prescription
1990 No 12:
In 1960, an unregistered parcel of land was mortgaged by owner O to M, a
family friend, as collateral for a loan. O acted through his attorney-in-fact, son S,
who was duty authorized by way of a special power of attorney, wherein O declared
that he was the absolute owner of the land, that the tax declarations/receipts were
all issued in his name, and that he has been In open, continuous and adverse
possession in the concept of owner.
As O was unable to pay back the loan plus interest for the past five [5) years,
M had to foreclose the mortgage. At the foreclosure sale, M was the highest bidder.
Upon issuance of the sheriffs final deed of sale and registration In January, 1966,
the mortgage property was turned over to M's possession and control M has since
then developed the said property. In 1967, O died, survived by sons S and P.
In 1977, after the tenth (10th) death anniversary of his father O. son P filed a
suit to annul the mortgage deed and subsequent sale of the property, etc., on the
ground of fraud. He asserted that the property in question was conjugal in nature
actually belonging, at the time of the mortgage, to O and his wife, W, whose
conjugal share went to their sons (S and P) and to O,
(a) Is the suit filed by P barred by prescription? Explain your answer.
(b) After the issuance of the sheriff's final deed of sale in 1966 in this case,
assuming that M applied for registration under the Torrens System and was issued a
Torrens Title to the said property in question, would that added fact have any
significant effect on your conclusion? State your reason.
Answer:
(a) Under Art. 173 of the Civil Code, the action is barred by prescription
because the wife had only ten (10) years from the transaction and during the
marriage to file a suit for the annulment of the mortgage deed.
Alternative Answers to (a) first Alternative Answer:
(a) The mortgage contract executed by O, if at all, is only a voidable contract
since it involves a conjugal partnership property. The action to annul the same
instituted in 1977, or eleven years after the execution of the sheriff's final sale, has
obviously prescribed because:
1. An action to annul a contract on the ground of fraud must be brought
within four (4) years from the date of discovery of the fraud. Since this is in essence
an action to recover ownership, it must be reckoned from the date of execution of
the contract or from the registration of the alleged fraudulent document with the
assessor's office for the purpose of transferring the tax declaration, this being
unregistered land, (Bael u. Intermediate Appellate Court G. R. L-74423 Jan.30, 1989
169 SCRA 617).
2. If the action is to be treated as an action to recover ownership of land, it
would have prescribed Just the same because more than 10 years have already
elapsed since the date of the execution of the sale.
Second Alternative Answer;
(a) The action to recover has been barred by acquisitive prescription in favor
of M considering that M has possessed the land under a claim of ownership for ten
(10) years with a just title.
(b) If M had secured a Torrens Title to the land, all the more S and P could
not recover because if at all their remedies would be:
Page 246 of 391
1. A Petition to Review the Decree of Registration. This can be availed of
within one (1) year from-the entry thereof, but only upon the basis of "actual fraud."
There is no showing that M committed actual fraud in securing his title to the land; or
2. An action in personam against M for the reconveyance of the title In their
favor. Again, this remedy is available within four years from the date of the discovery
of the fraud but not later than ten (10) years from the date of registration of the title
in the name of M.
06; Prescription
1978 No. IV-b
A signed a promissory note dated July 25, 1960 in favor of B, which reads:
"For value received I promise to pay B at his residence at 43 Caledonia St., Malate,
the sum of Ten Thousand Pesos (F10,000.00) with interest at twelve per cent (12%)
per annum upon demand." Twelve (12) years later on August 15, 1972, B brought
an action to collect the sum due under the promissory note. A interposed the
defense of prescription as more than ten (10) years had transpired- Decide the case
with reasons.
Answer
The action brought by B against A to collect the sum due under the
promissory note has already prescribed as more than ten years had already elapsed
from the time of the execution of the promissory note. Under our statute of
limitations, the law declares that actions upon a written contract must be brought
within ten years from the time the right of action accrues. Here, the promissory note
expressly states that A shall pay B the sum of P10,000 with interest at 12 % per
annum upon demand. Obviously B's right of action accrued immediately upon the
execution of the promissory note. Unfortunately for him, he brought the action for
collection twelve years later. The action, therefore, was filed out of time.
(NOTE: The above answer is based on Art. 1144 of the Civil Code and on
decided cases.)
06; Prescription; acquisitive; movable
1984 No. 7
On January l, 1978, A sold a typewriter to B. It turned out, however that A had
stolen the typewriter from C.
In February, 1982, when C discovered that his typewriter was in B's
possession, he immediately filed an action against B to recover it.
Will the action prosper? Answer:
A. Furnished by Office of Justice Palma,
No. While the sale of the typewriter by A to B is void, B has acquired
ownership of the typewriter because of the lapse of 4 years. He had possession of
the typewriter in good faith with just title for the period fixed by law. Ownership of
movables prescribes through uninterrupted possession for 4 years in good faith.
B. Comments and Suggested Answers
We agree with the answer of the Bar Examiner.
06; Prescription; just title
1985 No. 10:
A) A bought a parcel of land from B, adjoining that of C's. By mistake, A took
possession of C's land, thinking it was sold to him. He then possessed it openly,
peacefully, continuously and in the concept of owner for 10 years. When C
Page 247 of 391
discovered that A was possessing his land, C demanded its return and upon A's
refusal, brought an action for its recovery.
Will the action prosper? Discuss,
Answers:
A) 1. The action will prosper. A is not a possessor in good faith of the land
of C because A has no title or mode of acquisition with respect to the land of C.
What A bought was the land of B. A cannot claim acquisition of ownership by
ordinary "prescription of 10 years.
2) It depends whether it is under the Code of Civil Procedure or under the
New Civil Code. Under the Code of Civil Procedure, A has acquired the land by
prescription. Under the New Civil Code, there is no prescription because there is no
just title.
3) The action, absent the possible application of laches, can prosper. The
possession by A was without a just title; such possession can thus only ripen into
ownership by acquisitive prescription after 30 years of open, peaceful and
continuous possession in the concept of an owner (Art. 1137 and Art. 1129, in
relation to Art. 1127)
06; Prescription; laches
2000 No XVI
In an action brought to collect a sum of money based on a surety agreement,
the defense of laches was raised as the claim was filed more than seven years from
the maturity of the obligation. However, the action was brought within the ten-year
prescriptive period provided by law wherein actions based on written contracts can
be Instituted.
a) Will the defense prosper? Reason. (3%)
b) What are the essential elements of laches? (2%)
SUGGESTED ANSWER;
No, the defense will not prosper. The problem did not give facts from which
laches may be Inferred. Mere delay in filing an action, standing alone, does not
constitute laches (Agra v. PNB. 309 SCRA 509).
SUGGESTED ANSWER:
b) The four basic elements of laches are; (1) conduct on the part of the
defendant or of one under whom he claims, giving rise to the situation of which
complainant seeks a remedy; (2) delay In asserting the complainant's rights, the
complainant having had knowledge or notice of the defendant's conduct and having
been afforded an opportunity to institute suit; (3) lack of knowledge on the part of the
defendant that the complainant would assert the right on which he bases his suit;
and (4) injury or prejudice to the defendant in the event relief is accorded to the
complainant, or the suit is not held to be barred.
06; Prescription; laches
2002 No VII.
Way back in 1948, Windas husband sold in favor of Verde Sports Center
Corp. (Verde) a 10-hectare property belonging to their conjugal partnership. The
sale was made without Windas knowledge, much less consent. In 1950, Winda
learned of the sale, when she discovered the dead of sale among the documents in
her husbands vault after his demise. Soon after, she noticed that the construction of
the sports complex had started. Upon completion of the construction in 1952, she
tried but failed to get free membership privileges in Verde.
Page 248 of 391
Winda now files a suit against Verde for the annulment of the sale on the
ground that she did not consent to the sale. In answer, Verde contends that, in
accordance with the Spanish Civil Code which was then in force, the sale in 1948 of
the property did not need her concurrence. Verde contends that in any case the
action has prescribed or is barred by laches. Winda rejoins that her Torrens title
covering the property is indefeasible, and imprescriptible.
A. Define or explain the term laches. (2%)
B. Decide the case, stating your reasons for your decision. (3%)
SUGGESTED ANSWER:
A. Laches means failure or neglect, for an unreasonable and unexplained
length of time, to do what, by exercising due diligence, could or should have been
done earlier. It is negligence or omission to assert a right within a reasonable time.
(De Vera v. CA, 305 SCRA 624 [1999])
B. While Article 1413 of the Spanish Civil Code did not require the
consent of the wife for the validity of the sale, an alienation by the husband in fraud
of the wife is void as held in Uy Coque v. Navas, 45 Phil. 430 (1923). Assuming that
the alienation in 1948 was in fraud of Winda and, therefore, makes the sale to Verde
void, the action to set aside the sale, nonetheless, is already barred by prescription
and laches. More than 52 years have already elapsed from her discovery of the sale
in 1950.
ALTERNATIVE ANSWER:
B. Windas claim that her Torrens Title covering the property is indefeasible
and imprescriptible [does not hold water] is not tenable. The rule of indefeasibility of
a Torrens Title means that after one year from the date of issue of the decree of
registration or if the land has fallen into the hands of an innocent purchaser for
value, the title becomes incontestable and incontrovertible. Imprescriptibility, on the
other hand, means that no title to the land in derogation of that of the registered
owner may be acquired by adverse possession or acquisitive prescription or that the
registered owner does not lose by extinctive prescription his right to recover
ownership and possession of the land.
The action in this case is for annulment of the sale executed by the husband
over a conjugal partnership property covered by a Torrens Title. Action on contracts
are subject to prescription.
06; Prescription; real rights
1992 No 7:
A owned a parcel of unregistered land located on the Tarlac side of the
boundary between Tarlac and Pangasinan. His brother B owned the adjoining parcel
of unregistered land on the Pangasinan side.
A sold the Tarlac parcel to X in a deed of sale executed as a public
instrument by A and X. After X paid in full the, price of the sale, X took possession of
the Pangasinan parcel In the belief that it was the Tarlac parcel covered by the deed
of sale executed by A and X.
After twelve (12) years, a controversy arose between B and X on the issue of
the ownership of the Pangasinan parcel,
B claims a vested right of ownership over the Pangasinan parcel because B
never sold that parcel to X or to anyone else.
Page 249 of 391
On the other hand, X claims a vested right of ownership over the Pangasinan
parcel by acquisitive prescription, because X possessed this parcel for over ten (10]
years under claim of ownership.
Decide on these claims, giving your reasons. Answer;
At this point in time, X cannot claim the right of vested ownership over the
Pangasinan parcel by acquisitive prescription. In addition to the requisites common
to ordinary and extraordinary acquisitive prescription consisting of uninterrupted,
peaceful, public, adverse and actual possession in the concept of owner, ordinary
acquisitive prescription for ten (10) years requires (1) possession in good faith and
(2) just title. "Just title" means that the adverse claimant came into possession of the
property through one of the modes recognized by law for the acquisition of
ownership but the grantor was not the owner or could not transmit any right (Art.
1129. Civil Code). In this case, there is no "just title" and no "mode" that can be
Invoked by X for the acquisition of the Pangasinan parcel There was no constructive
delivery of the Pangasinan parcel because It was not the subject-matter of the deed
of sale.
Hence, B retains ownership of the Pangasinan parcel of land.
Page 250 of 391
07; Contracts; annulment of contracts; capacity to sue
1996 No 9;
Sometime in 1955, Tomas donated a parcel of land to his stepdaughter Irene,
subject to the condition that she may not sell, transfer or cede the same for twenty
years. Shortly thereafter, he died. In 1965, because she needed money for medical
expenses, Nene sold the land to Conrado. The following year, Irene died, leaving as
her sole heir a son by the name of Armando. When Armando learned that the land
which he expected to inherit had been sold by Irene to Conrado. he filed an action
against the latter for annulment of the sale, on the ground that it violated the
restriction imposed by Tomas. Conrado filed a motion to dismiss, on the ground that
Armando did not have the legal capacity to sue.
If you were the Judge, how will you rule on this motion to dismiss? Explain.
Answer:
As judge, 1 will grant the motion to dismiss. Armando has no personality to
bring the action for annulment of the sale to Conrado. Only an aggrieved party to the
contract may bring the action for annulment thereof (Art. 1397. NCC). While
Armando is heir and successor-in-interest of his mother (Art. 1311, NCC), he
[standing in place of his mother) has no personality to annul the contract. Both are
not aggrieved parties on account of their own violation of the condition of, or
restriction on, their ownership Imposed by the donation. Only the donor or his heirs
would have the personality to bring an action to revoke a donation for violation of a
condition thereof or a restriction thereon. (Garrido u. CA, 236 SCRA 450).
Consequently, while the donor or his heirs were not parties to the sale, they have
the right to annul the contract of sale because their rights are prejudiced by one of
the contracting parties thereof [DBP v. CA, 96 SCRA 342; Teves vs. PHHC. 23
SCRA 1141. Since Armando is neither the donor nor heir of the donor, he has no
personality to bring the action for annulment.
Alternative Answer:
As judge, I will grant the motion to dismiss. Compliance with a condition
imposed by a donor gives rise to an action to revoke the donation under Art. 764,
NCC. However, the right of action belongs to the donor. Is transmissible to his heirs,
and may be exercised against the donee's heirs. Since Armando is an heir of the
donee, not of the donor, he has no legal capacity to sue for revocation of the
donation. Although he is not seeking such revocation but an annulment of the sale
which his mother, the donee, had executed in violation of the condition imposed by
the donor, an action for annulment of a contract may be brought only by those who
are principally or subsidiarily obliged thereby (Art. 1397, NCC). As an exception to
the rule, it has been held that a person not so obliged may nevertheless ask for
annulment if he is prejudiced in his rights regarding one of the contracting parties
(DBP us. CA. 96 SCRA 342 and other cases) and can show the detriment which
would result to him from the contract in which he had no intervention, (Teves vs.
PHHC, 23 SCRA 1141).
Such detriment or prejudice cannot be shown by Armando. As a forced heir,
Armando's interest In the property was, at best, a mere expectancy. The sale of the
land by his mother did not impair any vested right. The fact remains that the
premature sale made by his mother (premature because only half of the period of
the ban had elapsed) was not voidable at all, none of the vices of consent under Art.
139 of the NCC being present. Hence, the motion to dismiss should be granted.
07; Contracts; annulment of contracts; prescriptive period
1979 No. IX
Page 251 of 391
On the basis of a document entitled "Deed of Absolute Sale", a certain lot and
building then, leased by its owner, PC, to JG with a monthly rental of P1,000.00, was
sold to, and thus registered in the latter's name. Six years after the issuance of the
title to JG, MC the sole heir of PC who had just died brought an action for recovery
of the property alleging in his complaint, among others, that PC then very old and
with weak eyesight was tricked by JG into signing the Deed of Absolute Sale upon
the fraudulent misrepresentation that said document was only a renewal of the lease
contract over the property; that the price stated in the document is only P10,000
although the property was then worth about P50,000 JG moved to dismiss the action
on the ground of prescription. Should the motion be granted? Why?
Answer
The motion should be granted. In reality, the action instituted by MC against
JG for recovery of the property is an action for annulment of the contract based on
fraud and/or mistake. This can be inferred from the allegations in the complaint.
Being an action for annulment of a voidable contract, the period, of prescription is
four years to be counted from the time of the discovery of the fraud or mistake.
When did PC and his heir MC discover the fraud or mistake? The discovery took
place when JG registered the property in his name. Registration of an instrument
constitutes constructive notice to the whole world, and therefore, discovery of the
fraud or mistake is deemed to have taken place at the time of registration. (Carantes
vs. CA, 76 SCRA 514) In the case at bar, since the action fop annulment was
instituted six years after registration of the property in JC's name, it follows that it is
now barred by the statute of limitation.
Answer No. II
The motion should be granted. In reality, the action instituted by MC against
JG for recovery of the property is an action for reconveyance of property based on
implied or constructive trust. The allegation in the complaint are clear. JG acquired
the property through fraud. He is, therefore, by force of law, considered a trustee of
an implied trust for the benefit of the person from whom the property comes (Art.
1456, Civil Code). It is now a well-settled rule in this jurisdiction that an action for
reconveyance based on implied trust is prescriptible and that the action prescribes
after ten years to be counted from the time of the discovery of the fraud. When did
PC and his heir MC discover the fraud? The discovery took place when JG
registered the property in his name. Registration of an instrument constitutes
constructive notice to the whole world, and therefore, discovery of the fraud is
deemed to have taken place at the time of registration. (Carantes vs. CA, 76 SCRA
514, and cases cited thereunder). In the case at bar, since the action for
reconveyance was instituted six years after registration of the property in JG's name,
it follows that it is not yet barred by the statute of limitations.
07; Contracts; consensual vs real contracts
1998 No XVIII.
2. Distinguish consensual from real contracts and name at least four (4)
kinds of real contracts under the present law. [3%|
Answer;
2. Consensual contracts are those which are perfected by mere consent
(Art. 1315. Civil Code). Real contracts are those which are perfected by the
delivery of the object of the obligation. (Art. 1316, Civil Code)
Examples of real contracts are deposit, pledge, commodatum and simple
loan (mutuum).
07; Contracts; consent; invitation to bid
Page 252 of 391
1980 No. IV
(a) "K" & Co. published in the newspaper an "Invitation To Bid" inviting
proposals to supply labor and materials for a construction project described in the
invitation. "L", "M", and "N" submitted bids. When the bids were opened, it appeared
that "L" submitted the lowest bid. However, "K" & Co. awarded the contract "N", the
highest bidder, on the ground that he was the most experienced and responsible
bidder. "L" brought an action against "K" & Co, to compel the award to him and to
recover damages.
Is "L's" position meritorious?
Answer
(a) "L's" position is not meritorious. According to the Civil Code,
advertisements for bidders are simply invitations to make proposals, and the
advertiser is not bound to accept the highest or lowest bidder, unless the contrary
appears (Art. 1326). It is clear that the general rule applies in the instant case. In its
advertisements, "K" & Co., for instance, did not state that it will award the contract to
the lowest bidder. Therefore, in awarding the contract to "N", the defendant
company acted in accordance with its rights.
07; Contracts; consideration; validity
2000 No XV
a) Lolita was employed in a finance company. Because she could not account
for the funds entrusted to her. she was charged with estafa and ordered arrested. In
order to secure her release from jail, her parents executed a promissory note to pay
the finance company the amount allegedly misappropriated by their daughter. The
finance company then executed an affidavit of desistance which led to the
withdrawal of the information against Lolita and her release from jail. The parents
failed to comply with their promissory note and the finance company sued them for
specific performance. Will the action prosper or not? (3%)
SUGGESTED ANSWER:
The action will prosper. The promissory note executed by Lolita's parents is
valid and binding, the consideration being the extinguishment of Lolita's civil liability
and not the stifling of the criminal prosecution.
ALTERNATIVE: ANSWER:
The action will not prosper because the consideration for the promissory note
was the non-prosecution of the criminal case for estafa. This cannot be done
anymore because the information has already been filed in court and to do it is
illegal. That the consideration for the promissory note is the stifling of the criminal
prosecution is evident from the execution by the finance company of the affidavit of
desistance immediately after the execution by Lolita's parents of the promissory
note. The consideration being illegal, the promissory note is invalid and may not be
enforced by court action.
07; Contracts; form of contracts
1977 No IV-c
H sold a parcel of land, with right to repurchase in a private instrument to C,
who sold it to D, who transferred it to E. E took possession, and because all the
transfers were in private instruments, E after 30 years filed suit to compel the heirs
of H to execute a formal deed of conveyance. Has prescription set in? Can E
prevail?
Answer
Page 253 of 391
The suit filed by E against the heirs of H cannot prosper. In the first place,
there is no privity of interest between E and H or the latter's heirs. E should have
directed the suit against D, his immediate predecessor. In the second place, said
action filed by E against the heirs of H, which is presumably based on Art. 1357 of
the Civil Code (to compel each other to execute the required form), has already
prescribed. An action based on a right provided by law prescribes in 10 years.
07; Contracts; forms of contracts
1982 No. 17
"A" and "B" entered into a verbal contract whereby "A" agreed to sell to "B"
his only parcel of land for P20,000, and "B" agreed to buy at the aforementioned
price. "B" went to the bank, withdrew the necessary amount, and returned to "A" for
the consummation of the contract. "A" however, had changed his mind and refused
to go through with the sale. Is the agreement valid? Will an action by "B" against "A"
for specific performance prosper? Reason.
Answer
It must be observed that there are two questions in the case at bar. They are:
(1) Is the agreement valid? The answer is yes. It is a time-honored rule that
even a verbal agreement to sell land is valid so long as there is already an
agreement with respect to the object and the purchase price.
(2) Will an action by "B" against "A" for specific performance prosper? The
answer is no, unless it is ratified. The reason is obvious. The agreement, being an
agreement of sale of real property, is covered by the Statute of Frauds It cannot,
therefore, be enforced by a court action because it is not evidenced by any note or
memorandum or writing properly subscribed by the party charged.
(Note: The above answer is based on No. 2 of Art. 1203 of the Civil Code and
on decided cases.)
07; Contracts; inducing another to violate a contract
1980 No. V
(a) "O", a very popular movie star was under contract with "P" Movie
Productions to star exclusively in the latter's films for two years. "O" was prohibited
by the contract to star in any film produced by another producer. "X" Film Co.
induced "O" to break her contract with "P" Movie Productions by giving her twice her
salary. "P" Movie Productions sued "X" Film Co. for damages "X" Film Co contended
that it had a right to compete for the services of "0" and that her contract with "P"
Movie Productions was in restraint of trade and a restriction on her freedom of
contract.
Whose contention would you sustain?
Answer
(a) The contention of "P" Movie Productions should be sustained.
According to the Civil Code, any third person who induces another to violate his
contract shall be liable for damages to the other contracting party (Art. 1314).
However, the following requisites must concur: (a) the existence of a valid contract;
(b) knowledge on the part of the third person of the existence of the contract; and
(c) interference of the third person without legal justification or excuse. All of these
requisites are present in the case at bar. (See Daywalt vs. Agustinos Recoletos, 39
Phil. 587).
07; Contracts; innominate contracts
1977 No. X-c
Page 254 of 391
What are the different kinds of innominate contracts and how are they
regulated?
Answer
There are four kinds of innominate contracts. They are:
(1) Do ut des I give that you give;
(2) Do ut facias I give that you do;
(3) Facio ut des - I do that you give; and
(4) Facio ut facias I do that you do. These contracts shall be regulated by
the stipulation of the parties, by the general provisions or principles of obligations
and contracts, by the rules governing the most analogous nominate contracts, and
by the custom of the place (Art. 1307, Civil Code).
1977 No. XI-a
A and B are spurious children of T, born in 1945 and 1947, respectively. T
died in 1955. Can A and B inherit from him? Would your answer be the same if T
died the year after B's birth?
Answer
If T died in 1955, A and B can inherit from him. Under the New Civil Code,
spurious children can inherit. This is, of course, based on the assumption that T had
recognized A and B as his spurious children either voluntarily or by means of a final
judgment of a competent court. If T had not recognized them, they cannot inherit
from him.
If T died the year after B's birth, A and B cannot inherit from him. The reason
is that in such case the right of A and B to inherit from T shall still be governed by
the Spanish Civil Code (Art. 2263, Civil Code). Under the facts presented, it is clear
that T died prior to the effectivity (Aug. 30, 1950) of the New Civil Code. Therefore,
what is applicable is the Spanish Civil Code. Under said Code, spurious children
cannot inherit
07; Contracts; obligatory nature of contracts
1991 No 9:
Roland, a basketball star, was under contract for one year to play-for-play
exclusively for Lady Love, Inc. However, even before the basketball season could
open, he was offered a more attractive pay plus fringes benefits by Sweet Taste,
Inc. Roland accepted the offer and transferred to Sweet Taste. Lady Love sues
Roland and Sweet Taste for breach of contract. Defendants claim that the restriction
to play for Lady Love alone Is void, hence, unenforceable, as It constitutes an undue
Interference with the right of Roland to enter into contracts and the impairment of his
freedom to play and enjoy basketball
Can Roland be bound by the contract he entered Into with Lady Love or can
he disregard the same? Is he liable at all? How about Sweet Taste? Is it liable to
Lady Love?
Answer;
Roland Is bound by the contract he entered into with Lady Love and he
cannot disregard the same, under the principles of obligatoriness of contracts.
Obligations arising from contracts have the force of law between the parties.
Yes, Roland is liable under the contract as far as Lady Love is concerned.
He is liable for damages under Article 1170 of the Civil Code since he
contravened the tenor of his obligation. Not being a contracting party, Sweet Taste is
Page 255 of 391
not bound by the contract but It can be held liable under Art. 1314. The basis of its
liability Is not prescribed by contract but is founded on quail-delict, assuming that
Sweet Taste knew of the contract. Article 1314 of the Civil Code provides that any
third person who induces another to violate his contract shall be liable for damages
to the other contracting party.
Alternative Answer:
It is assumed that Lady Love knew of the contract.
Neither Roland nor Sweet Taste would be liable, because the restriction in
the contract is violative of Article 1306 as being contrary to law. morals, good
customs, public order or public policy.
07; Contracts; perfection of contracts; obligations with a period
1988 No. 11:
(b) Merle offered to sell her automobile to Violy for P60,000.00. After
inspecting the automobile, Violy offered to buy it for P50,000.00. This offer was
accepted by Merle. The next day, Merle offered to deliver the automobile, but Violy
being short of funds, secured postponement of the delivery, promising to pay the
price "upon arrival of the steamer, Helena". The steamer however never arrived
because it was wrecked by a typhoon and sank somewhere off the Coast of Samar.
(1) Is there a perfected contract in this case? Why?
(2) Is the promise to pay made by Violy conditional or with a term? Why?
(3) Can Merle compel Violy to pay the purchase price and to accept the
automobile? Why?
Answer:
(b) (1) Yes, there is a perfected contract because there is already a
concurrence between the offer and the acceptance with respect to the object and
the cause which shall constitute the contract. Such concurrence is manifested by the
acceptance made by Merle of the offer made by Violy.
(2) I submit that the promise to pay made by Violy is not conditional, but with
a term. The promise is to pay the P50,000 upon arrival in this port of the steamer,
Helena, not if the steamer Helena shall arrive in this port. Hence, the promise is
with regard to the date of arrival and not with regard to the fact of arrival.
(3) Yes, Merle can compel Violy to pay the purchase price and to accept the
automobile. She will, however, have to wait for the date when the steamer, Helena,
would have arrived were it not for the shipwreck. After all, there is already a
perfected contract.
Suggested Alternative Answers To: No. 11 (b):
(b) (2) The promise to pay is subject to a term. When there is a pre-existing
obligation and the "condition" affects only the time of payment such "condition" can
be considered as a period. In other words, the parties must be deemed to have
contemplated a period,
(3) Yes Merle can compel Violy to pay the purchase price and to accept the
automobile but only after the parties would have fixed the period. Failing in that, the
courts may be asked to fix the period. Article 1180 provides that:
"When the debtor binds himself to pay when his means permit him to do so,
the obligation shall be deemed to be one with a period, subject to the provisions of
article 1197."
07; Contracts; privity of contract
Page 256 of 391
1996 No. 12:
Baldomero leased his house with a telephone to Jose. The lease contract
provided that Jose shall pay for all electricity, water and telephone services in the
leased premises during the period of the lease. Six months later. Jose surreptitiously
vacated the premises. He left behind unpaid telephone bills for overseas telephone
calls amounting to over P20,000.00. Baldomero refused to pay the said bills on the
ground that Jose had already substituted him as the customer of the telephone
company. The latter maintained that Baldomero remained as his customer as far as
their service contract was concerned, notwithstanding the lease contract between
Baldomero and Jose.
Who is correct, Baldomero or the telephone company? Explain.
Answer:
The telephone company is correct because as far as it is concerned, the only
person it contracted with was Baldomero. The telephone company has no contract
with Jose. Baldomero cannot substitute Jose in his stead without the consent of the
telephone company (Art. 1293, NCC). Baldomero is. therefore, liable under the
contract.
07; Contracts; reformation of instruments
1984 No. 15
On June 13, 1982, A sold to B in a public instrument a parcel of land for
P50,000, Simultaneously, B granted A an option to buy the same property for
P60,000 within one year. On June 13, 1983, B allowed A an extension of the option
to buy for another year, this time at the price of P72,000. All the while, A has
remained in possession of the land.
In May, 1984, A filed an action for the reformation of the deed of sale into a
real estate mortgage, alleging that the land covered thereby was given only as a
security for the repayment of a loan.
Under the circumstances, will the action prevail? Why?
Answer:
A. Furnished by the Office of Justice Palma,
The option was granted on the same date that the sale was executed. The
repurchase price increased at a rate of 20% a year, which could be equivalent to
interest at the rate of 20% a year. Moreover, the seller remained in possession of
the premises. All of these are indications that the real transaction between the
parties is a loan, not a sale. In case of doubt, under Article 1603, a contract
purporting to be a sale with right of repurchase shall be construed as an equitable
mortgage. The instrument should therefore be reformed.
B. Comments and Suggested Answer
We agree with the answer of the Bar Examiner.
07; Contracts; rescission of contracts; capacity to sue
1996 No. 13:
In December 1985, Salvador and the Star Semiconductor Company (SSC)
executed a Deed of Conditional Sale wherein the former agreed to sell his 2,000
square meter lot in Cainta, Rizal, to the latter for the price of P1,000,000.00, payable
P100,000.00 down, and the balance 60 days after the squatters in the property have
been removed. If the squatters are not removed within six months, the P100,000.00
down payment shall be returned by the vendor to the vendee,
Page 257 of 391
Salvador filed ejectment suits against the squatters, but in spite of the
decisions in his favor, the squatters still would not leave. In August, 1986, Salvador
offered to return the PIOO,OOO.OO down payment to the vendee, on the ground
that he is unable to remove the squatters on the property. SSC refused to accept the
money and demanded that Salvador execute a deed of absolute sale of the property
in its favor, at which time It will pay the balance of the price. Incidentally, the value of
the land had doubled by that time.
Salvador consigned the P 100,000.00 in court, and filed an action for
rescission of the deed of conditional sale, plus damages.
Will the action prosper? Explain.
Answer;
No, the action will not prosper. The action for rescission may be brought only
by the aggrieved party to the contract. Since it was Salvador who failed to comply
with his conditional obligation, he is not the aggrieved party who may file the action
for rescission but the Star Semiconductor Company, The company, however, is not
opting to rescind the contract but has chosen to waive Salvador's compliance with
the condition which it can do under Art. 1545, NCC.
Alternative Answer:
The action for rescission will not prosper. The buyer has not committed any
breach, let alone a substantial or serious one, to warrant the rescission/resolution
sought by the vendor. On the contrary, it is the vendor who appears to have failed to
comply with the condition imposed by the contract the fulfillment of which would
have rendered the obligation to pay the balance of the purchase price demandable.
Further, far from being unable to comply with what is incumbent upon it, ie., pay the
balance of the price - the buyer has offered to pay it even without the vendor having
complied with the suspensive condition attached to the payment of the price, thus
waiving such condition as well as the 60-day term in its favor The stipulation that the
PI00,000.00 down payment shall be returned by the vendor to the vendee if the
squatters are not removed within six months, is also a covenant for the benefit of the
vendee, which the latter has validly waived by Implication when it offered to pay the
balance of the purchase price upon the execution of a deed of absolute sale by the
vendor. (Art. 1545, NCCJ
07; Contracts; Statute of Frauds
1988 No. 15:
(a) Suppose that in an oral contract, which by its terms is not to be performed
within one year from the execution thereof, one of the contracting parties has
already complied within the year with the obligations imposed upon him by said
contract, can the other party avoid fulfillment of those incumbent upon him by
invoking the Statute of Frauds?
Answer:
(a) No, he cannot. This is so, because the Statute of Frauds aims to prevent
and not to protect fraud. It is well-settled that when the law declares that an
agreement which by its terms is not to be performed within a year from the making
thereof is unenforceable by action, unless the same, or some note or memorandum
thereof, be in writing, and subscribed by the party- charged, or by his agent, it refers
only to an agreement which by its terms is not to be performed on either side within
a year from the execution thereof. Hence, one which has already been fully
performed on one side within a year is taken out of the operation of the statute. (Phil.
Nat. Bank vs. Phil. Vegetable Oil Co., 49 Phil. 857; Shoemaker vs. La Tondena, 68
Phil. 24.)
Page 258 of 391
07; Contracts; Statute of Frauds
1979 No. XI
W and Colk a logging company, received a letter from M, the new owner of a
certain property, notifying it that the latter will close the road running thru his
property and thru which W & Co.'s trucks pass in hauling logs to its saw mill. W &
Co. therefore begged M not to do so and upon the latter's refusal, W & Co. filed an
action for injunction alleging among others that it had acquired a right of way through
M's land before by virtue of a verbal agreement with the previous owner. Will the
action prosper? Why?
Answer
Yes, the action will prosper. The right of way, although arising from a verbal
agreement with the former owner of the land, still subsists. Obviously, the Statute of
Frauds cannot be applied because the agreement is not an agreement for the sale
of real property or an interest therein. Therefore, the agreement is both valid and
enforceable. (Western Mindanao Lumber Co. vs. Medalle, 79 SCR A 702).
Alternative Answer
Yes, the action will prosper. The right of way, although arising from a verbal
agreement with the former owner of the land still subsists. It is a well-settled rule in
this jurisdiction that the Statute of Frauds is applicable only to purely executory
contracts and not to contracts which have already been executed either totally or
partially. Here, the verbal agreement with respect to the right of way had already
been totally executed. Besides, there was already acceptance of benefits.
Therefore, the agreement is now valid and enforceable at the same time.
07; Contracts; Statute of Frauds; ratification by acceptance
1981 No. 9
"O" verbally leased his house and lot to "L" for two years at a monthly rental
of P250 a month. After the first year, "0" demanded a rental of P500.00 claiming that
due to the energy crisis, with the sudden increase of the price of oil, which no one
expected, there was also a general increase in prices. "O" proved an inflation rate of
100%. When "L" refused to vacate the house, "0" brought an action for ejectment.
"O" denied that he had agreed to a lease for two years.
a) Can the lessee testify on a verbal contract of lease? Reasons.
b) Assuming that "O" admits the two-year contract, is he justified in
increasing the rental? Why?
Answer
(a) Yes, the lessee "L" may testify on the verbal contract of lease. Well-
settled is the rule that the Statute of Frauds by virtue of which oral contracts {such
as the contract in the instant case) are unenforceable by court action is applicable
only to those contracts which have not been consummated either totally or partially.
The reason for this rule is obvious. In effect, there is already a ratification of the
contract by acceptance of benefits. Here, "L" has been paying to "O" in monthly
rental of P250.00 for one year. The case is, therefore, withdrawn from the coverage
of the Statute of Frauds.
{Note: The above answer is based on Arts. 1403, No. 2 and 1405 of the Civil
Code and on decided cases.)
(b) Yes, "O" is justified in increasing the monthly rental. Since it is admitted
that the contract of lease is for a definite term or period of two years and since he
Page 259 of 391
has established an inflation rate of 100%, it is crystal clear that the case is
withdrawn from the coverage of the new rental law.
(Note: The above answer is based on Batas Pambansa Blg. 25.)
07; Contracts; stipulation pour atrui
2002 No X.
Printado is engaged in the printing business. Suplico supplies printing paper
to Printado pursuant to an order agreement under which Suplico binds himself to
deliver the same volume of paper every month for a period of 18 months, with
Printado in turn agreeing to pay within 60 days after each delivery. Suplico has been
faithfully delivering under the order agreement for 10 months but thereafter stopped
doing so, because Printado has not made any payment at all. Printado has also a
standing contract with publisher Publico for the printing of 10, 000 volumes of school
textbooks. Suplico was aware of said printing contract. After printing 1, 000 volumes,
Printado also fails to perform under its printing contract with Publico. Suplico sues
Printado for the value of the unpaid deliveries under their order agreement. At the
same time Publico sues Printado for damages for breach of contract with respect to
their own printing agreement. In the suit filed by Suplico, Printado counters that: (a)
Suplico cannot demand payment for deliveries made under their order agreement
until Suplico has completed performance under said contract; (b) Suplico should pay
damages for breach of contract; and (c) with Publico should be liable for Printados
breach of his contract with Publico because the order agreement between Suplico
and Printado was for the benefit of Publico. Are the contentions of Printado tenable?
Explain your answers as to each contention. (5%)
SUGGESTED ANSWER:
No, the contentions of Printado are untenable.
Printado having failed to pay for the printing paper covered by the delivery
invoices on time, Suplico has the right to cease making further delivery. And the
latter did not violate the order agreement (Integrated Packaging Corporation v. Court
of Appeals, (333 SCRA 170, G.R. No. 115117, June 8, [2000]).
Suplico cannot be held liable for damages, for breach of contract, as it was
not he who violated the order agreement, but Printado.
Suplico cannot be held liable for Printados breach of contract with Publico.
He is not a party to the agreement entered into by and between Printado and
Publico. Theirs is not a stipulation pour atrui. [Aforesaid] Such contracts do could not
affect third persons like Suplico because of the basic civil law principle of relativity of
contracts which provides that contracts can only bind the parties who entered into it,
and it cannot favor or prejudice a third person, even if he is aware of such contract
and has acted with knowledge thereof. (Integrated Packaging Corporation v. CA,
supra.)
07; Contracts; stipulation pour atrui
1977 No. VII-b
What is a stipulation pour atrui and what are its essential requisites? Give an
example.
Answer
A stipulation pour atrui is a stipulation in a contract, clearly and deliberately
conferred by the contracting parties as a favor upon a third person, who must
communicate his acceptance to the obligor before it is revoked. Before such a
stipulation may be enforced, it is necessary that the following requisites must
concur: (1) that it must be for the benefit or interest of the third person; (2) that such
Page 260 of 391
benefit or Interest must not be merely incidental; (3) that the contracting parties
should have clearly and deliberately conferred such benefit or interest upon the third
person; and (4) that the third person should have communicated his acceptance of
the benefit or interest to the obligor before its revocation. (Art. 1311, par. 2, Civil
Code.)
Example: In a contract with X Co., the Philippine National Bank, for a
valuable consideration, agreed to cause a sum of money to be paid to A in New
York City. If A communicates his acceptance of the benefit to X Co. before it could
be revoked by the latter, we have what is commonly known as a stipulation pour
atrui.
07; Contracts; void contracts; in pari delicto principle
1977 No, VII-c
Discuss the principle of in pari delicto and enumerate five (5) instances in the
Civil Code which are exceptions to the principle.
Answer
When the defect of a void contract consists in the illegality of the cause or
object of the contract, and both of the parties are at fault or in pari delicto, the law
refuses them every remedy and leaves them where they are. This rule which is
embodied in Arts. 1411 of the Civil Code is what is commonly known as the principle
of in pari delicto,
The exceptions to the principle of pari delicto are the following:
(1) Payment of usurious interest. In such case, the law allows the debtor to
recover the interest paid in excess of that allowed by the usury laws, with interest
thereon from the date of payment. (Art 1413, Civil Code).
(2) Payment of money or delivery of property for an illegal purpose, where
the party who paid or delivered repudiates the contract bore the purpose has been
accomplished, or before any damage has been caused to a third person. In such
case, the courts may allow such party to recover what he has paid or delivered if the
public interest will thus be sub served. (Art. 1414, Civil Code).s
(3) Payment of money or delivery of property by an incapacitated person. In
such case, the courts may allow such person to recover what he has paid or
delivered, if the interest of justice so demands. (Art, 1415, Civil Code).
(4) Agreement or contract which is not illegal per se but is merely prohibited
by law, and the prohibition is designed for the protection of the plaintiff. In such case,
such plaintiff, if public policy is thereby enhanced, may recover what he has paid or
delivered. (Art. 1416. Civil Code).
(5) Payment of any amount in excess of the maximum price of any article or
commodity fixed by law. In such case, the buyer may recover the excess. (Art. 1417,
Civil Code).
(6) Contract whereby a laborer undertakes to work longer than the
maximum number of hours fixed by law. In such case, the laborer may demand for
overtime pay. (Art. 1418, Civil Code).
(7) Contract whereby a laborer accepts a wage lower than the minimum
wage fixed by law. In such case, the laborer may demand for the deficiency, (Art.
1419, Civil Code).
(NOTE: The above exceptions may be tagged or labeled only either by their
codal numbers or by any means of identification.)
07; Contracts; void contracts; usurious interests
Page 261 of 391
1977 No. X-a
The Court found that the loan by C to D of P20,000.00 was usurious,
because it provided for the payment P5,000 as interest in 1 year.
(1) If the principal and the interest were paid, what can D recover from C?
(2) If no payment whatsoever was made, can D resist an action to collect by
C on the ground that the transaction is illegal and void?
Answer:
(1) D can recover from S the entire interest paid by him to the latter with
interest thereon from the date of payment. This is expressly directed by the Civil
Code (Art. 1418.) True, the Usury Law (Sec. 6) merely states that he can recover
only the whole interest paid, but the Civil Code (Art. 1413) adds that the same can
be recovered with interest thereon from the date of payment. (Angel Jose
Warehousing Co. vs. Ckelda Enterprises, 23 SCRA 119.)
(2) No, D cannot. According to the Civil Code (Art. 1420), in case of a
divisible contract, if the illegal terms can be separated from the legal ones, the latter
may be enforced. In a simple contract of a loan with usurious interest, the prestation
of the debtor to pay the principal debt is not illegal; what is illegal is to pay the
stipulated interest. Hence, being separable, the latter only should be deemed void.
(Angel Jose vs. Chelda, supra; Briones vs, Cammayo, 41 SCRA 404.)
07; Contracts; void vs voidable contracts
2004 No. II
A. Distinguish briefly but clearly between: 4. Inexistent contracts and
annullable contracts.
07; Contracts; void/unenforceable contracts
1976 No. X-c
C, husband of D, sold paraphernal property in her name without her (D's)
consent. Was such sale valid, void, voidable, rescissible or unenforceable? Explain.
Answer
Under the general principle on contracts, the contract is unenforceable if
entered into in the name of another without authority. (Article 1317)
Assuming that C, the husband of D was the letter's agent, under the law on
agency which was taken from the old Statute of Frauds (Art. 1874), if a sale of a
piece of land or any interest therein is through an agent, the authority of the latter
shall be in writing; otherwise, the sale shall be void.
To reconcile those two provisions, Article 1317 should apply if the property
sold was movable and Article 1874 shall apply if the property sold is immovable.
Hence it the property is movable, the contract is unenforceable, and if the property is
immovable, the contract is void.
07; Contracts; voidable contracts
1990 No 7:
X was the owner of a 10,000 square meter property. X married Y and out of
their union. A, B and C were born. After the death of Y, X married Z and they begot
as children, D, E and F. After the death of X. the children of the first and second
marriages executed an extrajudicial partition of the aforestated property on May 1,
1970. D, E and F were given a one thousand square meter portion of the property.
They were minors at the time of the execution of the document. D was 17 years old,
E was 14and F was 12; and they were made to believe by A, B and C that unless
Page 262 of 391
they sign the document they will not get any share. Z was not present then. In
January 1974, D,E and F filed an action in court to nullify the suit alleging they
discovered the fraud only in 1973.
(a) Can the minority of D, E and F be a basis to nullify the partition? Explain
your answer.
(b) How about fraud? Explain your answer.
Answer;
(a) Yes, minority can be a basis to nullify the partition because D, E and F
were not properly represented by their parents or guardians at the time they
contracted the extra-judicial partition. (Articles 1327. 1391, Civil Code).
(b) In the case of fraud, when through Insidious words or machinations of one
party the other is induced to enter into the contract without which he would not have
agreed to, the action still prosper because under Art, 1391 of the Civil Code, in case
of fraud, the action for annulment may be brought within four years from the
discovery of the fraud.
07; Contracts; voidable contracts
1976 No. VIII-a
A, 20, contracted in 1970 with B, 22, an operator of a driving range for
golfers, to supply the latter with golf balls. May B annul the contract on the ground
that A was incapable of giving consent to a contract? Explain.
Answer
No, under the law on contracts, only the incapacitated person can bring the
action to annul, based on the ground of estoppel. Hence, only A may bring the
action to annul. (Article 1397)
1976 No. VIII-b
If A claims that he (A) was drunk when the contract was signed, may an
action to annul the contract be filed by him in 1975? Explain.
Answer
No. Contracts agreed upon in a state of drunkenness are voidable but the
action to annul on that ground must be brought within four (4) years. (Article 1328,
1391,) Here it was brought beyond the 4-year period.
1976 No. VIII-c
If one week after the execution of the contract, B was told by A that A would
have the contract annulled as A was drunk at the time of the signing thereof and A
continued delivering golf balls for 3 years, will such action by A prosper? Explain.
Answer
No. The act of A in performing his obligation under the contract after he
became of age, constituted ratification and cleanses the contract of all its defects
from the time of execution of the agreement. (Article 1396) He may also be barred
from bringing the action on the ground of estoppel.
07; Contracts; voidable contracts
1978 No. VI-a
A purchased from XYZ subdivision company a lot of three hundred (300)
square meters in Cebu on December 3, 1973. Transfer Certificate of Title No. 2537
was issued in his name with the annotation at the back thereof that the lot is subject
to the condition that it cannot be resold within the period of ten (10) years and if he is
Page 263 of 391
forced to resell, he may resell it only to XYZ subdivision company. A year later, A
executed a real estate mortgage in favor of B upon a P5,000 loan, which was
registered with the Register of Deeds. A and B were aware of the condition in favor
of XYZ subdivision company. Upon A's failure to redeem the property in 1927, B was
the highest bidder at the foreclosure sale conducted by the sheriff and was issued a
certificate of sale which was registered with the Register of Deeds. A later brought
an action to annul the sale on the ground that it violated the condition. Decide the
case with reasons.
Answer:
The action brought by A against B to annul the sheriff's sale will not prosper.
Both parties are equally guilty of breaching the condition in favor of XYZ subdivision,
a condition known to both of them. Under both the principle of estoppel and of pari
delicto, neither one of sale. Besides, and this is decisive, the proper party who
should institute the action is XYZ subdivision and not A. Under the law on defective
contracts, in actions for annulment of rescission, it is the party who is damaged or
prejudiced who should institute the action and not the party who, with another or
others, was responsible for the breach or damage.
(NOTE: The above answer is based on Art. 1431, with respect to pari delicto,
and Art 1397, Civil Code, with respect to proper party in actions for annulment.)
07; Contracts; voidable contracts
1979 No. X
Mrs. S borrowed P20,000 from PG, She and her 19-year old son, Mario,
signed the promissory note for the loan, which note did not say anything- about the
capacity of the signers. Mrs. S made partial payments little by little. After seven (7)
years she died leaving a balance of P10,000.00 on the note. PG demanded
payment from Mario who refused to pay. When sued for the amount, Mario raised
the defense: that when he signed the note he was still a minor. Should the defense
be sustained? Why?
Answer
The defense should be sustained. Mario cannot be bound by his signature in
the promissory note. It must be observed that the promissory note does not say
anything about the capacity of the signers. In other words, there is no active fraud or
misrepresentation; there is merely silence or constructive fraud or
misrepresentation. It would have been different if the note says that Mario is of age.
The principle of estoppel would then apply. Mario would not be allowed to invoke the
defense of minority. The promissory note would then have all of the effects of a
perfectly valid note. Hence, as far as Mario's share in the obligation is concerned,
the promissory note is voidable because of minority or non-age. He cannot,
however, be absolved entirely from monetary responsibility. Under the Civil Code,
even if his written contract is voidable because of minority he shall make restitution
to the extent that he may have been benefited by the money received by him (Art.
1399, Civil Code). (Braganza vs. Villa Abrille, L-12471, April 13, 1959).
07; Estoppel
1987 No. 7:
Fred sold to Juan a parcel of land, belonging to his minor son, Lino, then
under his guardianship, without judicial approval. After the sale, Juan immediately
took possession of the land, built a house and religiously paid the taxes thereon.
Nine years thereafter, Lino, no longer a minor, rented the ground floor of the house
built by Juan. Lino paid the rent for the first month, then stopped paying. Two years
thereafter, when pressed for payment of the accrued rent. Lino refused, claiming
Page 264 of 391
ownership over the property, alleging that the sale of the property to Juan while he
was A minor without the approval of the guardianship court rendered the sale null
and void.
Is the claim of Lino valid and meritorious.? Explain. Answer
No, Lino's claim is not valid and not meritorious because Lino is in estoppel.
A lessee cannot assail the right and title of the lessor and cannot claim ownership as
against the lessor. The fact that the sale was made while Lino was a minor is of no
moment because he recognized and ratified the contract after he was already of
majority age.
Answer;
No, Lino's claim is not valid and not meritorious because Juan had already
become the owner of the land by ordinary acquisitive prescription through adverse
possession of the land for over ten (10) years.
Answer
No, Lino's claim is not valid and not meritorious. Lino can no longer recover
the land because of laches.
07; Estoppel
1989 No. 15:
(1) What do you understand by ESTOPPEL? What are the different kinds of
estoppel? Explain.
Answer:
The Civil Code enumerates only two (2) kinds of estoppel: estoppel in pais or,
by conduct and estoppel by deed. Estoppel in pais or by conduct arises when one
by his act, representation, oral admission or by his silence induces another to
believe certain facts to exist and the other realize an act on such belief.
Estoppel by deed is that by virtue of which a party to a deed and his privies
are precluded from asserting as against the other party any right or title in
derogation of the deed or any fact asserted therein.
Alternative Extended Answer:
The Civil Code gives two (2) kinds of estoppel, namely: estoppel in pais and
estoppel by deed; and jurisprudence gives a third, namely: estoppel by laches.
Estoppel in pais or by conduct arises when one by his act, representation,
oral admission or by his silence induces another to believe certain facts to exist and
the other realize an act on such belief.
Estoppel by deed is that by virtue of which a party to a deed and his privies
are precluded from asserting as against the other party by which any right or title in
derogation of the deed or any fact asserted therein.
Laches is negligence or omission to assert a right within a reasonable time
giving rise to the presumption that the party entitled to assert it either has
abandoned it or declined to assert it.
07; Estoppel; laches
1988 No. 2:
(c) Since 1935, Janice possessed alone a parcel of land which she co-
owned with Lenny. In 1970, with the knowledge of Lenny, Janice obtained a
Torrens title over the land in her own name alone. On August 1, 1988, Lenny
Page 265 of 391
brought an action against Janice for reconveyance of her share, Janice set up the
defense of laches. Will the defense prosper? Reasons.
Answer:
(c) It is submitted that the defense of laches will prosper. As held by the
Supreme Court in several notable decisions, in order that the doctrine of laches or
"stale demands" can be applied, the following elements must concur: (1) Conduct on
the part of the defendant, or of one under whom he claims, giving rise to the
situation of which complaint is made and for which the complaint seeks a remedy;
(2) delay in asserting the complainant's rights, the complainant having had
knowledge or notice, of the defendant's conduct and having been afforded an
opportunity to institute a suit (3) lack of knowledge or notice on the part of the
defendant that the complainant would assert the right on which he bases the suit-
and (4) injury or prejudice to the defendant in the event relief is accorded to the
complainant, or the suit is not held to be barred (Miguel vs. Catalino, 26 SCRA 234).
All of these elements are present in the instant case. As a matter of fact, the doctrine
was applied to a case wherein co-heir and another were able, through fraud, to
register a tract of land in their names. According to the Supreme Court, the action for
reconveyance brought by the other co-heirs more than twenty years later is now
barred not only by extinctive prescription but also by laches- (Fabian vs. Fabian, 22
SCRA 231).
07; Natural obligations
1977 No. IX-c
What are natural obligations? Give an example. Answer
Natural obligations are those based on equity and natural law, which are not
enforceable by means of a court action, but which, after voluntary fulfillment by the
obligor, authorize the retention by the obligee of what has been delivered or
rendered by reason thereof. In other words, they refer to those ''obligations without a
sanction susceptible of voluntary performance, but not through compulsion by legal
means". (4 Tolentino, Civil Code, 1956 Ed., p. 588, citing Colin & Capitant)
(NOTE: There are seven examples given in the Civil Code (Arts, 1424 to
1430). Any one of them may be given.)
07; Obligations; alternative obligations
1988 No. 9:
(a) Define alternative and facultative obligations.
Answer:
(a) Alternative obligations refer to those juridical relations which
comprehend several objects or prestations which are due, but the payment or
performance of one of them would be sufficient. On the other hand, facultative
obligations refer to those juridical relations where only one object or prestation has
been agreed upon by the parties to the obligation, but the obligor may deliver or
render another in substitution.
07; Obligations; alternative/facultative obligations
1977 No. VIII-c
Distinguish between alternative and facultative obligations: and between
suspensive and resolutory conditions in obligations.
Answer
Facultative obligations may be distinguished from alternative obligations in
the following ways:
Page 266 of 391
(1) As to object due: In facultative obligations only one object is due, whereas
in alternative obligations several objects are due.
(2) As to compliance: Facultative obligations may be complied with by the
delivery of another object or the performance of another prestation in substitution of
that which is due, whereas alternative obligations may be complied with by the
delivery of one of the objects or by the performance of the prestations which are
alternative due.
(3) As to right of choice: In the first, the right of choice pertains only to the
debtor, whereas in the second, the right of choice may pertain even to the creditor or
to a third person.
(4) As to effect of fortuitous loss: In the first, the loss or impossibility of the
object or prestation which is due without any fault of the debtor is sufficient to
extinguish the obligation, whereas in the second, the loss or impossibility of all of the
objects or prestations which are due without any fault of the debtor is necessary to
extinguish the obligation.
<6) As to effect of culpable loss: In the first, the culpable loss of the object
which the debtor may deliver in substitution before the substitution is effected does
not give rise to any liability on the part of such debtor; in the second, the culpable
loss of any of the objects which are alternately due before the choice is made may
give rise to a liability on the part of the debtor.
(NOTES: A brief statement of the first three distinctions should constitute a
sufficient answer.)
If the suspensive condition is fulfilled, the obligation fulfilled, the obligation is
extinguished. If the first arises or becomes effective; if the resolutory condition is not
fulfilled, no juridical relation is created; if the second is not fulfilled, the juridical
relation is consolidated. In other-words, in the first, rights are not yet acquired, but
there is a hope or expectancy that they will soon be acquired; in the second, rights
are already acquired, but subject to the threat of extinction.
(NOTE: A statement of the substance of the above distinctions should
constitute a sufficient answer.)
07; Obligations; civil vs natural oblig
2004 No. II
A. Distinguish briefly but clearly between: 3. Civil obligation and natural
obligation.
07; Obligations; civil vs natural obligation
1989 No. 15:
(2) How is a civil obligation distinguished from a natural obligation? Give an
example of a natural obligation.
Answer:
Civil obligations give a right of action to compel their performance. Natural
obligations, not being based on positive law but on equity and natural law, do not
grant a right of action to enforce their performance, but after voluntary fulfillment by
the obligor, they authorize the retention of what has been delivered on rendered by
reason thereof.
Example of a natural obligation (one example out of any of the following):
1. When a right to sue upon a civil obligation has lapsed by extinctive
prescription, the obligor who voluntarily performs the contract cannot recover what
he has delivered or the value of the service he has rendered.
Page 267 of 391
2. When without the knowledge or against the will of the debtor a third
person pays a debt which the obligor is not legally bound to pay because the
action thereon has prescribed, but the debtor later voluntarily reimburses the third
person, the obligor cannot recover what he has paid.
3. When a minor between eighteen and twenty-one years of age who has
entered into a contract without the consent of the parent or guardian, after the
annulment of the contract voluntarily returns the whole thing or price received,
notwithstanding the fact that he has not been benefited thereby, there is no right to
demand the thing or price thus returned.
4. When a minor between eighteen and twenty-one years of age, who has
entered into a contract without the consent of the parent or guardian, voluntarily
pays a sum of money or delivers a fungible thing in fulfillment of the obligation, there
shall be no right to recover the same from the obligee who has spent or consumed it
in good faith.
5. When, after an action to enforce a civil obligation has failed, the defendant
voluntarily performs the obligation, he cannot demand the return of what he has
delivered or the payment of the value of the service he has rendered,
6. When a testate or intestate heir voluntarily pays a debt of the decedent
exceeding the value of the property which he received by will or by the law of
intestacy from the estate of the deceased, the payment is valid and cannot be
rescinded by the payer.
7. When a will is declared void because it has not been executed in
accordance with the formalities required by law, but one of the intestate heirs, after
the settlement of the debts of the deceased, pays a legacy in compliance with a
clause in the defective will, the payment is effective and irrevocable.
07; Obligations; conditional obligations
1997 No, 14;
In two separate documents signed by him, Juan Valentino "obligated" himself
each to Maria and to Perla, thus -
'To Maria, my true love, I obligate myself to give you my one and only horse
when I feel like It."
- and -
'To Perla, my true sweetheart, I obligate myself to pay you the P500.OO I
owe you when I feel like it."
Months passed but Juan never bothered to make good his promises. Maria
and Perla came to consult you on whether or not they could recover on the basis of
the foregoing settings.
What would your legal advice be? Answer:
I would advise Maria not to bother running after Juan for the latter to make
good his promise. [This is because a promise is not an actionable wrong that allows
a party to recover especially when she has not suffered damages resulting from
such promise, A promise does not create an obligation on the part of Juan because
it is not something which arises from a contract, law, quasi-contracts or quasi-delicts
(Art, 1157)]. Under Art. 1182, Juan's promise to Maria is void because a conditional
obligation depends upon the sole will of the obligor.
As regards Perla, the document is an express acknowledgment of a debt,
and the promise to pay what he owes her when he feels like it Is equivalent to a
promise to pay when his means permits him to do so, and is deemed to be one with
Page 268 of 391
an indefinite period under Art. 1180. Hence the amount Is recoverable after Perla
asks the court to set the period as provided by Art. 1197, par. 2.
07; Obligations; conditional obligations
1999 No XI
In 1997, Manuel bound himself to sell Eva a house and lot which is being
rented by another person, if Eva passes the 1998 bar examinations. Luckily for Eva,
she passed said examinations.
(a) Suppose Manuel had sold the same house and lot to another before
Eva passed the 1998 bar examinations, is such sale valid? Why? (2%)
(b) Assuming that it is Eva who is entitled to buy said house and lot, is she
entitled to the rentals collected by Manuel before she passed the 1998 bar
examinations? Why? (3%)
ANSWER:
(a) Yes, the sale to the other person is valid as a sale with a resolutory
condition because what operates as a suspensive condition for Eva operates a
resolutory condition for the buyer.
FIRS T ALTERNATIVE ANS WER:
Yes, the sale to the other person is valid. However, the buyer acquired the
property subject to a resolutory condition of Eva passing the 1998 Bar Examinations.
Hence, upon Eva's passing the Bar, the rights of the other buyer terminated and Eva
acquired ownership of the property.
SECOND ALTERNATIVE ANSWER:
The sale to another person before Eva could buy it from Manuel is valid, as
the contract between Manuel and Eva is a mere promise to sell and Eva has not
acquired a real right over the land assuming that there is a price stipulated in the
contract for the contract to be considered a sale and there was delivery or tradition
of the thing sold.
(b) No, she is not entitled to the rentals collected by Manuel because at the
time they accrued and were collected, Eva was not yet the owner of the property.
FIRST ALTERNATIVE ANSWER:
Assuming that Eva is the one entitled to buy the house and lot, she is not
entitled to the rentals collected by Manuel before she passed the bar examinations.
Whether it is a contract of sale or a contract to sell, reciprocal prestations are
deemed imposed A for the seller to deliver the object sold and for the buyer to pay
the price. Before the happening of the condition, the fruits of the thing and the
interests on the money are deemed to have been mutually compensated under
Article 1187.
SECOND ALTERNATIVE ANSWER:
Under Art. 1164, there is no obligation on the part of Manuel to deliver the
fruits (rentals) of the thing until the obligation to deliver the thing arises. As the
suspensive condition has not been fulfilled, the obligation to sell does not arise.
07; Obligations; conditional obligations
2000 No XV
b) Pedro promised to give his grandson a car if the latter will pass the bar
examinations. When his grandson passed the said examinations, Pedro refused to
give the car on the ground that the condition was a purely potestative one. Is he
correct or not? (2%)
Page 269 of 391
SUGGESTED ANSWER:
No, he is not correct. First of all, the condition is not purely potestative,
because it does not depend on the sole will of one of the parties. Secondly, even if it
were, it would be valid because it depends on the sole will of the creditor (the donee)
and not of the debtor (the donor).
07; Obligations; conditional obligations
2003 No XIII.
Are the following obligations valid, why, and if they are valid, when is the
obligation demandable in each case?
(a) If the debtor promises to pay as soon as he has the means to pay;
(b) If the debtor promises to pay when he likes;
(c) If the debtor promises to pay when he becomes a lawyer;
(d) If the debtor promises to pay if his son, who is sick with cancer, does
not die within one year. 5%
SUGGESTED ANSWER:
(a) The obligation is valid. It is an obligation subject to an indefinite period
because the debtor binds himself to pay when his means permit him to do so (Article
1180, NCC). When the creditor knows that the debtor already has the means to pay,
he must file an action in court to fix the period, and when the definite period as set
by the court arrives, the obligation to pay becomes demandable 9Article 1197,
NCC).
(b) The obligation to pay when he likes is a suspensive condition the
fulfillment of which is subject to the sole will of the debtor and, therefore the
conditional obligation is void. (Article 1182, NCC).
(c) The obligation is valid. It is subject to a suspensive condition, i.e. the
future and uncertain event of his becoming a lawyer. The performance of this
obligation does not depend solely on the will of the debtor but also on other factors
outside the debtors control.
(d) The obligation is valid. The death of the son of cancer within one year
is made a negative suspensive condition to his making the payment. The obligation
is demandable if the son does not die within one year (Article 1185, NCC).
07; Obligations; conditional obligations
1988 No. 8:
(b) Distinguish between the effects of suspensive and resolutory conditions
upon an obligation.
Answer:
(b) It is evident that a resolutory condition affects the obligation to which it is
attached in a manner which is diametrically opposed to that of a suspensive
condition. If the suspensive condition is fulfilled, the obligation arises or becomes
effective if the resolutory condition is fulfilled, the obligation is extinguished. If the
first is not fulfilled, the juridical relation is created; if the second is not fulfilled, the
juridical relation is consolidated. In other words, in the first, rights are not yet
acquired, but there is a hope or expectancy that they will soon be acquired; in the
second, rights are already acquired, but subject to the threat of extinction (8
Manresa, 5th Ed., Bk. 1, p. 311.)
07; Obligations; conditional obligations
Page 270 of 391
1975 No. XIII
A owed B a certain sum of money. C wrote B a letter stating that he would be
the one to take care of A's debt as soon as A had made a shipment of logs to Japan.
A never made such shipment, C did not pay B. Is C liable to B? Explain.
Answer
No, C is not liable to B. C did not assume the obligation of A. C merely stated
that he will "take care" of A's debt. Moreover, even if C assumed liability, the
suspensive conditionnamely, the shipment by A of logs to Japan, was never
fulfilled. C's obligation never arose. Under Article 1181 of the Civil Code, in
conditional obligations, the acquisition of rights, as well as the extinguishment or
loss of those already acquired, shall depend upon the happening of the event which
constitutes the condition. (Villanueva v. Girged, 110 Phil. 478)
07; Obligations; exemption from liability due to fortuitous events; exceptions
1983 No. 11
Cite three instances where a person is made civilly liable for failure to comply
with his obligations although he was prevented from doing so by a fortuitous event.
Answer
(a) When the loss is due to the debtor's fault.
(b) When the debtor has incurred in delay,
(c) When he has promised to deliver the same thing to 2 or more different
persons who do not have the same interest,
(d) When it is expressly stipulated.
(e) When the law expressly provides so,
(f) When the nature of the obligation requires the assumption of risk, and
(g) When the thing to be delivered is indeterminate, the loss of a thing of
the same kind, even if due to a fortuitous event, does not extinguish the
obligation.
07; Obligations; extinguishment of a cause of action
2004 No. I
B. TX filed a suit for ejectment against BD for non-payment of condominium
rentals amounting to P150,000. During the pendency of the case, BD offered and TX
accepted the full amount due as rentals from BD, who then filed a motion to dismiss
the ejectment suit on the ground that the action is already extinguished.
Is BDs contention correct? Why or why not? Reason. (5%)
07; Obligations; extinguishment; assignment of rights
2001 No XI
The sugar cane planters of Batangas entered into a long-term milling contract
with the Central Azucarera de Don Pedro Inc. Ten years later, the Central assigned
its rights to the said milling contract to a Taiwanese group which would take over the
operations of the sugar mill. The planters filed an action to annul the said
assignment on the ground that the Taiwanese group was not registered with the
Board of Investments. Will the action prosper or not? Explain briefly. (5%)
(Note: The question presupposes knowledge and requires the application of
the provisions of the Omnibus Investment Code, which properly belongs to
Commercial law)
Page 271 of 391
SUGGESTED ANSWER
The action will prosper not on the ground invoked but on the ground that the
farmers have not given their consent to the assignment. The milling contract
imposes reciprocal obligations on the parties. The sugar central has the obligation to
mill the sugar cane of the farmers while the latter have the obligation to deliver their
sugar cane to the sugar central. As to the obligation to mill the sugar cane, the sugar
central is a debtor of the farmers. In assigning its rights under the contract, the sugar
central will also transfer to the Taiwanese its obligation to mill the sugar cane of the
farmers. This will amount to a novation of the contract by substituting the debtor with
a third party. Under Article 1293 of the Civil Code, such substitution cannot take
effect without the consent of the creditor. The formers, who are creditors as far as
the obligation to mill their sugar cane is concerned, may annul such assignment for
not having given their consent thereto.
ALTERNATIVE ANSWER
The assignment is valid because there is absolute freedom to transfer the
credit and the creditor need not get the consent of the debtor. He only needs to
notify him.
07; Obligations; extinguishment; compensation
2002 No IX.
Stockton is a stockholder of Core Corp. He desires to sell his shares in Core
Corp. In view of a court suit that Core Corp. has filed against him for damages in the
amount of P 10 million, plus attorneys fees of P 1 million, as a result of statements
published by Stockton which are allegedly defamatory because it was calculated to
injure and damage the corporations reputation and goodwill.
The articles of incorporation of Core Corp. provide for a right of first perusal in
favor of the corporation. Accordingly, Stockton gave written notice to the corporation
of his offer to sell his shares of P 10 million. The response of Core corp. was an
acceptance of the offer in the exercise of its rights of first refusal, offering for the
purpose payment in form of compensation or set-off against the amount of damages
it is claiming against him, exclusive of the claim for attorneys fees. Stockton rejected
the offer of the corporation, arguing that compensation between the value of the
shares and the amount of damages demanded by the corporation cannot legally
take effect. Is Stockton correct? Give reason for your answer. (5%)
SUGGESTED ANSWERS:
Stockton is correct. There is no right of compensation between his price of P
10 million and Core Corp.s unliquidated claim for damages. In order that
compensation may be proper, the two debts must be liquidated and demandable.
The case for the P 10 million damages being still pending in court, the corporation
has as yet no claim which is due and demandable against Stockton.
ANOTHER MAIN ANSWER:
The right of first refusal was not perfected as a right for the reason that there
was a conditional acceptance equivalent to a counter-offer consisting in the amount
of damages as being credited on the purchase price. Therefore, compensation did
not result since there was no valid right of first refusal (Art. 1475 & 1319, NCC)
ANOTHER MAIN ANSWER:
Even [if] assuming that there was a perfect right of first refusal, compensation
did not take place because the claim is unliquidated.
07; Obligations; extinguishment; compensation
Page 272 of 391
1981 No. 8
"B" borrowed from "C" Pl,000.00 payable in one year. When "C" was in the
province, "C's" 17-year old son, borrowed P500 from "B" for his school tuition.
However, the son spent it instead nightclubbing. When the debt to "C" fell due, "B"
tendered only P500, claiming compensation on the P500 borrowed by "C's" son,
a) Is there legal compensation? Why?
b) Suppose the minor son actually used the money for school tuition, would
the answer be different? Reasons.
Answer
(a) There is no legal compensation. Under the Civil Code, in order that there
will be a valid and effective compensation, it is essential that there must be two
parties, who in their own right, are principal creditors and principal debtors of each
other. In the instant case, "C" cannot be considered as a party to the act of his 17-
year old son in borrowing P500.00 from "B ". Consequently, he did not become a
principal debtor of "B"; neither did "B" become a principal creditor of "C". Therefore,
there can be no partial compensation of the P1 ,000,00 borrowed by "B" from "C".
(Note: The above answer is based on Arts. 1278 and 1279, No. (1), of the
Civil Code and on decided cases.)
(b) There would be no difference in any answer. There will still be no legal
compensation. The fact that "C's" son actually used the P500.00 for his school
tuition did not make "C" a party to the contract between his son and "B". Therefore,
"C" is not the principal debtor of "B" and "B" is not the
principal creditor of "C" with respect to said amount.

(Note: The above answer is based on Arts. 1278 and 1279, No. UK Civil
Code.)
07; Obligations; extinguishment; compensation vs payment
1998 No XIV.
1. Define compensation as a mode of extinguishing an obligation, and
distinguish it from payment. |2%]
2. X, who has a savings deposit with Y Bank in the sum of P1 ,000,000.00,
incurs a loan obligation with the said Bank in the sum of P800.000.00 which has
become due. When X tries to withdraw his deposit, Y Bank allows only P200.000.00
to be withdrawn, less service charges, claiming that compensation has extinguished
its obligation under the savings account to the concurrent amount of X's debt. X
contends that compensation is improper when one of the debts, as here, arises from
a contract of deposit. Assuming that the promissory note signed by X to evidence
the loan does not provide for compensation between said loan and his savings
deposit, who is correct? [3%]
Answer:
1. Compensation is a mode of extinguishing to the concurrent amount, the
obligations of those persons who In their own right are reciprocally debtors and
creditors of each other (Tolentino, 1991 ed., p. 365, citing 2 Castan 560 and Francia
vs. IAC. 162 SCRA 753). It involves the simultaneous balancing of two obligations in
order to extinguish them to the extent in which the amount of one is covered by that
of the other. (De Leon, 1992 ed., p. 221, citing 8 Manresa 401).
Payment means not only delivery of money but also performance of an
obligation (Article 1232, Civil Code). In payment, capacity to dispose of the thing
Page 273 of 391
paid and capacity to receive payment are required for debtor and creditor,
respectively: in compensation, such capacity is not necessary, because the
compensation operates by law and not by the act of the parties. In payment, the
performance must be complete; while in compensation there may be partial
extinguishment of an obligation (Tolentino, supra)
Answer:
2. Y bank is correct. An. 1287, Civil Code, does not apply. All the requisites
of Art. 1279, Civil Code are present. In the case of Gullas vs. PNB [62 Phil. 519), the
Supreme Court held: "The Civil Code contains provisions regarding compensation
(set off) and deposit. These portions of Philippine law provide that compensation
shall take place when two persons are reciprocally creditor and debtor of each other.
In this connection, it has been held that the relation existing between a depositor
and a bank is that of creditor and debtor, x x x As a general rule, a bank has a right
of set off of the deposits in its hands for the payment of any indebtedness to it on the
part of a depositor." Hence, compensation took place between the mutual
obligations of X and Y bank.
07; Obligations; extinguishment; compensation: payment: confusion: set-off
1977 No X-b
Differentiate compensation from payment, from confusion and from set-off.
Answer
Compensation may be distinguished from payment in the following ways:
(1) The requisites prescribed by law for compensation are different from
those prescribed by law for payment.
(2) Compensation takes effect by operation of law, whereas payment takes
effect by act of the parties.
(3) Capacity to give and to acquire is not necessary in compensation, but it
is essential in payment.
(4) Compensation is, as a rule, partial, whereas payment is, as a rule,
complete and indivisible.
Compensation may be distinguished from confusion in the following ways;
(1) As to number of persons, in compensation there must be two persons,
who, in their own right, are creditors and debtors of each other, whereas in
confusion there is only one person in whom is merged the qualities of creditor and
debtor.
(2) As to number of obligations, in compensation there must be at least two,
whereas in confusion there is only one.
Compensation may be distinguished from set-off or counterclaim in the
following ways:
(1) Compensation requires that the two debts must consists in money, or if
the things due are fungibles, they must be of the same kind and quality, but in
counterclaim this is not necessary.
(2) Compensation, as a general rule, requires that the debts must be
liquidated, but counterclaim does not.
(3) Compensation need not be pleaded, whereas a counterclaim must be
pleaded to be effectual.
07; Obligations; extinguishment; condonation
Page 274 of 391
2000 No VII
a) Arturo borrowed P500,000.00 from his father. After he had paid
P300,000.00, his father died. When the administrator of his father's estate requested
payment of the balance of P200,000.00. Arturo replied that the same had been
condoned by his father as evidenced by a notation at the back of his check payment
for the P300,000.00 reading: "In full payment of the loan". Will this be a valid
defense in an action for collection? (3%)
SUGGESTED ANSWER;
It depends. If the notation "in full payment of the loan" was written by Arturo's
father, there was an implied condonation of the balance that discharges the
obligation. In such case, the notation is an act of the father from which condonation
may be inferred. The condonation being implied, it need not comply with the
formalities of a donation to be effective. The defense of full payment will, therefore,
be valid.
When, however, the notation was written by Arturo himself. It merely proves
his intention in making that payment but in no way does it bind his father (Yam v.
CA, G.R No. 104726. 11 February 1999). In such case, the notation was not the act
of his father from which condonation may be Inferred. There being no condonation
at all. the defense of full payment will not be valid.
ALTERNATIVE ANSWER:
If the notation was written by Arturo's father, it amounted to an express
condonation of the balance which must comply with the formalities of a donation to
be valid under the 2nd paragraph of Article 1270 of the New Civil Code. Since the
amount of the balance is more than 5,000 pesos, the acceptance by Arturo of the
condonation must also be in writing under Article 748. There being no acceptance in
writing by Arturo, the condonation is void and the obligation to pay the balance
subsists. The defense of full payment is, therefore, not valid. In case the notation
was not written by Arturo's father, the answer is the same as the answers above.
07; Obligations; extinguishment; dation
1986 No. 9.
On due date, Mayutang, finding himself unable to pay Makaragdag his
P5OO,OOO.OO obligation, proposed in a letter to Makaragdag that he would deed
over to Makaragdag his Mercedes Benz car, "to be applied to the amount which I
owe you." The following week, Mayutang sent the car to Makaragdag with the proper
deed of conveyance. It was accepted.
Was the arrangement a valid way of settling the obligation? Explain.
After the delivery of the car would Makaragdag have any further claim against
Mayutang if the value of the car is found to be Less than the P500,000.00
obligation? Explain.
Answer
The general rule according to commentators (Castan and Manresa) is that
dation extinguishes in full the obligation-the exception is a contrary agreement
because then it becomes assignment of rights - hence valid only up to the value.
Answer - The arrangement was a valid way of settling the obligation. The law
provides that payment for an obligation may be made by delivery of other property.
Answer - The creditor can claim the deficiency because the-debt shall only be
extinguished up to extent of the value of the property given to him, since the
agreement was that the car was "to be applied "to the amount owed.
Page 275 of 391
Answer - Yes, the arrangement was a valid way of settling the obligation.
Under the Civil Code, there are several special forms of payment which will have all
of the effects of a valid payment. One of them is dation in payment (dacion en pago).
It is defined as the transmission of the ownership of a thing by the debtor to the
creditor as the accepted equivalent of the performance of an obligation, According to
the Civil Code, the law on sales shall govern the transaction. Thus, in the instant
problem, the Mercedes Benz car is considered the object of the contract of sale,
while the debt of P500,000 is considered the purchase price.
If the value of the car is found to be less than the P500,000.00 obligation,
would Makaragdag have any further monetary claim against Mayutang? Normally,
dacion en pago has the effect of extinguishing the obligation to the extent of the
value of the thing delivered either as agreed upon or as may be proved, unless the
silence of the parties signifies that they consider the delivery of the thing as the
equivalent of the performance of the obligation. It must be observed, however, that
Makaragdag accepted the delivery of the car without any protest or objection. The
entire obligation, therefore, is deemed fully complied with.
(Note: The above answer is based upon Arts. 1245 and 1235 of the Civil
Code and upon Lopez vs. CA, 114 SCRA 671.)
Answer - The arrangement is a valid way of settling the obligation which is
known as "dacion en pago." As to whether the delivery of the car was in "full
satisfaction" of the debt or to be merely "applied" to the whole indebtedness is
another question. The interpretation of their agreement would depend on the
difference between the value of the car and P500,000.00. If the difference is very
great, the intention of the parties would be that the car is not in "full satisfaction" of
the debt.
07; Obligations; extinguishment; dation in payment vs assignment
1989 No. 8:
(2) What is dation in payment and how is it distinguished from assignment of
property?
Answer:
Dation in payment is a special form of payment whereby property is alienated
to the creditor in satisfaction of a debt in money.
Assignment of property, or payment by cession, is a special form of payment
whereby the debtor cedes or assigns his property to his creditors so that the
proceeds thereof will be applied in payment of his debts.
Alternative Answer;
In dation in payment whereby property is given by the debtor to the creditor in
payment of a debt in money, there is only one creditor. In assignment of property,
there are several creditors.
In the former, the debtor may be solvent. In the latter, there may be partial
insolvency.
In the former, particular property is ceded. In the latter, all the property of the
debtor is ceded.
In the former, the particular obligation is extinguished in whole or in part as
agreed upon. In the latter, it releases the debtor from the net proceeds only, unless
otherwise agreed or intended.
07; Obligations; extinguishment; extraordinary inflation or deflation
2001 No X
Page 276 of 391
On July 1, 1998, Brian leased an office space in a building for a period of five
years at a rental rate of P1,000.00 a month. The contract of lease contained the
proviso that "in case of inflation or devaluation of the Philippine peso, the monthly
rental will automatically be Increased or decreased depending on the devaluation or
inflation of the peso to the dollar." Starting March 1, 2001, the lessor increased the
rental to P2,000 a month, on the ground of inflation proven by the fact that the
exchange rate of the Philippine peso to the dollar had Increased from P25.00=$1.00
to P50.00=$1.00. Brian refused to pay the increased rate and an action for unlawful
detainer was filed against him. Will the action prosper? Why? (5%)
SUGGESTED ANSWER
The unlawful detainer action will not prosper. Extraordinary inflation or
deflation is defined as the sharp decrease in the purchasing power of the peso. It
does not necessarily refer to the exchange rate of the peso to the dollar. Whether or
not there exists an extraordinary inflation or deflation is for the courts to decide.
There being no showing that the purchasing power of the peso had been reduced
tremendously, there could be no inflation that would justify the increase in the
amount of rental to be paid. Hence, Brian could refuse to pay the increased rate.
ALTERNATIVE ANSWER.
The action will not prosper. The existence of inflation or deflation requires an
official declaration by the Bangko Sentral ng Pilipinas.
ALTERNATIVE ANSWER:
The unlawful detainer action will prosper. It is a given fact in the problem, that
there was Inflation, which caused the exchange rate to double. Since the contract
itself authorizes the increase in rental in the event of an inflation or devaluation of
the Philippine peso, the doubling of the monthly rent is reasonable and is therefore a
valid act under the very terms of the contract. Brian's refusal to pay is thus a ground
for ejectment.
07; Obligations; extinguishment; loss
1994 No, 17;
Dino sued Ben for damages because the latter had failed to deliver the
antique Marcedes Benz car Dino had purchased from Ben, which wasby
agreementdue for delivery on December 31,1993. Ben, in his answer to Dino's
complaint, said Dino's claim has no basis for the suit, because as the car was being
driven to be delivered to Dino on January 1, 1994, a reckless truck driver had
rammed into the Mercedes Benz. The trial court dismissed Dino's complaint, saying
Ben's obligation had. indeed, been extinguished by force majeure.
Is the trial court correct? Alternative Answers:
a) No. Article 1262, New Civil Code provides, "An obligation which consists
in the delivery of a determinate thing shall be extinguished if it should be lost or
destroyed without the fault of the debtor, and before he has incurred in delay.
b) The judgment of the trial court is incorrect. Loss of the thing due by
fortuitous events or force majeure is a valid defense for a debtor only when the
debtor has not incurred delay. Extinguishment of liability for fortuitous event
requires that the debtor has not yet incurred any delay. In the present case, the
debtor was in delay when the car was destroyed on January 1,1993 since it was due
for delivery on December 31, 1993. (Art. 1262 Civil Code)
c) It depends whether or not Ben the seller, was already in default at the
time of the accident because a demand for him to deliver on due date was not
complied with by him. That fact not having been given in the problem, the trial court
Page 277 of 391
erred in dismissing Dino's complaint. Reason: There is default making him
responsible for fortuitous events Including the assumption of risk or loss.
If on the other hand Ben was not in default as no demand has been sent to
him prior to the accident, then we must distinguish whether the price has been paid
or not. If it has been paid, the suit for damages should prosper but only to enable the
buyer to recover the price paid. It should be noted that Ben. the seller, must bear the
loss on the principle of res perit domino. He cannot be held answerable for damages
as the loss of the car was not imputable to his fault or fraud. In any case, he can
recover the value of the car from the party whose negligence caused the accident. If
no price has been paid at all, the trial court acted correctly in dismissing the
complaint.
07; Obligations; extinguishment; loss; impossible service
1993 No. 11
In 1971, Able Construction, Inc. entered into a contract with Tropical Home
Developers, Inc. whereby the former would build for the latter the houses within Its
subdivision. The cost of each house, labor and materials included, was
P100,000.00. Four hundred units were to be constructed within five years. In 1973,
Able found that it could no longer continue with the job due to the increase in the
price of oil and its derivatives and the concomitant worldwide spiraling of prices of all
commodities, including basic raw materials required for the construction of the
houses. The cost of development had risen to unanticipated levels and to such a
degree that the conditions and factors which formed the original basis of the contract
had been totally changed. Able brought suit against Tropical Homes praying that the
Court relieve it of its obligation.
Is Able Construction entitled to the relief sought?
Answer;
Yes, the Able Construction. Inc. is entitled to the relief sought under Article
1267, Civil Code. The law provides: "When the service has become so difficult as to
be manifestly beyond the contemplation of the parties, the obligor may also be
released therefrom, in whole or in part."
07; Obligations; extinguishment; novation
1977 No. XIV-b
R borrowed P5,000 from H and he authorized his bank to pay the loan. The
bank agreed. Eventually, the bank paid only P2,550.00 H sued both R and the bank.
Discuss the bank's liability.
Answer
The bank cannot be held liable for the remaining P2,500. Even assuming that
H gave his consent to R's proposal that the bank shall pay his indebtedness of
P5,000, in reality, there was no substitution of debtor by delegation resulting in a
novation of the obligation. There was merely an authorization, which was accepted
by the bank, that the latter shall pay R's debt. As it turned out, the bank paid only
P2,500.00 to H. Beyond that amount, the bank cannot be held liable. (Hodges vs.
Key, 111 Phil. 219).
07; Obligations; extinguishment; novation
1988 No. 11:
(a) Suppose that under an obligation imposed by a final judgment, the
liability of the judgment debtor is to pay the amount of P6,000.00 but both the
judgment debtor and the judgment creditor subsequently entered into a contract
Page 278 of 391
reducing the liability of the former to only P4,000.00, is there an implied novation
which will have the effect of extinguishing the judgment obligation and creating a
modified obligatory relation? Reasons.
Answer:
There is no implied novation in this case. We see no valid objection to the
judgment debtor and the judgment creditor in entering into an agreement regarding
the monetary obligation of the former under the judgment referred to. The payment
by the judgment debtor of the lesser amount of P4,000, accepted by the creditor
without any protest or objection and acknowledged by the latter as in full satisfaction
of the money judgment, completely extinguished the judgment debt and released the
debtor from his pecuniary liability.
Novation results in two stipulationsone to extinguish an existing obligation,
the other to substitute a new one in its place. Fundamental it is that novation effects
a substitution or modification of an obligation by another or an extinguishment of one
obligation by the creation of another. In the case at hand, we fail to see what new or
modified obligation arose out of the payment by judgment debtor of the reduced
amount of P4,000 to the creditor. Additionally, to sustain novation necessitates that
the same be so declared in unequivocal terms clearly and unmistakably shown by
the express agreement of the parties or by acts of equivalent importor that there is
complete and substantial incompatibility between the two obligations. (Sandico vs.
Piguing, 42 SCRA322.)
Suggested Alternative Answers To: No. 11(a):
(a) (1) There remains an obligation on the basis of the facts given. There is
no showing in the facts that the P4,000 has been paid so it created a modified
obligatory obligation no longer based on the judgment but based on the novatory
agreement.
(2) There is no implied novation. Instead there has been a partial remission
in the amount of P2,000 leaving P4,000 still enforceable under the judgment.
(3) It can amount to a compromise. A final judgment which has not yet been
fully satisfied may be the subject of a compromise. The compromise partakes the
nature of a novation. Article 204; provides that:
"If one of the parties fails or refuses to abide by the compromise, the other
party may either enforce the compromise or regard it as rescinded and insist upon
his original demand."
(Gatchalian vs, Arlegui 75 SCRA 234; Dormitorio vs. Fernandez 72 SCRA
388).
07; Obligations; extinguishment; novation
1982 No. 19
ABC Trading Co., a domestic corporation engaged in the sale of automobile
spare parts, opened with "X" Bank letter of credit up to the extent of P450,000.00 for
a period of one year. To secure payment thereof, it executed a chattel mortgage
over its stocks-in-trade valued at 7500,000.00. On May 15, and June 15r 1981, Mr.
"Y", president and general manager of ABC Trading drew against this letter of credit
by means of promissory notes in the total amount of P430,000.00, payable within 30
days from the respective dates of the promissory notes with interest of 10%. Upon
maturity of said notes, ABC Trading failed to pay, but was able to negotiate for an
extension of six (6) months within which to pay said amount, in return for the
additional security posted by Mr. "Y" consisting- of a real estate mortgage over his
land in Manila. At the end of 6 months, ABC Trading Co. failed to pay the amount
due despite repeated demands by "X" Bank. "X" Bank filed an action for foreclosure
Page 279 of 391
of the chattel mortgage executed by ABC Trading. ABC Trading opposed said action
contending that the chattel mortgage has been novated by the real estate mortgage
executed by Mr. "Y" in favor of "X" Bank. Is the contention of ABC Trading Co.
tenable? Reasons.
Answer
The contention of ABC Trading Co. that the chattel mortgage has been
novated by the real estate mortgage executed by Mr. "Y" in favor of "X" Bank is
untenable. Well-settled is the rule that in order that there will be a novation, there
must be complete incompatibility between the two obligations. And the test of
incompatibility is simple. All that we have to ask is: Can the two obligations stand
together. If they can then there is no incompatibility. If there is no incompatibility,
then there is no novation. However, if they cannot stand together, then there is
incompatibility. If there is incompatibility, then there is a novation. Applying the test
to the instant case, it is clear that the two obligations can stand together. Therefore,
there is no novation*
(Note: The above answers is based on Arts. 1291(1) and 1292 of the Civil
Code and on decided cases, such as Bank of PI. vs. Herridge, 47 Phil. 57;
Ynchausti & Co. vs. Yulo, 34 Phil. 978; Pascual vs. Lacsamana,, 400 Phil. 381; La
Tondena, vs. Alto Surety & Ins. Co., 101 Phil. 879.)
07; Obligations; extinguishment; novation
1978 No. V-a
A bought from B a parcel of land and paid the purchase price except for an
unpaid balance of P6,000. A, therefore, executed a promissory note for the balance
of P6,000 with interest at ten per cent (10%) to be paid within sixty (60) days. On the
same date, C surety company executed a bond in favor of B for the amount of
P6,000 representing the unpaid balance of the purchase price of the parcel of land,
without any stipulation regarding payment of interest. On the due date, A failed to
pay, and C surety company paid P6,000 to B. B then sued A for the accumulated
interest on the principal of P6,000. A claimed novation of the obligation when B
accepted unqualifiedly the surety bond which merely guaranteed payment of
P6,000. Is A correct? Explain your answer.
Answer
A is not correct. There is no novation so long as there is no agreement that
the principal debtor (a) shall be released from responsibility, Here, there is no such
agreement. True, C surety company executed a bond in favor of B for the amount of
P6,000, but that did not have the effect of releasing A from the obligation. The surety
bond is not a new and separate contract but is merely an accessory of the original
contract entered into by and between A and C surety company on one hand and B
on the other hand. It provided merely for a more definite and solid arrangement for
payment. Therefore, A and B are still bound under their old contract. The former is
still liable for accumulated interests on the principal of P6,000.
(NOTE: The above answer is based on Arts. 1291, No. 1, and 1292, Civil
Code, and on Dungo vs. Lopena, L-18377, Dec. 29, 1962, and Magdalena Estate
vs. Rodriguez, 18 SCRA 967.)
Alternative Answer
A is not correct. The defense of implied novation requires clear and
convincing proof of complete incompatibility between the two obligations. The test is
whether the two obligations can stand together. If they cannot, incompatibility arises,
and the second obligation novates the first. If they can stand together, no
incompatibility results and novation does not take place. Applying this test to the
Page 280 of 391
instant-case, it is clear that the original contract between A and B and the surety
bond executed by C surety company can stand together. The bond is merely an
accessory of the original contract. Therefore, there is no novation.
(NOTE: The above answer-is based on decided cases applying Arts. 1291,
No. 1, and 1292 of the Civil Code.)
07; Obligations; extinguishment; novation
1994 No. 11:
In 1978, Bobby borrowed Pl,000,000.00 from Chito payable in two years. The
loan, which was evidenced by a promissory note, was secured by a mortgage on
real property. No action was filed by Chito to collect the loan or to foreclose the
mortgage. But in 1991, Bobby, without receiving any amount from Chito, executed
another promissory note which was worded exactly as the 1978 promissory note,
except for the date thereof, which was the date of its execution.
1) Can Chito demand payment on the 1991 promissory note in 1994?
2) Can Chito foreclose the real estate mortgage if Bobby fails to make good
his obligation under the 1991 promissory note?
Answer:
1) Yes, Chito can demand payment on the 1991 promissory note in 1994.
Although the 1978 promissory note for P1 million payable two years later or in 1980
became a natural obligation after the lapse of ten (10) years, such natural obligation
can be a valid consideration of a novated promissory note dated in 1991 and
payable two years later, or in 1993.
All the elements of an implied real novation are present:
a) an old valid obligation;
b) a new valid obligation;
c) capacity of the parties;
d) animus novandi or intention to novate; and
e) The old and the new obligation should be incompatible with each other on
all material points (Article 1292). The two promissory notes cannot stand together,
hence, the period of prescription of ten (10) years has not yet lapsed.
2) No. The mortgage being an accessory contract prescribed with the loan.
The novation of the loan, however, did not expressly include the mortgage, hence,
the mortgage is extinguished under Article 1296 of the NCC. The contract has been
extinguished by the novation or extinction of the principal obligation insofar as third
parties are concerned.
07; Obligations; extinguishment; novation
1979 No. XII
T borrowed P10,000.00 from a bank executing a promissory note therefore
wherein it is expressly stated that the note is due 120 days thereafter. On the date of
maturity T pays only the very small amount of P200.00 plus interest in advance for
the renewal of the note for another 60 days. For almost two years the bank was very
lenient with T and allowed him to renew his note in the same manner more than 10
times until finally the bank management had to make demand for payment of the
balance of P8,000.00 after the expiration of the date of last renewal. Upon T's failure
to pay, the bank filed an action for collection of said balance. T raised the defense
that the action is premature because the bank by its conduct had impliedly agreed
Page 281 of 391
that the payment of the note is to be made as the financial means of T warrants.
Should the defense be sustained? Why?
Answer
The defense should not be sustained. The acts of leniency of the bank in
accepting partial payments for a period of two years by virtue of BO many renewals
or extensions should not be interpreted as a novation of the original obligation. The
bank was merely compassionate. It is a well-settled rule in this jurisdiction that the
extension of the period for payment or postponement of the date of payment does
not result in a novation. There is no clear case of incompatibility between such
extension or postponement and the original obligation; neither is there a change in
the obligatory relation of the parties which will alter the essence of the original
obligation.
07; Obligations; extinguishment; payment
1986 No. 8.
Mr. Magaling obtained a judgment against Mr. Mayaman in the amount of
P500,000.00. A writ of execution was issued pursuant to which various personal
properties of Mayaman were levied upon by the sheriff. An auction sale was
scheduled.
Before the appointed day of the auction. Mayaman delivered to the sheriff a
cashier's check of Far East Bank in the amount of .P200,000.00 and enough cash to
cover the remainder of the total amount due. Magaling refused to accept the check
and asked the sheriff to proceed with the auction sale.
Did Magaling have the right to refuse the payment of part of the obligation
with a cashier's check? Explain.
Answer:
Magaling did not have the right to refuse the payment of part of the obligation
with a cashier's check.
The Central Dank Act provides that a check which has been cleared and
credited to the account of the creditor shall be equivalent to a delivery to the creditor
in cash in an amount equal to the amount credited to his account.
Analyzing the above provision, it is clear that the cashier's check of Far East
Bank, a reputable bank, and credited to the account of Mayaman has legal tender
power. Therefore, there was no basis for Magaling in refusing payment of the
obligation.
(Note: The above answer is based on Art. 1249 of the Civil Code. The
exception in New Pacific Timber and Supply -Co. vs. Seneris, 101 SCRA 686, refers
to "Certified Check" because the reason given in that case was based on Sec. 63 of
the Central Bank Act. However, a contrary answer that "Cashier's Check" is
payment in cash may be considered correct because the above case confused
cashier's check with certified check - hence examinee should be given benefit of
doubt.)
Answer - It is well settled that a cashier's check is as good as cash. The law,
however, provides that a creditor may refuse payment if it is other than legal tender.
Although the creditor has the right to refuse the payment in the form of the
check, he cannot do so without any justifiable reason. Under the Chapter on Human
Relations in the Civil Code, a person, in the exercise of his rights, must act with
justice x x x. Therefore, although technically the creditor can refuse the check, he
could be made liable if he refused the cashier's check for no good reason or in bad
faith.
Page 282 of 391
Answer - Although a cashier's check may be as good as cash, still it is not
legal tender, and the sheriff may refuse to accept the check, as it does not produce
the effect of payment until cashed.
07; Obligations; extinguishment; payment
1981 No. 11
"S", an American resident of Manila, about to leave on a vacation, sold his
car to "B" for U.S. $2,000.00, the payment to be made ten days after delivery to "X",
a third party depositary agreed upon, who shall deliver the car to "B" upon receipt by
"X" of the purchase price. It was stipulated that ownership is retained by "S" until
delivery of the car to "X". Five days after delivery of the car to "X", it was destroyed
in a fire which gutted the house of "X", without the fault of either "X" or "B".
a) Is buyer "B" still legally obligated to pay the purchase price? Explain.
b) May seller "S" demand payment in U.S. dollar? Why?
Answer
(a) Yes, B is still legally obligated to pay the purchase price...
(b) The seller "S" cannot demand payment in U.S. dollars. According to the
law, an agreement that payment shall be made in currency other than Philippine
currency is void because it is contrary to public policy. That does not mean,
however, that "S" cannot demand payment from "B." He can demand payment, but
not in American dollars. Otherwise, there would be unjust enrichment at the expense
of another. Payment, therefore, should be made in Philippine currency,
(Note: The above answer is based on R.A. No. 529 and on Ponce vs. CA,
May 31, 1979.)
07; Obligations; extinguishment; payment
1983 No. 13
A owes B P20,000 which became due and payable last October 1, 1983. On
that date, A offered B P10,000 the only money he then had, but B refused to accept
the payment. A thereafter met C, B's 22-year old son, to whom he gave the P10,000
with the request that he turn the money over to B. The money was stolen while in
C's possession.
Was B justified in refusing to accept the payment of A? May he still recover
the full amount of his debt of P20,000? Why?
Answer
Yes, the creditor cannot be compelled to receive partial payments of the
obligation due him, there being no stipulation to the contrary.
B may still demand full payment of the sum due him. The payment to his son,
who does not appear to have been authorized to receive it, is invalid, the creditor not
having received any benefit therefrom.
07; Obligations; extinguishment; payment
1995 No. 12;
In 1983 PHILCREDJT extended loans to Rivett-Strom Machineries, Inc.
(RIVETTT-STROM), consisting of US$10 Million for the cost of machineries
imported and directly paid by PHTLCREDIT, and 5 Million in cash payable in
installments over a period of ten (10) years on the basis of the value thereof
computed at the rate of exchange of the U.S. dollar vis--vis the Philippine peso at
the time of payment.
Page 283 of 391
RIVETT-STROM made payments on both loans which if based on the rate of
exchange In 1983 would have fully settled the loans.
PHILCREDIT contends that the payments on both loans should be based on
the rate of exchange existing at the time of payment, which rate of exchange has
been consistently increasing, and for which reason there would still be a
considerable balance on each loan.
Is the contention of PHILCREDIT correct? Discuss fully. Answer:
As regards the loan consisting of dollars, the contention of PHILCREDIT is
correct. It has to be paid in Philippine currency computed on the basis of the
exchange rate at the time of payment of each installment, as held in Kalalo v. Luz,
34 SCRA 337. As regards the P5 Million loan in Philippine pesos, PHILCREDIT Is
wrong. The payment thereof cannot be measured by the peso-dollar exchange rate.
That will be violative of the Uniform Currency Act (RA, 529] which prohibits the
payment of an obligation which, although to be paid In Philippine currency, Is
measured by a foreign currency. (Palanca v. CA, 238 SCRA 593).
07; Obligations; extinguishment; payment, consignation, set-off
1988 No. 10:
(a) Under the Civil Code, what are the different special forms of payments?
(b) What are the special requisites of consignation in order that it shall
produce the effect of payment?
(c) A treasury warrant payable to Rosenne and indorsed by Boni was
cashed at the Philippine National Bank. The warrant was subsequently dishonored
by the Philippine Treasury. The Bank then applied the deposit of Boni to the
payment of the amount paid for the warrant. Is the action of the Bank in accordance
with law? Reasons.
Answer:
(a) Under the Civil Code, there are actually four special forms of payment.
They are (1) application of payment (Arts. 1252-1254); (2) dation in payment (Art.
1245); (3) payment by cession (Art. 1255): and (4) tender of payment and
consignation (Arts. 1256-1261). Strictly speaking, however, application of payment,
by its very nature, is not a special form of payment,
(b) In order that consignation shall produce the effect of payment, it is not
only essential that it must conform with all of the requisites of payment, but it is also
essential that certain special requirements prescribed by law must be complied with.
The debtor must show;
(1) That there is a debt due;
(2) That the consignation has been made either because the creditor to
whom tender of payment was made refused to accept the payment without just
cause, or because any of the cause stated by law for effective consignation without
previous tender of payment exists (Art. 1256, CC);
(3) The previous notice of the consignation had been given to the persons
interested in the fulfillment of the obligation (Art, 1256, CC);
(4) That the thing or amount due had been placed at the disposal of judicial
authority (Art. 1258, par. 1, CC); and
(5) That after the consignation had been made, the persons interested in the
fulfillment of the obligation had been notified thereof (Art. 1258, par. 2, CC).
Page 284 of 391
(c) Yes, the action of the Bank is in accordance with law. The facts stated in
the above problem are exactly the same as those in the case of Cullas vs. National
Bank, 62 Phil. 519, where the Supreme Court held that a bank has a right of set-off
of the deposit in its hands for the payment of any indebtedness to it on the part of
the depositor. When a person deposits his money at a bank, whether such deposit is
fixed, savings or current, a relationship of creditor and debtor is established between
the depositor and bank. It is, therefore, evident that all of the requisites for
compensation are present in this case.
Committee's Recommendation Re: No. 10(a):
(a) The committee recommends that if application for payment is not
mentioned by the examinee, full credit must likewise be given,
07; Obligations; extinguishment; payment; application
1982 No. 16
The debtor owes his creditor several debts, all of them due, to wit: (1) an
unsecured debt; (2) a debt secured with a mortgage of the debtor's property; (3) a
debt bearing interest; (4) a debt in which the debtor is solidarily liable with another.
Partial payment was made by the debtor. Assuming that the debtor had not
specified the debts to which the payment should be applied and, on the other hand,
the creditor had not specified in the receipt he issued the application of payment,
state the order in which the payment should be applied and your reasons therefore.
Answer
In this case, according to the Civil Code, the debt, which is most onerous to
the debtor, among those due, shall be deemed satisfied.
Analyzing the four debts stated in the problem, the most onerous is No. 4, the
second most onerous is No. 2, the third most onerous is No. 3, and the last onerous
is No. 1, Consequently, the payment should be applied in that order.
(Note: The above answer is based on Art. l254 of the Civil Code and on
decided cases and commentaries of recognized commentators.)
07; Obligations; extinguishment; payment; consignation, when applicable
1984 No. 12
A sold to B a parcel of land with the right to repurchase the same within three
years. A tendered the repurchase of price to B within the prescribed period, but B
refused to accept it. A then brought an action in court for specific performance.
B contends that since A did not deposit the money in court within the
stipulated period for repurchase and the period has now lapsed, A can no longer
repurchase the property. Is this contention correct? Explain.
Answer:
A. Furnished by Office of Justice Palma
No. Consignation is not necessary to compel B to make the resale if he
refused to accept the repurchase price tendered. The provisions of consignation
refer only to obligations. They are not applicable to the right of repurchase which is
not an obligation but a right exercisable purely at the option of A.
B. Comments and Suggested Answer
We agree with the answer of the Bar Examiner. However, if the bar candidate
will attack the problem by holding that there was no default or mora of A because of
the previous tender of payment which was refused by B without any justifiable
Page 285 of 391
cause, and consequently. A can still repurchase the property, it is recommended
that said answer should be properly credited.
07; Obligations; fortuitous events
1988 No. 8:
(a) Mario received from Edgar a pendant with diamonds valued at
P5,OOO.OO to be sold on commission basis or to be returned on demand. In the
evening of August 31, 1987, while he was walking home, two men snatched his
clutch bag containing the pendant and ran away. Subsequently, the snatchers were
apprehended and charged. During the pendency of the criminal case, Edgar brought
an action against Mario for the recovery of the pendant or its value and damages.
Mario interposed the defense of fortuitous event but Edgar contends
(1) That the defense of fortuitous event is untenable because there was
negligence on the part of the defendant; and
(2) That if the defense is untenable, there must be a prior conviction of
robbery before it can be availed of, Decide the case.
Answer:
(a) The factual setting of the above problem is identical to that of Austria vs.
Court of Appeals (39 SCRA 527). In that case the Supreme Court held that
defendant is not liable.
To constitute a caso fortuito that would exempt a person from responsibility, it
is necessary (1) that the event must be independent of the will of the debtor; (2) that
it must be either unforeseeable or unavoidable; (3) that the occurrence must render
it impossible for the debtor to fulfill the obligation in a normal manner; and (b) that
the debtor must be free of, participation in, or aggravation of, the injury to the
creditor.
All of the above requisites or conditions are present in this case. It is
undeniable that in order to completely exonerate the debtor by reason of a fortuitous
event, such debtor must, in addition into the causes itself, be free of any concurrent
or contributory fault or negligence. We believe, however, that her act in traveling
alone in the evening, carrying jewelry of considerable value, cannot be considered
as either concurrent or contributory negligence. While it may be so considered now,
we are not persuaded that the same rule should obtain ten years previously when
the robbery in question took place, for at that time criminality had not by far reached
the levels attained in the present day.
There is likewise no merit in the contention that to allow the fact of robbery to
be recognized in this case before conviction is secured in the criminal action, would
prejudice the latter case, or would result in inconsistency should the accused obtain
an acquittal or should the criminal case be dismissed. It must be realized that a court
finding that a robbery has happened would not necessarily mean that those accused
in the criminal action would be found guilty of the crime; nor would a ruling that those
actually accused did not commit the robbery be inconsistent with a finding that a
robbery did take place. The evidence to establish these facts would not necessarily
be the same.
Suggested Alternative Answers to No. 8(a):
(a) (1) We would like to call attention to the fact that the question says
"contends." So perhaps we should make a distinction if negligence is proven and if
negligence is not proven. If the negligence of the defendant is not proven as Edgar
contends, then the defense of fortuitous event is tenable. However, if negligence is
proven to be present then the defense of fortuitous event is not tenable here and the
defendant will be liable.
Page 286 of 391
(2) There is no need of prior conviction in either case.
07; Obligations; joint/ solidary liability; joint
2001 No VII
Four foreign medical students rented the apartment of Thelma for a period of
one year. After one semester, three of them returned to their home country and the
fourth transferred to a boarding house. Thelma discovered that they left unpaid
telephone bills in the total amount of P80,000.00. The lease contract provided that
the lessees shall pay for the telephone services in the leased premises. Thelma
demanded that the fourth student pay the entire amount of the unpaid telephone
bills, but the latter is willing to pay only one fourth of it. Who is correct? Why? (5%)
SUGGESTED ANSWER:
The fourth student is correct. His liability is only joint, hence, pro rata. There is
solidary liability only when the obligation expressly so states or when the law or
nature of the obligation requires solidarity (Art. 1207, CC). The contract of lease in
the problem does not, in any way, stipulate solidarity.
07; Obligations; joint/ solidary liability; solidary
1998 No XV.
Joey, Jovy and Jojo are solidary debtors under a loan obligation of
P300,000.00 which has fallen due. The creditor has, however, condoned Jojo's
entire share in the debt. Since Jovy has become insolvent, the creditor makes a
demand on Joey to pay the debt.
1. How much, if any, may Joey be compelled to pay? [2%]
2. To what extent, if at all. can Jojo be compelled by Joey to contribute to
such payment? [3%]
Answer:
1. Joey can be compelled to pay only the remaining balance of
P2OO.OOO, in view of the remission of Jojo's share by the creditor. (Art. 1219, Civil
Code)
Answer;
2. Jojo can be compelled by Joey to contribute P5O.OOO. Art. 1217. par.
3, Civil Code provides. "When one of the solidary debtors cannot, because of his
insolvency, reimburse his share to the debtor paying the obligation, such share shall
be borne by all his co-debtors, in proportion to the debt of each."
Since the insolvent debtor's share which Joey paid was P100,000, and there
are only two remaining debtors - namely Joey and Jojo - these two shall share
equally the burden of reimbursement. Jojo may thus be compelled by Joey to
contribute P50.0OO.OO.
07; Obligations; joint/ solidary liability; solidary
2003 No XIV.
A,B,C,D, and E made themselves solidarity indebted to X for the amount of
P50,000.00. When X demanded payment from A, the latter refused to pay on the
following grounds.
(a) B is only 16 years old.
(b) C has already been condoned by X
(c) D is insolvent.
Page 287 of 391
(d) E was given by X an extension of 6 months without the consent of the
other four co-debtors.
State the effect of each of the above defenses put up by A on his obligation to
pay X, if such defenses are found to be true.
SUGGESTED ANSWER:
(a) A may avail the minority of B as a defense, but only for Bs share of P
10, 000.00. A solidary debtor may avail himself of any defense which personally
belongs to a solidary co-debtor, but only as to the share of that co-debtor.
(b) A may avail of the condonation by X of Cs share of P 10, 000.00. A
solidary debtor may, in actions filed by the creditor, avail himself of all defenses
which are derived from the nature of the obligation and of those which are personal
to him or pertain to his own share. With respect to those which personally belong to
others, he may avail himself thereof only as regards that part of the debt for which
the latter are responsible. (Article 1222, NCC).
(c) A may not interpose the defense of insolvency of D as a defense.
Applying the principle of mutual guaranty among solidary debtors, A guaranteed the
payment of Ds share and of all the other co-debtors. Hence, A cannot avail of the
defense of Ds insolvency.
(d) The extension of six (6) months given by X to E may be availed of by A
as a partial defense but only for the share of E. there is no novation of the obligation
but only an act of liberality granted to E alone.
07; Obligations; joint/ solidary liability; solidary obligations
1988 No. 9:
(b) Define joint and solidary obligations.
(c) A, B, and C borrowed P12,000 from X. This debt is evidenced by a
promissory note wherein the three bound themselves to pay the debt jointly and
severally. However, according to the note, A can be compelled to pay only on June
15, 1962, B can be compelled to pay only on June 15, 1964, while C can be
compelled to pay only on June 15T 1966. On June 15, 1962, X made a demand
upon A to pay the entire indebtedness but the latter aid only P4,OOO.OO,
Subsequently, because of A's refusal to pay the balance, X brought an action
against him for collection of the amount. Will such an action prosper? Reasons.
Answer:
(b) When there is a concurrence of two or more creditors or of two or more
debtors in one and the same obligation, such obligation may be either joint
(obligation mancomunada) or solidary (obligacion solidaria). A joint
obligation may be defined as an obligation where there is a concurrence of several
creditors or several debtors, or of several creditors and debtors, by virtue which
each of the creditors has a right to demand, while each of debtors is bound to render
compliance with his proportionate part of the prestation which constitutes the object
of the obligation. In other words, each of the creditors is entitled to demand the
payment of only a proportionate part of the credit, while each of the debtors is liable
for the payment of only a proportionate part of the debt. A solidary obligation, on the
other hand, may be defined as an obligation where there is a concurrence of several
creditors, or several debtors, or of several creditors and debtors, by virtue which
each of the creditors has a right to demand, while each of the debtors is bound to
render entire compliance with the prestation which constitutes the object of the
obligation. In other words, each of the creditors is entitled to demand the payment of
the entire credit/ while each of the debtors is liable for the payment of the entire
debt. (See Art. 1207, CC; 3 Castan, 7th Ed,, pp. 65-66.)
Page 288 of 391
(c) For the present, the action will not prosper. It is of course true that the
obligation here is solidary and that its solidary character is not destroyed by the fact
that the debtors are bound by different periods for payment is expressly
provided for in Art. 1211 of the Civil Code. However, in solidary obligations of
this type, the right of the creditor is limited to the recovery of the amount owed by the
debtor whose obligation has already matured, leaving in suspense his right to
recover the shares corresponding to the other debtors whose obligations have not
yet matured. This restriction upon the creditor's right does not destroy the solidary
character of the obligation, because ultimately, he can still compel one and the same
debtor, if that is his wish, to pay the entire obligation Therefore, in the instant case, X
shall have to wait for June 15, 1964, when B's obligation shall have matured, and for
June 15, 1966, when C's obligation shall have also matured. On June 15, 1966, he
can collect P4,000 from either A or B. On June 15, 1966, he can again collect
another P4,000 from either A or B or C. (See Ynchausti vs. Yulo, 34 Phil. 978.)
Suggested Alternative Answer To: No. 9 (c):
(c) It now being 1988, the action can no longer prosper because it has
already prescribed. Actions upon written contracts prescribe in 10 years.
07; Obligations; joint/ solidary liability; solidary obligations
1992 No. 3:
In June 1988, X obtained a loan from A and executed with Y as solidary co-
maker a promissory note in favor of A for the sum of P200,OOO.OO. The loan was
payable at P20,000.00 with interest monthly within the first week of each month
beginning July 1988 until maturity in April 1989. To secure the payment of the loan.
X put up as security a chattel mortgage on his car, a Toyota Corolla sedan. Because
of failure of X and Y to pay the principal amount of the loan, the car was
extrajudicially foreclosed. A acquired the car at A's highest bid of P120,000.00
during the auction sale.
After several fruitless letters of demand against X and Y, A sued Y alone for
the discovery of P80.000.00 constituting the deficiency.
Y resisted the suit raising the following defenses:
a) That Y should not be liable at all because X was not sued together with Y.
b) That the obligation has been paid completely by A's acquisition of the car
through "dacion en pago" or payment by cession.
c) That Y should not be held liable for the deficiency of P80,000.00 because
he was not a co-mortgagor in the chattel mortgage of the car. which contract was
executed by X alone as owner and mortgagor.
d) That assuming that Y is liable, he should only pay the proportionate sum
of P40,000.00.
Decide each defense with reasons.
Answer:
(a) This first defense of Y is untenable. Y is still liable as solidary debtor. The
creditor may proceed against any one of the solidary debtors. The demand against
one does not preclude further demand against the others so long as the debt is not
fully paid.
(b) The second defense of Y is untenable. Y is still liable. The chattel
mortgage is only given as a security and not as payment for the debt in case of
failure to pay. Y as a solidary co-maker is not relieved of further liability on the
promissory note as a result of the foreclosure of the chattel mortgage.
Page 289 of 391
{c} The third defense of Y is untenable. Y is a surety of X and the extrajudicial
demand against the principal debtor is not inconsistent with a judicial demand
against the surety. A suretyship may co-exist with a mortgage.
(d) The fourth defense of Y is untenable. Y is liable for the entire prestation
since Y incurred a solidary obligation with X.
(Arts. 1207, 1216. 1252 and 2047 Civil Code; Bicol Savings and Loan
Associates vs. Guinhawa 188 SCRA 642)
07; Obligations; joint/ solidary liability; solidary obligations
1980 No. IX
(a) "FF" and "GG" executed a promissory note binding themselves, jointly
and severally, to pay "X" Bank P10,000.00 within 90 days from January 10, 1979,
"FF" signed the note as principal and "GG" as guarantor. Upon failure to pay the
note on due date "X" bank sued "FF" and "GG" for payment. "GG" interposed the
defense that he was just a guarantor and the Bank must first exhaust all the
remedies against the principal "FF"
Is "GG's" defense tenable?
Answer
(a) "GG's" defense is untenable. Had he not bind himself solidarity with "FF"
to pay the obligation, undoubtedly, as guarantor, he could have availed of the
defense of benefit of excussion. In other words, he cannot be compelled to pay the
creditor unless the latter has exhausted all the property of the debtor and he
resorted to all the legal remedies against the said debtor. But then in the promissory
note, he bound himself jointly and severally with "FF" to pay the obligation to the
creditor. According to the law, such a defense now invoked by "GG" is no longer
available.
(NOTE: The above answer is based on Arts. 2058, 2059, Civil Code,)
07; Obligations; joint/ solidary liability; solidary obligations
1984 No. 11
A, B and C solidarity promised to pay D the amount of P3,000.00.
Unfortunately, C became insolvent.
What recourse does B have against A and B? What are the rights of A and B
as against each other?
Answer:
A. Furnished by the Office of Justice Palma
D may sue either A or B, or both, and recover the whole amount of P3,000
(Art. 1207) from either or both of them. Either party paying the entire amount may
recover the amount of P1,5000 from the other party. (Art. 1217).
B. Comments and Suggested Answer
We agree with the answer of the Bar Examiner. However, if the bar candidate
will answer the problem by invoking the provisions of Arts, 1216 and 1217 of the
Civil Code instead of Arts. 1207 and 1217, it is recommended that the answer
should be considered a correct Answer.
07; Obligations; joint/ solidary obligations; joint
1983 No. 12
A and B sold 1,000 sacks of rice to X and Y and, on X's request, delivered
them to him. X resold the rice, without turning over any part of it or its price to Y.
Page 290 of 391
May Y compel A and B to deliver what he bought? If so, to what extent?
Answer
Yes, Y may compel A to deliver 250 sacks of rice and B the same quantity,
the obligation being joint, not solidary.
07; Obligations; loss of the thing due
1986 No. 1:
Mr. Mekanico leased some automobile repair equipment to Mr. Masipag, who
was opening his auto repair shop. The lease agreement was executed on February
15, 1985. It stipulated that the period was one month only, at the expiration of which
Masipag was to return the equipment of Mekanico. The equipment was delivered on
February 15, 1985. On March 15, 1985 Mekanico, in a telephone call, asked
Masipag to return the leased property that same day. Because his truck broke down,
Masipag was unable to comply. Early the next morning, the equipment was burned
in an accidental fire that started in a nearby restaurant and gutted Masipag's auto
repair shop. Mekaniko seeks to hold Masipag liable for the value of the lost property
plus damages on the ground that he did not return it as agreed upon. Is Mekanico's
claim tenable? Explain.
Answer:
Mekaniko's claim in untenable.
The fire of accidental origin which destroyed the equipment which is the
object of the obligation in the instant case is clearly fortuitous in character.
Therefore, the doctrine of fortuitous events is applicable. The debtor or obligator,
Masipag, is not liable. In other words, the obligation is extinguished.
The fact that the loss took place on March 16, 1985. which is one day after
Mekaniko had made a demand upon Masipag to return the leased property, does
not mean that the loss took place after the obligor had already incurred in delay. It
must be noted that the lease agreement was executed on February 15, 1985,
Obviously, on March 16, 1985. Masipag had not yet incurred in delay.
(Note: The above answer is based upon Arts. 13 and 1174 of the Civil Code.
We recommend that an answer based on Arts. 13 and 1262 of the Civil Code or on
Arts. 13, 1665 and 1667, arriving at the same conclusion should also be considered
correct)
Answer - The loss occurred by fortuitous event before Masipag incurred in
delay. Therefore, the general rule applies; No one shall be held liable for loss due to
fortuitous event.
The one-month period of lease would expire only on March 18th while the fire
occurred on the 29th day. The C.C. provides that one-month consists of 30 days and
February
1985 had only 28 days, not being a leap year.
07; Obligations; loss of the thing due; force majeure
2000 No XIV
Kristina brought her diamond ring to a jewelry shop for cleaning. The jewelry
shop undertook to return the ring by February 1, 1999." When the said date arrived,
the jewelry shop informed Kristina that the Job was not yet finished. They asked her
to return five days later. On February 6, 1999, Kristina went to the shop to claim the
ring, but she was informed that the same was stolen by a thief who entered the shop
the night before. Kristina filed an action for damages against the jewelry shop which
put up the defense of force majeure. Will the action prosper or not? (5%)
Page 291 of 391
SUGGESTED ANSWER:
The action will prosper. Since the defendant was already in default not having
delivered the ring when delivery was demanded by plaintiff at due date, the
defendant is liable for the loss of the thing and even when the loss was due to force
majeure,
07; Obligations; nature and effect of obligations
1986 No. 11:.
By means of a public instrument, Mr. Nagbibili sold his mango plantation to
Abenturero effective immediately. The document stipulated, however, that delivery
would be effected six months from the execution of the deed of sale. When the said
period arrived, Abenturero demanded delivery in writing but Nagbibili dilly-dallied. It
was not until a month afterwards that Nagbibili finally gave the land to Abenturero. In
the three weeks before delivery, Nagbibili sold and delivered the entire produce of
the mango plantation to Mr.Commerciante for P200.000.00. Commerciante knew
nothing of the contract between Nagbibili and Abenturero. Abenturero now seeks to
recover from Commerciante either the full value of the mangoes or a similar amount
and quality of the mangoes sold.
Does Abenturero have this right against Commerciante? Explain.
Answer:
Abenturero does not have any right, whether personal or real, to proceed
against Commerciante.
In obligations to give, the creditor has a right to the thing which is the object
of the obligation and to the fruits thereof when the obligation to deliver arises. The
obligation to deliver arises from the moment of the perfection of the contract. In
sales, once the contract is perfected, the vendor is bound to deliver the thing sold
and the fruits, unless there is a stipulation to the contrary. In the instant problem,
there is a stipulation that delivery will be effected six months from the execution of
the deed of sale. Instead of delivering the mango plantation and the fruits of
Abenturero in accordance with the agreement, Nagbibili breached the contract by
delaying the delivery of the plantation beyond the period agreed upon and by selling
the fruits to Commerciante. However, since there was still no delivery of the
plantation and the fruits, it is obvious that the only right which Abenturero had
acquired was a personal right enforceable against Nagbibili, not a real right
enforceable against the whole world. Ergo, he can now proceed against Nagbibili for
indemnification for damages. He cannot proceed against Commerciante who was
not even aware of the existence of the contract between Nagbibili and Abenturero.
(Note - The above answer is based upon Arts. 1164 and 1537 of the Civil
Code and upon decided cases.)
Answer - Although the buyer is entitled to the fruits of the land from the time
of perfection of the contract, still he did not acquire a real right over the fruits until
they are delivered to him. Therefore, Abenturero has no right of action against
Commerciante.
Answer - The seller is supposed to deliver to the buyer the thing sold as well
as the fruits and accessions that accrue from the moment of perfection. Abenturero
is entitled to the fruits of the mango plantation from the time of the execution of the
public instrument which provided that the sale was to be effective immediately.
Delivery may be either actual or constructive. The execution of public instrument is
one of the modes of constructive delivery.
07; Obligations; obligation to deliver a determinate thing; effect of loss
Page 292 of 391
1984 No. 14
For value received, Pedro promised to deliver to Juan on or before August
15, 1984 a Mercedes Benz with Plate No. 123 which he (Pedro) had just brought
home from Germany, as well as a 1984 18" Sony television set. Unfortunately,
before the scheduled delivery date, the Mercedes Benz and the television set which
Pedro had intended to deliver to Juan were destroyed by an accidental fire.
Has the obligation of Pedro been extinguished? Explain.
Answer:
A. Furnished by Office of Justice Palma
The obligation to deliver the Mercedes-Benz is an obligation to deliver a
determinate thing, because the particular car to be delivered had been designated.
The obligation to deliver that particular car was therefore extinguished by the
occurrence of the fire, a fortuitous event.
On the other hand, the obligation to deliver the television set is an obligation
to deliver a generic thing. The particular television set to be delivered has not been
determined. Only the kind of television set to be delivered was agreed upon. Pedro
is therefore still obligated to deliver a television set of the kind and quality agreed
upon, since generic obligations are never extinguished by fortuitous events.
E. Comments and Suggested Answer
We agree with the answer of the Bar Examiner.
07; Obligations; obligation to deliver a generic thing
1985 No. 15
A) On September 1, 1982, A sold to B 50 heads of cattle for P150,000.00
and 60 heads of carabao for the same price, the cattle to be loaded in Davao City on
December 1, 1982, on the SS "Argus" and delivered upon her arrival in Manila
5 days later while the carabao were be loaded in the same city and shipped on
December 15 and delivered likewise to B upon her arrival in Manila 5 days later.
Because of the breakdown of his cargo truck, A was able to ship the cattle and the
carabaos only on December 15. On her way to Manila, the SS "Argus" ran into a
storm and all the animals were thrown into the sea to prevent her from sinking. B,
who paid one half of the price upon the execution of the contract, demanded its
return while A sought from B the full payment of the price.
Decide the controversy, giving the reasons for your decision.
Answers:
A) 1. The obligation of A to deliver the cattle or carabao is a generic
obligation, therefore, it is not extinguished by loss, so the seller is liable. However,
the question of damages will depend on who is at fault. Since, however, there is
nothing stated here as to whether the price is for a lump sum or for each cattle, the
problem here is on the partial payment. And the partial payment of one-half is a
demand. If the one-half payment is equivalent to the value of 1/2, it is a demand for
one-half and, therefore, when A delayed the delivery of the cattle, he is "in mora"
and will be liable for damages.
2. The question with respect to the sale is whether ownership passed to the
buyer at the time of the loss. The place of delivery and the sale is Manila. The things
sold were destroyed on the way to Manila. The ownership did not pass to the buyer
and therefore the buyer is not yet answerable for the payment of the price. B can
demand therefore its return. Aside from that there was a delay on the part of the
vendor and being in delay, he bears the loss through fortuitous -event.
Page 293 of 391
3. The heads of cattle and carabaos are to be considered specific, so this
refers to the sale of specific things, and when they were lost, the obligation was
extinguished because of fortuitous event.
4. Res perit creditori applies. The risk, therefore, would lie with the buyer
from the perfection of the contract until the delivery of the thing sold. Accordingly,
the buyer must pay to the seller the remaining unpaid price of the goods.
6. The seller is liable to the buyer for the loss of the cattle because the
delivery to the carrier was late.
The buyer is liable for the loss of the carabaos since delivery to the carrier is
equivalent to delivery to the buyer. The buyer bears the loss and is liable under the
principle of "res pent domino" when the subject matter is lost through fortuitous
event.
07; Obligations; obligation to give; obligation to do
1983 No. 10
A bound himself to deliver to B a 21-inch 1983 model TV set, and the 13
cubic feet White Westinghouse refrigerator, with Motor No. WERT-385, which B saw
in A's store, and to repair B's piano. A did none of these things.
May the court compel A to deliver the TV set and the refrigerator and repair
the piano? Why? If not, what relief may the court grant B? Why?
Answer
(Examiner's Answer)
Yes, in so far as his obligation to deliver the Westinghouse refrigerator is
concerned, the thing to be given being determinate, but no in so far as the 2 other
obligations are concerned, one being an obligation to give an indeterminate thing,
and the other being an obligation to do. In these 2 cases, the court shall order the
obligations to be performed at A's expense.
(Committee's Answer)
As far as the refrigerator is concerned, the Court may compel A to deliver the
refrigerator to B. The obligation to give is a determinate obligation to give. Under the
law, in this type of obligation, the principal right of the creditor against the debtor is
to compel the debtor to make the delivery.
(Note; The above answer is based on Art. 1165, par. 1, Civil Code)
As far as the TV set is concerned, the Court may compel A to deliver
although not specifically. The obligation of A is a generic obligation to give. Under
the law, in this type of obligation, once all of the circumstances of the obligation have
been taken into consideration, the Court may compel A to deliver to B a TV set
which must be neither of superior nor inferior quality.
(Note: The above answer is based on Art, 1246, Civil Code.)
As far as the repair of the piano is concerned, the court cannot compel A to
repair said piano. The obligation here is a purely personal obligation, an obligation to
do. Under the law, in this type of obligation to compel A to repair the piano of B
would constitute an infringement of A's liberty,
(Note: The above answer is based by implication on Art. 1165, Civil Code.)
Anent the TV set, if the debtor refuses or is unable to comply with his
obligation to deliver a 21 inch TV get which must be neither of superior nor inferior
quality, B may ask the court to order the performance of the obligation at the
expense of A. Additionally, he can ask for damages.
Page 294 of 391
Anent the repair of the piano, B may also ask the court to order the
performance of the obligation at the expense of A.
(The above answers are based on Arts. 1165, par. 2 and 1170, Civil Code.)
07; Obligations; obligations with a penal clause
1986 No. 10:
The Betis Furniture Co. undertook to deliver to Mr. Bagongkasal specified
pieces of living room, dining room and bedroom furniture, all made of narra, for a
price stated in the contract. The agreement had a penal clause that any violation of
the contract would entitle the aggrieved party to damages in the amount of
P100,000.00. The furniture delivered by Betis was made, not of narra, but of inferior
wood.
In a suit to recover damages, Bagongkasal was able to prove that the actual
damages he sustained amounted to P200,000.00, He demanded that amount plus
the P100,000.00 penalty or a total of P300,000.00. Betis, however, countered that if
it were liable for damages at all, the maximum award should not exceed
P100,000.00 as stated in the penal clause of the contract.
Whose claim would you sustain? Why? Answer:
I would sustain the claim of Betis. A penal clause is supposed to answer for
damages without the introduction of proof as to actual damages. It is to be noted
that the penal clause has been freely agreed upon between the parties precisely
with the intention of doing away with having to present proof of damages.
Answer I shall sustain the claim of the defendant Betis Furniture Co. The
Civil Code is explicit. According to said Code, in obligations with a penal clause, the
penalty shall substitute the indemnity for damages and the payment of interests in
case of non-compliance. There are only three exceptions to this rule. They are: first,
when there is a stipulation to the contrary: second, when the creditor is compelled to
sue the debtor because of the latter's refusal to pay the agreed penalty; and third,
when the debtor is guilty of fraud. It is clear that the instant case does not fall within
the purview of any of the three exceptions Therefore, the award in favor of the
plaintiff should be for P100,000.00 only.
Answer -- I shall not sustain the claim of Beds Furniture Co. The Civil Code is
explicit. According to said Code, in obligations with a penal clause, the penalty shall
substitute the indemnity for damages and the payment of interests in case of non-
compliance. There are three exceptions to this rule. They are: first, when there is a
stipulation to the contrary; second, when the creditor is compelled to sue the debtor
because of the latter's refusal to pay the agreed penalty: and third, when the debtor
is guilty of fraud. It is clear that the instant case falls within the purview of the third
exception. The furniture delivered by Betis was made, not of narra. but of inferior
wood. That indicates that the debtor commuted fraud in performing its obligation.
Therefore, the award in favor of the plaintiff should be for P200,000 actual damages
plus the P100,000 penalty.
Answer - Under the doctrine of G.A. Machineries, Inc. vs. Yaptinchay. 126
SCRA 78 (ponente J. Gutierrez) there is breach of contract not breach of warranty.
The breach is fraudulent. Under the doctrine of Mariano Pamintuan vs. CA. SCRA
556, since debtor is guilty of fraud - "The proven damages supersede the stipulated
damages (penalty)" citing Art. 1226 Civil Code of the Philippines).
Answer ~ Here there is a breach of warranty and Betis Furniture Co. must
know the difference between "narra" and other kinds of wood. Therefore, in
delivering the furniture made of inferior quality, it undoubtedly acted fraudulently,
and under the C.C, the buyer in this case would be entitled to be reimbursed the
Page 295 of 391
difference in value of the furniture and the price 'he paid (P200,000) and damages
(penalty -P100,000),
07; Obligations; obligations with a penal clause
1985 No, 12
A, agreed to build a house for B on the condition that it shall be completed
within 6 months from January 1, 1980, that work shall begin when B pays A
P30,000.00, and that A shall pay a penalty of P200.00 for each day of delay in the
fulfillment of the contract. A finished the construction of the house on August 31,
1980, and turned it over the same day to B who demanded payment of the penalty
of P200.00 for every day of delay in its completion. A refused to pay, alleging the
delay was caused by B who gave him the first payment only on March 1. B's answer
was that he had the money ready since January 1 but it was not until March 1, after
calling A, that he went to his house to get it
a) Rule on the respective contentions of both parties.
b) Was there delay in the completion of the house? State your reasons.
Answers:
(a) and (b)
1. B was bound to pay A the amount of P30.000 only upon demand for such
payment by A. Hence, there was no default on the part of B who was ready to
comply with the obligation. Since the agreement was that the house was to be
completed within 6 months from Jan. 1, 1980 and the building was not completed
within the specified time, the builder is liable for the penalty according to the terms of
the agreement.
2. Even though there is a penalty, that is dependent on the fact whether time
is of the essence of the contract or not. The mere fixing of the date is not considered
necessarily to mean that time is of the essence. The date is only for maturity.
Generally, time is not of the essence.
3. Although B had the money, he never informed the other party. So the
other party did not get to know that he had the money. Therefore, B was not in
default. A is not entitled to collect penalty.
4. I would rule in favor of B, since the payment of P30,000.00 does not
appear to be a condition for the completion of the house within six months from
January 1, 1980. But, assuming otherwise, there was no default in. the payment of
the P30.000.00 since there was no demand made for the payment thereof (Art.
1169). In addition, unless otherwise expressed in the agreement, payment shall
be made at the domicile of the obligor (Art. 1251).
There was a delay in the completion of the house; accordingly, the payment
of P200 for each day of delay could be sought since this penalty is not
unconscionable considering the total value of the contract.
07; Obligations; obligations with a period
1980 No. IV
(b) "M" and "N" were very good friends. "N" borrowed P10,000 from "M".
Because of their close relationship, the promissory note executed by "N" provided
that he would pay the loan "whenever his means permit." subsequently, "M" and "N"
quarreled. "M" now asks you to collect the loan because he is in dire need of money.
What legal action, if any, would you take in behalf of "M"?
Answer
Page 296 of 391
(b) "M" must bring an action against "N" for the purpose of asking the court to
fix the duration of the term or period for payment (Arts. 1180, 1197, Civil Code).
Once the court has fixed the duration of the term or period, it becomes a part of the
covenant of the two contracting parties. If the debtor defaults in the payment of the
obligation after the expiration of the period fixed by the court, the creditor can then
bring an action against him for collection. Any action for collection brought before
that would be premature. This is well-settled. (Gonzales vs. Jose, 66 Phil. 369) ;
Conception vs. People of the Phil, 74 Phil. 62; Pages vs. Basilan, 104 Phil. 882).
Alternative Answer: Normally, before an action for collection may be
maintained by the creditor against the debtor, the former must first bring an action
against the latter asking the court to fix the duration of the term or period for
payment (Art. 1197, Civil Code). However, an action combining such action with that
of an action for collection may be allowed if it can be shown that a separate action
for collection would be a. mere formality because no additional proofs other than the
admitted facts will be presented and would serve no purpose other than to delay.
Here, there is no legal obstacle to such course of action. (Borromeo vs. Court of
Appeals, 47 SCRA 65).
07; Obligations; obligations with a period
1984 No. 13
A obtained from B a loan payable within a year. As security for its repayment,
A mortgaged his uninsured house.
Three months after the loan was given, A's house was gutted by an
accidental fire. Thereupon, B demanded immediate payment from A, who refused to
pay contending that the loan was for a one-year period.
Is A's contention valid? Explain. Answer:
A. Furnished by Office of Justice Palma
No. Under Art. 1198, the debtor shall low every right to make use of a period
when the securities disappear through a fortuitous event. A has to give satisfactory
substitute collateral.
B. Comments and Suggested Answer
We agree with the answer of the Bar Examiner.
07; Obligations; obligations with a period
1982 No 15
"A" Corporation, engaged in the sale of subdivision residential lots, sold to "B"
a lot of 1,000 square meters. The contract provides that the corporation should put
up an artesian well with tank, within a reasonable time from the date thereof and
sufficient for the needs of the buyers. Five years thereafter, and no well and tank
have been put up by the corporation, "B" sued the corporation for specific
performance. The corporation set up a defense that no period having been fixed, the
court should fix the period. Decide with reason.
Answer
First Answer: The action for specific performance should be dismissed on the
ground that it is premature. It is clear that the instant case falls within the purview of
obligations with a term or period which must be judicially fixed. Thus, "B", instead of
bringing an action for specific performance, should bring an action asking the court
to determine the period within which "A" Corporation shall put up the artesian well
with tank. Once the court has fixed the period, once the court, let us say, has
declared that the period is six months, then that will become a part of the covenant
Page 297 of 391
between the contracting parties. It can no longer be changed by them. If the
Corporation does not put up the artesian well with tank within the period fixed by the
court, "B" can then bring an action for specific performance.
Second Answer: Normally, before an action for specific performance may be
maintained by "B" against "A" Corporation, the former must first bring an action
against the latter asking the court to fix the duration of the term or period to install
the artesian well with tank. However, an action combining such action with that of an
action for specific performance may be allowed if it can be shown that a separate
action for specific performance would be a mere formality because no additional
proofs other than the admitted facts will be presented and would serve no purpose
other than to delay. Here, there is no obstacle to such course of action.
(Note: The above answers are based on Art. 1197 of the Civil Code and on
decided cases. The Committee respectfully recommends that either answers should
be considered correct.)
07; Obligations; obligations with a period; suspensive period dependent upon
will of debtor
1991 No 13:
In a deed of sale of a realty, it was stipulated that the buyer would construct a
commercial building on the lot while the seller would construct a private passageway
bordering the lot. The building was eventually finished but the seller failed to
complete the passageway as some of the squatters, who were already known to be
there at the time they entered into the contract, refused to vacate the premises, In
fact, prior to its execution, the seller filed ejectment cases against the squatters.
The buyer now sues the seller for specific performance with damages. The
defense is that the obligation to construct the passageway should be with a period
which, incidentally, had not been fixed by them, hence, the need for fixing a judicial
period.
Will the action for specific performance of the buyer against the seller
prosper?
Answer:
No. the action for specific performance filed by the buyer is premature under
Art. 1197 of the Civil Code. If a period has not been fixed although contemplated by
the parties, the parties themselves should fix that period, failing in which, the Court
maybe asked to fix it taking into consideration the probable contemplation of the
parties. Before the period is fixed, an action for specific performance is premature.
Alternative Answer;
It has been held in Borromeo vs. CA (47 SCRA 69), that the Supreme Court
allowed the simultaneous filing of action to fix the probable contemplated period of
the parties where none is fixed in the agreement if this would avoid multiplicity of
suits. In addition, technicalities must be subordinated to substantial justice.
Alternative Answer;
The action for specific performance will not prosper. The filing of the
ejectment suit by the seller was precisely in compliance with his obligations and
should not, therefore, be faulted if no decision has yet been reached by the Court on
the matter.
07; Trust; implied resulting trust
1995 No. 17:
Page 298 of 391
In I960. Maureen purchased two lots in a plush subdivision registering Lot 1
In her name and Lot 2 In the name of her brother Walter with the latter's consent.
The idea was to circumvent a subdivision policy against the acquisition of more than
one lot by one buyer. Maureen constructed a house on Lot 1 with an extension on
Lot 2 to serve as a guest house. In 1987. Walter who had suffered serious business
losses demanded that Maureen remove the extension house since the lot on which
the extension was built was his property. In 1992, Maureen sued for the
reconveyance to her of Lot 2 asserting that a resulting trust was created when she
had the lot registered in Walter's name even if she paid the purchase price. Walter
opposed the suit arguing that assuming the existence of a resulting trust the action
of Maureen has already prescribed since ten years have already elapsed from the
registration of the title in his name.
Decide. Discuss fully Answer:
This is a case of an implied resulting trust. If Walter claims to have acquired
ownership of the land by prescription or If he anchors his defense on extinctive
prescription, the ten year period must be reckoned from 1987 when he demanded
that Maureen remove the extension house on Lot No. 2 because such demand
amounts to an express repudiation of the trust and It was made known to Maureen.
The action for reconveyance filed in 1992 is not yet barred by prescription. (Spouses
Huang v. Court of Appeals, Sept. 13, 1994).
07; Trust; implied trust;
1998 No VII.
Juan and his sister Juana inherited from their mother two parcels of farmland
with exactly the same areas. For convenience, the Torrens certificates of title
covering both lots were placed In Juan's name alone. In 1996, Juan sold to an
innocent purchaser one parcel in Its entirety without the knowledge and consent of
Juana, and wrongfully kept for himself the entire price paid.
1. What rights of action, if any, does Juana have against and/or the buyer?
|3%]
2. Since the two lots have the same area, suppose Juana flies a complaint
to have herself declared sole owner of the entire remaining second lot, contending
that her brother had forfeited his share thereof by wrongfully disposing of her
undivided share in the first lot. will the suit prosper? [2%]
Answer:
1. When, for convenience, the Torrens title to the two parcels of land were
placed in Joan's name alone, there was created an implied trust (a resulting trust) for
the benefit of Juana with Juan as trustee of one-half undivided or ideal portion of
each of the two lots. Therefore, Juana can file an action for damages against Joan
for having fraudulently sold one of the two parcels which he partly held in trust for
Juana's benefit. Juana may claim actual or compensatory damage for the loss of her
share in the land; moral damages for the mental anguish, anxiety, moral shock and
wounded feelings she had suffered; exemplary damage by way of example for the
common good, and attorney's fees.
Juana has no cause of action against the buyer who acquired the land for
value and in good faith, relying on the transfer certificate of title showing that Juan is
the registered owner of the land.
Another Answer:
1. Under Article 476 of the Civil Code, Juana can file an action for quieting of
title as there is a cloud in the title to the subject real property. Second, Juana can
also file an action for damages against Juan, because the settled rule is that the
Page 299 of 391
proper recourse of the true owner of the property who was prejudiced and
fraudulently dispossessed of the same is to bring an action for damages against
those who caused or employed the same. Third, since Juana had the right to her
share in the property by way of inheritance, she can demand the partition of the
thing owned in common, under Article 494 of the Civil Code, and ask that the title to
the remaining property be declared as exclusively hers.
However, since the farmland was sold to an innocent purchaser for value,
then Juana has no cause of action against the buyer consistent with the established
rule that the rights of an innocent purchaser for value must be respected and
protected notwithstanding the fraud employed by the seller in securing his title.
(Eduarte vs. CA, 253 SCRA 391)
Additional Answer:
1. Juana has the right of action to recover (a) her one-half share in the
proceeds of the sale with legal interest thereof, and (b) such damages as she may
be able to prove as having been suffered by her, which may include actual or
compensatory damages as well as moral and exemplary damages due to the breach
of trust and bad faith (Imperial vs. CA, 259 SCRA 65). Of course, if the buyer knew
of the co-ownership over the lot he was buying, Juana can seek (c) reconvenyance
of her one-half share instead but she must implead the buyer as co-defendant and
allege his bad faith in purchasing the entire lot. Finally, consistent with the ruling in
Imperial us. CA. Juana may seek instead (d) a declaration that she is now the sole
owner of the entire remaining lot on the theory that Juan has forfeited his one-half
share therein.
Additional Answer:
1. Juana can file an action for damages against Juan for having
fraudulently sold one of the two parcels which he partly held In trust for Juana's
benefit. Juana may claim actual or compensatory damage for the loss of her share
in the land; moral damages for the mental anguish, anxiety, moral shock and
wounded feelings she had suffered; exemplary damage by way of example for the
common good, and attorney's fees.
Juana has no cause of action against the buyer who acquired the land for
value and in good faith, relying on the transfer certificate showing that Juan is the
registered owner of the land.
Answer:
2. Juana's suit to have herself declared as sole owner of the entire
remaining area will not prosper because while Juan's act in selling the other lot was
wrongful. It did not have the legal effect of forfeiting his share in the remaining lot.
However, Juana can file an action against Juan for partition or termination of the co-
ownership with a prayer that the lot sold be adjudicated to Juan, and the remaining
lot be adjudicated and reconveyed to her.
Another Answer:
2. The suit will prosper, applying the ruling in Imperial vs. CA cited above.
Both law and equity authorize such a result, said the Supreme Court.
Strictly speaking, Juana's contention that her brother had forfeited his share
in the second lot is incorrect. Even if the two lots have the same area, It does not
follow that they have the same value. Since the sale of the first lot on the Torrens
title in the name of Juan was valid, all that Juana may recover is the value of her
undivided Interest therein, plus damages. In addition, she can ask for partition or
reconveyance of her undivided interest in the second lot, without prejudice to any
Page 300 of 391
agreement between them that in lieu of the payment of the value of Juana's share In
the first lot and damages, the second lot be reconveyed to her.
Alternative Answer:
2. The suit will not prosper, since Juan's wrongful act of pocketing the entire
proceeds of the sale of the first lot is not a ground for divesting him of his rights as a
co-owner of the second lot. Indeed, such wrongdoing by Juan does not constitute,
for the benefit of Juana, any of the modes of acquiring ownership under Art. 712,
Civil Code.
07; Trusts; express trust; prescription
1997 No. 13:
On 01 January 1980. Redentor and Remedies entered into an agreement by
virtue of which the former was to register a parcel of land in the name of Remedies
under the explicit covenant to reconvey the land to Remigio, son of Redentor, upon
the son's graduation from college. In 1981, the land was registered in the name of
Remedies.
Redentor died a year later or in 1982. In March 1983, Remigio graduated
from college. In February 1992, Remigio accidentally found a copy of the document
so constituting Remedies as the trustee of the land. In May 1994, Remigio filed a
case against Remedies for the reconveyance of the land to him. Remedies, in her
answer, averred that the action already prescribed.
How should the matter be decided? Answer:
The matter should be decided in favor of Remigio (trustee) because the
action has not prescribed. The case at bar involves an express trust which does not
prescribe as long as they have not been repudiated by the trustee (Diaz vs.
Gorricho. 103 Phil, 261).
Page 301 of 391
08; Sales & Donation; ownership of the thing sold
2003 No XV.
(a) May a person sell something that does not belong to him? Explain.
(b) May a person donate something that does not belong to him? Explain.
5%
SUGGESTED ANSWERS:
(a) Yes, a person may sell something which does not belong to him. For
the sale to be valid, the law does not require the seller to be the owner of the
property at the time of the sale. (Article 1434, NCC). If the seller cannot transfer
ownership over the thing sold at the time of delivery because he was not the owner
thereof, he shall be liable for breach of contact.
(b) As a general rule, a person cannot donate something which he cannot
dispose of at the time of the donation (Article 751, New Civil Code).
08; Sales; Art. 1592
2003 No XVI.
X sold a parcel of land to Y on 01 January 2002, payment and delivery to be
made on 01 February 2002. It was stipulated that if payment were not to be made by
Y on 01 February 2002, the sale between the parties would automatically be
rescinded. Y failed to pay on 01 February 2002, but offered to pay three days later,
which payment X refused to accept, claiming that their contract of sale had already
been rescinded. Is Xs contention correct? Why? 5%
SUGGESTED ANSWER:
No, X is not correct. In the sale of immovable property, even though it may
have been stipulated, as in this case, that upon failure to pay the price at the time
agreed upon the rescission of the contract shall of right take place, the vendee may
pay, even after the expiration of the period, as long as no demand for rescission of
the contract has been made upon him either judicially or by a notarial act (Article
1592, New Civil code). Since no demand for rescission was made on Y, either
judicially or by a notarial act, X cannot refuse to accept the payment offered by Y
three (3) days after the expiration of the period.
ANOTHER SUGGESTED ANSWER:
This is a contract to sell and not a contract of absolute sale, since as there
has been no delivery of the land. Article 1592 of the New Civil code is not applicable.
Instead, Article 1595 of the New Civil Code applies. The seller has two alternative
remedies: (1) specific performance, or (2) rescission or resolution under Article 1191
of the New Civil code. In both remedies, damages are due because of default.
ALTERNATIVE ANSWER:
Yes, the contract was automatically rescinded upon Ys failure to pay on 01
February 2002. By the express terms of the contract, there is no need for X to make
a demand in order for rescission to take place. (Article 1191, New Civil Code, Suria
v. IAC 151 SCRA 661 [1987]; U.P. v. de los Angeles 35 SCRA 102 [1970]).
08; Sales; Art. 1592
1988 No. 13:
(a) A sold to B a house and lot for P50,000.00 payable 30 days after the
execution of the deed of sale. It was expressly agreed in the deed that the sale
would ipso facto be of no effect upon the buyer's failure to pay as agreed. B failed to
pay on maturity, and A sued to declare the contract of no force and effect. If B
Page 302 of 391
tendered payment before the action was filed, but subsequent to the stipulated date
of payment, would the action prosper? Why?
Answer:
(a) The action would not prosper in such a case. According to the law, "in
the sale of immovable property, even though it may have been stipulated that upon
failure to pay the price at the time agreed upon the rescission of the contract shall of
right take place, the vendee may pay, even after the expiration of the period, so long
as no demand for the rescission of the contract has been made upon him either
judicially or by notarial act. After the demand, the court may not grant him a new
term." (Art. 1592, CC.) Here, at the time B tendered payment of the purchase price,
there was still no demand made upon him by A for the payment of said purchase
price either judicially or by notarial act.
08; Sales; assignment of credit
1993 No, 14:
Peter Co, a trader from Manila, has dealt business with Allied Commodities in
Hongkong for five years. All through the years. Peter Co accumulated an
indebtedness of P50O,OOO.OO with Allied Commodities. Upon demand by its
agent in Manila, Peter Co paid Allied Commodities by check the amount owed. Upon
deposit in the payee's account in Manila, the check was dishonored for insufficiency
of funds. For and In consideration of P1.00, Allied Commodities assigned the credit
to Hadji Butu who brought suit against Peter Co in the RTC of Manila for recovery of
the amount owed. Peter Co moved to dismiss the complaint against him on the
ground that Hadji Butu-was not a real party in interest and, therefore, without legal
capacity to sue and that he had not agreed to a subrogation of creditor.
Will Peter Co's defense of absence of agreement to a subrogation of creditor
prosper?
Answer:
No, Co's defense will not prosper. This is not a case of subrogation, but an
assignment of credit. Assignment of credit is the process of transferring the right of
the assignor to the assignee. The assignment may be done either gratuitously or
onerously, in which case, the assignment has an effect similar to that of a sale (Nyco
Sales Corp.v.BA Finance Corp. G.R No.71694. Aug.16, 1991 200 SCRA 637). As a
result of the assignment, the plaintiff acquired all the rights of the assignor including
the right to sue in his own name as the legal assignee. In assignment, the debtor's
consent is not essential for the validity of the assignment (Art. 1624; 1475. CC;
Rodriguez v. CA, et al, G. R No. 84220, March 25. 1992 207 SCRA 553).
Alternative Answer:
No. the defense of Peter Co will not prosper. Hadji Butu validly acquired his
right by an assignment of credit under Article 1624 of the Civil Code. However, the
provisions on the contract of sale (Article 1475 Civil Code) will apply, and the
transaction is covered by the Statute of Frauds. (Art. 1403 par. (2) Civil Code)
08; Sales; conditional sale vs absolute sale
1997 No. 15:
(b) Between a conditional sale, on the one hand, and an absolute sale, on
the other hand.
Answer:
(b) A conditional sale is one where the vendor is granted the right to
unilaterally rescind the contract predicated on the fulfillment or non-fulfillment, as the
Page 303 of 391
case may be, of the prescribed condition. An absolute sale is one where the title to
the property is not reserved to the vendor or if the vendor Is not granted the right to
rescind the contract based on the fulfillment or non-fulfillment, as the case may be,
of the prescribed condition.
08; Sales; contract of sale vs agency to sell
1999 No XV.
(b) A granted B the exclusive right to sell his brand of Maong pants in Isabela,
the price for his merchandise payable within 60 days from delivery, and promising B
a commission of 20% on all sales. After the delivery of the merchandise to B but
before he could sell any of them, BOs store in Isabela was completely burned
without his fault, together with all of A's pants. Must B pay A for his lost pants? Why?
(5%)
ANSWER:
(b) The contract between A and B is a sale not an agency to sell because the
price is payable by B upon 60 days from delivery even if B is unable to resell it. If B
were an agent, he is not bound to pay the price if he is unable to resell it.
As a buyer, ownership passed to B upon delivery and, under Art. 1504 of the
Civil Code, the thing perishes for the owner. Hence, B must still pay the price.
08; Sales; contract of sale vs contract to sell
1988 No. 15:
(c) Distinguish between a contract of sale and a contract to sell.
Answer:
(c) The two may be distinguished from each other in the following ways:
(1) In the first, title passes to the vendee upon delivery of the thing sold,
whereas in the second, by agreement, ownership is reserved in the vendor and is
not to pass until full payment of the price.
(2) In the first, nonpayment is a negative resolutory condition, whereas in the
second, full payment is a positive suspensive condition.
(3) In the first, the vendor has lost and cannot recover ownership until and
unless the contract is resolved or rescinded, whereas in the second, title remains in
the vendor, and when he seeks to eject the vendee because of noncompliance by
such vendee with the suspensive condition stipulated, he is enforcing the contract
and not resolving the same. (Santos vs. Santos, CA,47 Off, Gaz,6372.)
08; Sales; contract to sell
2001 No XVI
Arturo gave Richard a receipt which states:

"Receipt
Received from Richard as down payment
For my 1995 Toyota Corolla with
plate No. XYZ-1 23.............. P50.000.00
Balance payable: 12/30/01........ P50 000.00
September 15, 2001.

Page 304 of 391
(Sgd.) Arturo

Does this receipt evidence a contract to sell? Why? (5%)

SUGGESTED ANSWER
It is a contract of sale because the seller did not reserve ownership until he
was fully paid.
08; Sales; contract to sell vs contract of sale
1997 No. 15:
State the basic difference (only in their legal effects) -
(a) Between a contract to sell, on the one hand, and a contract of sale, on
the other;
Answer:
(a) In a contract of sale, ownership is transferred to the buyer upon delivery
of the object to him while in a contract to sell, ownership is retained by the seller until
the purchase price is fully paid. In a contract to sell, delivery of the object does not
confer ownership upon the buyer. In a contract of sale, there Is only one contract
executed between the seller and the buyer, while in a contract to sell, there are two
contracts, first the contract to sell (which is a conditional or preparatory sale) and a
second, the final deed of sale or the principal contract which is executed after full
payment of the purchase price.
08; Sales; double sales
2001 No XII
On June 15,1995, Jesus sold a parcel of registered land to Jaime. On June
30. 1995, he sold the same land to Jose. Who has a better right if:
a) the first sale is registered ahead of the second sale, with knowledge of
the latter. Why? (3%)
b) the second sale is registered ahead of the first sale, with knowledge of
the latter? Why? (5%)
SUGGESTED ANSWER:
(a) The first buyer has the better right if his sale was first to be registered,
even though the first buyer knew of the second sale. The fact that he knew of the
second sale at the time of his registration does not make him as acting in bad faith
because the sale to him was ahead in time, hence, has a priority in right. What
creates bad faith in the case of double sale of land is knowledge of a previous sale.
b) The first buyer is still to be preferred, where the second sale is registered
ahead of the first sale but with knowledge of the latter. This is because the second
buyer, who at the time he registered his sale knew that the property had already
been sold to someone else, acted in bad faith. (Article 1544, C.C.)
08; Sales; double sales
2004 No. IV
A. JV, owner of a parcel of land, sold it to PP. But the deed of sale was not
registered. One year later, JV sold the parcel again to RR, who succeeded to
register the deed and to obtain a transfer certificate of title over the property in his
own name.
Page 305 of 391
Who has a better right over the parcel of land, RR or PP? Why? Explain the
legal basis for your answer. (5%)
08; Sales; double sales
1986 No. 18:
Mapusok sold his lot to Masugid under a pacto de retro sale. The lot was
registered under the Torrens system but the pacto de retro sale was not registered.
Subsequently, Masigla obtained a money judgment against Mapusok. Pursuant to a
writ of execution, the lot was attached, the attachment being annotated on the
certificate of title. The purchaser at the public auction was Masigla himself. When
Masigla sought to register his title, Masugid opposed the registration on the ground
of the prior pact de retro sale to him.
Who as between Masugid and Masigla has the better right to the land?
Explain.
Answer:
Under the doctrine of Campillo vs. CA, 129 SCRA 513, Masigla has a better
right because at the time of attachment and sale at public auction, the property was
still registered in name of Mapusok - hence the rule on Torrens Titled land and Art.
1544 Civil Code of the Philippines (double sale) will apply:
Answer - Masigla has a better right because he is an innocent purchaser for
value. He cannot be required to go beyond or outside of the four corners of the
certificate of title presented to him.
Answer - Although the Torrens system requires registration of conveyances
and other instruments affecting registered lands as the "operative act" to convey and
affect the property, and if not registered, the contract is binding only as between the
parties, still the purchaser at an execution sale under the Rules of Court merely
acquires the rights of the judgment debtor in the property, and "steps into the shoes"
of the judgment debtor (Mapusok). Therefore, Masugid is entitled to the land as a
vendee a retro and Masigla (purchaser) merely acquired Mapusok's (judgment
debtor) right to redeem the land under the pacto de retro sale, from Masugid.
08; Sales; double sales
1987 No. 8:
Miguel, Carlos and Lino are neighbors. Miguel owned a piece of registered
land which both Carlos and Lino wanted to buy. Miguel sold the land to Carlos. The
sale was not registered upon the request of Miguel. Later on, the same property was
sold by Miguel to Lino. Miguel told Carlos about the second sale. Carlos immediately
tried to see Lino to discuss the matter and inform him of the previous sale to him
(Carlos) of the same property but Lino refused to see Carlos. Thereupon Carlos
annotated in the Registry of Property his adverse claim on the property. A week
later, Lino registered the sale on his favor and had a new transfer certificate of title
issued in his name. However, the adverse claim of Carlos was duly annotated in the
title. Notwithstanding, Lino took possession of the property and built a small
bungalow thereon.
(a) Who is the rightful owner of the property? Explain.
(b) To whom would the bungalow built by Lino on the property belong?
Explain,
A rawer:
a, In double sales, under Article 1544 the land sold belongs to the first
registrant in good faith. If none, it belongs to the first possessor in good faith. If none
Page 306 of 391
it belongs to the person with the oldest title, provided there is good faith. Carlos, who
has the oldest title, is therefore the rightful owner of the' property, because there was
no registration in good faith by Lino.
b. The bungalow built by Lino belongs to Carlos. Lino is a builder in bad faith.
Article 449 provides that he who builds in bad faith on the land of another loses what
it built without right to indemnity.
08; Sales; double sales
1988 No. 13:
(b) In 1950, A executed a power of attorney authorizing B to sell a parcel of
land consisting of more than 14 hectares. A died in 1954. In 1956, his four children
sold more than 12 hectares of the land to C. In 1957, B sold 8 hectares of the same
land to D, It appears that C did not register the sale executed by the children. D, who
was not aware of the previous sale, registered the sale executed by B, whose
authority to sell was annotated at the back of the Original Certificate of Title.
(1) What was the effect of the death of A upon B's authority to sell the land?
(2) Assuming that B still had the authority to sell the landwho has a better
right over the said land, C or D?
Answer:
(b) (1) Bs authority subsisted notwithstanding the principals death because
he was unaware of such death and he contracted w/ 3
rd
persons who apparently
acted in good faith.
(2) As the case at bar is a case of double sale of registered land he who
recorded the sale in good faith has a better right in conformity with Art. 1544 of the
Civil Code. Since D was not aware of the previous sale, he had to rely on the face of
the certificate of title of the registered owner. Hence, he now has a better right to the
land. (Buason vs. Panuyas, supra.)
08; Sales; double sales
1989 No. 9:
(1) If the same thing should have been sold to different vendees, to whom
shall the ownership be transferred?
Answer:
If the same thing should have been sold to different vendees, the ownership
shall be transferred to the person who may have first taken possession thereof in
good faith, if it should be movable property.
Should it be immovable property, the ownership shall belong to the person
acquiring it who in good faith first recorded it in the Registry of Property.
Should there be an inscription, the ownership shall pertain to the person who
in good faith was first in the possession; and, in the absence thereof, to the person
who presents the oldest title, provided there is good faith.
08; Sales; effect of oral sale
1988 No. 15:
(b) One-half of a parcel of land belonging to A and B was sold by X to Y for
the amount of P1,500.00. The sale was executed verbally. One year later, A and B
sold the entire land to X. Is the sale executed verbally by X to Y valid and binding?
Reasons.
Answer:
Page 307 of 391
(b) The sale, although not contained in a public instrument or formal writing,
is nevertheless valid and binding for the time-honored rule is that even a verbal
contract of sale of real estate produces legal effects between the parties. In the
premises, Art. 1434 of the Civil Code, which declares that when a person who is not
the owner of a thing sells or alienates and delivers it, and later the seller or grantor
acquires title thereto, such title passes by operation of law to the buyer or grantee, is
applicable, (Bucton vs. Gabar, 55 SCRA 499.)
Suggested Alternative Answers To: No, 15 (b):
(b) 1) The contract of sale is valid and enforceable in view of the payment of
the price of P1,500 but there is no showing the problem that there was delivery of
the land. Accordingly, Article 1434 does not apply. However, Y can compel under
Article 1357 to observe the proper form of a deed of sale involving real property and
simultaneously compel specific performance to deliver.
2) The verbal sale of land is unenforceable since there is no statement in the
problem that the agreed price of P1,500 was paid, nor was the land delivered.
Being, Article 1434 will not apply since it is predicated on a valid or enforceable
contract of sale.
08; Sales; equitable mortgage
1991 No 10;
On 20 December 1970, Juliet, a widow, borrowed from Romeo P4,000.00
and, as security therefore, she executed a deed of mortgage over one of her two (2)
registered lots which has a market value of P15,000.00. The document and the
certificate of title of the property were delivered to Romeo.
On 2 June 1971, Juliet obtained an additional sum of P3,000 from Romeo.
On this date, however, Romeo caused the preparation of a deed of absolute sale of
the above property, to which Juliet affixed her signature without first reading the
document. The consideration indicated is P7,000.00. She thought that this document
was similar to the first she signed. When she reached home, her son X, after
reading the duplicate copy of the deed, informed her that what she signed was not a
mortgage but a deed of absolute sale. On the following day, 3 June 1971, Juliet,
accompanied by X, went back to Romeo and demanded the reformation it, Romeo
prepared and signed a document wherein, as vendee In the deed of sale above
mentioned, he obligated and bound himself to resell the land to Juliet or her heirs
and successors for the same consideration as reflected in the deed of sale (P7,000)
within a period of two (2) years, or until 3 June 1973. It Is further stated therein that
should the Vendor (Juliet) fail to exercise her right to redeem within the said period,
the conveyance shall be deemed absolute and irrevocable. Romeo did not take
possession of the property. He did not pay the taxes thereon.
Juliet died in January I973 without having repurchased the property. Her only
surviving heir, her son X, failed to repurchase the property on or before 3 June 1973.
In 1975, Romeo sold the property to Y for P50,000.00. Upon learning of the sale, X
filed an action for the nullification of the sale and for the recovery of the property on
the ground that the so-called deed of absolute sale executed by his mother was
merely an equitable mortgage, taking into account the inadequacy of the price and
the failure of Romeo to take possession of the property and to pay the taxes
thereon. Romeo and Y maintain that there was a valid absolute sale and that the
document signed by the former on 3 June 1973 was merely a promise to sell.
(a) If you were the Judge, would you uphold the theory of X?
(b) If you decide in favor of Romeo and Y, would you uphold the validity of the
promise to sell?
Page 308 of 391
Answer:
A. I will not uphold the theory of X for the nullification of the sale and for the
recovery of the property on the ground that the so-called sale was only an equitable
mortgage. An equitable mortgage may arise only if, in truth, the sale was one with
the right of repurchase. The facts of the case state that the right to repurchase was
granted after the absolute deed of sale was executed. Following the rule in Cruzo
vs. Carriaga (174 SCRA 330), a deed of repurchase executed independently of the
deed of sale where the two stipulations are found in two instruments instead of one
document, the right of repurchase would amount only to one option granted by the
buyer to the seller. Since the contract cannot be upheld as a contract of sale with
the right to repurchase, Art. 1602 of the Civil Code on equitable mortgage will not
apply. The rule could have been different if both deeds were executed on the same
occasion or date, in which case, under the ruling in spouses Claravall v. CA (190
SCRA 439), the contract may still be sustained as an equitable mortgage, given the
circumstances expressed in Art. 1602. The reserved right to repurchase Is then
deemed an original intention.
B, If I were to decide in favor of Romeo and Y, I would not uphold the
validity of the promise to sell, so as to enforce it by an action for specific
performance. The promise to sell would only amount to a mere offer and, therefore,
It is not enforceable unless it was sought to be exercised before a withdrawal or
denial thereof
Even assuming the facts given at the end of the case, there would have been
no separate consideration for such promise to sell. The contract would at most
amount to an option which again may not be the basis for an action for specific
performance.
Obligations and contracts; 1991 No 11:
A is the lessee of an apartment owned by Y. A allowed his married but
employed daughter B, whose husband works in Kuwait, to occupy it. The
relationship between Y and A soured. Since he has no reason at all to eject A, Y, in
connivance with the City Engineer, secured from the latter an order for the
demolition of the building. A Immediately filed an action in the Regional Trial Court to
annul the order and to enjoin its enforcement. Y and A were able to forge a
compromise agreement under which A agreed to a twenty percent (20%) increase in
the monthly rentals. They further agreed that the lease will expire two (2) years later
and that in the event that Y would sell the property, either A or his daughter B shall
have the right of first refusal. The Compromise Agreement was approved by the
court. Six (6) months before the expiration of the lease, A died. Y sold the property
to the Visorro Realty Corp. without notifying B. B then filed an action to rescind the
sale in favor of the corporation and to compel Y to sell the property to her since
under the Compromise Agreement, she was given the right of first refusal which, she
maintains is a stipulation pour atrui under Article 1311 of the Civil Code.
Is she correct?
Answer;
B is not correct. Her action cannot prosper. Article 1311 requires that the third
person intended to be benefited must communicate his acceptance to the obligor
before the revocation. There is no showing that B manifested her acceptance to Y at
any time before the death of A and before the sale. Hence. B cannot enforce any
right under the alleged stipulation pour atrui.
08; Sales; equitable mortgage
1977 No. XVI-c
Page 309 of 391
When may a contract of sale of realty be presumed to be an equitable
mortgage? Cite five (5) instances,
Answer
The contract shall be presumed to be an equitable mortgage, in any of the
following cases:
(1) When the price of sale with right to repurchase is unusually inadequate;
(2) When the vendor remains in possession as lessee or otherwise;
(3) When upon or after the expiration of the right to repurchase another
instrument extending the period of redemption or granting a new period is executed;
(4) When the purchaser retains for himself a part of the purchase price;
(5) When the vendor binds himself to pay the taxes on the thing sold;
(6) In any other case where it may be fairly inferred that the real intention of
the parties is that the transaction shall secure the payment of a debt or the
performance of any other obligation; and
(7) When there is a doubt as to whether the contract is a contract of sale
with right of repurchase or an equitable mortgage. (Arts. 1602, 1603, Civil Code).
08; Sales; equitable mortgage
1979 No. VIII
In a document dated June 10, 1960 and expressly denominated "Deed of
Sale with Right to Repurchase," AB sold his land to CD. Substantially, the document
provided among others: "I, AB, being in great need of money, hereby sell my 10-
hectare coconut land to CD for P2.000 00. It is agreed that I have the right to
repurchase this land in 10 years. If I fail to buy back the property, I shall deliver
possession thereof to CD." Upon failure of AB to repurchase the property, CD, in
1971, consolidated his title and files an action to recover possession, AB files an
answer offering to return the P2,000.00 plus interest at the legal rate. Will the action
of CD prosper? Why?
Answer
The action of CD will not prosper. The contract in the instant case is not a
true contract of sale with right of repurchase. The purchase price is unusually
inadequate and the vendor is still in possession of the property. There is now a
presumption that the real covenant or agreement is an equitable mortgage. This is
strengthened by the fact that AB, the vendor, was in dire straights: he was in great
need of money. The land, therefore, is merely the security for the loan.
Alternative Answer
The action of CD will not prosper. Whether we look at the deed of sale as a
true contract of sale with right of repurchase or a mere contract of equitable
mortgage, the effect in the instant case will be the same.
If it is a true contract of sale with right of repurchase, according to the Civil
Code, the vendor may still exercise the right to repurchase within thirty days from
the time final judgment was rendered in a civil action on the basis that the contract
was a true sale with right to repurchase.
If it is a mere equitable mortgage, which we believe it is because of the gross
inadequacy of the purchase, the fact that the vendor is still in possession of the
property, and the fact that AB was in great need of money, the vendor (mortgagor)
may still exercise his right to redeem the property by paying the mortgage loan plus
interest.
Page 310 of 391
08; Sales; equitable mortgage
1980 No. VI
(a) "S" executed a Deed of Sale of a parcel of land in favor of "T" reserving
for himself the right to repurchase the same within five years from the date of the
contract. The contract provided that during the repurchase period "S" will retain
possession of the land as lessee and pay the land taxes thereon. The consideration
for the sale was P10,000.00 but the land was worth double the price. "S" failed to
repurchase the land within the agreed period and "T" applied to the Court for the
consolidation of his title. "S" opposed the application and claimed that he had the
right to repurchase the land.
Whose stand should be upheld?
Answer
(a) The stand of "S" should be upheld.
In reality, the contract in the instant case is an equitable mortgage. The land
is merely the collateral or security for the payment of a loan of P10,000.00. This is
obvious from the deed of sale itself. In the first place, it says that "S" will retain
possession of the land as lessee; in the second place, it says that "S", the vendor,
shall pay the taxes thereon; and in the third place, the purchase price is unusually
inadequate. According to the Civil Code, the presence of any of these will be
sufficient to raise the presumption that the contract is an equitable mortgage.
(Note: The above answer is based on Art 1602, Civil Code. See also Gardner
vs. CA, 80 SCRA 399; Gloria-Diaz vs. CA, 84 SCRA 483; Labasan vs. Lacuesta, 86
SCRA 16.)
08; Sales; Maceda law
2000 No XIII
Priscilla purchased a condominium unit in Makati City from the Citiland
Corporation for a price of P10 Million, payable P3 Million down and the balance with
interest thereon at 14% per annum payable in sixty (60) equal monthly installments
of P198,333.33. They executed a Deed of Conditional Sale in which it is stipulated
that should the vendee fail to pay three (3) successive installments, the sale shall be
deemed automatically rescinded without the necessity of judicial action and all
payments made by the vendee shall be forfeited in favor of the vendor by way of
rental for the use and occupancy of the unit and as liquidated damages. For 46
months, Priscilla paid the monthly installments religiously, but on the 47th and 48th
months, she failed to pay. On the 49th month, she tried to pay the installments due
but the vendor refused to receive the payments tendered by her. The following
month, the vendor sent her a notice that it was rescinding the Deed of Conditional
Sale pursuant to the stipulation for automatic rescission, and demanded that she
vacate the premises. She replied that the contract cannot be rescinded without
judicial demand or notarial act pursuant to Article 1592 of the Civil Code.
a) Is Article 1592 applicable? (3%)
b) Can the vendor rescind the contract? (2%)
SUGGESTED ANSWER:
a) Article 1592 of the Civil Code does not apply to a conditional sale. In
Valarao v. CA, 304 SCRA 155, the Supreme Court held that Article 1592 applies
only to a contract of sale and not to a Deed of Conditional Sale where the seller has
reserved title to the property until full payment of the purchase price. The law
applicable is the Maceda Law.
Page 311 of 391
SUGGESTED ANSWER;
b) No, the vendor cannot rescind the contract under the circumstances.
Under the Maceda Law, which is the law applicable, the seller on installment may
not rescind the contract till after the lapse of the mandatory grace period of 30 days
for every one year of installment payments, and only after 30 days from notice of
cancellation or demand for rescission by a notarial act. In this case, the refusal of
the seller to accept payment from the buyer on the 49th month was not Justified
because the buyer was entitled to 60 days grace period and the payment was
tendered within that period. Moreover, the notice of rescission served by the seller
on the buyer was not effective because the notice was not by a notarial act. Besides,
the seller may still pay within 30 days from such notarial notice before rescission
may be effected. All these requirements for a valid rescission were not complied with
by the seller. Hence, the rescission is invalid.
08; Sales; Maceda law
1976 No. IX-b
If A and B fix the price at F50.000.00 payable in installment, secured by a
chattel mortgage on the car and a real estate mortgage by a third party, upon
foreclosure of the chattel mortgage, may A foreclose the real estate mortgage for the
unpaid balance? Explain.
Answer
No, according to the decided cases of Cruz and Reyes v. Filipinas Investment
and Financing Corporation and Pascual v. Universal Motors, the seller cannot
recover the deficiency by foreclosing the real estate mortgage given by the 3rd party
because the latter would have a right to be indemnified by B and therefore indirectly
the seller would be recovering the deficiency from B which is prohibited by law,
(Article 1484).
08; Sales; Maceda law
1977 No. V-a
A bought on installment a residential subdivision lot, but after the 5th year,
was unable to make further payments. Can the developer cancel the sale
unilaterally, or must he go to court to obtain rescission? Is A entitled to any refund?
Answer
Yes, the developer can cancel the sale unilaterally. He need not go to court in
order to obtain rescission, provided that the actual cancellation of the contract shall
take place after thirty days from receipt by the buyer of the notice of cancellation or
the demand for rescission of the contract by a notarial act and upon full payment of
the cash surrender value to the buyer. (Rep. Act No. 6552, Sec. 8, (b).)
A shall be entitled to the cash surrender value, which is fifty per cent (60%) of
the total payment made by him to the seller (Ibid).
08; Sales; Maceda law; Recto law
1999 No XIII
What are the so-called "Maceda" and "Recto" laws in connection with sales
on installments? Give the most important features of each law. (5%)
ANSWER:
The Maceda Law (R.A. 655) is applicable to sales of immovable property on
installments. The most important features are (Rillo v. CA, 247 SCRA 461):
Page 312 of 391
(1) After having paid installments for at least two years, the buyer is entitled
to a mandatory grace period of one month for every year of installment payments
made, to pay the unpaid installments without interest.
If the contract is cancelled, the seller shall refund to the buyer the cash
surrender value equivalent to fifty percent (50%) of the total payments made, and
after five years of installments, an additional five percent (5%) every year but not to
exceed ninety percent (90%) of the total payments made.
(2) In case the installments paid were less than 2 years, the seller shall give
the buyer a grace period of not less than 60 days. If the buyer fails to pay the
installments due at the expiration of the grace period, the seller may cancel the
contract after 30 days from receipt by the buyer of the notice of cancellation or
demand for rescission by notarial act.
The Recto Law (Art. 1484} refers to sale of movables payable in installments
and limiting the right of seller, in case of default by the buyer, to one of three
remedies:
(a) exact fulfillment;
(b) cancel the sale if two or more installments have not been paid;
(c) foreclose the chattel mortgage on the things sold, also in case of default
of two or more installments, with no further action against the purchaser.
08; Sales; option contract
2002 No XIV.
A. Explain the nature of an option contract. (2%)
SUGGESTED ANSWERS:
A. An option contract is one granting a privilege to buy or sell within an
agreed time and at a determined price. It must be supported by a consideration
distinct from the price. (Art. 1479 and 1482, NCC)
08; Sales; option contract
1975 No. XV
A agreed to sell to B a parcel of land for P5,000. B was given up to May 6,
1975 within which to raise the necessary funds. It was further agreed that if B could
not produce the money on or before said date, no liability would attach to him.
Before May 6, 1976, A backed out of the agreement Is A obliged to sell the property
to B? Explain.
Answer
This is an option given by A to B for the latter to buy A's property. As it is not
supported by a consideration distinct from the price of the sale, the option can be
withdrawn at anytime before it is accepted. (Art. 1324, 1479).
On the other hand, if D bound himself to buy it for the price stated at the time
the agreement was entered into, then it became a bilateral promise to buy and sell
which is reciprocally demandable.
08; Sales; option; earnest money; Art. 1592
1993 No. 8:
LT applied with BPI to purchase a house and lot In Quezon City, one of its
acquired assets. The amount offered was Pl,000,000.00 payable, as follows:
P200,000.00 down payment, the balance of P800,000.00 payable within 90 days
from June 1, 1985. BPI accepted the offer, whereupon LT drew a check for
Page 313 of 391
P200,000.00 in favor of BPI which the latter thereafter deposited in its account. On
September 5, 1985, LT wrote BP'I requesting extension until October 10, 1985.
within which to pay the balance, to which BPI agreed. On October 5, 1985, due to
the expected delay in the remittance of the needed amount by his financier from the
United States, LT wrote BPI requesting a last extension until October 30, 1985,
within which to pay the balance. BPI denied LTs request because another had
offered to buy the; same property for P1,500,000.OO. cancelled its agreement with
LT and offered to return to him the amount of P200,200.00 that LT had paid to it. On
October 20, 19!85, upon receipt of the amount of P800,000.00 from his US financier,
LT offered to pay the amount by tendering a cashier's check therefor but which BPI
refused to accept. LT then filed a complaint against BPI in the RTC for specific
performance and deposited in court the amount of P800,OOO.OO.
Is BPI legally correct in canceling its contract with LT?
Answer;
BPI is not correct in canceling the contract with LT. In Lina Topacio v Court of
Appeals and BPI Investment (G. R No. 102606, July 3. 1993, 211 SCRA 291). the
Supreme Court held that the earnest money is part of the purchase price and is
proof of the perfection of the contract. Secondly, notarial or judicial rescission under
Art. 1592 and 1991 of the Civil Code is necessary (Taguba v. de Leon, 132 SCRA
722.)
Alternative Answer:
BPI is correct in canceling its contract with LT but BPI must do so by way of
Judicial rescission under Article 1191 Civil Code. The law requires a judicial action,
and mere notice of rescission is insufficient if it is resisted. The law also provides
that slight breach is not a ground for rescission (Song Fo & Co, vs, Hawaiian Phil
Co., 47 Phils. 821), Delay in the fulfillment of the obligation (Art. 1169, Civil Code) is
a ground to rescind, only if time is of the essence. Otherwise, the court may refuse
the rescission if there is a just cause for the fixing of a period.
08; Sales; pacto de retro; when not
1977 No. XIV-a
For only P100,000.00, V sold to C a house and lot valued at P200,000.00. A
month later, C stipulated in writing that V may repurchase in 2 years for P120,000.
After 4 years, C refused to reconvey and V sued for reformation. What legal grounds
should be sustained? How should C resist the suit? Who should prevail and why?
Answer
V may try to sustain his position by claiming that the instrument executed by
C stipulating that V may repurchase the property for P120,000.00 is a part or
continuation of the previous absolute sale, and that the two transactions taken
together constitute a contract of sale with right of repurchase. Hence, because the
price is unusually inadequate, the contract is in reality a contract of equitable
mortgage.
C, on the other hand, should resist the suit by claiming that the two
transactions taken together are separate and distinct from each other. The first is an
absolute sale, while the second, wherein C stipulated that V may repurchase the
property, is merely an option to buy. Hence, the presumption of an equitable
mortgage cannot be sustained. Besides, even assuming arguendo that there is
indeed a contract of sale with right of repurchase, such right has already prescribed
(Art. 1606, Civil Code).
Because of the ground stated by C, he should prevail. (Villarica vs. Court of
Appeals, 26 SCRA 189)
Page 314 of 391
08; Sales; perfected sale
2002 No XIV.
Bert offers to buy Simeons property under the following terms and
conditions: P 1 million purchase price, 10% option money, the balance payable in
cash upon the clearance of the property of all illegal occupants. The option money is
promptly paid and Simeon clears the property of illegal occupants in no time at all.
However, when Bert tende4rs payment of the balance and ask Simeon for the deed
for absolute sale, Simeon suddenly has a change of heart, claiming that the deal is
disadvantageous to him as he has found out that the property can fetch three time
the agreed purchase price. Bert seeks specific performance but Simeon contends
that he has merely given Bert an option to buy and nothing more, and offers to
return the option money which Bert refuses to accept.
B. Will Berts action for specific performance prosper? Explain. (4%)
C. May Simeon justify his refusal to proceed with the sale by the fact that
the deal is financially disadvantageous to him? Explain. (4%)
SUGGESTED ANSWERS:
B. Berts action for specific performance will prosper because there was a
binding agreement of sale, not just an option contract. The sale was perfected upon
acceptance by Simeon of 10% of the agreed price. This amount is in really earnest
money which, under Art. 1482, shall be considered as part of the price and as proof
of the perfection of the contract. (Topacio v. CA, 211 SCRA 291 [1992]; Villongco
Realty v. Bormaheco, 65 SCRA 352 [1975]).
C. Simeon cannot justify his refusal to proceed with the sale by the fact
that the deal is financially disadvantageous to him. Having made a bad bargain is
not a legal ground for pulling out a biding contract of sale, in the absence of some
actionable wrong by the other party (Vales v. Villa, 35 Phil 769 [1916]), and no such
wrong has been committed by Bert.
08; Sales; perfected sale
1989 No. 13:
(1) "X" offered to buy the house and lot of "Y" for P300,000. Since "X" had
only P200,000 in cash at the time, he proposed to pay the balance of P1OO.OOO in
four (4) equal monthly installments. As the title to the property was to be immediately
transferred to the buyer, "X", to secure the payment of the balance of purchase
price, proposed to constitute a first mortgage on the property in favor of "Y". "Y"
agreed to the proposal so that on April 15, 1987, the contract of sale in favor of "X"
was executed and on the same date (April 15,1987), "X" constituted the said first
mortgage. When the first installment became due. "X" defaulted in the payment
thereof, "Y" now brings an action to rescind the contract of sale, which "X" opposed.
How would you decide the conflict? Give your reasons.
Answer:
Either of the following answers should be given full credit:
A. "Y" cannot rescind. The relationship is no longer that of buyer and seller
because the sale was already perfected and consummated. The relationship is
already that of mortgagor and mortgagee. Rescission is not a principal action
retaliatory in character but a subsidiary one available only in the absence of any
other legal remedy. Foreclosure is not only a legal but a contractual remedy. The
debtor must pay and, in case of breach, the mortgagee may foreclose.
B. "Y" can rescind. Specific performance and rescission are alternative
remedies in breach of reciprocal obligations. The contract is only partly
Page 315 of 391
consummated. The price is not fully paid. The mortgage is an accessory contract of
guarantee and can be waived by the creditor who can avail of his remedies in the
principal contract.
Alternative Answers:
C. Considering that the default covers only P25,000.00 and the sum of
P206,000.00 has already been paid, there is only, a slight or casual breach negating
the right of the seller to rescind the contract of sale.
D. Rescission is available provided that the vendor give the vendee the 60-
day period as required by the Maceda Law or the Realty Installment Buyers Law.
08; Sales; perfected sale
1989 No. 13:
(2) "X" came across an advertisement in the "Manila Daily Bulletin" about the
rush sale of three slightly used TOYOTA cars, Model 1989 for only P200,000 each.
Finding the price to be very cheap and in order to be sure that he gets one unit
ahead of the others, "X" immediately phoned the advertiser "Y" and place an order
for one car. "Y" accepted the order and promised to deliver the ordered unit on July
15,1989. On the said date, however, "Y" did not deliver the unit. "X" brings an action
to compel "Y" to deliver the unit. Will such action prosper? Give your reasons.
Answer;
The contract in this case has been perfected. However, the contract is
unenforceable under the statute of -frauds, The action will prosper if there is no
objection to the oral evidence, which amounts to a waiver of the statute of frauds.
08; Sales; perfected sale
1980 No. V
(b) "Q", the owner of a house and lot in Quezon City, gave an option to "R" to
purchase said property for P100,000.00 within ninety days from May 1, 1979. "R"
gave "Q" one (Pl.00) peso as option money. Before the expiration of the ninety-day
period, "R" went to "Q" to exercise his option and to pay the purchase price but "Q"
refused because somebody wanted to buy his property for P150,000 and because
there was no sufficient consideration for the option. "R" sued "Q" to compel him to
accept payment and execute a deed of sale in his favor.
Decide the case.
Answer
(b) "Q" should be compelled to accept the purchase price of P100,000 and
to execute a deed of sale of the subject property hi favor of "R". The reason is that
there is already a perfected contract of sale.
Undoubtedly, in the instant case, there is a unilateral offer of "Q" to sell the
subject property to "R". For that purpose, the latter is given an option of ninety days
from May 1, 1979 within which to exercise the option. The consideration for the
option is P1.00. Since there is a consideration for the option, "Q" is now bound by
his promise to sell the property to "R" so long as the latter will exercise the option
within the agreed period of ninety days (Arts. 1324, 1479, par. 2, Civil Code). "R"
exercised his option. Therefore, there is already a perfected contract of sale.
At any rate, even assuming that there is indeed an insufficient consideration,
or that there is no consideration whatsoever, the result would still be the same.
Since "R" accepted the offer before it could be withdrawn or re-voiced by "Q", there
is already a perfected contract of sale. (Sanchez vs. Rigos 45 SCRA 368).
08; Sales; Recto law
Page 316 of 391
1976 No. IX-c
May it be stipulated that in a foreclosure of the chattel mortgage to secure the
purchase of a car on installment, the installments paid will not be refunded? Explain,
Answer
Yes, such a stipulation may be construed as a penalty clause and shall be
valid insofar as the sum is not unconscionable. (Article 1486)
08; Sales; Recto Law
1981 No. 10
"O", owner of a copying machine, leased it to "L" at a rental of P4,OOO.OO a
month for a period of one year with option on the part of "L" to buy the copying
machine at the end of the year for P80,000.00, to be paid by applying the rentals, so
that "L" needs only to pay P32,OOO.OO.
"L" failed to pay rentals for the 4th, 5th and 6th months so that "0" terminated
the lease and repossessed the copying machine, the sued "L" for the unpaid rental
of three months, or P12,000.00.
Is "0s suit legally tenable? Explain.
Answer
"0's" suit is legally untenable.
By express provisions of Art. 1485 of the Civil Code, the preceding article
(Art. 1484) shall be applied to contracts purporting to be leases of personal property
with option to buy, when the lessor has deprived the lessee of the possession or
enjoyment of the thing. Consequently, applying Art. 1484, upon taking possession of
the copying machine, "O" has no further action against "L" to recover the unpaid
rents.
(Note: The above answer is based on the Recto Law (Arts. 1484, No. 3, and
1485, Civil Code) and on U.S. Commercial Co, vs. Halili, 93, Phil. 371.1
08; Sales; Recto law; recovery of deficiency
1984 No 16
A bought a truck from B payable in installment secured by a chattel mortgage
executed by A on the truck. As additional security, A's brother, C, executed a real
estate mortgage in favor of B.
A defaulted in the payment of several installments. Consequently, B filed an
action for replevin, repossessed the truck, and foreclosed the chattel mortgage.
Can B proceed against the other properties of A and the real estate mortgage
executed by C to recover the deficiency, if any, after the chattel mortgage
foreclosure sale? Explain.
Answer;
A. Furnished by the Office of Justice Planet.
No. Under Art. 1484, in a contract of sale of personal property the price of
which is payable in installments, if the seller elects to foreclose after buyer defaults,
he shall have no further action against the purchaser to recover any unpaid balance.
Since the principal obligation is extinguished, the mortgage executed by C as
security therefore will also necessarily be released. (Art. 2086).
B. Comments and Suggested Answer
We agree with the answer of the Bar Examiner.
Page 317 of 391
08; Sales; redemption
2002 No XII.
Adela and Beth are co-owners of a parcel of land. Beth sold her undivided
share of the property to Xandro, who promptly notified Adela of the sale and
furnished the latter a copy of the deed of absolute sale. When Xandro presented the
deed for registration, the register of deeds also notified Adela of the sale, enclosing
a copy of the deed with the notice. However, Adela ignored the notices. A year later,
Xandro filed a petition for the partition of the property. Upon receipt of summons,
Adela immediately tendered the requisite amount for the redemption. Xandro
contends that Adela lost her right of redemption after the expiration of 30 days from
her receipt of the notice of the sale given by him. May Adela still exercise her right of
redemption? Explain. (5%)
SUGGESTED MAIN ANSWER:
Yes, Adela may still exercise her right of redemption notwithstanding the
lapse of more than 30 days from notice of the sale given to her because Article 1623
of the New Civil Code requires that the notice in writing of the sale must come from
the prospective vendor or vendor as the case may be. In this case, the notice of the
sale was given by the vendee and the Register of Deeds. The period of 30 days
never tolled. She can still avail of that right.
(FIRST) ALTERNATIVE MAIN ANSWER:
Adela can no longer exercise her right of redemption. As co-owner, she had
only 30 days from the time she received written notice of the sale which in this case
took the form of a copy of the deed of sale being given to her (Conejero v. CA, 16
SCRA 775 [1966]). The law does not prescribe any particular form of written notice,
nor any distinctive method for notifying the redemptioner (Etcuban v. CA, 148 SCRA
507 [1987]). So long as the redemptioner was informed in writing, he has no cause
to complain (Distrito v. CA, 197 SCRA 606, 609 [1991]). In fact, in Distrito, a written
notice was held unnecessary where the co-owner had actual knowledge of the sale,
having acted as middleman and being present when the vendor signed the deed of
sale.
08; Sales; redemption; conventional and legal
1977 No. XVI-b
When do conventional redemptions and legal redemptions take place?
Answer
Conventional redemption takes place when the vendor reserves the right to
repurchase the thing sold with the obligation to reimburse to the vendee the price of
the sale, the expenses of the contract, other legitimate payments made by reason of
the sale, as well as necessary and useful expenses made on the thing sold. (Arts.
1601, 1616, Civil Code).
Legal redemption takes place when there is a right to be subrogated upon the
same terms and conditions stipulated in the contract, in the place of one who
acquires a thing by purchase or dation in payment, or by any other transaction
whereby ownership is transmitted by onerous title. (Art. 1619, Civil Code).
(NOTE: Enumeration of the different instances when the right of redemption
takes place should also be accepted as sufficient answers.)
08; Sales; redemption; legal
2001 No XIX
Page 318 of 391
Betty and Lydia were co-owners of a parcel of land. Last January 31, 2001,
when she paid her real estate tax, Betty discovered that Lydia had sold her share to
Emma on November 10, 2000. The following day, Betty offered to redeem her share
from Emma, but the latter replied that Betty's right to redeem has already prescribed.
Is Emma correct or not? Why? (5%)
SUGGESTED ANSWER:
Emma, the buyer, is not correct. Betty can still enforce her right of legal
redemption as a co-owner. Article 1623 of the Civil Code gives a co-owner 30 days
from written notice of the sale by the vendor to exercise his right of legal redemption.
In the present problem, the 30-day period for the exercise by Betty of her right of
redemption had not even begun to run because no notice in writing of the sale
appears to have been given to her by Lydia.
08; Sales; redemption; legal
1982 No. 18
"A", "B" and "C" bought a parcel of land. Subsequently, "A" sold his share to
"X".
(a) What right, if any, do "B" and "C" have with respect to the sale? Reason.
(b) May "B" exercise the same right if "A" had sold his share to "C" instead of
to "X"? Reason,
(c) Assume that in question (a) neither "B" nor "C" had exercised the right
and later "B" sold his share to "Y", may "X" exercise that right referred to in question
(a) ? Reason.
Answer
(a) "B" and "C" may exercise the right of legal redemption. In other words,
they can be subrogated to all of the rights of "X" under the same terms and
conditions stipulated in the contract. Should the two desire to exercise the right, they
may only do so in proportion to their respective shares in the thing owned in
common.
(b) No, "B" cannot exercise the same right if "A" had sold his share to "C"
instead of to "X". The reason is obvious. "C" cannot be classified as a third person
within the meaning of the law.
(c) Yes, "X" may exercise the right of legal redemption. For all legal
purposes, he has already become a co-owner. Being a co-owner, he is, therefore
entitled to all of the rights of a co-owner, including the right of legal redemption.
(Note: The above answers are based on Arts. 1619 and 1620 of the Civil
Code.)
08; Sales; redemption; legal; by co-owners
1986 No. 17:
Mayroon, Magari and Kilalanin Sr. are co-owners in equal shares of a piece
of land. Kilalanin Sr. sold his undivided interest to his son Kilalanin Jr. A week later,
Mayroon and Magari served notice on Kilalanin Jr. of their intention to redeem the
portion sold. However, Kilalanin Jr. refused to allow redemption arguing that being
the son of Kilalanin Sr., he was not a third person in contemplation of law with
respect to redemption by co-owners.
Is the refusal by Kilalanin Jr. justified? Explain. Answer:
Page 319 of 391
The son is still a stranger, and under the C.C. when a share of a co-owner is
sold to a third person, the other co-owners may exercise the right of legal
redemption.
A third person is defined by the court in one case as "one who is not a co-
owner."
Answer - No. He is a 3rd person in contemplation of law. The law considers
as a 3rd person any purchaser who is not one of the co-owners. The fact that he is
the son of the vendor co-owner does not make him a co-owner as in fact the son
had acquired the interest of his father by purchase.
Answer - Yes. the son is not a third person (Villanueva vs. Florendo, 139
SCRA 329).
08; Sales; right of first refusal in favor of lessee; effect thereof
1998 No X.
In a 20-year lease contract over a building, the lessee is expressly granted a
right of first refusal should the lessor decide to sell both the land and building.
However, the lessor sold the property to a third person who knew about the lease
and in fact agreed to respect it. Consequently, the lessee brings an action against
both the lessor-seller and the buyer (a) to rescind the sale and (b) to compel specific
performance of his right of first refusal in the sense that the lessor should be ordered
to execute a deed of absolute sale In favor of the lessee at the same price. The
defendants contend that the plaintiff can neither seek rescission of the sale nor
compel specific performance of a "mere" right of first refusal. Decide the case. [5%]
Answer:
The action filed by the lessee, for both rescission of the offending sale and
specific performance of the right of first refusal which was violated, should prosper.
The ruling in Equatorial Realty Development, Inc. vs. Mayfair Theater, Inc. (264
SCRA 483), a case with similar facts, sustains both rights of action because the
buyer in the subsequent sale knew the existence of right of first refusal, hence in
bad faith.
Another Answer:
The action to rescind the sale and to compel the right to first refusal will not
prosper. (Ang Yu Asuncion vs. CA, 238 SCRA 602). The Court ruled in a unanimous
en banc decision that the right of first refusal is not founded upon contract but on a
quasi-delictual relationship covered by the principles of human relations and unjust
enrichment (Art. 19, et seq. Civil Code). Hence the only action that will prosper
according to the Supreme Court is an "action for damages in a proper forum for the
purpose."
08; Sales; right of first refusal in favor of the lessee; effect thereof
1996 No. 14:
Ubaldo is the owner of a building which has been leased by Remigio for the
past 20 years. Ubaldo has repeatedly assured Remigio that if he should decide to
sell the building, he will give Remigio the right of first refusal. On June 30, 1994,
Ubaldo informed Remigio that he was willing to sell the building for P5 Million. The
following day, Remigio sent a letter to Ubaldo offering to buy the building at P4.5
Million. Ubaldo did not reply. One week later, Remigio received a letter from Santos
Informing him that the building has been sold to him by Ubaldo for P5 Million, and
that he will not renew Remigio's lease when it expires. Remigio filed an action
against Ubaldo and Santos for cancellation of the sale, and to compel Ubaldo to
execute a deed of absolute sale in his favor, based on his right of first refusal.
Page 320 of 391
1) Will the action prosper? Explain. Answer:
No, the action to compel Ubaldo to execute the deed of absolute sale will not
prosper. According to Ang Yu v. Court of Appeals (238 SCRA 602), the right of first
refusal is not based on contract but is predicated on the provisions of human
relations and, therefore, its violation is predicated on quasi-delict. Secondly, the right
of first refusal implies that the offer of the person in whose favor that right was given
must conform with the same terms and conditions as those given to the offeree. In
this case, however. Remigio was offering only P4.5 Million instead of P5 Million.
Alternative Answer:
No, the action will not prosper. The lessee's right of first refusal does not go
so far as to give him the power to dictate on the lessor the price at which the latter
should sell his property. Upon the facts given, the lessor had sufficiently complied
with his commitment to give the lessee a right of first refusal when he offered to sell
the property to the lessee for P5 Million, which was the same price he got in selling it
to Santos. He certainly had the right to treat the lessee's counter-offer of a lesser
amount as a rejection of his offer to sell at P5 Million. Thus, he was free to find
another buyer upon receipt of such unacceptable counter-offer (Art. 1319. NCC).
2) If Ubaldo had given Remigio an option to purchase the building instead of
a right of first refusal, will your answer be the same? Explain.
Answer;
Yes, the answer will be the same. The action will not prosper because an
option must be supported by a consideration separate and distinct from the
purchase price. In this case there is no separate consideration. Therefore, the option
may be withdrawn by Ubaldo at any time. (Art. 1324, NCC)
08; Sales; right of repurchase
1993 No. 12:
On January 2, 1980, A and B entered into a contract whereby A sold to B a
parcel of land for and in consideration of P10.000.00. A reserving to himself the right
to repurchase the same. Because they were friends, no period was agreed upon for
the repurchase of the property.
1) Until when must A exercise his right of repurchase?
2) If A fails to redeem the property within the allowable period, what would
you advise B to do for his better protection?
Answer:
1) A can exercise his right of repurchase within four (4) years from the date of
the contract (Art. 1606, Civil Code).
2} I would advise B to file an action for consolidation of title and obtain a
judicial order of consolidation which must be recorded in the Registry of Property
(Art. 1607. Civil Code).
08; Sales; tradition
1977 No. VI-b
What is tradition and give five (5) kinds of tradition which are provided and
recognized in the Civil Code.
Answer
Tradition is a derivative mode of acquiring ownership and other real rights by
virtue of which they are transmitted from the patrimony of the grantor, in which they
had previously existed, to that of the grantee by means of a just title, there being
Page 321 of 391
both the intention and the capacity on the part of both parties (3 Sanchez Roman
238).
The different kinds of tradition which are recognized in the Civil Code are:
(1) Real tradition, which takes place by the delivery or transfer of a thing from
hand to hand if it is movable, or by certain material and possessory acts of the
grantee performed in the presence and with the consent of the grantor if it is
immovable.
(2) Constructive tradition, which takes place by the delivery of a movable or
immovable thing by means of acts or signs indicative thereof. This delivery may take
place in the following ways:
a. Traditio symbolica, which consists in the delivery of a symbol representing
the thing which is delivered, such as the key to a warehouse;
b. Traditio longa manu, which consists in the grantor pointing out to the
grantee the thing which is delivered which at the time must be within sight;
c. Traditio brevi manu, which takes place when the grantee is already in
"possession of the thing under a title which is not of ownership, such as when the
lessee purchases from the lessor the object of the lease; and
d. Traditio constitutum possessorium, which takes place when the
grantor alienates a thing belonging to him, but continues in possession thereof under
a different title, such as that of a lessee, pledgee or depositary.
(3) Quasi-tradicion, which is used to indicate the exercise of a right by the
grantee with the acquiescence of the grantor; and
(4) Tradicion por ministerio de la ley, which refers to delivery that takes
place by operation of law. (See Arts. 1497-1501, Civil Code; 2 Castan 208-209; 3
Sanchez Roman 209-210).
(NOTE: The above kinds of tradition may be stated only, without defining
them.)
08; Sales; transfer of ownership
1990 No 5:
D sold a second-hand car to E for P150,000.00 The agreement between D
and E was that half of the purchase price, or P75,000.00, shall be paid upon delivery
of the car to E and the balance of P75,000.00 shall be paid in five equal monthly
installments of P15,000.00 each. The car was delivered to E. and E paid the amount
of P75.000.00 to D. Less than one month thereafter, the car was stolen from E's
garage with no fault on E's part and was never recovered. Is E legally bound to pay
the said unpaid balance of P75.000.0O? Explain your answer.
Answer:
Yes, E is legally bound to pay the balance of P75,000.OO. The ownership of
the car sold was acquired by E from the moment it was delivered to him. Having
acquired ownership, E bears the risk of the loss of the thing under the doctrine of res
peril domino. [Articles 1496. 1497, Civil Code).
08; Sales; transfer of ownership
1991 No 17:
Pablo sold his car to Alfonso who issued a postdated check in full payment
therefor. Before the maturity of the check. Alfonso sold the car to Gregorio who later
sold it to Gabriel. When presented for payment, the check issued by Alfonso was
Page 322 of 391
dishonored by the drawee bank for the reason that he, Alfonso, had already closed
his account even before he issued his check.
Pablo sued to recover the car from Gabriel alleging that he (Pablo) had been
unlawfully deprived of it by reason of Alfonso's deception.
Will the suit prosper?
Answer:
No. The suit will not prosper because Pablo was not unlawfully deprived of
the car although he was unlawfully deprived of the price. The perfection of the sale
and the delivery of the car was enough to allow Alfonso to have a right "of ownership
over the car, which can be lawfully transferred to Gregorio. Art. 559 applies only to a
person who is in possession in good faith of the property, and not to the owner
thereof. Alfonso, in the problem, was the owner, and. hence, Gabriel acquired the
title to the car.
Non-payment of the price in a contract of sale does not render ineffective the
obligation to deliver.
The obligation to deliver a thing is different from the obligation to pay its price.
EDCA Publishing Co. v. Santos (1990)
08; Sales; vendors lien
1985 No. 13
A) A sold to B a piano for P10,000, payable in monthly installments of
P1,000 each. After paying the first installment, B resold the piano to C who paid
P2,-000.00, leaving a balance of F8,000.00. Thereafter, X sued B for the value of
services rendered to him and had the credit of P8,000, due B from C garnished. A
thereupon filed a third-party claim with the sheriff for P20,000.00, representing the
balance of the price of the piano still unpaid and a loan of P11,000.00 he gave B.
Discuss who between A or X should prevail.
Answers:
A) 1, This is a question of priority between the vendor's lien and the
garnishment order. Concurrences and preferences of credits are not applicable for
the simple reason that there is no special proceeding to convene the creditors, but
since both are preferred in the sense that one is the vendor and the other has a
garnishment order, it is a question of who is preferred between the two. Therefore,
the vendor's lien of A is superior because the sale occurred before. The moment he
sold the piano, there already arose the vendor's lien.
2. The right of X should prevail over that of A. The problem relates to the
efficacy of the garnishment order on B's receivables from the sale and not on the
piano itself. Accordingly, the unpaid seller's lien which is a lien on the piano as the
object of A's sale, not being really involved in the garnishment order, will not allow A
to question said garnishment order.
08; Sales; who bears risk of loss
1981 No. 11
"S", an American resident of Manila, about to leave on a vacation, sold his
car to "B" for U.S. $2,000.00, the payment to be made ten days after delivery to "X",
a third party depositary agreed upon, who shall deliver the car to "B" upon receipt by
"X" of the purchase price. It was stipulated that ownership is retained by "S" until
delivery of the car to "X". Five days after delivery of the car to "X", it was destroyed
in a fire which gutted the house of "X", without the fault of either "X" or "B".
Page 323 of 391
a) Is buyer "B" still legally obligated to pay the purchase price? Explain.
Answer
(a) Yes, buyer "B" is still legally obligated to pay the purchase price. It must
be observed that "S" had already delivered the car to "X", the third party depositary
or bailee. It was agreed that ownership is retained by "S" until delivery to "X".
Therefore, in effect, there was already a transfer of the right of ownership over the
car to "B". Consequently, "B" shall assume the fortuitous loss of the car. As a matter
of fact, even if it was agreed that "S" shall retain the ownership of the car until the
purchase price has been paid by "B", the end result will still be the same. Since
eventually, the purpose is to secure performance by the buyer of his obligation to
pay the purchase price, by express mandate of the law, the fortuitous loss of the car
shall be assumed by "B".
(Note: The above answer is based on Art. 1504 of the Civil Code.)
Page 324 of 391
09; Lease
1980 No. VIII
(b) "CC" owned a two-story commercial building which he leased to
"DD" with the agreement that all improvements made on the building shall belong to
"CC". "DD" made improvements on said building purchasing on credit the materials
needed for said improvements from "EE". "DD" failed to pay rentals to "CC" and
failed to pay the value of the materials to "EE".
"EE" sued "DD" for the unpaid value of the materials. "CC" also sued "DD" for
the unpaid rentals. "CC" levied on the materials. "EE" objected to the levy claiming
that the materials belong to him.
Who is the owner of the materials"CC", the unpaid lessor, or "EE", the
unpaid vendor?
Answer
(b) "CC", the unpaid lessor, is the owner of the materials.
When the materials were delivered and installed in the leased premises, "DD"
became the owner thereof. Ownership is not transferred by perfection of the contract
but by delivery. This is true even if the purchase is an credit, as in the case at bar.
Under the contract of lease entered into by and between "CC" and "DD",
"CC" became the owner of the above materials by virtue of the agreement in said
contract that all improvements shall belong to the lessor. Therefore, when levy was
made by "EE" on the materials, "DD", the judgment debtor, was no longer the owner
thereof.
(NOTE: The above answer is based on Sampaguita Pictures vs. Jalwinder,
93 SCR A 420,)
09; Lease
1983 No. 14
After leasing his restaurant to B, A leased the adjoining room to C knowing
fully well that C was going to put up another restaurant, which he did.
Is A liable to B for the damage he may have suffered as a result of the
opening of C's restaurant? Why?
Answer
(Examiner's Answer)
No. A has not obligated himself not to allow the establishment of another
restaurant adjoining that leased to B nor is there any law which imposes such an
obligation upon him. On the contrary, the Civil Code (Art, 1656) permits the lessor of
a business to continue engaging in the same kind of business to which the thing
leased is devoted unless there is a stipulation to the contrary. If the lessor is not
forbidden, with less reason should a stranger, not a party to the lease be prohibited.
(Committee's Answer)
In addition to the answer of the Bar Examiner, the following answer, we
believe, should also be considered correct:
A is liable to B for the damages he may have suffered as a result of the
opening of C's restaurant. In the exercise of his rights and the performance of his
obligations, A did not observe honesty and good faith. A is, therefore, liable for
damages.
Page 325 of 391
(Note: The above answer is based on Art. 19, Civil Code, If the bar candidate
invokes Art, 21 of the Civil Code, the Committee respectfully recommends that the
answer should be considered correct.)
09; Lease
2000 No XIX.
a) A leased his house to B with a condition that the leased premises shall be
used for residential purposes only. B subleased the house to C who used it as a
warehouse for fabrics. Upon learning this, A demanded that C stop using the house
as a warehouse, but C ignored the demand, A then filed an action for ejectment
against C, who raised the defense that there is no privity of contract between him
and A, and that he has not been remiss in the payment of rent. Will the action
prosper? (3%)
SUGGESTED ANSWER:
Yes, the action will prosper. Under Article 1651 of the Civil Code, the
sublessee is bound to the lessor for all acts which refer to the use and preservation
of the thing leased in the manner stipulated between the lessor and the lessee.
09; Lease of urban lands
1981 No. 12
"S", a landowner of an urban lot covered by a Torrens title, sold it to "B". "B"
saw that the land was occupied by lessees who paid rent on a month to month
basis. "S" told "B" that some lessees had been renting the parcels of land for twelve
(12) years, other for eight (8) years. Unknown to "S" and "B" is that the area had
been declared as urban land reform area.
a) May "S" terminate the lease contracts and eject the lessees? Reasons,
b) If the lessees went to you for legal assistance, what would be your advice?
Explain briefly.
c) Does "B" have a right against "S" in the event he is unable to obtain
possession? Why?
Answer
(a) It is respectfully submitted that there is a typographical error here. The
question should read "may "B" terminate the lease contracts and eject the
lessees? At any rate, let us answer both questions.
"S", of course, cannot because he has already sold the property to "B". He
does not have the necessary legal personality to do so. "B" cannot because he was
aware of the existence of the lease contracts at the time when he bought the
property from "S". Under the law, this is one of the exceptions to the rule that the
purchaser of a piece of land which is under a lease that is not recorded in the
Registry of Property may terminate the lease.
(Note: The above answer with respect to the revised question is based on
Art. 1676 of the Civil Code and on Divina Gratia Agro Commercial vs. CA. April 21,
1981.) The Committee, however, respectfully recommends that if the bar candidate
answers the above question as is or in its revised form, it should be considered
correct,
(b) I would advice them to avail of the provisions of the Civil Code which
declares that the courts may fix a longer term or period for the lease. Under the law
on obligations in general (Art. 1197. Civil Code and under the law on lease (Art.
1687, Civil Code which declare that the courts may fix a longer term to fix or extend
the period of the lease is explicitly recognized.
Page 326 of 391
(Note: The above doctrine was stated in Divina Gracia Agro Commercial,
April 21,1981.)
(c) "B" does not have any right against "S" in the event he is unable to obtain
possession of the property. He was aware of the existence of the lease contracts.
He must, therefore, assume the consequences of his act in buying the property.
(Note: The Committee respectfully recommends that if the bar candidate
comes out with a contrary answer and supports his answer by saying that "B" would
never have bought the property had he known that the area had been declared a
land reform area, it should also be considered a correct answer.)
09; Lease; contract for a piece of work
1987 No. 10
Lino entered into a written agreement for the repair of his private plane with
Airo Repair Works, Inc. for P500,000. Additional work was done upon the verbal
request and authority of a duly recognized representative of Linn. Lino refused to
pay for the additional work, interposing as a defense the absence of a written
contract for the additional work done.
Is the defense put up by Lino valid? Explain? Answer:
The defense put up by Lino is valid under Article 1724 of the Civil Code. The
change must have been authorized by the proprietor in writing, and the additional
price to be paid the contractor must have been determined in writing by both parties.
Answer
!t appearing that the additional work was done upon verbal request and
authority of a duly authorized representative of Lino, and the benefits have been
received by Lino in consequence of the actual repair and the additional work, the
defense put up by Lino is not valid on the ground that no person may unjustly enrich
himself at the expense of another.
09; Lease; contract for a piece of work; liability of architects/contractors
1981 No. 13
"0", lot owner, contracted with "B", builder, to build a multi-story office building
designed by "A", architect. "A" was paid a fee to supervise the construction and
execution of his design. When completed, "O" accepted the work and occupied the
building, but within one year, it collapsed in an earthquake that destroyed only the
building and not the surrounding buildings. Construction was faulty. The building
cost P3,000,000.00, but reconstruction cost would reach P10,000,000.00.
a) What are the rights of "O" against "A" and "B"? Explain briefly.
b) Could "O" demand reconstruction of the building? On what ground?
Amplify.
Answer
(a) "O" can hold "A" and "B" solidarily liable for damages. This is clear from
the Civil Code, which declares that the contractor is liable for damages if within
fifteen years from the completion of the edifice or structure, the same should
collapse on account of defects in the construction. If the engineer or architect who
drew up the plans and specifications of the building supervises the construction, he
shall be solidarily liable with the contractor. Acceptance of the building, after
completion, does not imply waiver of the cause of action. However, the action must
be brought within ten years following the collapse of the building.
(Note: The above answer is based on Art. 1723 of the Civil Code. The
Committee respectfully recommends that if the bar candidate attacks the problem
Page 327 of 391
from the point of view of fortuitous events (Art.1174, Civil Code), he should be
properly credited.)
(b) "O" can demand reconstruction of the building. The obligation of both "A"
and "B" is an obligation to do. Consequently, Art. 1167 of the Civil Code is
applicable. According to this article, if a person obliged to do something does it in
contravention of the tenor of the obligation, the same shall be executed at his cost. It
is obvious that the builder "B" and the architect "A" performed their jobs in
contravention of the tenor of obligation. As a matter of fact, had' the building not
collapsed, under the same article, it may even be decreed that what has been poorly
done be undone. Consequently, "O" can now demand for the reconstruction of the
building by "A" and "B" or by another at their cost.
(Note: The above answer is based on Art. 1167 of the Civil Code and on
Manresa, Vol. 8, pp. 116-117.)
09; Lease; contract of labor
1977 No. XV-a
50 employees of A and Co. brought suit to collect living allowance and
Christmas bonus from their employer. A and Co. resisted, alleging lack of a cause of
action. Which party should be sustained?
Answer
Assuming that they meet all of the conditions prescribed by the law (P.D No.
85, and other decrees), the plaintiffs should be sustained.
As regards the -claim for cost-of-living allowance, the law makes it an
obligation of an employer to pay cost-of-living or emergency allowance to each
employee who is receiving less than P600.00 a month. As regards the Christmas
bonus, the law requires all covered employers to pay their employees a "13-month-
pay" not later than December 24 of each year where the employee's salary is not
more than Pl,000 00 a month.
09; Lease; effect of death of lesee
1997 No. 17:
Stating briefly the thesis to support your answer to each of the following
cases, will the death - a) of the lessee extinguish the lease agreement?
Answer:
a) No. The death of the lessee will not extinguish the lease agreement, since
lease is not personal in character and the right is transmissible to the heirs. (Heirs of
Dimaculangan vs. IAC, 170 SCRA 393).
09; Lease; ejectment
1977 No. XV-b
L leased a house to J and the contract stipulated that in the event of non-
payment of the rent L could eject J without a court action. J defaulted for 2 months
and L ejected him. Can J claim damages because the renunciation of his day in
court is void?
Answer
J cannot claim damages because the renunciation of his day in court is void.
True, under the law, in reciprocal obligations there is always a tacit resolutory
condition that if one party is unable to comply with what is incumbent upon him, the
injured has the power to rescind the obligation (Art. 1191, Civil Code). This rule is
reiterated under the law on lease (Art. 1659, Civil Code). And it is a settled rule that
Page 328 of 391
such injured party must invoke judicial aid. But then, these rules can be applied only
to a case where the obligation is silent with respect to the power to rescind. The right
to rescind is implied only it not expressly granted; no right can be said to be implied
if expressly recognized. This is also well-settled. In the instant case, the right of L to
eject J without a court action in case of non-payment of the rent was expressly
recognized. What L did was merely to enforce what was agreed upon in their
contract.
09; Lease; ejectment
1982 No. 20
By virtue of a written contract, "X" rented an apartment belonging to "Y" on
June 5, 1979, for a monthly rental of P280.00. There was no stipulation with respect
to the duration of the lease. In August 1980, desirous of increasing his rental
income, "Y" devised a scheme to eject "X" from said apartment by refusing to accept
the monthly rental paid by "X". Hence, "X" deposited the rental money with the PNB
in the name of "Y" giving the latter notice of such deposit. After the lapse of 3
months, "Y" filed a complaint against "X" for ejectment on the ground of non-
payment of rent. After trial, the court rendered judgment, dismissing "Y"'s complaint,
but fixed the period of lease to one year. "X" appealed from that portion of the
decision fixing the period of lease. Decide "X"' s appeal with reasons.
Answer
"X"' s appeal is meritorious. True, arrears in payment of rent for three months
at any one time is a ground for ejectment under Sec, 5 (b) of Batas Pambansa Big.
25, but then the law continues; provided, that in case of refusal to accept payment of
the rental agreed upon, the lessee shall either deposit, by way of consignation, the
amount in court or in a bank in the name of and with notice to the lessor. "X",
according to the facts, deposited the amount in the PNB in the name of "Y" giving
the latter notice of the deposit. Therefore, under the law, the lease is still covered by
the rental law (B.P. Blg. 23). "X" cannot be ejected, The order of the court fixing the
period of the lease at one year is contrary to law.
(Note: The above answer is based on the rental law Batas Pambansa Blg.
25.)
09; Lease; extinguishment
1993 No. 9:
A is the owner of a lot on which he constructed a building in the total cost of
P1O,OOO,OOO.OO. Of that amount B contributed P5.000.000,00 provided that the
building as a whole would be leased to him (B) for a period of ten years from
January 1. 1985 to December 31, 1995 at a rental of P100.000.00 a year. To such
condition, A agreed. On December 20, 1990, the building was totally burned. Soon
thereafter, A's workers cleared the debris and started construction of a new building.
B then served notice upon A that he would occupy the building being constructed
upon completion, for the unexpired portion of the lease term, explaining that he had
spent partly for the construction of the building that was burned. A rejected B's
demand.
Did A do right in rejecting B's demand?
Answer;
Yes. A was correct in rejecting the demand of B. As a result of the total
destruction of the building by fortuitous event, the lease was extinguished. (Art.
1655, Civil Code.)
09; Lease; implied new lease
Page 329 of 391
1999 No XIV.
(b) Under what circumstances would an implied new lease or a tacita
reconduccion arise? (2%)
ANSWER:
(b) An implied new lease or tacita reconduccion arises if at the end of the
contract the lessee should continue enjoying the thing leased for 15 days with the
acquiescence of the lessor, and unless a notice to the contrary by either parties has
previously been given (Art. 1670). In short, in order that there may be tacita
reconduccion there must be expiration of the contract; there must be continuation of
possession for 15 days or more; and there must be no prior demand to vacate.
09; Lease; lease of rural lands
2000 No XIX.
b) In 1995, Mark leased the rice land of Narding in Nueva Ecija for an annual
rental of P1,000.00 per hectare. In 1998, due to the El Nino phenomenon, the rice
harvest fell to only 40% of the average harvest for the previous years. Mark asked
Narding for a reduction of the rental to P500.00 per hectare for that year but the
latter refused. Is Mark legally entitled to such reduction? (2%)
SUGGESTED ANSWER:
No, Mark is not entitled to a reduction. Under Article 1680 of the Civil Code,
the lessee of a rural land is entitled to a reduction of the rent only in case of loss of
more than 1/2 of the fruits through extraordinary and unforeseen fortuitous events.
While the drought brought about by the "El Nino" phenomenon may be classified as
extraordinary, it is not considered as unforeseen.
ALTERNATIVE ANSWER:
Yes, Mark is entitled to a reduction of the rent. His loss was more than 1/2 of
the fruits and the loss was due to an extraordinary and unforeseen fortuitous event.
The "El Nino" phenomenon is extraordinary because it is uncommon; it does not
occur with regularity. And neither could the parties have foreseen its occurrence.
The event should be foreseeable by the parties so that the lessee can change the
time for his planting, or refrain from planting, or take steps to avoid the loss. To be
foreseeable, the time and the place of the occurrence, as well as the magnitude of
the adverse effects of the fortuitous event must be capable of being predicted. Since
the exact place, the exact time, and the exact magnitude of the adverse effects of
the "El Nino" phenomenon are still unpredictable despite the advances in science,
the phenomenon is considered unforeseen.
09; Lease; lease with a term
1977 No. IX-b
O leased a building to T, the later to pay P2.000.00 monthly and the light and
water bills. The contract stipulated that non-payment of the rent would automatically
cancel the contract, but otherwise T could stay on indefinitely. After 5 years, O tried
to eject T because he planned to tear the building down and put up another. Can O
eject T?
Answer
O can eject T. T cannot successfully set up the defense that under the
contract of lease, he can continue occupying the building so long as he will faithfully
fulfill his obligation of paying the rentals. It is clear that under this stipulation, the
continuance and fulfillment of the contract would then depend solely and exclusively
upon his uncontrolled choice between continuing paying the rentals or not,
Page 330 of 391
completely depriving the owner of any say on the matter. If this defense is allowed,
so long as defendant elected to continue the lease by continuing the payment of the
rentals, the owner would never be able to discontinue it; conversely, although the
owner should desire the lease to continue, the lessee could effectively thwart his
purpose by the simple expedient of stopping payment of the Civil Code.
(Encarnacion vs. Baldemar, 77 Phil. 470. To the same effect see: General
Enterprises, Inc. vs. Lianga Bay Logging Co., L-18487, August 31, 1964; Garcia vs.
Rita Legarda,, Inc., L-20175, Oct. 30, 1967.)
(NOTE: Another way of answering the above problem would be to say that 0
can eject T on the ground that, In reality, the lease is subject to a term, although
indefinite in character. That will bring into play Arts. 1673 and 1197 of the Civil
Code.)
09; Lease; liability for hidden defects
1978 No VII-b
A leased a cold storage plant to B at a daily rental. B stored its foodstuffs in
said storage plant while the refrigeration facilities of one (1) of its vessels was out of
order. After seven (7) days B found out that the foodstuffs stored in said storage
plant were contaminated by rats and not fit for consumption. They were, therefore,
thrown out to the sea, B sued A for recovery of the loss of the foodstuffs disposed of.
Is A liable? Reasons?
Answer
Yes, A is liable for the loss of the foodstuffs disposed of. According to the
Civil Code, the provisions of the Civil Code on warranty, contained in the law on
sales, shall be applicable to lease, Now, according to the law on sales with respect
to warranty against hidden defects, the vendor (lessor) is responsible to the vendee
(lessee) for any hidden faults or defects in the thing sold (leased), even though he
was not aware thereof. Under this provision, it is clear that A is liable to B for the
loss of the foodstuffs.
(NOTE: The above answer is based on the decision of the Supreme Court in
United States Lines, Inc. vs. San Miguel Brewery, 10 SCRA 808, applying Arts.
1653, 1566 and 1567 of the Civil Code.)
Alternative Answer
Yes, A is liable. According to the Civil Code, the lessor Is obliged to deliver
the thing leased in such a condition as to render it fit for the use intended and to
make on the same during the lease all the necessary repairs in order to keep it
suitable to the use to which it is devoted. In the instant case, it is clear that A has not
complied with both obligations. Therefore, the lessee B may now ask for rescission
of the contract and indemnification for damages, or only the latter, allowing the
contrast to remain in force.
(NOTE: The above answer is based on Arts. 1664 and 1659 of the Civil
Code.)
1978 No. VIII-a
Is an agreement limiting the common carrier's liability for delay in case of a
strike valid?
la a stipulation limiting the common carrier's liability to the value of the goods
declared by the shipper in the bill of lading- valid? Give reasons for your answer.
Answer
(to first question)
Page 331 of 391
Yes, an agreement limiting a common carrier's liability for delay in case of a
strike is valid. This is expressly recognized in the Civil Code under the law on
common carriers. Besides, and this is the reason behind the law, not only natural
disasters or acts of God but even acts of men or force majeure which are
unforeseeable or unavoidable, such as strikes and riots, are classified as fortuitous
events. Under the doctrine of fortuitous events, such an agreement would be
perfectly valid.
(NOTE: The above answer is based on Arts. 1748 and 1174 of the Civil
Code. It is suggested that an answer based on either Art, 1748 or on the doctrine of
fortuitous events above should be considered as a correct answer,)
Answer
(to second question)
I distinguish. If the stipulation limits the carrier's liability to an agreed valuation
such as the value of the goods appearing in the bill of lading, unless the shipper or
owner declares a greater value, it is valid. As a matter of fact, this is stated in the law
on common carriers. If the stipulation limits the carrier's liability to an agreed
valuation without any qualification whatsoever such as the value of the goods
appearing in the bill of lading, as a rule, it is contrary to public policy, and therefore,
void. However, if it can be shown to be reasonable and just under the
circumstances, and had been fairly and freely agreed upon, then it is perfectly valid.
(NOTE: The above answer is based on Arts. 1749 and 1750 of the Civil Code
and on decided cases. It is suggested that an answer based on Art. 1749 alone
should be considered a perfect answer.)
09; Lease; option to buy
2001 No XIV
On January 1, 1980, Nestor leased the fishpond of Mario for a period of three
years at a monthly rental of P1,000.00, with an option to purchase the same during
the period of the lease for the price of P500,000.00. After the expiration of the three-
year period, Mario allowed Nestor to remain in the leased premises at the same
rental rate. On June 15, 1983, Nestor tendered the amount of P500,000 to Mario
and demanded that the latter execute a deed of absolute sale of the fishpond in his
favor. Mario refused, on the ground that Nestor no longer had an option to buy the
fishpond. Nestor filed an action for specific performance. Will the action prosper or
not? Why? (5%)
SUGGESTED ANSWER:
No, the action will not prosper. The implied renewal of the lease on a month-
to-month basis did not have the effect of extending the life of the option to purchase
which expired at the end of the original lease period. The lessor is correct in refusing
to sell on the ground that the option had expired.
09; Lease; reduction of rent
1985 No. 15
B) A leased to B a fishpond for 5 years. During the second year of the
lease, he received only 1/2 of the usual harvest from the fishpond as he could visit
and supervise it only occasionally due to the presence of armed men who were
extorting money from him and other fishpond operators by threats to their lives.
May B demand the reduction of the rent for the second year and the
extension of the lease for one year on the ground that the lessor failed to comply
with his obligation to maintain the lessee in the peaceful and adequate enjoyment of
the lease?
Page 332 of 391
Answers:
B) 1. The reduction of rental only applies to extraordinary fortuitous events.
This is trespass in fact, not trespass in law. With regard to the extension of lease, in
the case of Victorias Milling Co., fortuitous event does not extend the lease
anymore.
2. The law authorizes a reduction of the rent whenever, among other cases,
there is a loss of fruits due to extraordinary and unforeseen fortuitous event, such as
fire, war, pestilence, unusual floods or other calamity (Art 1680). The presence of
armed men who extort money and threaten fishpond operators, in my view, is not
included in the intendment of the law. Neither will be the extension of the lease for
one year be demanded (see Gandoy vs. Tapucar, 76 Phil. 31).
3. B cannot demand a reduction of the rent and ex-tension of the lease. The
warranty of the lessor refers to his own acts and not to the acts of third persons who
are beyond his control. The maintenance of the lessee in the peaceful and adequate
enjoyment of the lease refers to the enjoyment of his rights over the property as
lessee, and not to personal physical disturbance or bodily threat.
09; Lease; rights and obligations of leasee and lessor
1990 No 1;
A vacant lot several blocks from the center of the town was leased by its
owner to a young businessman B. for a term of fifteen (15) years renewal upon
agreement of the parties. After taking possession of the lot, the lessee built thereon
a building of mixed materials and a store. As the years passed, he expanded his
business, earning more profits. By the tenth (10th) year of his possession, he was
able to build a three (3)-story building worth at least P300,000.00. Before the end of
the term of the lease, B negotiated with the landowner for its renewal, but despite
their attempts to do so, they could not agree on the new conditions for the renewal.
Upon the expiration of the term of the lease, the landowner asked B to vacate the
premises and remove his building and other improvements. B refused unless he was
reimbursed for necessary and useful expenses. B claimed that he was a possessor
and builder in good faith, with right of retention. This issue is now before the court for
resolution in a pending litigation.
a) What are the rights of B?
b) What are the rights of the landowner? Answer;
a) B has the right to remove the building and other improvements unless
the landowner decides to retain the building at the time of the termination of the
lease and pay the lessee one-half of the value of the improvements at that time. The
lessee may remove the building even though the principal thing may suffer damage
but B should not cause any more impairment upon the property leased than is
necessary. The claim of B that he was a possessor and builder in good faith with
the right of retention is not tenable. B is not a builder in good faith t because as
lessee he does not claim ownership over the property leased,
b) The landowner/lessor may refuse to reimburse 1/2 of the value of the
improvements and require the lessee to remove the improvements. [Article 1678,
Civil Code),
09; Lease; sublease
1999 No XIV.
(a) May a lessee sublease the property leased without the consent of the
lessor, and what are the respective liabilities of the lessee and sub-lessee to the
lessor in case of such sublease? (3%)
Page 333 of 391
ANSWER:
(a) Yes, provided that there is no express prohibition against subleasing.
Under the law, when in the contract of lease of things there is no express prohibition,
the lessee may sublet the thing leased without prejudice to his responsibility for the
performance of the contract toward the lessor. [Art, 1650)
In case there is a sublease of the premises being leased, the sublessee is
bound to the lessor for all the acts which refer to the use and preservation of the
thing leased in the manner stipulated between the lessor and the lessee. (Art. 1651}
The sublessee is subsidiarily liable to the lessor for any rent due from the
lessee. However, the sublessee shall not be responsible beyond the amount of the
rent due from him. (Art. 1652)
As to the lessee, the latter shall still be responsible to the lessor for the rents;
bring to the knowledge of the lessor every usurpation or untoward act which any
third person may have committed or may be openly preparing to carry out upon the
thing leased; advise the owner the need for all repairs; to return the thing leased
upon the termination of the lease just as he received it, save what has been lost or
impaired by the lapse of time or by ordinary wear and tear or from an inevitable
cause; responsible for the deterioration or loss of the thing leased, unless he proves
that it took place without his fault.
09; Lease; sublease
1990 No 6:
A leased a parcel of land to B for a period of two years. The lease contract
did not contain any express prohibition against the assignment of the leasehold or
the subleasing of the leased premises. During the third year of the lease, B
subleased the land to C. In turn, C, without A's consent, assigned the sublease to D.
A then filed an action for the rescission of the contract of lease on the ground that B
has violated the terms and conditions of the lease agreement. If you were the judge.
how would you decide the case, particularly with respect to the validity of:
(a) B's sublease to C? and
(b) C's assignment of the sublease to D? Explain your answers.
Answer:
(a) B's sublease to C is valid. Although the original period of two years for
the lease contract has expired, the lease continued with the acquiescence of the
lessor during the third year. Hence, there has been an implied renewal of the
contract of lease. Under Art. 1650 of the Civil Code, the lessee may sublet the thing
leased, in whole or in part, when the contract of lease does not contain any express
prohibition. [Articles 1650, 1670 Civil Code). A's action for rescission should
not prosper on this ground.
(b) C's assignment of the sublease to D is not valid. Under Art. 1649, of the
Civil Code, the lessee cannot assign the lease without the consent of the lessor,
unless there is a stipulation to the contrary. There is no such stipulation in the
contract. If the law prohibits assignment of the lease without the consent of the
lessor, all the more would the assignment of a sublease be prohibited without such
consent. This is a violation of the contract and is a valid ground for rescission by A,
09; Lease; sublease; delay in payment of rentals
1994 No, 8:
In January 1993, Four-Gives Corporation leased the entire twelve floors of
the GQS Towers Complex, for a period of ten years at a monthly rental of
Page 334 of 391
P3,000,000.00. There is a provision in the contract that the monthly rentals should
be paid within the first five days of the month. For the month of March, May, June,
October and December 1993, the rentals were not paid on time with some rentals
being delayed up to ten days. The delay was due to the heavy paper work Involved
in processing the checks.
Four-Gives Corporation also subleased five of the twelve floors to wholly-
owned subsidiaries. The lease contract expressly prohibits the assignment of the
lease contract or any portion thereof. The rental value of the building has Increased
by 50% since its lease to Four-Gives Corporation.
1) Can the building owner eject Four-Gives Corporation on grounds of the
repeated delays in the payment of the rent?
2} Can the building owner ask for the cancellation of the contract for violation
of the provision against assignment?
Alternative Answers:
1) a) The "repeated delays" in the payment of rentals would, at best, be a
slight or casual breach which does not furnish a ground for ejectment especially
because the delays were only due to heavy paper work. Note that there was not
even a demand for payment obviously because the delay lasted for only a few days
(10 days being the longest), at the end of which time payments were presumably
made and were accepted. There was, therefore, no default. Note also that there was
no demand made upon the lessee to vacate the premises for non-payment of the
monthly rent. There is, therefore, no cause of action for ejectment arising from the
"repeated delays".
b) The building owner cannot eject Four-Gives Corporation on the ground of
repeated delays in the payment of rentals. The delay in the payment off the rentals
is minimal and cannot be made the basis of an ejectment suit. The delay was due to
the heavy paperwork involved In processing the checks. It would be otherwise if the
lease contract stated that in the payment of rentals within the first five days of the
month, time is of the essence or that the lessee will be in delay if he falls to pay
within the agreed period without need of demand. In this case he can judicially eject
the tenant on the ground of lack of payment of the price stipulated after a demand to
vacate, (Article 1673(2), New Civil Code),
c) No. Resolution of a contract will not be permitted for a slight or casual
breach, but only for such substantial and fundamental breach as would defeat the
very object of the parties in making the agreement. (Zepeda v. CA, 216 SCRA
293], The delay of ten (10)) days is not such a substantial and fundamental breach
to warrant the resolution of the contract of lease specially so when the delay was
due to the heavy paperwork in processing the checks.
2) a) No. Sublease is different from assignment of lease. Sublease, not being
prohibited by the contract of lease is therefore allowed and cannot be invoked as a
ground to cancel the lease,
b) No, the lessor cannot have the lease cancelled for alleged violation of the
provision against assignment. The lessee did not assign the lease, or any portion
thereof, to the subsidiaries. It merely subleased some floors to its subsidiaries. Since
the problem does not state that the contract of lease contains a prohibition against
sublease, the sublease is lawful, the rule being that in the absence of an express
prohibition a lessee may sublet the thing leased, in whole or in part, without
prejudice to his/its responsibility to the lessor for the performance of the contract.
Page 335 of 391
10; Common carriers
1988 No 1;
(b) Mojar, a passenger in a bus operated by Times Transit Co., suffered
serious physical injuries as a result of a vehicular accident. An information was filed
against Ailes, driver of the bus, for serious physical injuries through reckless
imprudence, Ailes was, however, acquitted on the merits of the case because,
according to the judgment of acquittal, he was not negligent. Subsequently, Moja's
instituted an action against Times Transit Co,, to recover damages. Will the action
prosper? Give your reasons.
Answer:
(b) If Mojar can prove the negligence of Ailes by preponderance of
evidence, the action will prosper. He can still recover damages from the operator of
the bus even if the driver had been acquitted in the criminal action, because it is
clear that the action to recover is based on culpa contractual and not on the act or
omission complained of as a felony (Bernaldez vs. Bohol Trans. Co. 7 SGRA 276).
According to article 31 of the Civil Code, when the civil action is based on an
obligation not arising from the act or omission complained of as a felony, such civil
action may proceed independently of the criminal proceedings and regardless of the
result of the latter.
10; Common carriers; death of passengers
1975 No. XVI
A taxicab passenger was deliberately killed by the driver. Is the operator of
the taxicab civilly liable? Explain.
Answer:
Yes, the taxicab operator is civilly liable on the basis of breach of the contract
of carriage. Article 1769 of the Civil Code states that common carriers are liable for
the death of or injuries to passengers through the negligence or willful acts of the
former's employees, although such employees may have acted beyond the scope of
their authority or in violation of the orders of the common carriers. This liability does
not cease upon proof that the common carrier exercised all the diligence of a good
father of a family in the selection and supervision of their employees.
In other words, the liability of the employer is not based on delict or quasi-
delict. The liability of the common carrier is primary and cannot be eliminated or
limited by stipulation (Art. 1760). [Maranan v. Perez, 20 SCRA 412]
10; Common carriers; defenses and liability of employers
1986 No. 15:
Sumakay, a passenger on a bus owned and operated by Kanlungan Bus
Company suffered serious injuries when the vehicle went out of control and rammed
an electric post. Tsuper, the bus driver, was, at the time of the accident, doing 100
kilometers per hour in a school zone and hit the post because he was trying to avoid
hitting school children crossing the street.
In a suit for damages against the bus company for the driver's gross
negligence, Kanlungan interposed the defenses that all its drivers were under strict
injunction to observe speed limits in their particular routes and that in any event, the
driver should first have been sued, held liable, and found insolvent before
Kanlungan could be proceeded against.
Are the bus company's defenses tenable? Explain. Answer:
Page 336 of 391
I beg to qualify my answer. As far as the first defense (that the drivers of the
bus company were under strict injunction to observe speed limits "in their particular
routes) is concerned, it is submitted that said defense is untenable. This is clearly
stated by the Civil Code itself. In this jurisdiction, we have never adhered to the
principle of respondent superior. We adhere instead to the principle that there is
always an implied duty on the part of a common carrier to carry a passenger safely
to his place of destination.
Anent the second defense, if the civil action for damages is impliedly
instituted in the criminal action itself, then, the defense is tenable. Under the
principle of subsidiary liability of employers, it is essential that there must be a
finding that the driver is insolvent before the bus company can be proceeded
against. However, if the civil action for damages is separated from the criminal
action, then, the defense is untenable. Under the Civil Code, the liability of the bus
company is always directed and primary.
(Note: The above answer is based upon Art. 1759 of the Civil Code, upon
Arts. 100 to 103 of the Revised Penal Code, and upon Maranan vs. Perez, 20 SCRA
412.)
Answer - The bus company, if sued based on culpa contractual, cannot raise
the defense of diligence in the supervision of its employees. The mere fact that there
was a breach of the contract of carriage makes the company liable for damages.
If the bus company was sued based on culpa aquiliana, it cannot raise the
defense that the driver should have been first sued, held liable, and found insolvent
before it could be proceeded against because the company is a joint tort-feasor. The
company, therefore, has a primary liability, because of culpa aquiliana and not a
subsidiary one.
Subsidiary liability on the part of the bus company only results if there is an
action based on culpa criminal against the driver. If the latter is held liable but is
insolvent, then the bus company may raise the defense that is only subsidiarily liable
for the damages.
Answer - Liability of the Bus Company is contractual. Due supervision of
driver by the Bus Company, as well as violations of instructions, is not a defense.
(Art. 1759 Civil Code of the Philippines) Liability of the Bus Company is direct and
primary hence the Bus driver need not be impleaded first.
Answer The cause of action of the passenger against the bus company is
based on the contract of transportation. Here such defenses are not available to the
bus company as it is bound to exercise extraordinary diligence in the transport of its
passengers. The driver is not a party to the contract of transportation and need not
be joined in the same action. The driver may be criminally charged.
10; Common carriers; extraordinary diligence
2000 No XX.
b) Despite a warning from the police that an attempt to hijack a PAL plane
will be made in the following week, the airline did not take extra precautions, such as
frisking of passengers, for fear of being accused of violating human rights. Two days
later, an armed hijacker did attempt to hijack a PAL flight to Cebu. Although he was
subdued by the other passengers, he managed to fire a shot which hit and killed a
female passenger. The victim's parents sued the airline for breach of contract, and
the airline raised the defense of force majeure. Is the airline liable or not? (2%)
SUGGESTED ANSWER:
The airline is liable. In case of death of a passenger, common carriers are
presumed to have been at fault or to have acted negligently, unless they prove that
Page 337 of 391
they observed extraordinary diligence (Article 1756, Civil Code). The failure of the
airline to take extra precautions despite a police warning that an attempt to hijack
the plane would be made, was negligence on the part of the airline. Being negligent,
it is liable for the death of the passenger. The defense of force majeure is not
tenable since the shooting incident would not have happened had the airline taken
steps that could have prevented the hijacker from boarding the plane.
ALTERNATIVE ANSWER:
Under Article 1763 of the Civil Code, the common carrier is not required to
observe extraordinary diligence in preventing injury to its passengers on account of
the willful acts or negligence of other passengers or of strangers. The common
carrier, in that case, is required to exercise only the diligence of a good father of a
family; hence, the failure of the airline to take EXTRA precautions in frisking the
passengers and by leaving that matter to the security personnel of the airport, does
not constitute a breach of that duty so as to make the airline liable. Besides, the use
of Irresistible force by the hijackers was farce majeure that could not have been
prevented even by the observance of extraordinary diligence.
10; Common carriers; liability of carriers for loss, damage, destruction of
goods
1983 No. 15
On October 15, 1983, goods were loaded on a vessel owned by a common
carrier for transportation from Manila to Cebu under a bill of lading which provided
that the carrier would not be responsible for loss arising from theft or robbery. The
goods were stolen while the vessel was docked in Manila.
Sued for damages, the common carrier invoked the cited stipulation to avoid
liability for the loss of the cargo. Additionally, the carrier, which is a corporation,
argued that it could not be held liable because it had done all it could to prevent the
loss by exercising the utmost diligence in the selection and supervision of its
employees.
How valid are these defenses?
Answer
The defenses are not valid.
Common carriers cannot escape liability by stipulation in the bill of lading
relieving them for responsibility for the acts of thieves or robbers who do not act with
grave or irresistible threat or force.
Common carriers are similarly forbidden from exempting themselves from
liability for the acts or omissions of its employees by stipulations to that effect in the
bill of lading
10; Common carriers; limitation of liability for loss
1977 No. VIII-b
The bill of lading specified in fine print that the carrier's liability was limited to
P100 00 per box unless a higher value was declared and extra freight paid. No
higher value was declared and no extra freight was paid and the shipment was lost.
Can shipper S collect the true value of the lost "shipment (P2,000) on the ground
that the loss was caused by the lack of care of carrier C?
Answer
I distinguish. If the shipper S read the stipulation specified in fine print in the
bill of lading or was aware thereof, then he cannot collect the true value of the lost
shipment. Such a stipulation is considered by law valid and binding (Arts. 1749,
Page 338 of 391
1750, Civil Code). If he was not aware thereof, then he can collect the true value of
the lost shipment. There can be no presumption that S read the stipulation. It was
written in fine print.
Page 339 of 391
11; Agency
2003 No V
Jo-Ann asked her close friend, Aissa, to buy some groceries for her in the
supermarket. Was there a nominate contract entered into between Jo-Ann and
Aissa? In the affirmative, what was it? Explain. 5%
Suggested Answer:
Yes, there was a nominate contract. On the assumption that Aissa accepted
the request of her close friend Jo-Ann to but some groceries for her in the
supermarket, what they entered into was a nominate contract of Agency. Article
1868 of the New Civil Code provides that by the contract of agency a person binds
himself to render some service or to do something in representation or on behalf of
another, with the consent or authority of the latter.
Alternative Answer:
Yes, they entered into a nominate contract of lease to service in the absence
of a relation of principal and agent between them (Article 1644, New Civil Code).
11; Agency
2004 No. IV
B. CX executed a special power of attorney authorizing DY to secure a loan
from any bank and to mortgage his property covered by the owners certificate of
title. In securing a loan from MBank, DY did not specify that he was acting for CX in
the transaction with said bank.
Is CX liable for the bank loan? Why or why not? Justify your answer. (5%)
11; Agency; agency vs sale
2000 No XVIII
A foreign manufacturer of computers and a Philippine distributor entered into
a contract whereby the distributor agreed to order 1,000 units of the manufacturer's
computers every month and to resell them In the Philippines at the manufacturer's
suggested prices plus 10%. All unsold units at the end of the year shall be bought
back by the manufacturer at the same price they were ordered. The manufacturer
shall hold the distributor free and harmless from any claim for defects in the units. Is
the agreement one for sale or agency? (5%)
SUGGESTED ANSWER:
The contract is one of agency, not sale. The notion of sale is negated by the
following indicia: (1) the price is fixed by the manufacturer with the 10% mark-up
constituting the commission; (2) the manufacturer reacquires the unsold units at
exactly the same price; and (3) warranty for the units was borne by the
manufacturer. The foregoing indicia negate sale because they indicate that
ownership over the units was never intended to transfer to the distributor.
11; Agency; appointment of sub-agent
1999 No XV.
(a) X appoints Y as his agent to sell his products in Cebu City. Can Y appoint
a sub-agent and if he does, what are the effects of such appointment? (5%)
ANSWER:
a. Yes, the agent may appoint a substitute or sub-agent if the principal has
not prohibited him from doing so, but he shall be responsible for the acts of the
substitute:
Page 340 of 391
(1) when he was not given the power to appoint one;
(2) when he was given such power, but without designating the person, and
the person appointed was notoriously incompetent or insolvent.
11; Agency; authority to sell does not include authority to collect
2004 No. VI
B. As an agent, AL was given a guarantee commission, in addition to his
regular commission, after he sold 20 units of refrigerators to a customer, HT Hotel.
The customer, however, failed to pay for the units sold. ALs principal, DRBI,
demanded from AL payment for the customers accountability. AL objected, on the
ground that his job was only to sell and not to collect payment for units bought by the
customer.
Is ALs objection valid? Can DRBI collect from him or not? Reason. (5%)
11; Agency; commission
1978 No. VII-b
A authorized B to sell her property for P20,000 subject to the condition that
the purchaser would assume the mortgage existing in favor of Plaridel Bank and
agreed to pay B a commission of six per cent (6%) on the purchase price plus
whatever over price he may obtain for the property. B found a buyer C who was
willing to buy the property under the terms stipulated by A. When B introduce C to A,
A told B that she was no longer interested in selling the property and a document
was signed canceling the written authority to sell with the agreement of B. One (1)
month later, A sold the same property directly to C for P22,000, A refused to pay B
his commission, contending that when the property was sold to C the authority to
sell of B was already cancelled. B sued to collect his commission. Is B entitled to his
agent's commission? Give reasons for your answer.
Answer
B is entitled to his agent's commission. C, the buyer of B, was willing to buy
the property under the terms stipulated by A. Despite this, A told B that she was no
longer interested in selling the property. As a result, B's authority was cancelled. And
yet, all of a sudden, one month later, A sold the property directly to C for P22,000. It
is clear that there was bad faith on the part of A. This act of bad faith cannot serve
as a basis for him to evade payment of the commission of B.
(NOTE: The above answer is based on Infante vs. Cunanan, 49 Off. Gaz.
3320.)
11; Agency; coupled with an interest
2001 No XV
Richard sold a large parcel of land in Cebu to Leo for P100 million payable in
annual installments over a period of ten years, but title will remain with Richard until
the purchase price is fully paid. To enable Leo to pay the price, Richard gave him a
power-of-attorney authorizing him to subdivide the land, sell the Individual lots, and
deliver the proceeds to Richard, to be applied to the purchase price. Five years later,
Richard revoked the power of attorney and took over the sale of the subdivision lots
himself. Is the revocation valid or not? Why? (5%)
SUGGESTED ANSWER
The revocation is not valid. The power of attorney given to the buyer is
irrevocable because it is coupled with an interest: the agency is the means of
fulfilling the obligation of the buyer to pay the price of the land (Article 1927, CC). In
Page 341 of 391
other words, a bilateral contract (contract to buy and sell the land) is dependent on
the agency.
11; Agency; coupled with an interest
1980 No. VIII
(a) "AA" had an option to purchase a vessel. He entered into a contract
with "BB" wherein he assigned his option to "BB" under the condition that "BB"
would appoint him as agent of the vessel for five years. "BB" purchased the vessel
and appointed "AA" as agent in accordance with the contract. After three years of
operation "BB" revoked the appointment of "AA" as agent for loss of confidence.
"AA" sued "BB" for damages.
Would you hold "BB" liable for damages?
Answer
(a) "BB" should be held for damages. True, according to the Civil Code, the
principal may revoke the agency at will. But there are exceptions. These
exceptions are sometimes denominated as agency coupled with an interest.
One of them is when the agency is the means of fulfilling an obligation already
contracted. It is obvious that the agency is the means of fulfilling an obligation
already contracted in favor of "AA". "BB" has clearly breached his contract
or undertaking by revoking the agency before the expiration of the term or
period of five years.
(NOTE: The above answer is based upon Arts. 1927, 1930, Civil Code.)
11; Agency; coupled with an interest
1979 No. XVI
DT borrowed P50,000.00 from a bank and to secure the payment thereof,
signed a Deed of Real Estate Mortgage in favor of the bank in the usual printed form
wherein it is provided among others that "for the purpose of extra-judicial
foreclosure, the mortgagor hereby appoints the mortgagee his attorney-in-fact to sell
the property mortgaged under Act 3135, as amended, to sign all documents and
perform any act requisite and necessary to accomplish said purpose." Upon failure
of DT to pay the loan, the bank foreclosed and bought the property at the
foreclosure sale. During the one year period of redemption DT died and the property
was not redeemed despite the lapse of one year. The bank, despite its actual
knowledge, of DT's death, consolidated its title by executing the affidavit of
consolidation and Deed of Sale of the land in its favor as empowered in the Deed of
Real Estate Mortgage. After the bank had consolidated its title the heirs of DT asked
the bank to allow them to redeem the property by paying only the P50,000.00 plus
accrued interest and expense of foreclosure, contending that the sale in favor of the
bank was invalid due to the prior death of DT which therefore revoked the power of
attorney inserted in the Deed of Mortgage but the bank demanded payment of
P200,000.00, the then fair market value of the property. Can the bank be compelled
to accept the tender of redemption by the heirs of DT? Why?
Answer
The bank cannot be compelled to accept the tender of redemption by the
heirs of DT. True, agency is extinguished by the death of the principal, but there are
two well-known exceptions. The first exception is where the agency is coupled with
an interest and the second is where the agent, unaware of the death of his principal,
enters into a contract in behalf of his principal with a third person who is also
unaware of the death of the principal (Arts. 1930, 1931, Civil Code) The instant case
falls squarely within the purview of agency coupled with an interest. According to the
Civil Code, the agency shall remain in full force and effect even after the death of the
Page 342 of 391
principal, if it has been constituted in the common interest of the latter and of the
agent, or in the interest of a third person who has accepted the stipulation in his
favor. Hence, despite the death of DT, the power granted by him to the bank to sell
the property mortgaged and to sign all documents and perform any act requisite and
necessary to accomplish the extra-judicial foreclosure in case he is unable to pay
the loan is still of full force and effect. The foreclosure, therefore, and the
consolidation by the bank of its title over the mortgaged property are perfectly valid.
11; Agency; general vs special agency
1992 No 8:
A as principal appointed B is his agent granting him general and unlimited
management over A's properties, stating that A withholds no power from B and that
the agent may execute such acts as he may consider appropriate.
Accordingly, B leased A's parcel of land in Manila to C for four (4) years at
P60,000.00 per year, payable annually in advance.
B leased another parcel of land of A in Caloocan City to D without a fixed
term at P3,000.00 per month payable monthly.
B sold to E a third parcel of land belonging to A located in Quezon City for
three (3) times the price that was listed in the inventory by A to B.
All those contracts were executed by B while A was confined due to illness in
the Makati Medical Center.
Rule on the validity and binding effect of each of the above contracts upon A
the principal. Explain your answers,
Answer:
The agency couched in general terms comprised only acts of administration
(Art. 1877, Civil Code). The lease contract on the Manila parcel is not valid, not
enforceable and not binding upon A. For B to lease the property to C, for more than
one (1) year, A must provide B with a special power of attorney (Art. 1878. Civil
Code).
The lease of the Caloocan City property to D is valid and binding upon A.
Since the lease is without a fixed term, it is understood to be from month to month,
since the rental is payable monthly (Art. 1687, Civil Code).
The sale of the Quezon City parcel to E is not valid and not binding upon A. B
needed a special power of attorney to validly sell the land (Arts. 1877 and 1878, Civil
Code). The sale of the land at a very good price does not cure the defect of the
contract arising from lack of authority
11; Agency; liability of an agent
1975 No. XIV
A borrowed from B the sum of P3,000.00. Three days after A in a letter
authorized the Philippine National Bank to pay his debt to B out of whatever crop
loan might be granted to him by said Bank. On the same day, the Bank agreed but
the Bank paid B only P2,000.00. On the date of the maturity, B sued the Bank and A
for the remaining P 1,000.00. Is the Bank liable to B? Explain.
Answer
No, the Bank is not liable to B. The letter of A to PNB is merely an authority
given to PNB to pay B. PNB, therefore, is merely an agent of A, and an agent cannot
be personally liable as long as be acts within the scope of his authority.
Page 343 of 391
Moreover, the Bank did not assume the obligation to pay A's indebtedness to
B, either as co-principal, surety or guarantor. (Hodges v. Rey, 111 Phil. 219)
11; Agency; liability of an agent to render an account
1981 No. 15
"A", an official of a mining company, was appointed by the company as its
buying agent for the acquisition of mining rights in a designated area for operation
by the company. "A" proceeded to enter into contracts with the claim owners. Claim
owner "B", an illiterate, was helped by "A" in locating and perfecting his rights and
for which "A", by contract, obtained a participation in the royalty paid by the
company to the claim owner.
a) The mining company goes to you for advice as to whether it is entitled to
the royalty obtained by "A" from "B". What would your advice be and why?
b) May "B", the claim owner, question the royalty obtained by "A"? On what
grounds? Explain,
Answer
(a) I would advice the mining company to withhold the payment of the part of
the royalty corresponding to "A". This is so because of the explicit mandate of the
Civil Code. According to the law: Every agent is bound to render an account of his
transactions and to deliver to the principal whatever he may have received by virtue
of the agency, even though it may not be owing to the principal. It is crystal dear that
the act of "A", agent of the mining company, falls squarely within the purview or
coverage of this rule.
(Note: The above answer is based on Art. 1891 of the Civil Code.)
(b) "B", the claim owner, may question the royalty obtained by "A" on the
ground that it is "not owing to the principal." It must be observed that the obligation
of the agent to deliver to his principal anything which he has received by virtue of the
agency is followed by the phrase "even though it may not be owing to the principal"
This means that the action for recovery by "B" on the ground of undue payment
would be directed against the mining company and not against the agent.
(Note: The above answer is based on Art. 1891 of the Civil Code and on
Manresa's opinion Vol. 11, p. 512,}
11; Agency; powers of the agent
1994 No. 18:
Prime Realty Corporation appointed Nestor the exclusive agent in the sale of
lots of its newly developed subdivision. Prime Realty told Nestor that he could not
collect or receive payments from the buyers. Nestor was able to sell ten lots to
Jesus and to collect the down payments for said lots. He did not turn over the
collections to Prime Realty. Who shall bear the loss for Nestor's defalcation. Prime
Realty or Jesus?
Alternative Answer:
a) The general rule is that a person dealing with an agent must inquire into
the authority of that agent. In the present case, if Jesus did not inquire into that
authority, he is liable for the loss due to Nestor's defalcation unless Article 1900,
Civil Code governs, in which case the developer corporation bears the loss.
Art. 1900 Civil Code provides: "So far as third persons are concerned, an act
is deemed to have been performed within the scope of the agent's authority, if such
act is within the terms of the power of attorney, as written, even if the agent has in
Page 344 of 391
fact exceeded the limits of his authority according to an understanding between the
principal and the agent.
However, if Jesus made due inquiry and he was not informed by the principal
Prime Realty of the limits of Nestor's authority. Prime Realty shall bear the loss.
b) Considering that Prime Realty Corporation only "told" Nestor that he
could not receive or collect payments, it appears that the limitation does not appear
in his written authority or power of attorney. In this case, insofar as Jesus, who is a
third person. Is concerned, Nestor's acts of collecting payments is deemed to have
been performed within the scope of his authority {Article 1900. Civil Code).
Hence, the principal is liable.
However, if Jesus was aware of the limitation of Nestor's power as an agent,
and Prime Realty Corporation does not ratify the sale contract, then Jesus shall be
liable (Article 1898. Civil Code).
11; Agency; termination; death or principal: double sales
1988 No. 13:
(b) In 1950, A executed a power of attorney authorizing B to sell a parcel of
land consisting of more than 14 hectares. A died in 1954. In 1956, his four children
sold more than 12 hectares of the land to C. In 1957, B sold 8 hectares of the same
land to D, It appears that C did not register the sale executed by the children. D, who
was not aware of the previous sale, registered the sale executed by B, whose
authority to sell was annotated at the back of the Original Certificate of Title.
(1) What was the effect of the death of A upon B's authority to sell the land?
(2) Assuming that B still had the authority to sell the landwho has a better
right over the said land, C or D?
Answer:
(b) (1) While the death of the principal in 1954 ended the authority of the
agent to sell the land, it has not been shown that he was aware of his principal's
demise. Hence, the act of such agent is valid and shall be fully effective with respect
to third persons which may have contracted with him in good faith in conformity with
Art. 1931 of the Civil Code. (Buason vs. Panuyas, 105 Phil. 795, Herrera vs. Luy,
110 Phil. 1020.)
(2) D has better right since he registered first in good faith.
Alternative Answer to: No, 13 (b):
(b) (1) The agency is terminated upon the death of either the principal or
agent. Exceptionally, a transaction entered into by the agent with a third person
where both had acted in good faith is valid. Article 1930 of the Civil Code provides
that:
"The agency shall remain in full force and effect even after the death of the
principal, if it has been constituted in the common interest of the latter and of the
agent, or in the interest of a third person who has accepted the stipulation in his
favor.'* and Article 1931 provides that;
"Anything done by the agent, without knowledge of the death of the principal
or of any other cause which extinguishes the agency, is valid and shall be fully
effective with respect to third persons who may have contracted with him in good
faith,"
11; Agency; termination; effect of death of agent
1997 No. 17:
Page 345 of 391
Stating briefly the thesis to support your answer to each of the following
cases, will the death - (c) of an agent end an agency?
Answer:
(c) Yes. The death of an agent extinguishes the agency, by express
provision of par. 3, Art 1919 of the Civil Code.
Page 346 of 391
11; Partnership
1993 No. 10;
A, B and C formed a partnership for the purpose of contracting with the
Government in the construction of one of its bridges. On June 30, 1992, after
completion of the project, the bridge was turned over by the partners to the
Government. On August 30, 1992. D, a supplier of materials used in the project
sued A for collection of the indebtedness to him. A moved to dismiss the complaint
against him on the ground that it was the ABC partnership that is liable for the debt.
D replied that ABC partnership was dissolved upon completion of the project for
which purpose the partnership was formed.
Will you dismiss the complaint against B If you were the Judge?
Answer:
As Judge, I would not dismiss the complaint against A. because A is still
liable as a general partner for his pro rata share of 1/3 (Art. 1816, C. C.J. Dissolution
of a partnership caused by the termination of the particular undertaking specified in
the agreement does not extinguish obligations, which must be liquidated during the
"winding up" of the partnership affairs (Articles 1829 and 1830. par. 1-a, Civil Code).
11; Partnership; conveyance of a partners share dissolution
1998 No XVII.
Dielle, Karlo and Una are general partners in a merchandising firm. Having
contributed equal amounts to the capital, they also agree on equal distribution of
whatever net profit is realized per fiscal period. After two years of operation,
however, Una conveys her whole interest in the partnership to Justine, without the
knowledge and consent of Dielle and Karlo.
1. Is the partnership dissolved? 12%]
2. What are the rights of Justine, If any. should she desire to participate in
the management of the partnership and in the distribution of a net profit of
P360.000.00 which was realized after her purchase of Una's interest? [3%]
Answer:
1. No, a conveyance by a partner of his whole interest in a partnership
does not of itself dissolve the partnership in the absence of an agreement. (Art.
1813. Civil Code)
Answer;
2. Justine cannot interfere or participate in the management or
administration of the partnership business or affairs. She may, however, receive the
net profits to which Una would have otherwise been entitled. In this case. P120.0OO
(Art. 1813, Civil Code)
11; Partnership; dissolution
1995 No 8:
Pauline, Patricia and Priscilla formed a business partnership for the purpose
of engaging in neon advertising for a term of five (5) years. Pauline subsequently
assigned to Philip her interest in the partnership. When Patricia and Priscilla learned
of the assignment, they decided to dissolve the partnership before the expiration of
its term as they had an unproductive business relationship with Philip in the past. On
the other hand, unaware of the move of Patricia and Priscilla but sensing their
negative reaction to his acquisition of Pauline's interest, Philip simultaneously
petitioned for the dissolution of the partnership.
Page 347 of 391
1. Is the dissolution done by Patricia and Priscilla without the consent of
Pauline or Philip valid? Explain.
2. Does Philip have any right to petition for the dissolution of the partnership
before the expiration of Its specified term? Explain.
Answer;
1, Under Art. 1830 (1) (c) of the NCC, the dissolution by Patricia and Priscilla
Is valid and did not violate the contract of partnership even though Pauline and
Philip did not consent thereto. The consent of Pauline is not necessary because she
had already assigned her Interest to Philip. The consent of Philip is not also
necessary because the assignment to him of Pauline's Interest did not make him a
partner, under Art, 1813 of the NCC.
Alternative Answer:
Interpreting Art. 1830 (1) (c) to mean that if one of the partners had assigned
his interest on the partnership to another the remaining partners may not dissolve
the partnership, the dissolution by Patricia and Priscilla without the consent of
Pauline or Philip Is not valid.
2. No, Philip has no right to petition for dissolution because he does not
have the standing of a partner (Art. 1813 NCC).
11; Partnership; effect of death of partner
1997 No. 17:
Stating briefly the thesis to support your answer to each of the following
cases, will the death - b) of a partner terminate the partnership?
Answer:
b) Yes. The death of a partner will terminate the partnership, by express
provision of par. 5, Art. 1830 of the Civil Code.
11; Partnership; obligations of a partner
2001 No XIII
Joe and Rudy formed a partnership to operate a car repair shop in Quezon
City. Joe provided the capital while Rudy contributed his labor and industry. On one
side of their shop, Joe opened and operated a coffee shop, while on the other side,
Rudy put up a car accessories store. May they engage in such separate
businesses? Why? [5%]
SUGGESTED ANSWER
Joe, the capitalist partner, may engage in the restaurant business because it
is not the same kind of business the partnership is engaged in. On the other hand,
Rudy may not engage in any other business unless their partnership expressly
permits him to do so because as an industrial partner he has to devote his full time
to the business of the partnership [Art. 1789, CC).
11; Partnership; obligations of a partner
1992 No 12:
W. X. Y and Z organized a general partnership with W and X as industrial
partners and Y and Z as capitalist partners. Y contributed P50,000.00 and Z
contributed P20,000.00 to the common fund. By a unanimous vote of the partners,
W and X were appointed managing partners, without any specification of their
respective powers and duties.
Page 348 of 391
A applied for the position of Secretary and B applied for the position of
Accountant of the partnership.
The hiring of A was decided upon by W and X, but was opposed by Y and Z.
The hiring of B was decided upon by W and Z, but was opposed by X and Y.
Who of the applicants should be hired by the partnership? Explain and give
your reasons.
Answer:
A should be hired as Secretary. The decision for the hiring of A prevails
because it is an act of administration which can be performed by the duly appointed
managing partners, W and X.
B cannot be hired, because in case of a tie in the decision of the managing
partners, the deadlock must be decided by the partners owning the controlling
interest. In this case, the opposition of X and Y prevails because Y owns the
controlling Interest (Art. 1801, Civil Code).
11; Partnership; partner by estoppel
1987 No. 5:
Tomas, Rene and Jose entered into a partnership under the firm name
"Manila Lumber." Subsequently, upon mutual agreement, Tomas withdrew from the
partnership and the partnership was dissolved. However, the remaining partners,
Rene and Jose, did not terminate the business of "Manila Lumber." Instead of
winding up the business of the partnership and liquidating its assets, Rene and Jose
continued the business in the name of "Manila Lumber" apparently without objection
from Tomas. The withdrawal of Tomas from the partnership was not published in the
newspapers.
Could Tomas be held liable for any obligation or indebtedness Rene and
Jose might incur while doing business in the name of "Manila Lumber" after his
withdrawal from the partnership? Explain.
Answer:
Yes. Tomas can be held liable under the doctrine of estoppel. But as regards
the parties among themselves, only Rene and Jose are liable. Tomas cannot be
held liable since there was no proper notification or publication.
In the event that Tomas is made to pay the liability to third person, he has the
right to seek reimbursement from Rene and Jose (Articles 1837 to 1840; Goquiolay
vs. Sycip, 9 SCRA 663).
11; Partnership; partner vs lender (debtor)
1989 No. 9:
(2) "X" used his savings from his salaries amounting to a little more than
P2,000 as capital in establishing a restaurant. "Y" gave the amount of P4,000 to "X"
as "financial assistance" with the understanding that "Y" would be entitled to 22% of
the annual profits derived from the operation of the restaurant. After the lapse of 22
years, "Y" filed a case demanding his share in the said profits, "X" denied that there
was a partnership and raised the issue of prescription as "Y" did not assert his rights
anytime within ten (10) years from the start of the operation of the restaurant. Is "Y"
a partner of "X" -in the business? Why? What is the nature of the right to demand
one's share in the profits of a partnership? Does this right prescribe?
Answer:
Page 349 of 391
Yes, because there is an agreement to contribute to a common fund and an
intent to divide profits. It is founded upon an express trust. It is imprescriptible unless
repudiated,
Alternative Answer:
No, "Y" is not a partner because the amount is extended in the form of a
financial assistance and therefore it is a loan, and the mere sharing of profits does
not establish a partnership. The right is founded upon a contract of loan whereby the
borrower is bound to pay principal and interest like all ordinary obligations. Yes, his
right prescribes in six or ten years depending upon whether the contract is oral or
written.
11; Partnership; revocation of managers power; liability of an industrial
partner
1981 No. 14
"A", "B" and "C" formed a partnership under the following terms and
conditions:
(a) Participation: "A" - 40%; "B" - 40%;
"C" - 20%.
(b) "A" and "B" would supply the entire capital. "C" would contribute his
management expertise and be manager for the first five years without
compensation.
(c) "C" shall not be liable for losses. The partnership became bankrupt.
1. Could "A" alone, opposed by "B" and "C", have "C" removed as manager?
Explain.
2. Could "C" be personally held liable for debts of the partnership not
satisfied with the assets of the partnership. Amplify.
Answer
(a) "A", alone, opposed by "B" and "C", cannot have "C" removed as manager
of the partnership. According to the Civil Code, the vote of the partners representing
the controlling interest shall be necessary for such revocation of power. Under the
partnership agreement, it is crystal clear that the vote of "A" does not represent the
controlling interest.
(Note: The above answer is based on Art. 1800 of the Civil Code. The
Committee respectfully recommends that if the bar candidate attacks the problem
from the point of view of the fact that "C" was appointed manager in the articles of
partnership and arrives at the same conclusion, it should be considered a correct
answer.)
(b) Yes, "C" eon be held personally, although jointly, liable for debts of the
partnership not satisfied with the assets of the partnership.
Under our partnership law, as among themselves, the industrial partner is
always excluded from any participation in the losses in the absence of an agreement
to the contrary Hence, the agreement that "C", the industrial partner, shall not be
liable for losses is valid. It merely affirms the law. The rule that a stipulation which
excludes one or more partners in the profits or losses is void, is applicable only to
capitalist partners, not to industrial partners. However, as far as third persons are
concerned, the rule is different. An industrial partner can be held personally liable.
Of course, this is without prejudice to his right to hold his co-partners proportionately
liable for what he paid to partnership creditors.
Page 350 of 391
Thus, in the instant case, the liability of "A"', "B" and "C" is joint (pro rata} and
subsidiary. The facts merely state that their participation is: "A 40%; "B" 40%;
"C" -20%. Therefore, since by agreement "C" is excluded from any participation in
the losses, the agreement that "C's" participation is 20% applies only to his
participation in the profits. In the case of "A" and "B", the agreement applies to both
profits and losses. Despite the exclusion of "C" in the losses, such agreement is not
applicable insofar as partnership creditors are concerned. Consequently, the liability
of the three partners for partnership debts shall also be: 40% for "A", 40% for "B";
and 20% for "C". Hence, "C" can now be compelled to pay 20% of the partnership
debts. After payment, he can then proceed against his co-partners "A" and "B" for
reimbursement of the amount paid by him.
(Note: The above answer is based on Art. 1816 of the Civil Code in relation to
Arts. 1797 and 1799.)
11; Partnership; who can form partnerships; spouses; corporations
1994 No. 5:
1) Can a husband and wife form a limited partnership to engage in real
estate business, with the wife being a limited partner?
2) Can two corporations organize a general partnership under the Civil Code
of the Philippines?
3) Can a corporation and an individual form a general partnership?
Alternative Answers;
1) a) Yes. The Civil Code prohibits a husband and wife from constituting a
universal partnership. Since a limited partnership is not a universal partnership, a
husband and wife may validly form one.
b) Yes. While spouses cannot enter into a universal partnership, they can
enter into a limited partnership or be members thereof (CIR u. Suter, etal. 27 SCRA
152).
2) a) No, A corporation Is managed by its board of directors. If the
corporation were to become a partner, co-partners would have the power to make
the corporation party to transactions in an irregular manner since the partners are
not agents subject to the control of the Board of Directors, But a corporation may
enter into a joint venture with another corporation as long as the nature of the
venture is in line with the business authorized by its charter. (Tuason & Co., Inc. v.
Bolano, 95 Phil. 106).
b) As a general rule a corporation may not form a general partnership with
another corporation or an individual because a corporation may not be bound by
persons who are neither directors nor officers of the corporation.
However, a corporation may form a general partnership with another
corporation or an individual provided the following conditions are met:
1) The Articles of Incorporation of the corporation expressly allows the
corporation to enter into partnerships;
2) The Articles of Partnership must provide that all partners will manage the
partnership, and they shall be jointly and severally liable; and
3) In case of a foreign corporation, it must be licensed to do business in the
Philippines.
c) No. A corporation may not be a general partner because the principle of
mutual agency in general partnership allowing the other general partner to bind the
Page 351 of 391
corporation will violate the corporation law principle that only the board of directors
may bind the corporation.
3) No, for the same reasons given in the Answer to Number 2 above.
Page 352 of 391
12; Commodatum
1993 No. 18:
A, upon request, loaned his passenger Jeepney to B to enable B to bring his
sick wife from Paniqui. Tarlac to the Philippine General Hospital in Manila for
treatment. On the way back to Paniqul, after leaving his wife at the hospital, people
stopped the passenger Jeepney. B stopped for them and allowed them to ride on
board, accepting payment from them just as in the case of ordinary passenger
Jeepneys plying their route. As B was crossing Bamban, there was an onrush of
Lahar from Mt Pinatubo, the Jeep that was loaned to him was wrecked.
1) What do you call the contract that was entered into by A and B with
respect to the passenger Jeepney that was loaned by A to B to transport the latter's
sick wife to Manila?
2) Is B obliged to pay A for the use of the passenger jeepney?
3) Is B liable to A for the loss of the Jeepney?
Answer:
1) The contract is called "commodatum". [Art. 1933. Civil Code)
2) No, B is not obliged to pay A for the use of the passenger Jeepney
because commodatum Is essentially gratuitous. (Art. 1933. Civil Code]
3) Yes, because B devoted the thing to a purpose different from that for
which it has been loaned (Art. 1942, par. 2, Civil Code)
Alternative Answer:
No, because an obligation which consists in the delivery of a determinate
thing shall be extinguished if it should be lost or destroyed without the fault of the
debtor, and before he has incurred in delay. (Art. 1262. Civil Code)
12; Commodatum
1977 No. XVI-a
Differentiate mutuum from commodatum. Answer
In mutuum, the object is money or any consumable (fungible) thing; in
commodatum, the object is, as a general rule, a non-consumable (non-fungible)
thing.
The former may or may not be gratuitous; the latter is essentially gratuitous.
The purpose of the former is consumption; the purpose of the latter is use.
In the former, ownership passes to the debtor; in the latter, ownership
remains with the bailor.
In the former, the debtor must pay or return an equal amount of the same
kind or quality; in the latter, the bailee must return the specific thing loaned.
12; Commodatum vs usufruct
1998 No XVIII.
1. Distinguish usufruct from commodatum and state whether these may be
constituted over consumable goods. [2%]
Answer:
1. Usufruct is a right given to a person (usufructuary) to enjoy the property of
another with the obligation of preserving its form and substance. (Art. 562. Civil
Code)
Page 353 of 391
On the other hand, commodatum is a contract by which one of the parties
(bailor) delivers to another (bailee) something not consumable so that the latter may
use it for a certain time and return it.
In usufruct the usufructuary gets the right to the use and to the fruits of the
same, while in commodatum, the bailee only acquires the use of the thing loaned
but not its fruits.
Usufruct may be constituted on the whole or a part of the fruits of the thing.
(Art. 564. Civil Code). It may even be constituted over consumables like money
(Alunan v. Veloso, 52 Phil. 545). On the other hand, in commodatum, consumable
goods may be subject thereof only when the purpose of the contract is not the
consumption of the object, as when it is merely for exhibition. (Art. 1936, Civil Code)
Another Answer:
1. There are several points of distinction between usufruct and
commodatum. Usufruct is constituted by law, by contract, by testamentary
succession, or by prescription (Art. 1933. Civil Code). Usufruct creates a real right to
the fruits of another's property, while commodatum creates only a purely personal
right to use another's property, and requires a stipulation to enable the bailee to
"make use" of the fruits (Arts. 1939& 1940, Civil Code). Usufruct maybe onerous
while commodatum is always or essentially gratuitous (Arts. 1933 & 1935, Civil
Code). The contract constituting usufruct is consensual, while commodatum is a
real contract (perfected only by delivery of the subject matter thereof). However,
both involve the enjoyment by a person of the property of another, differing only as
to the extent and scope of such enjoyment [jus fruendi in one and Jus utendi in the
other); both may have as subject matter either an immovable or a movable; and,
both maybe constituted over consumable goods (Arts. 574 & 1936, Civil Code).
A consumable thing may be the subject-matter of an abnormal usufruct but in
a normal usufruct, the subject-matter may be used only for exhibition. A
commodatum of a consumable thing may be only for the purpose of exhibiting, not
consuming it.
12; Commodatum; liabilties of a bailee
1983 No. 16
A borrowed B's truck. During a fire which broke out in A's garage, he had time
to save only one vehicle and he saved his car instead of the truck. Is he liable for the
loss of B's truck? Why?
Answer
Yes, The bailee in a commodatum is liable for the loss of the thing loaned
even if thru a fortuitous event where, being able to save it or his own thing, he chose
to save the latter.
12; Mutuum vs commodatum
2004 No. II
A. Distinguish briefly but clearly between: 1. Mutuum and commodatum.
12; Mutuum; interest; usury
1975 No. XVII
A partnership borrowed P20,000 from A at clearly usurious interest. Can the
creditor recover anything from the debtor? Explain.
Answer
Page 354 of 391
Yes, the creditor can recover the principal together with legal interest thereon
from the date of demand (Art. 2209), and legal interest on the interests paid in
excess of the lawful rate from the date of payment (Art. 1413),
The usurious interest, that is to say, the whole usurious interest can not be
recovered, because of Article 1413 of the Civil Code and Section 6 of the Usury
Law. However, the illegality of the stipulation concerning the usurious interests does
not affect the creditor's right to recover the principal, inasmuch as a contract of loan
with usurious interest is a divisible contract. The illegal terms can be separated from
the legal ones (Art. 1420). [Angel Jose v. Chelda, 23 SCR A 119; Briones v,
Cammayo, 41 SCRA 404}
12; Mutuum; interests
2001 No IX
Samuel borrowed P300,000.00 housing loan from the bank at 18% per
annum Interest. However, the promissory note contained a proviso that the bank
"reserves the right to increase interest within the limits allowed by law," By virtue of
such proviso, over the objections of Samuel, the bank increased the interest rate
periodically until it reached 48% per annum. Finally, Samuel filed an action
questioning the right of the bank to increase the interest rate up to 48%. The bank
raised the defense that the Central Bank of the Philippines had already suspended
the Usury Law. Will the action prosper or not? Why? (5%)
SUGGESTED ANSWER:
The action will prosper. While it is true that the interest ceilings set by the
Usury Law are no longer in force, it has been held that PD No. 1684 and CB Circular
No. 905 merely allow contracting parties to stipulate freely on any adjustment in the
interest rate on a loan or forbearance of money but do not authorize a unilateral
increase of the interest rate by one party without the other's consent (PNB v. CA,
238 SCRA 2O [1994]]). To say otherwise will violate the principle of mutuality of
contracts under Article 1308 of she Civil Code. To be valid, therefore, any change of
interest must be mutually agreed upon by the parties (Dizon v, Magsaysay, 57
SCRA 25O [1974]). In the present problem, the debtor not having given his consent
to the increase in interest, the increase is void.
12; Mutuum; interests
2002 No XV.
Carlos sues Dino for (a) collection on a promissory note for a loan, with no
agreement on interest, on which Dino defaulted, and (b) damages caused by Dino
on his (Carlos|) priceless Michaelangelo painting on which Dino liable on the
promissory note and awards damages to Carlos for the damaged painting, with
interests for both awards. What rates of interest may the court impose with respect
to both awards? Explain. (5%)
SUGGESTED ANSWER:
With respect to the collection of money or promissory note, it being a
forbearance of money, the legal rate of interest for having defaulted on the payment
of 12% will apply. With respect to the damages to the painting, it is 6% from the time
of the final demand up to the time of finality of judgment until judgment credit is fully
paid. The court considers the latter as a forbearance of money. (Eastern Shipping
Lines, Inc. v. CA, 234 SCRA 78 [1994]; Art 2210 and 2211, CC)
12; Mutuum; interests
2004 No. IX
Page 355 of 391
A. The parties in a contract of loan of money agreed that the yearly interest
rate is 12% and it can be increased if there is a law that would authorize the
increase of interest rates. Suppose OB, the lender, would increase by 5% the rate
of interest to be paid by TY, the borrower, without a law authorizing such increase,
would OBs action be just and valid? Why? Has TY a remedy against the
imposition of the rate increase? Explain. (5%)
Page 356 of 391
13; Deposit
1997 No. 18:
In order to secure a bank loan. XYZ Corporation surrendered its deposit
certificate, with a maturity date of 01 September 1997 to the bank. The corporation
defaulted on the due repayment of the loan, prompting the bank to encash the
deposit certificate. XYZ Corporation questioned the above action taken by the bank
as being a case of pactum commissorium. The bank disagrees.
What is your opinion? Answer:
We submit that there is no pactum commissorium here. Deposits of money in
banks and similar institutions are governed by the provisions on simple loans (Art.
1980. Civil Code). The relationship between the depositor and a bank is one of
creditor and debtor. Basically this is a matter of compensation as all the elements of
compensation are present in this case (BPI vs. CA, 232 SCRA 302).
Additional Answer:
Where the security for the debt is also money deposited in a bank, it is not
illegal for the creditor to encash the time deposit certificates to pay the debtor's
overdue obligation. (Chu us. CA, et al., G.R 78519, September 26, 1989).
13; Deposit
1987 No. 11:
Ana rented a safety deposit box at the Alto Bank, paid the rental fee and was
given the key. Ana put her jewelry and gold coins in the box. Days after, three armed
men gained entry into the Alto Bank, opening its vault and several safety deposit
boxes, including Ana's and emptied them of their contents.
Could Ana hold the Alto Bank liable for the toss of the contents of her deposit
box? Explain.
Answer:
No, because under Article 1990 of the Civil Code, if the depository by force
majeure loses the thing and receives money or another thing in its place, he shall
deliver the sum or other thing to be depositor. There being no showing that there
was anything received in place of the things deposited, the Alto Bank is not liable for
the contents of the safety box.
Answer:
The Alto Bank is not liable because the contract is not a deposit but a rental
of the safety deposit box. Hence, the Alto Bank is not liable for the loss of the
contents of the box.
13; Deposit
1992 No 9:
X and Y staged a daring bank robbery in Manila at 10:30 AM. in the morning
of a regular business day, and escaped with their loot of two (2) bags, each bag
containing P50,000,00.
During their flight to elude the police, X and Y entered the nearby locked
house of A, then working in his Quezon City office. From A's house, X and Y stole a
box containing cash totaling P50,OOO.OO which box A had been keeping in deposit
for his friend B.
In their hurry. X and Y left in A's bedroom one (1) of the bags which they had
taken from the bank.
Page 357 of 391
With X and Y now at large and nowhere to be found, the bag containing
P50.000.00 is now claimed by B, by the Mayor of Manila, and by the bank.
B claims that the depository. A, by force majeure had obtained the bag of
money in place of the box of money deposited by B.
The Mayor of Manila, on the other hand, claims that the bag of money should
be deposited with the Office of the Mayor as required of the finder by the provisions
of the Civil Code.
The bank resists the claims of B and the Mayor of Manila.
To whom should A deliver the bag of money? Decide with reasons.
Answer:
B would have no right to claim the money. Article 1990 of the Civil Code is
not applicable. The law refers to another thing received in substitution of the object
deposited and is predicated upon something exchanged.
The Mayor of Manila cannot invoke. Article 719 of the Civil Code which
requires the finder to deposit the thing with the Mayor only when the previous
possessor is unknown.
In this case , a must return the bag of money to the bank as the previous
possessor and known owner (Arts. 719 and 1990. Civil Code.
Page 358 of 391
14; Surety
1975 No. XVIII
A debtor pledged to his surety pieces of jewelry to indemnify the Utter in case
the surety would be obliged to pay the creditor. The surety paid P2,800 00 to the
creditor. To recover the amount, the surety sold at public auction the jewelry but
realized only P500. May the surety recover the deficiency from the debtor? Explain.
Answer
No, the surety is not entitled to recover the deficiency. Article 2115 of the Civil
Code provides that in the foreclosure of a pledge, if the price of the sale is less than
the indebtedness secured by the pledge, the creditor shall not be entitled to recover
the deficiency, notwithstanding any stipulation to the contrary. By electing to sell the
articles pledged, the creditor waived any other remedy, and must abide by the
results of the sale. No deficiency is recoverable, [Manila Surety v. Velayo, 21 SCRA
615]
14; Surety; recovery of deficiency
1997 No. 16:
AB sold to CD a motor vehicle for and in consideration of P120,000.00. to be
paid in twelve monthly equal installments of P10,000,00, each Installment being due
and payable on the 15th day of each month starting January 1997.
To secure the promissory note, CD (a) executed a chattel mortgage on the
subject motor vehicle, and (b) furnished a surety bond issued by Philam life, CD
failed to pay more than two (2) installments, AB went after the surety but he was
only able to obtain three-fourths (3/4) of the total amount still due and owing from
CD. AB seeks your advice on how he might, if at all, recover the deficiency.
How would you counsel AB? Answer:
Yes, he can recover the deficiency. The action of AB to go after the surety
bond cannot be taken to mean a waiver of his right to demand payment for the
whole debt, The amount received from the surety is only payment pro tanto, and an
action may be maintained for a deficiency debt.
Page 359 of 391
15; Antichresis
1995 No. 5:
Olivia owns a vast mango plantation which she can no longer properly
manage due to a lingering illness. Since she is indebted to Peter in the amount of
P500.000.00 she asks Peter to manage the plantation and apply the harvest to the
payment of her obligation to him, principal and interest, until her indebtedness shall
have been fully paid. Peter agrees.
1. What kind of contract is entered Into between Olivia and Peter? Explain.
2. What specific obligations are imposed by law on Peter as a consequence
of their contract?
3. Does the law require any specific form for the validity of their contract?
Explain
4. May Olivia re-acquire the plantation before her entire indebtedness shall
have been fully paid? Explain.
Answer;
1. A contract of antichresis was entered into between Olivia and Peter.
Under Article 2132 of the New Civil Code, by a contract of antichresis the creditor
acquires the right to receive the fruits of an immovable of his debtor, with the
obligation to apply them to the payment of the Interest, and thereafter to the principal
of his credit.
2. Peter must pay taxes and charges upon the land and bear the necessary
expenses for preservation and repair which he may deduct from the fruits. (Art,
2135, NCC)
3. The amount of the principal and interest must be specified in writing,
otherwise the antichresis will be void. (Art. 2134, NCC)
4. No. Art. 2136 specifically provides that the debtor cannot re-acquire the
enjoyment of the immovable without first having totally paid what he owes the
creditor. However, it is potestative on the part of the creditor to do so in order to
exempt him from his obligation under Art. 2135, NCC, The debtor cannot re-acquire
the enjoyment unless Peter compels Olivia to enter again the enjoyment of the
property.
15; Antichresis vs pledge and mortgage
1989 No. 14:
(1) What do you understand by ANTICHRESIS? How is it distinguished from
pledge and mortgage?
Answer;
Antichresis is a contract whereby the creditor acquires the right to receive the
fruits of an immovable of his debtor with the obligation to apply them to the payment
of interest if owing and thereafter to the principal.
Pledge is an accessory and real contract whereby the debtor delivers to the
creditor movable property as security for the performance of a principal obligation
upon the fulfillment of which the thing pledged shall be returned to the debtor.
A real estate mortgage is an accessory contract whereby the debtor
guarantees the performance of the principal obligation by subjecting real property or
real right as security for the performance of such obligation.
Alternative Extended Answer;
Page 360 of 391
By the contract of antichresis the creditor acquires the right to receive the
fruits of an immovable of his debtor, with the obligation to apply them to the payment
of the interest, if owing, and thereafter to the principal of his credit.
Antichresis distinguished from pledge;
1. Antichresis is consensual, pledge is a real contract,
2. Antichresis involves real property, pledge involves personal property.
3. In antichresis, the principal and the interest must be provided in writing for
validity. In pledge, the date and description of the pledge must be in a public
instrument to affect third persons.
Antichresis distinguished from mortgage;
1, In antichresis the fruits that are applied to the interest and thereafter to
the principal. In mortgage the fruits are not applied to the principal obligation.
2, In antichresis, the creditor is in possession. In mortgage, the debtor is
in possession.
3. The principal and interest must be in writing for validity. In mortgage,
registration is required to bind third persons.
4, In antichresis, the creditor pays the taxes. In mortgage, taxes are not
imposed on the creditor.
RECOMMENDATION OF THE COMMITTEE:
If the above alternative answer is given, two (2) distinctions for each should
be given full credit.
15; Chattel mortgage
1995 No. 16:
Lawrence, a retired air force captain, decided to go into the air transport
business. He purchased an aircraft in cash except for an outstanding balance of
P500,000.00. He incurred an indebtedness of P300,000.00 for repairs with an
aircraft repair company. He also borrowed P1 Million from a bank for additional
capital and constituted a chattel mortgage on the aircraft to secure the loan.
While on a test flight the aircraft crashed causing physical injuries to a third
party who was awarded damages of P200,000.00.
Lawrence's insurance claim for damage to the aircraft was denied thus
leaving him nothing else but the aircraft which was then valued only at P1 Million.
Lawrence was declared insolvent.
Assuming that the aircraft was sold for Pl Million, give the order of preference
of the creditors of Lawrence and distribute the amount of P1 Million.
Answer:
Assuming that the aircraft was sold for P1 Million, there is no order of
preference. The P1 Million will all go to the bank as a chattel mortgagee because a
chattel mortgage under Art. 2241 (4) NCC. defeats Art. 2244 (12) and (14}. Art. 2241
(3) and (5) are not applicable because the aircraft is no longer In the possession of
the creditor.
15; Mortgage; chattel mortgage vs pledge
1999 No XVI.
(a) Distinguish a contract of chattel mortgage from a contract of pledge.
(2%)
Page 361 of 391
ANSWER:
(a) In a contract of chattel mortgage possession belongs to the creditor,
while in a contract of pledge possession belongs to the debtor.
A chattel mortgage is a formal contract while a pledge is a real contract.
A contract of chattel mortgage must be recorded in a public instrument to
bind third persons while a contract of pledge must be in a public instrument
containing description of the thing pledged and the date thereof to bind third
persons.
15; Mortgage; effect of loss of thing mortgaged upon principal oblig
1977 No. VIII-a
F obtained a loan from W, to secure the payment of which he mortgaged his
sugar crop. F delivered to W the warehouse quedans and authorized W to sell the
sugar in the event of default. The warehouse and the sugar were burned. Who bears
the loss?
Answer
The debtor F bears the loss of the sugar. Although the sugar was
constructively delivered by F to W, it was only a security for the payment of the debt.
The ownership of the sugar remained with F. By reason of the principle of res perit
domino, the owner F should bear the loss.
15; Mortgage; pactum commissorium
1999 No XVI.
(c) X borrowed money from Y and gave a piece of land as security by way
of mortgage. It was expressly agreed between the parties in the mortgage contract
that upon nonpayment of the debt on time by X, the mortgaged land would already
belong to Y. If X defaulted in paying, would Y now become the owner of the
mortgaged land? Why? (3%)
(d) Suppose in the preceding question, the agreement between X and Y was
that if X failed to pay the mortgage debt on time, the debt shall be paid with the land
mortgaged by X to Y. Would your answer be the same as in the preceding question?
Explain. (3%)
ANSWER:
(c) No, Y would not become the owner of the land. The stipulation is in the
nature of pactum commissorium which is prohibited by law. The property should be
sold at public auction and the proceeds thereof applied to the indebtedness. Any
excess shall be given to the mortgagor.
(d) No, the answer would not be the same. This is a valid stipulation and does
not constitute pactum commissorium. In pactum commissorium, the acquisition is
automatic without need of any further action. In the instant problem another act is
required to be performed, namely, the conveyance of the property as payment
(dacion en pago).
15; Mortgage; pactum commissorium
2001 No VIII
To secure a loan obtained from a rural bank, Purita assigned her leasehold
rights over a stall in the public market in favor of the bank. The deed of assignment
provides that in case of default in the payment of the loan, the bank shall have the
right to sell Purita's rights over the market stall as her attorney-in-fact, and to apply
the proceeds to the payment of the loan.
Page 362 of 391
1) Was the assignment of leasehold rights a mortgage or a cession? Why?
(3%)
2) Assuming the assignment to be a mortgage, does the provision giving
the bank the power to sell Purita's rights constitute pactum commissorium or not?
Why? (2%)
SUGGESTED ANSWER
1) The assignment was a mortgage, not a cession, of the leasehold rights.
A cession would have transferred ownership to the bank. However, the grant of
authority to the bank to sell the leasehold rights in case of default is proof that no
such ownership was transferred and that a mere encumbrance was constituted.
There would have been no need for such authority had there been a cession.
2) No, the clause in question is not a pactum commissorium. It is pactum
commissorium when default in the payment of the loan automatically vests
ownership of the encumbered property in the bank. In the problem given, the bank
does not automatically become owner of the property upon default of the mortgagor.
The bank has to sell the property and apply the proceeds to the indebtedness.
15; Mortgage; real & chattel; future loans
1985 No. 10:
B) A constituted in 1980 a real estate mortgage on his lot and a chattel
mortgage on his car to secure the payment of a debt of P200,000.00 which he then
owed to B, as well as other loans he may receive from him in the future. A paid his
debt of P200,000.00 but not the loan of P30,000.00 which he obtained in 1982.
May B foreclose both mortgages to satisfy A's unpaid obligation to him?
Reasons.
Answers:
B) 1. With regard to the real estate mortgage, B can foreclose the same
because it includes future loans. But with regard to the chattel mortgage, B cannot
foreclose because of the affidavit of good faith which requires that it be a just and
valid debt, and, therefore, the chattel mortgage can not cover future loans.
2. The mortgage is indivisible and therefore it answers for both debts.
Therefore, both mortgages can be foreclosed.
15; Mortgage; real estate mortgage vs sale with repurchase
1989 No. 12:
(1) Distinguish between a contract of real estate mortgage and a contract of
sale with right of repurchase.
Answer;
1. Real estate mortgage is an accessory contract. A contract of sale with
right of repurchase is a principal contract.
2. Real estate mortgage involves no transfer of title. A contract of sale
involves a conditional transfer of title.
3. Real estate mortgage involves no transfer of possession. A contract of
sale involves a conditional transfer of possession.
4. In a real estate mortgage the creditor has no rights to the fruits. In a
contract of sale, the vendee is entitled to the fruits.
5. In a real estate mortgage, upon default the creditor is not the owner. In a
contract of sale, upon consolidation, the vendee is the owner.
Page 363 of 391
RECOMMENDATION OF THE COMMITTEE:
Any three (3) of the foregoing distinctions should be given full credit.
15; Mortgage; right of redemption vs equity of redemption
1999 No XVI.
(b) Are the right of redemption and the equity of redemption given by law to
a mortgagor the same? Explain. (2%)
ANSWER:
(b) The equity of redemption is different from the right of redemption. Equity
of redemption is the right of the mortgagor after judgment in a judicial foreclosure to
redeem the property by paying to the court the amount of the judgment debt before
the sale or confirmation of the sale. On the other hand, right of redemption is the
right of the mortgagor to redeem the property sold at an extra-judicial foreclosure by
paying to the buyer in the foreclosure sale the amount paid by the buyer within one
year from such sale.
15; Pledge
1986 No. 16:
Mr. Matunod lent Mr. Maganaka the amount of P1000,000 As security of the
payment of said amount, Maganaka delivered to Matunod two rings in pledge. When
Maganaka failed to pay, Matunod foreclosed, and had the rings sold at auction. The
proceeds of the sale, after deducting expenses, amounted to only P70,000.00.
(a) May Matunod demand the deficiency from Maganaka? Explain.
(b) Assume that the proceeds, after deducting expenses, had come up to
P150,000.00. Would Matunod have been entitled to the excess? Explain.
(c) Suppose the rings, instead of being pledged, had been mortgaged to
Matunod, would Matunod have been entitled to the deficiency if the sale's proceeds
were less than the indebtedness or to the excess, if the proceeds were more?
Explain.
Answer:
The C.C. on pledge provides that the foreclosure of the pledge extinguishes
the principal obligation, whether the proceeds of the sale are more, or less than the
obligation. Hence,
a) Matunod cannot recover the deficiency.
b) Matunod is entitled to keep the excess, unless there is a stipulation to
the contrary.
c) if it is a chattel mortgage, Matunod can still recover the deficiency as
there is no prohibition in the Chattel Mortgage Law similar to pledge and the excess,
if any should be returned to the mortgagor (Maganaka),
15; Pledge
1989 No. 14:
(2) A diamond ring and a female cow were pledged to secure a loan in the
amount of P100,000. The pledge appeared in a public instrument. A month later, the
cow gave birth. When the amount of the loan was not paid upon its maturity date,
the pledged caused to be sold at a public auction the ring, the cow and the cow's
offspring and the amount of P150,000 as realized. The pledgor, upon learning of the
sale, demanded from the pledgee the excess in the price over and above the
amount of the principal obligation, claiming that he is entitled to the excess and that
Page 364 of 391
the offspring was not included in the pledge. The pledgee refused to comply with the
demand. How-would you decide this conflict? Give your reasons.
Answer:
Debtor/pledgor is not entitled to the excess unless the contrary is agreed
upon. The offspring shall pertain to the pledgor but is subject to the pledge if there is
no stipulation to the contrary.
15; Pledge
1994 No. 19:
In 1982. Steve borrowed P400.000.00 from Danny, collateralized by a pledge
of shares of stock of Concepcion Corporation worth P800,000,00, In 1983, because
of the economic crisis, the value of the shares pledged fell to only P100,000.00. Can
Danny demand that Steve surrender the other shares worth P700,000.00?
Alternative Answers:
a) No. Bilateral contracts cannot be changed unilaterally. A pledge is only a
subsidiary contract, and Steve Is still indebted to Danny for the amount of
P400,000.00 despite the fall in the value of the stocks pledged.
b) No. Danny's right as pledgee is to sell the pledged shares at a public
sale and keep the proceeds as collateral for the loan. There is no showing that the
fall in the value of the pledged property was attributable to the pledger's fault or
fraud. On the contrary, the economic crisis was the culprit. Had the pledgee been
deceived as to the substance or quality of the pledged shares of stock, he would
have had the right to claim another thing In their place or to the immediate payment
of the obligation. This is not the case here.
15; Pledge
2004 No. VI
A. ABC loaned to MNO P40,000 for which the latter pledged 400 shares of
stock in XYZ Inc. It was agreed that if the pledgor failed to pay the loan with 10%
yearly interest within four years, the pledgee is authorized to foreclose on the shares
of stock. As required, MNO delivered possession of the shares to ABC with the
understanding that the shares would be returned to MNO upon the payment of the
loan. However, the loan was not paid on time.
A month after 4 years, may the shares of stock pledged be deemed owned by
ABC or not? Reason. (5%)
15; Pledge; effect of loss of thing pledged upon principal obligation:
immediate demandability of principal oblig
1979 No. XVIII
DL borrowed P200,000 from T & Co. with which he imported 400 heads of
breeding cows from-New Zealand. Upon their arrival in the Philippines, the cattle
were sent to the ranch of T & Co. for pasture under the agreement that for as long
as DL has not paid the P200,000.00, he cannot get his cattle from the ranch; that the
cattle is to be taken care of by T & Co.'s personnel there, and that one-half of all the
offspring shall go to T & Co. for these services. Without anybody's fault, all the cattle
died of disease and the P200,000 00 remained unpaid. T & Co. therefore sued DL
for the P200,000.00 plus interest. Will the action prosper? Why?
Answer
The action will prosper. It is submitted that the 400 heads of New Zealand
cows were pledged by DL to T & Co. in order to secure the payment of the loan of
P200,000.00. As an adjunct of the accessory contract of pledge, the parties also
Page 365 of 391
agreed that the cows shall be kept at the ranch of T & Co. for pasture, that the
personnel of the latter shall take care of them, and that for these services, the latter
shall be entitled to one-half of the offspring. Unfortunately, they all died without the
fault of anybody. Now, what is the effect of the loss? Since the loss was due to a
fortuitous event, it is clear that both the accessory contract of pledge and the
corollary contract of services are totally extinguished. The principal contract of loan,
however, still subsists. As a matter of fact, the obligation of DL to pay the loan plus
interest has become immediately demandable (see Art. 1198, par. 3, Civil Code).
15; Pledge; mortgage; antichresis
1996 No. 15;
In the province, a farmer couple borrowed money from the local merchant. To
guarantee payment, they left the Torrens Title of their land with the merchant, for
him to hold until they pay the loan. Is there a -
a) contract of pledge,
b) contract of mortgage,
c) contract of antichresis, or
d) none of the above?
Explain. Answer:
None of the above. There is no pledge because only movable property may
be pledged (Art. 2094. NCC). If at all, there was a pledge of the paper or document
constituting the Torrens Title, as a movable by itself, but not of the land which the
title represents. There is no mortgage because no deed or contract was executed in
the manner required by law for a mortgage (Arts. 2085 to 2092, NCC; 2124 to 2131,
NCC). There is no contract of antichresis because no right to the fruits of the
property was given to the creditor (Art. 2132 NCC).
A contract of simple loan was entered into with security arrangement agreed
upon by the parties which is not one of those mentioned above,
Alternative Answer:
There is a contract of mortgage constituted over the land. There is no
particular form required for the validity of a mortgage of real property. It is not
covered by the statute of frauds in Art. 1403, NCC and even assuming that it is
covered, the delivery of the title to the creditor has taken it out of the coverage
thereof. A contract of mortgage of real property Is consensual and is binding on the
parties despite absence of writing. However, third parties are not bound because of
the absence of a written instrument evidencing the mortgage and, therefore the
absence of registration. But this does not affect the validity of the mortgage between
the parties (Art. 2125, NCC), The creditor may compel the debtor to execute the
mortgage in a public document in order to allow its registration (Art. 1357.NCC in
relation to Art. 1358. NCC).
15; Pledge; use of the thing pledged
1984 No. 17
On January 1, 19S3, A borrowed P10,000 from B payable on December 1,
1983. As security therefore, A pledged his car to B with an agreement that B could
use it. On June 30, 1983, A offered to pay the loan in full and asked for the return of
his car.
Can A compel B to accept the payment and to return the car? Why?
Answer:
Page 366 of 391
A. Furnished by Office of Justice Palma,
No. Under the agreement with A, B is authorized to use the car. The creditor
may use the thing pledged with the consent of the owner (Art. 2104). A period for the
payment of the obligation was also stipulated. Under Article 1196, it is presumed
that whenever a period is designated, it is presumed to have been established for
the benefit of both the creditors and the debtor. Hence, A cannot prepay the loan
and demand the return of the pledged property until the term had arrived.
B. Comments and Suggested Answer
We agree with the answer of the Bar Examiner.
Page 367 of 391
16; Quasi-contracts
1989 No. 8:
(1) Distinguish an implied contract from a quasi-contract.
Answer:
Any of the following answers should be given full credit:
A. An implied contract requires consent of the parties. A quasi-contract is
not predicated on consent, being a unilateral act.
B. The basis of an implied contract is the will of the parties. The basis of a
quasi-contract is law to the end that there be no unjust situation.
16; Quasi-contracts, negotiorium gestio;liabilities of a gestor
1985 No. 13
C) While A was abroad, the manager of his factory suddenly died and B and
C, A's friends, took over its management, without his knowledge. However, since
they were businessmen themselves, they had to entrust most of their duties to X and
as a result, the factory suffered considerable loss.
Are they liable for said losses? If they are, why and what is the nature of their
liability?
Answers:
C) 1. B and C are solidarity liable for the losses. They are so bound under the
law on negotiorum gestio and must comply with the duties of a gestor in good faith.
2. B and C are only jointly liable because the assumption of the negotiorum
gestio was not to avoid an imminent danger.
3. The law on negotiorum gestio which should apply, renders an officious
manager, who delegates to another person all or some of his duties, liable for the
acts of the delegate. The nature of their liability is solidary. (Art. 2146).
16; Quasi-contracts; negotiorium gestio
1992 No 13:
In fear of reprisals from lawless elements besieging his barangay, X
abandoned his fishpond, fled to Manila and left for Europe. Seeking that the fish in
the fishpond were ready for harvest, Y, who is in the business of managing
fishponds on a commission basis, took possession of the property, harvested the
fish and sold the entire harvest to Z.
Thereafter, Y borrowed money from W and used the money to buy new
supplies of fish fry and to prepare the fishpond for the next crop.
a) What is the Juridical relation between X and Y during X's absence?
b) Upon the return of X to the barangay. what are the obligations of Y to X
as regards the contract with Z?
c) Upon X's return, what are the obligations of X as regards Y's contract with
W?
d) What legal effects will result if X expressly ratifies Y's management and
what would be the obligations of X in favor of Y?
Explain all your answers.
Answer;
Page 368 of 391
(a) The juridical relation is that of the quasi-contract of "negotiorum gestio". Y
is the "gestor" or "officious manager" and X is the "owner" (Art. 2144, Civil Code).
(b) Y must render an account of his operations and deliver to X the price he
received for the sale of the harvested fish (Art, 2145, Civil Code).
(c) X must pay the loan obtained by Y from W because X must answer for
obligations contracted with third persons in the interest of the owner (Art. 2150, Civil
Code),
(d) Express ratification by X provides the effects of an express agency and
X is liable to pay the commissions habitually received by the gestor as manager (Art.
2149, Civil Code).
16; Quasi-contracts; negotiorium gestio
1993 No. 13:
In September, 1972, upon declaration of martial rule in the Philippines. A,
together with his wife and children. disappeared from his residence along A. Mabini
Street. Ermita, Manila. B, his immediate neighbor, noticing that mysterious
disappearance of A and his family, closed the doors and windows of his house to
prevent it from being burglarized. Years passed without B hearing from A and his
family, B continued taking care of A's house, even causing minor repairs to be done
at his house to preserve it. In 1976, when business began to perk up in the area, an
enterprising man. C, approached B and proposed that they build stores at the
ground floor of the house and convert its second floor into a pension house. B
agreed to Cs proposal and together they spent for the construction of stores at the
ground floor and the conversion of the second floor into a pension house. While
construction was going on, fire occurred at a nearby house. The houses at the entire
block, including A's. were burned. After the EDSA revolution in February 1986, A
and his family returned from the United States where they took refuge in 1972. Upon
learning of what happened to his house. A sued B for damages. B pleaded as a
defense that he merely took charge of his house under the principle of negotiorum
gestio. He was not liable as the burning of the house is a fortuitous event.
Is B liable to A for damages under the foregoing circumstances?
Answer:
No. B is not liable for damages, because he is a gestor in negotiorum gestio
(Art. 2144, Civil Code)
Furthermore, B Is not liable to A because Article 2147 of the Civil Code is not
applicable.
B did not undertake risky operations which the owner was not accustomed to
embark upon:
a) he has not preferred his own interest to that of the owner;
b) he has not failed to return the property or business after demand by the
owner; and
c) he has not assumed the management in bad faith. Alternative Answer;
He would be liable under Art. 2147 (1) of the Civil Code, because he used the
property for an operation which the operator is not accustomed to, and in so doing,
he exposed the house to increased risk, namely the operation of a pension house on
the second floor and stores on the first floor
16; Quasi-contracts; negotiorium gestio
1995 No. 11:
Page 369 of 391
Armando owns a row of residential apartments in San Juan, Metro Manila,
which he rents out to tenants. On 1 April 1991 he left for the United States without
appointing any administrator to manage his apartments such that uncollected rentals
accumulated for three (3) years. Amparo, a niece of Armando, concerned with the
interest of her uncle, took it upon herself to administer the property. As a
consequence, she incurred expenses in collecting the rents and in some instances
even spent for necessary repairs to preserve the property.
1. What Juridical relation between Amparo and Armando, if any. has
resulted from Amparo's unilateral act of assuming the administration of Armando's
apartments? Explain.
2. What rights and obligations, if any, does Amparo have under the
circumstances? Explain.
Answer:
1. Negotiorum gestio existed between Amparo and Armando, She
voluntarily took charge of the agency or management of the business or property of
her uncle without any power from her uncle whose property was neglected. She is
called the gestor negotiorum or officious manager, (Art. 2144, NCC)
2. It is recommended by the Committee that an enumeration of any two (2)
obligations and two (2) rights as enumerated in Arts. 2145 to 2152, NCC, would
entitle the examinee to full credit.
Art. 2145. The officious manager shall perform his duties with all the diligence
of a good father of a family, and pay the damages which through his fault or
negligence may be suffered by the owner of the property or business under
management.
The courts may. however. Increase or moderate the indemnity according to
the circumstances of each case.
Art. 2146. If the officious manager delegates to another person all or some of
his duties, he shall be liable for the acts of the delegate, without prejudice to the
direct obligation of the latter toward the owner of the business.
The responsibility of two or more officious managers shall be solidary, unless
management was assumed to save the thing or business from imminent danger.
Art. 2147. The officious manager shall be liable for any fortuitous event:
(1) If he undertakes risky operations which the owner was not accustomed to
embark upon;
(2) If he has preferred his own interest to that of the owner;
(3) If he fails to return the property or business after demand by the owner,
(4) If he assumed the management in bad faith.
Art. 2148. Except when the management was assumed to save the property
or business from imminent danger, the officious manager shall be liable for fortuitous
events
(1) If he is manifestly unfit to carry on the management;
(2) If by his Intervention h e prevented a more competent person from taking
up the management.
Art. 2149. The ratification of the management by the owner of the business
produces the effects of an express agency, even if the business may not have been
successful.
Page 370 of 391
Art. 2150, Although the officious management may not have been expressly
ratified, the owner of the property or business who enjoys the advantages of the
same shall be liable for obligations incurred in his interest, and shall reimburse the
officious manager for the necessary and useful expenses and for the damages
which the latter may have suffered in the performance of his duties.
The same obligation shall be incumbent upon him when the management
had for its purpose the prevention of an imminent and manifest loss, although no
benefit may have been derived.
Art. 2151. Even though the owner did not derive any benefit and there has
been no imminent and manifest danger to the property or business, the owner is
liable as under the first paragraph of the preceding article, provided:
(1) The officious manager has acted in good faith, and
(2) The property or business is Intact, ready to be returned to the owner.
Art. 2152. The officious manager is personally liable for contracts which he
has entered into with third persons, even though he acted in the name of the owner,
and there shall be no right of action between the owner and third persons. These
provisions shall not apply:
(1) If the owner has expressly or tacitly ratified the management, or
(2) When the contract refers to things pertaining to the owner of the
business,
(NOTE: It is recommended by the Committee that an enumeration of any two
(2) obligations and any two (2) rights as enumerated la Arts. 2145 to 2152, NCC
would entitle the examinee to full credit.)
16; Quasi-contracts; 1474; reasonable value
1976 No. IX-a
A sells his 1976 Colt Lancer Sedan to B, a compadre and leaves it to B to
determine the price. If B refuses to fix a price and simply takes the car, is he still
obliged to pay the price? Explain.
Answer
Yes, he is bound to pay the reasonable value thereof on the basis of quasi-
contract.
Article 1474, provides that where the price has not been fixed by the parties
and the thing or any part thereof has been delivered to an appropriate buyer, he
must pay a reasonable price therefore. What is a reasonable price is a question of
fact dependent on the circumstances of each particular case.
16; Quasi-contracts; solutio indebiti
2004 No. V
A. DPO went to a store to buy a pack of cigarettes worth P225.00 only. He
gave the vendor, RRA, a P500-peso bill. The vendor gave him the pack plus
P375.00 change. Was there a discount, an oversight, or an error in the amount
given? What would be DPOs duty, if any, in case of an excess in the amount of
change given by the vendor? How is this situational relationship between DPO and
RRA denominated? Explain. (5%)
16; Quasi-contracts; solutio indebiti
1980 No. II
Page 371 of 391
(a) "C", a Filipino resident of the U.S., sent to his father "D" in Manila
1500.00 through "X" Bank which had a branch in Manila. Due to mistake of the
employees of the Bank, "D" was paid $5,000.00 instead of $500.00. Upon discovery
of the mistake, the Bank demanded from "D" the return of the $4,500.00. "D" refused
and the Bank sued him.
Is the Bank entitled to recover from "D"?
Answer
(a) Yes, the Bank is entitled to recover the $4,500 from "D". We have in this
case an example of a quasi-contract of solutio indebiti which arises whenever a
person unduly delivers a thing through mistake to another who has no right to
demand it (Art 2154, Civil Code). Its requisites are:
(1) There mast be a payment or delivery made by one person to another;
(2) The person who made the payment or delivery was under no obligation
to do so; and
(3) The payment or delivery was made by reason of mistake.
It is obvious that the above requisites are present in the instant case.
16; Quasi-contracts; solutio indebiti, negotiorum gestio
1977 No. XIV-c
What is solutio indebiti, negotiorum gestio? Give an example of each.
Answer
Solutio indebiti refers to the juridical relation which arises whenever a person
unduly delivers a thing through mistake to another who has no right to demand it
(Art. 2154, Civil Code),
Negotiorum gestio refers to the juridical relation which arises when a person
voluntarily takes charge of the agency or management of the business or property of
another, without any power from the latter, as a consequence of which he is obliged
to continue the same until the termination of the affair and its incidents, or to require
tile person concerned to substitute him, if the owner is in a position to do go (Art.
2144, Civil Code),
(NOTE: Just come out with any example applying literally the above
definitions.)
Page 372 of 391
16; Torts and damages
1987 No. 2:
A to was the registered owner of a passenger jeepney, which was involved in
a collision accident with a vegetable truck, resulting in the death of four passengers
and injuries to three. At the time of the accident, Ato was legally married to Maria but
was cohabiting with Tonia in a relationship akin to that of husband and wife.
Could the heirs of the dead passengers and the injured persons recover
damages from:
(a) Ato?
(b) Maria?
(c) Tonia?
Explain each case. Answer:
a. Ato - Yes. Insofar as the dead passengers are concerned, the heirs can
recover damages on the basis of culpa contractual. If the injured persons are also
passengers, Ato is likewise liable on the same basis of culpa contractual. However,
if the injured persons are not passengers, then the liability for damages of Ato will
be on the basis of a quasi-delict.
b. Maria - In view of the ruling in Juaniza v. Jose (89 SCRA 306) that the
passenger jeepney acquired by the husband during an illicit cohabitation with
the paramour is conjugal property, Maria is liable to the same extent as Ato insofar
as the conjugal property in the marriage between Ato and Maria could be
answerable. But as regards her paraphernal property, Maria cannot be held
answerable.
c. Tonia - No, In Juaniza v. Jose the paramour of the owner of the passenger
jeepney that figured in an accident was held to be not a co-owner, and therefore not
liable for damages. Article 144 is inapplicable.
16; Torts and damages
1987 No. 15:
The X Electric Cooperative services a small town where the roads are lined
with lush acacia trees- Normally these trees are pruned before the onset of the rainy
season by the cooperative itself since the power lines of the cooperative are not
infrequently affected by falling branches. This year, for financial reasons, the electric
cooperative omitted the pruning in spite of reminders from the townspeople. In
August this year a strong typhoon hit the town and live wires fell to the ground. While
the cooperative made a preliminary survey of the damages, it did not immediately
take precautionary measures against possible harm. Thus, the attention of one of its
employees was called to the fallen wire in the center of the town. Before the
cooperative could make the necessary repairs, a four-year old boy crossed the
street and was electrocuted by the live wire.
His parents sued the electric cooperative for damages,
(a) If you were counsel for the parents, what arguments would you advance
to support your claim for damages and how much damages would you demand?
(b) If you were counsel for the electric cooperative, what defenses would
you offer?
(c) If you were judge, how would you decide the case?
Answer:
a. The damages that can be claimed by the parents are the following:
Page 373 of 391
1. civil indemnity for death - P30,000.00,
The People of the Philippines v. Leopoldo Traya, L-48065, Jan. 29, 1987;
2. actual and compensatory damages;
3. moral damages for mental anguish;
4. exemplary or corrective damages.
b. As counsel for the electric cooperative, 1 would offer the defense of
"fortuitous event," because the strong typhoon could not be foreseen and even if
foreseen, could not be avoided.
c. As judge, I would rule for the parents. The attention of the cooperative
through its employee was called to the fallen live wire. If there had been care and
diligence, the death could have been avoided. The cooperative could have made the
necessary repairs before the 4 year old boy crossed the street and was electrocuted
by the live wire. It failed to do so, hence it is liable.
16; Torts and damages
1981 No. 18
(a) True or false? -- In cases of defamation, a civil action for civil liability can
be commenced and prosper even while a criminal case is pending.
Answer
(a) True. The civil action for civil liability is an independent civil action under
the Civil Code. (Art. 33).
16; Torts and damages; abuse of right
1984 No. 4
A entered into a twenty-year lease contract with 6 for the use of B's
warehouse in connection with his (A's) business. After ten years, A's business had
so prospered that he needed to move to a larger place. Upon learning of As
intention to transfer his business elsewhere, B offered to terminate the lease
contract, as the rental rate for the warehouse had by then tripled the stipulated
rental. A, whose relationship with B had soured over the years, refused the offer so
as to prevent B from leasing the premises to another party. A padlocked the
warehouse after he had transferred his business to another place, although he
continued paying B the stipulated rental.
Under the circumstances, does B have any cause of action against A?
Explain.
Answer:
A. Furnished by Office of Justice Palma
B can file an action for damages and other relief, including a declaration of
the termination of the lease contract, against B for abuse of rights under Article 19,
which provides that every person must, in the exercise of his rights and in the
performance of his duties, act with justice, give everyone his due, and observe
honesty and good faith.
B. Comments and Suggested Answer
We agree with the answer of the Bar Examiner. However, we suggest that
the following should also be accepted as a correct answer:
First Alternative Answer: B can file an action against A for damages, including
a declaration of the termination of the lease contract. The act of A is not only willful
Page 374 of 391
but also contrary to morals, good customs and public policy (Arts. 21, 2219, No.
(10), Civil Code).
Second Alternative Answer: B can file an action against A for ejectment. A's
act constitutes a violation of one of the essential conditions of the contract -of lease
to use the thing leased for the purpose intended (Art 1673, No. (3), Civil Code).
16; Torts and damages; acts contrary to morals
1996 No. 2:
Rosa was leasing an apartment in the city. Because of the Rent Control Law,
her landlord could not increase the rental as much as he wanted to, nor terminate
her lease as long as she was paying her rent. In order to force her to leave the
premises, the landlord stopped making repairs on the apartment, and caused the
water and electricity services to be disconnected. The difficulty of living without
electricity and running water resulted in Rosa's suffering a nervous breakdown. She
sued the landlord for actual and moral damages.
Will the action prosper? Explain. Answer;
Yes, based on quasi-delict under the human relations provisions of the New
Civil Code (Articles 19, 20 and 21) because the act committed by the lessor is
contrary to morals. Moral damages are recoverable under Article 2219 (10) in
relation to Article 21. Although the action is based on quasi-delict and not on
contract, actual damages may be recovered if the lessee is able to prove the losses
and expenses she suffered.
Alternative Answers;
a) Yes, based on breach of contract. The lessor has the obligation to
undertake repairs to make the apartment habitable and to maintain the lessee in the
peaceful and adequate enjoyment of the lease for the entire duration of the contract
(Article 1654. NCCJ. Since there was willful breach of contract by the lesson the
lessee is entitled to moral damages under Article 3220, NCC. She is also entitled to
actual damages, e. g. loss of income, medical expenses, etc., which she can prove
at the trial.
b) Yes, based on contract and/or on tort. The lessor willfully breached his
obligations under Article 1654. NCC, hence, he is liable for breach of contract. For
such breach, the lessee may recover moral damages under Art. 2220 of the NCC,
and actual damages that she may have suffered on account thereof. And since the
conduct of the lessor was contrary to morals, he may also be held liable for quasi-
delict. The lessee may recover moral damages under Article 2219 (10) in relation to
Article 21, and all actual damages which she may have suffered by reason of such
conduct under Articles 9, 20 and 21.
c) Yes, the action should prosper for both actual and moral damages. In fact,
even exemplary damages and attorney's fees can be claimed by Rosa, on the
authority of Magbanua us, IAC (137 SCRA 328), considering that, as given, the
lessor's willful and illegal act of disconnecting the water and electric services
resulted in Rosa's suffering a nervous breakdown. Art. 20 NCC and Art, 21, NCC
authorize the award of damages for such willful and illegal conduct.
16; Torts and damages; actual and moral damages
2004 No. II
B. DT and MT were prominent members of the frequent travelers club of
FX Airlines. In Hongkong, the couple were assigned seats in Business Class for
which they had bought tickets. On checking in, however, they were told they were
upgraded by computer to First Class for the flight to Manila because the Business
Section was overbooked.
Page 375 of 391
Both refused to transfer despite better seats, food, beverage and other
services in First Class. They said they had guests in Business Class they should
attend to. They felt humiliated, embarrassed and vexed, however, when the
stewardess allegedly threatened to offload them if they did not avail of the upgrade.
Thus they gave in, but during the transfer of luggage DT suffered pain in his arm and
wrist. After arrival in Manila, they demanded an apology from FXs management as
well as indemnity payment. When none was forthcoming, they sued the airline for a
million pesos in damages.
Is the airline liable for actual and moral damages? Why or why not? Explain
briefly. (5%)
16; Torts and damages; breach of contract
1980 No. X
(a) "KK" sued "LL" for damages for breach of contract. At the trial "KK"
proved the breach of contract while "LL" proved that he acted in good faith.
Give the law governing the damages which "KK" is entitled to recover.
If "KK" proved that "LL" acted in bad faith, give the law governing the
damages which "KK" is entitled to recover.
Answer
(a) If "LL" acted in good faith, "KK" is entitled to recover from him as
compensatory damages those consequences which are natural, probable and which
the parties had foreseen at the time when the obligation was constituted.
However, if "LL" acted in bad faith, "KK" is entitled to recover from him as
compensatory damages those consequences which may be reasonably attributed to
the non-performance of the obligation. Additionally, he may even be entitled to
recover moral damages, exemplary damages and attorney's fees,
(NOTE: The above answer is based on Arts. 2201, 2220, 2232, 220S, Civil
Code.)
16; Torts and damages; collapse of structures
1990 No 4:
Mr and Mrs R own a burned-out building, the firewall of which collapsed and
destroyed the shop occupied by the family of Mr and Mrs S, which resulted in
injuries to said couple and the death of their daughter. Mr and Mrs S had been
warned by Mr & Mrs R to vacate the shop in view of its proximity to the weakened
wall but the former failed to do so.
Mr & Mrs S filed against Mr and Mrs R an action for recovery of damages the
former suffered as a result of the collapse of the firewall. In defense, Mr and Mrs R
rely on the doctrine of last clear chance alleging that Mr and Mrs S had the last clear
chance to avoid the accident if only they heeded the formers warning to vacate the
shop, and therefore Mr and Mrs Rs prior negligence should be disregarded.
If you were the judge, how would you decide the case? State your reasons.
Answer:
I would decide in favor of Mr & Mrs S. The proprietor of a building or structure
is responsible for the damages resulting from its total or partial collapse, if it should
be due to the lack of necessary repairs (Art 2190 Civil Code)
As regards the defense of last clear chance, the same is not tenable
because according to the SC in one case (De Roy v CA L-80718, Jan 29, 1988, 157
Page 376 of 391
S 757) the doctrine of last clear chance is not applicable to instances covered by Art
2190 of the Civil Code.
Further, in Phoenix Construction, Inc. v. Intermediate Appellate Court (G.R.
L-65295, March 10, 1987. 148 SCRA 353). the Supreme Court held that the role of
the common law "last clear chance" doctrine in relation to Article 2179 of the Civil
Code is merely to mitigate damages within the context of contributory negligence.
16; Torts and damages; common carriers
1977 No, VII-a
Taxi driver D, driving recklessly, killed pedestrian P and his passenger Y.
Discuss the source of the obligation of D and of his employer to P and to Y, and the
defenses available to the employer.
Answer
There are three interrelated and overlapping sources of the obligation of D
and of his employer. They are:
{1) Under the Revised Penal Code: The heirs of P and Y may proceed
against D and his employer. In this case, the source of the liability of D and of his
employer is the crime committed by D (culpa contractual). The liability of D is direct
and primary (Art. 100, RPC); the liability of his employer is subsidiary (Art. 103,
RPC). The latter cannot relieve himself of liability by proving due diligence of a good
father of a family. This is so because of the nature of the obligation itself.
(2) Under the Civil Code: The heirs of the pedestrian P may proceed against
both D and his employer, or against the latter only. In this case, the source of the
liability of D and his employer is the quasi-delict (culpa aquilana) committed by D
(Arts. 2176, 2180, Civil Code). The liability of both is direct and primary. D's
employer can relieve himself of liability by proving due diligence of a good father of a
family in the selection and supervision of his drivers (Art. 2180, Civil Code).
On the other hand, the heirs of Y may proceed against D's employer only.
The source of the liability of D's employer, in this case, is the breach of his contract
of carriage with Y (culpa contractual). His liability is direct and primary. He cannot
relieve himself of liability by proving due diligence of a good father of a family (Art.
1759, Civil Code). This is so because there is always an implied duty of common
carriers to carry the passenger safely to his place of destination.
16; Torts and damages; damages
1994 No 14:
On January 5, 1992, Nonoy obtained a loan of Pl,000,000.00 from his friend
Rafly. The promissory note did not stipulate any payment for Interest. The note was
due on January 5, 1993 but before this date the two became political enemies.
Nonoy, out of spite, deliberately defaulted in paying the note, thus forcing Raffy to
sue him.
1) What actual damages can Rafly recover?
2) Can Rafly ask for moral damages from Nonoy?
3) Can Rafly ask for nominal damages?
4) Can Raffy ask for temperate damages?
5) Can Rafly ask for attorney's fees?
Answer:
1) Raffy may recover the amount of the promissory note of P1 million,
together with interest at the legal rate from the date of judicial or extrajudicial
Page 377 of 391
demand. In addition, however, inasmuch as the debtor is in bad faith, he is liable
for all damages which may be reasonably attributed to the non-performance of the
obligation. (Art. 2201(2). NCC).
2) Yes, under Article 2220, NCC moral damages are recoverable incase of
breach of contract where the defendant acted fraudulently or in bad faith.
3) Nominal damages may not be recoverable in this case because Rafly
may already be indemnified of his losses with the award of actual and compensatory
damages. Nominal damages are adjudicated only in order that a right of the plaintiff,
which has been violated or invaded by the defendant may be vindicated or
recognized, and not for the purpose of Indemnifying the plaintiff for any loss suffered
by him. (Article 2231. Civil Code)
4) Raffy may ask for, but would most likely not be awarded temperate
damages, for the reason that his actual damages may already be compensated
upon proof thereof with the promissory note. Temperate damages may be
awarded only when the court finds that some pecuniary loss has been suffered but
its amount cannot, from the nature of the case, be proved with certainty. (Article
2224, Civil Code)
5) Yes. under paragraph 2, Article 2208 of the Civil Code, considering that
Nonoy's act or omission has compelled Raffy to litigate to protect his interests.
Furthermore. attorneys' fees may be awarded by the court when it is just and
equitable. (Article 2208(110) Civil Code).
16; Torts and damages; damages arising from death of unborn child
2003 No IV
If a pregnant woman passenger of a but were to suffer an abortion following a
vehicular accident due to the gross negligence of the bus driver, may she and her
husband claim damages from the bus company for the death of their unborn child?
Explain. 5%
Suggested Answer:
No, the spouses cannot recover actual damages in the form of indemnity for
the loss of life of the unborn child. This is because the unborn child is not yet
considered a person and the law allows indemnity only for loss of life of person. The
mother, however may recover damages for the bodily injury she suffered from the
loss of the fetus which is considered part of her internal organ. The parents may
also recover damages for injuries that are inflicted directly upon them, e.g., moral
damages for mental anguish that attended the loss of the unborn child. Since there
is gross negligence, exemplary damages can also be recovered. (Gelus v. CA, 2
SCRA 801 [1961])
16; Torts and damages; damages; loss of an unborn child
1991 No 2:
On her third month of pregnancy, Rosemarie, married to Boy, for reasons
known only to her, and without informing Boy, went to the clinic of X, a known
abortionist, who for a fee, removed and expelled the fetus from her womb, Boy
learned of the abortion six (6) months later.
Availing of that portion of Section 12 of Article II of the 1987 Constitution
which reads;
The State x xx shall equally protect the life of the mother and the life of the
unborn from conception, "xxx" which he claims confers a civil personality on the
unborn from the moment of conception. Boy filed a case for damages against the
abortionist, praying therein that the latter be ordered to pay him: (a) P30,000.00 as
Page 378 of 391
indemnity for the death of the fetus, (b) P100.000.00 as moral damages for the
mental anguish and anxiety he suffered, (c) P50,OOO.OO as exemplary damages,
(d) P20,000.00 as nominal damages, and (e) P25,000.00 as attorney's fees.
(a) Is Boy's interpretation of the above constitutional provision correct?
(b) Is Boy entitled to the foregoing damages and attorney's fees?
(c) Should exemplary damages be proved?
(d) May actual damages be also recovered? If so, what facts should be
alleged and proved?
D. Yes, provided that the pecuniary loss suffered should be substantiated and
duly proved.
16; Torts and damages; damages; moral damages & atty fees
2002 No XVI.
Ortillo contracts Fabricato, Inc. to supply and install tile materials in a building
he is donating to his province. Ortillo pays 50% of the contract price as per
agreement. It is also agreed that the balance would be payable periodically after
every 10% performance until completed. After performing about 93% of the contract,
for which it has been paid an additional 40% as per agreement, Fabricato, Inc. did
not complete the project due to its sudden cessation of operations. Instead,
Fabricato, Inc. demands payment of the last 10% of the contract despite its non-
completion of the project. Ortillo refuses to pay, invoking the stipulation that payment
of the last amount 10% shall be upon completion. Fabricato, Inc. brings suit for the
entire 10%. Plus damages, Ortillo counters with claims for (a) moral damages for
Fabricato, Inc.s unfounded suit which has damaged his reputation as a
philanthropist and respect businessman in his community, and (b) attorneys fees.
A. Does Ortillo have a legal basis for his claim for moral damages? (2%)
B. How about his claim for attorneys fees, having hired a lawyer to defend
him? (3%)
SUGGESTED ANSWERS:
A. There is no legal basis to Ortillos claim for moral damages. It does not
fall under the coverage of Article 2219 of the New Civil Code.
B. Ortillo is entitled to attorneys fees because Fabricatos complaint is a
case of malicious prosecution or a clearly unfounded civil action. (Art. 2208 [4] and
[11], NCC).
16; Torts and damages; death indemnity
1994 No. 16;
Johnny Maton's conviction for homicide was affirmed by the Court of Appeals
and. In addition, although the prosecution had not appealed at all. the appellate
court Increased the indemnity for death from P30,000.00 to P50,OOO.OO. On his
appeal to the Supreme Court, among the other things Johnny Maton brought to the
high court's attention, was the increase of indemnity imposed by the Court of
Appeals despite the clear fact that the People had not appealed from the appellate
court's judgment.
Is Johnny Maton correct? Alternative Answers;
a) In Abejam v. Court of Appeals, the Supreme Court said that even if the
issue of damages were not raised by the appellant in the Court of Appeals but the
Court of Appeals in its findings increased the damages, the Supreme Court will not
disturb the findings of the Court of Appeals.
Page 379 of 391
b) No, the contention of the accused is not correct because upon appeal to
the Appellate Court, the court acquired jurisdiction over the entire case, criminal as
well as civil. Since the conviction of homicide had been appealed, there is no finality
in the amount of indemnity because the civil liability arising from the crime and the
judgment on the crime has not yet become final
c) Yes. Since the civil indemnity is an award in the civil action arising from
the criminal offense, the rule that a party cannot be granted affirmative relief unless
he himself has appealed should apply. Therefore, it was error for the Court of
Appeals to have expanded the indemnity since the judgment on the civil liability had
become final.
d) No. Courts can review matters not assigned as errors. (Hydro Resource
vs. CA . 204 SCRA 309).
16; Torts and damages; defense; due diligence in selection
2003 No XVII.
As a result of a collision between the taxicab owned by A and another taxicab
owned by B, X, a passenger of the first taxicab, was seriously injured. X later filed a
criminal action against both drivers.
(b) May both taxicab owners raise the defense of due diligence in the
selection and supervision of their drivers to be absolved from liability for damages to
X? Reason. 5%
SUGGESTED ANSWER:
(b) It depends. If the civil action is based on a quasi-delict the taxicab
owners may raise the defense of diligence of a good father of a family in the
selection and supervision of the driver; if the action against them is based on culpa
contractual or civil liability arising from a crime, they cannot raise the defense.
16; Torts and damages; filing of separate civil action; need for reservation
2003 No XVII.
As a result of a collision between the taxicab owned by A and another taxicab
owned by B, X, a passenger of the first taxicab, was seriously injured. X later filed a
criminal action against both drivers.
(a) Is it necessary for X to reserve his right to institute a civil action for
damages against both taxicab owners before he can file a civil action for damages
against them? Why
SUGGESTED ANSWER:
(a) It depends. If the separate civil action is to recover damages arising
from the criminal act, reservation is necessary. If the civil action against the taxicab
owners is based on culpa contractual, or on quasi-delict, there is no need for
reservation.
ALTERNATIVE ANSWER:
(a) No, such reservation is not necessary. Under Section 1 of Rule 111 of
the 2000 Rules on Criminal Procedure, what is deemed instituted with the criminal
action is only the action to recover civil liability arising from the crime or ex delicto.
All the other civil actions under Articles 32, 33, 34 and 2176 of the New Civil Code
are no longer deemed instituted, and may be filed separately and prosecuted
independently even without any reservation in the criminal action (Section 3, Rule
111, Ibid). The failure to make a reservation in the criminal action is not a waiver of
the right to file a separate and independent civil action based on these articles of the
New Civil Code (Casupanan v. Laroya GR No. 145391, August 26, 2002).
Page 380 of 391
16; Torts and damages; human relations
1975 No. III
The award of moral damages in favor of the husband against the wife is
assailed on the ground that her refusal to perform her wifely duties, her denial of
consortium and desertion of her husband are not included in the enumeration of
cases where moral damage may lie. Is the contention meritorious? Why?
Answer
No, the contention is not meritorious. The right of the aggrieved husband to
recover moral damages would fall under Article 2219 itself, granting moral damages
in the cases enumerated therein which includes Article 21 and analogous cases.
(Tenchavez v. Escano, 15 SCRA 3557)
In the case of Tenchavez v. Escano, the husband was allowed to recover
moral damages from his wife on the ground of desertion.
16; Torts and damages; human relations
1977 No. II-a
Mention four (4) acts in human relations though they may not constitute a
criminal offense shall produce a cause of action for damages, prevention and other
relief.
Answer
The following and similar acts, though they may not constitute a criminal
offense; shall produce a cause of action for damages, prevention and other relief;
(1) Prying into the privacy of another's residence;
(2) Meddling with or distributing the private life or family relations of another;
(3) Intriguing to cause another to be alienated from his friends; and
(4) Vexing or humiliating another on account of his .religious beliefs, lowly
station in life, place of birth, physical defect, or other personal condition. (Art. 26,
Civil Code)
16; Torts and damages; human relations
1982 No. 11
"A" courted "B", a beautiful girl, 25 years old. Because of "A's" persistence
and repeated promises to marry her, she submitted her body to him. After "A" had
satisfied his lust, he became indifferent to the girl. Finally, he refused to comply with
his promise notwithstanding her demands. "B" filed an action against "A", for moral,
temperate and exemplary damages, alleging that she had been seduced by
defendant's false promises to marry her, and that she suffered "social humiliation,
mental anguish, besmirched reputation, wounded feelings and moral shock. Is "B's"
action tenable? Reason.
Answer
"B's" action is tenable. It must be observed that "A's" acts are clearly willful in
character and, at the same time, contrary to morals, good custom and public policy.
Under the Civil Code, she is entitled to moral, temperate and exemplary damages.
Of course, it must be noted that breach of promise to marry is not actionable. This is
well-settled. But then, it must also be noted that "B's" action is founded not on "A's"
breach of his promise to marry but on the tort or quasi-delict committed by him. It is
clear that everything that he did, including his promise to marry "B", were done
precisely for one purpose to satisfy his lust. Under the Civil Code, such acts are
actionable.
Page 381 of 391
(Note: The above answer-is based on Art. 21 of the Civil Code in relation to
pertinent provision of the Code under the law on damages. The Committee
respectfully recommends that if the bar candidate bases his or her answer on
Tanjanco vs. Court of Appeals, 18 SCRA 994, he or she should properly be credited,
provided that the conditions set forth in said case are presented.)
16; Torts and damages; human relations; abuse of right
1981 No. 1
"S", a fourth year medical student, having failed to pay rent for the room he
occupied for three months, despite repeated demands of "0", the house owner, was
warned that unless he paid not later than 6:00 P.M. that day, he would be locked out
of the house. "S" said that the money from the province had not yet arrived and
asked for an extension of ten days, as the semester was ending and it was
examination time. "O" refused.
When "S" got back from school at 9:00 P.M.. the house was locked and all
his clothes, toothbrush, etc., were outside the house, but his books and notes were
kept by "O" until he could pay his rent. "S" became angry and started banging the
door and called "O" and his mother ugly names, which was heard by the neighbors.
"S" was so upset that he failed in the examinations.
a) "S" sued "0" for damages. Can he recover? Reasons,
b) Suppose that "O" hired you as his lawyer, what defense would you invoke
for him, if any? Explain.
Answer
(a) Yes, "S" can recover damages. There is a clear abuse of rights on the
part of "O". He did not act with justice, he did not give "S" his due and he did not
observe honesty and good faith. His act is also willful thus causing injury to "S" in a
manner that is contrary to morals, good customs and public policy. "S" had already
explained to him that money from the province was forthcoming. He, therefore,
asked for an extension of ten days within which to pay his rent. He also explained
that it was the end of the semester and it was examination time. Despite this
explanation and plea, his room was locked, his personal belongings were placed
outside the house, and worst of all, his books and notes were kept by "O" until he
could pay his rent. As a result, he failed in the examinations. Under the Civil Code,
"O" is clearly liable for damages.
(Note: The above answer is based on Arts. 19 and 21 of the Civil Code. The
Committee, however, respectfully recommends that if the bar candidate continues
his answer by discussing the damages recoverable, such as moral damages under
Art. 2219 and exemplary damages under Art. 2219, and invokes recent decisions
such as Manila Gas Corp. vs. CA, Oct. 30, 1980; Grand Union Supermarket vs.
Espino, Dec. 28, 1979, and others, he should be properly credited.)
(b) I would invoke as defenses the oral defamation committed by "S" against
both "0" and the latter's mother as well as "S's" contributory fault.
It must be observed that "S", in anger, called "O" and his mother ugly names,
which was heard by the neighbors. There is a clear case of oral defamation, at least,
as far as "O's" mother is concerned. She is absolutely innocent.
It must also be observed that there was contributory fault in the part of "S".
He failed to pay his rent for three months despite repeated demands of "O". Under
the law on quasi-delicts and under the law on damages, the amount of damages
recoverable should be reduced.
Page 382 of 391
(Note' The above answer is based on general principles of the law on quasi-
delicts as well as on Arts. 2179, 2214 and other provisions of the Civil Code. Even
the decisions in Manila Gas Corporation (supra) and Grand Union Supermarket
(supra) with respect to contributory fault or negligence may be invoked.)
16; Torts and damages; insurance; subrogation
1987 No. 6.
M/S Philippines, operated by United Shipping Lines, loaded in Japan for
shipment to Manila 50 crates of pipes consigned to Standard Blooming Mills. The
shipment was insured against marine risks with Marine Insurance Company.
Enroute, the ship caught fire resulting in the total-loss of ship and cargo. The
insurance company paid the consignee and thereafter sought recovery and
reimbursement from the United Shipping Lines as subrogee unto the rights of the
insured. Evidence was presented establishing the fact that from the time the goods
were stored in the ship's hatch, no regular inspection was made during the voyage
such that the fire must have started 24 hours before it was noticed.
Could the insurance company claim reimbursement of the amount it had paid
its insured from the United Shipping Lines? Explain.
Answer:
Yes. Under Article 2207, the insurer is subrogated to the rights of the insured
against the wrongdoer or the person who violated the contract when the insurer
pays or indemnifies the insured for the injury or loss arising out of the wrong or
breach of contract complained of. There being a breach of contract of carriage in
view of total loss of the cargo insured, Marine Insurance Company may claim
reimbursement of the amount paid the insured from the United Shipping Lines.
16; Torts and damages; loss of an unborn child
1986 No. 4:
Mrs. Napintas, five months pregnant, had to undergo an emergency
appendectomy but, in the course of the surgery, through the negligence of the
surgeon, she suffered an abortion.
Mr. Napintas, as father, filed a suit for damages against the bungling surgeon
for the loss of the child. Will the suit prosper? Is it possible for Mr, Napintas to get
damages of some sort? Explain.
Answer:
The action for damages for loss of the child (fetus) will not prosper because
as held by the Court in the case of Geluz v. C.A., the fetus is not a person. But moral
damages may be recovered for mental anguish for loss of parental expectancy.
Perhaps, even exemplary damages may be recovered.
Answer - It would be possible for Mr. Napintas to claim moral damages for
whatever hurt feelings and remorse he may have felt by reason of the loss of the
child which was a result of the negligent act of the doctor.
The act of negligence would be the basis for damages that he may claim.
As for the child per se, no damages may be claimed on behalf of the child or
for the death of the child as civil personality begins from the moment of birth. Here,
the child was never born.
Answer The suit instituted, by Mr. Napintas will not prosper. The basis of
the suit is the loss or death of the child. True, damages may be awarded for death
caused by a crime or a quasi-delict. However, in the instant case, the unborn child
never died because it never acquired a juridical personality.
Page 383 of 391
The law expressly limits the provisional personality of a conceived child by
imposing the condition that the child should be subsequently born alive. Here, the
child was not alive when separated from its mother's womb.
Although Mr. Napintas cannot recover damages from the bungling surgeon
for the loss or death of the unborn child, it is submitted that he can recover actual
damages, moral damages, exemplary damages, and attorney's fees, but the basis
will be the quasi-delict committed by the surgeon.
(Note The first paragraph of the above answer is based upon the case of
Geluz vs. CA. 2 SCRA 801, and upon Art. 40 and 41 of the Civil Code in relation to
Art. 2206 of the same Code, white the second paragraph is based upon the law on
quasi-delicts and the law on damages.)
16; Torts and damages; product liability
1976 No, X-a
X, a customer, drinks a bottle of soft drink in a restaurant and is hospitalized
due to harmful substances in the beverage. May the customer sue the manufacturer
for damages, although there is no contractual relation between the customer and the
manufacturers?
Answer
Yes, the liability is based on tort. More specifically, Article 2187 provides that
manufacturers and processors of foodstuffs, drinks, toilet articles and similar goods
shall be liable for death or injuries caused by any noxious or harmful substances
used, although no contractual relation exists between them and the consumers.
16; Torts and damages; quasi- delict
1988 No. 13:
(3) What are the requisites in order that the defendant can be held liable for
damages in a quasi-delict case?
Answer:
(c) In actions based on quasi-delicts, before the person injured can recover
damages from the defendant, it is necessary that he must be able to prove the
following facts:
(1) The fault or negligence of the defendant;
(2) The damages suffered or incurred by the plaintiff; and
(3) The relation of cause and effect between the fault or negligence of the
defendant and the damage incurred by the plaintiff. (Taylor vs. Manila Electric Co.,
16 Phil. 8.)
16; Torts and damages; quasi-delict
1992 No 10:
As the result of a collision between a public service passenger bus and a
cargo truck owned by D, X sustained physical injuries and V died. Both X and Y
were passengers of the bus. Both drivers were at fault, and so X and Z, the only heir
and legitimate child of the deceased Y, sued the owners of both vehicles.
a) May the owner of the bus raise the defense of having exercised the
diligence of a good father of a family?
b) May D raise the same defense?
c) May X claim moral damages from both defendants?
Page 384 of 391
d) May Z claim moral damages from both defendants? Give reasons for all
your answers,
Answer:
(a) No. The owner of the bus cannot raise the defense because the carrier's
liability is based on breach of contract
(b) Yes. D can raise the defense because his liability is based on a quasi-
delict.
(c) Because X suffered physical Injuries, X can claim moral damages
against D, But as against the owner of the bus. X can claim moral damages only if X
proves reckless negligence of the carrier amounting to fraud.
(d) Z can claim moral damages against both defendants because the rules
on damages arising from death due to a quasi-delict are also applicable to death of
a passenger caused by breach of contract by a common carrier (Arts. 1755. 1756,
1764, 2206 and 2219. Civil Code).
16; Torts and damages; vicarious liability
2004 No. V
B. OJ was employed as professional driver of MM Transit bus owned by Mr.
BT. In the course of his work, OJ hit a pedestrian who was seriously injured and
later died in the hospital as a result of the accident. The victims heirs sued the
driver and the owner of the bus for damages.
Is there a presumption in this case that Mr. BT, the owner, had been
negligent? If so, is the presumption absolute or not? Explain. (5%)
16; Torts and damages; vicarious liability
1991 No 16:
Romano was bumped by a minivan owned by the Solomon School of
Practical Arts (SSPA). The minivan was driven by Peter, a student assistant whose
assignment was to clean the school passageways dally one hour before and one
hour after regular classes, in exchange for free tuition. Peter was able to drive the
school vehicle after persuading the regular driver, Paul, to turn over the wheel to him
(Peter). Romano suffered serious physical Injuries, The accident happened at night
when only one headlight of the vehicle was functioning and Peter only had a student
driver's permit.
As a consequence, Peter was convicted in the criminal case. Thereafter,
Romano sued for damages against Peter and SSPA.
(a) Will the action for damages against Peter and SSPA prosper?
(b) Will your answer be the same if, Paul, the regular driver, was impleaded
as party defendant for allowing Peter to drive the minivan without a regular driver's
license.
(c) Is the exercise of due diligence in the selection and supervision of Peter
and Paul a material issue to be resolved in this case?
Answer:
A. Yes. It will prosper (Art, 2180) because at the time he drove the vehicle,
he was not performing his assigned tasks as provided for by Art. 2180. With respect
to SSPA, it is not liable for the acts of Peter because the latter was not an employee
as held by Supreme Court in Filamer Christian Institute vs. CA. (190 SCRA 485).
Peter belongs to a special category of students who render service to the
school in exchange for free tuition fees.
Page 385 of 391
B. I would maintain the same answer because the incident did not occur
while the employee was in the performance of his duty as such employee. The
incident occurred at night time, and. in any case, there was no indication in the
problem that he was performing his duties as a driver.
C. In the case of Peter, if he were to be considered as employee, the
exercise of due diligence in the selection and supervision of peter would not be a
material issue since the conviction of Peter would result in a subsidiary liability
where the defense would not be available by the employer.
In the case of Paul, since the basis of subsidiary liability is the pater familias
rule under Art. 2180, the defense of selection and supervision of the employee
would be a valid defense.
Alternative Answer:
C. In the case of Peter, if he were to be considered an employee, the
exercise of due diligence in the selection and supervision of Peter would not be a
material issue since the conviction of Peter would result in a subsidiary liability
where the defense would not be available by the employer,
In the case of Paul, since he was in the performance of his work at the time
the incident occurred, the school may be held subsidiarily liable not because of the
conviction of Peter, but because of the negligence of Paul under Art. 2180.
16; Torts; defenses; fortuitous event
2002 No XVII.
A van owned by Orlando and driven by Diego, while negotiating a downhill
slope of a city road, suddenly gained speed, obviously beyond the authorized limit in
the are, and bumped a car in front of it, causing severed damage to the care and
serious injuries to its passengers. Orlando was not in the car at the time of the
incident. The car owner and the injured passengers sued Orlando and Diego for
damages caused by Diegos negligence. In their defense, Diego claims that the
downhill slope caused the van to gain speed and that, as he stepped on the brakes
to check the acceleration, the brakes locked, causing the van to go even faster and
eventually to hit the car in front of it. Orlando and Diego contend that the sudden
malfunction of the vans brake system is a fortuitous even and that, therefore, they
are exempt from any liability.
A. Is this contention tenable? Explain. (2%)
SUGGESTED ANSWERS:
A. No. Mechanical defects of a motor vehicle do not constitute fortuitous
event, since the presence of such defects would have been readily detected by
diligent maintenance check. The failure to maintain the vehicle in safe running
condition constitutes negligence.
16; Torts; liability of employer for damage caused by employees; defense of
due diligence
1997 No. 19;
(a) When would an employer's liability for damage, caused by an employee
in the performance of his assigned tasks, be primary and when would it be
subsidiary in nature?
(b) Would the defense of due diligence in the selection and supervision of
the employee be available to the employer in both instances?
Answer:
Page 386 of 391
(a) The employer's liability for damage based on culpa aquiliana under Art,
2176 and 2180 of the Civil Code is primary; while that under Art. 103 of the Revised
Penal Code is subsidiary.
(b) The defense of diligence in the selection and supervision of the employee
under Article 2180 of the Civil Code is available only to those primarily liable
thereunder, but not to those subsidiarily liable under Article 103 of the Revised
Penal Code (Yumul vs. Juliano, 72 Phil. 94).
16; Torts; motor vehicle mishaps; liability of owner who was in the vehicle
1998 No XIII.
A Gallant driven by John and owned by Art, and a Corolla driven by its owner,
Gina, collided somewhere along Adriatico Street. As a result of the accident, Gina
had a concussion. Subsequently. Gina brought an action for damages against John
and Art. There is no doubt that the collision is due to John's negligence. Can Art,
who was in the vehicle at the time of the accident, be held solidarily liable with his
driver, John? (5%)
Answer:
Yes. Art may be held solidary liable with John, if it was proven that the former
could have prevented the misfortune with the use of due diligence. Article 2184 of
the Civil Code states: "In motor mishaps, the owner is solidary liable with his driver,
if the former, who was in the vehicle, could have, by the use of due diligence,
prevented the misfortune, x x x"
Alternative Answer:
1. It depends. The Supreme Court in Chapman vs, Underwood (27 Phil 374),
held: "An owner who sits in his automobile, or other vehicle, and permits his driver to
continue in a violation of law by the performance of negligent acts, after he has had
a reasonable opportunity to observe them and to direct that the driver cease
therefrom, becomes himself responsible for such acts, x x x On the other hand, if the
driver, by a sudden act of negligence, and without the owner having a reasonable
opportunity to prevent the act or its continuance, injures a person or violates the
criminal law, the owner of the automobile, although present therein at the time the
act was committed is not responsible, either civilly or criminally, therefor. The act
complained of must be continued in the presence of the owner for such a length of
time that the owner, by his acquiescence, makes his driver's act his own."
16; Torts; motor vehicle mishaps; solidary liability of owner who was present
2002 No XVII.
C. Does the presence of the owner inside the vehicle causing damage to
a third party affect his liability for his drivers negligence? Explain (2%)
SUGGESTED ANSWERS:
C. In mot or vehicle mishaps, the owner is made solidarily liable with his
driver if he (the owner) was in the vehicle and could have, by the use of due
diligence, prevented the mishap. (Caedo v. Yu Khe Thai, 26 SCRA 410 [1968]).
However, this question has no factual basis in the problem given, in view of the
express given fact that Orlando was not in the car at the time of the incident.
16; Torts; motor vehicle mishaps; solidary liability of owner who was in the
vehicle
1996 No. 16:
Marcial, who does not know how to drive, has always been driven by Ben, his
driver of ten years whom he had chosen carefully and has never figured in a
Page 387 of 391
vehicular mishap. One day, Marcial was riding at the back seat of his Mercedes
Benz being driven along EDSA by Ben. Absorbed in reading a book, Marcial did not
notice that they were approaching the corner of Quezon Avenue, when the traffic
light had just turned yellow. Ben suddenly stepped on the gas to cross the
intersection before the traffic light could turn red. But, too late. Midway in the
intersection, the traffic light changed, and a Jeepney full of passengers suddenly
crossed the car's path. A collision between the two vehicles was inevitable. As a
result, several jeepney passengers were seriously Injured. A suit for damages based
on culpa aquiliana was filed against Marcial and Ben, seeking to hold them Jointly
and severally liable for such injuries.
May Marcial be held liable? Explain. Answer:
Marcial may not be liable because under Art. 2184, NCC, the owner who is in
the vehicle is not liable with the driver if by the exercise of due diligence he could
have prevented the injury. The law does not require the owner to supervise the
driver every minute that he was driving. Only when, through his negligence, the
owner has lost an opportunity to prevent the accident would he be liable (Caedo v.
Ytt Khe Thai, 26 SCRA 410 citing Chapman u. Underwood and Manlangit v. Mauler,
250 SCRA 560). In this case, the fact that the owner was absorbed in reading a
book does not conclusively show that he lost the opportunity to prevent the accident
through his negligence.
Alternative Answer;
Yes, Marcial should be held liable. Art. 2164. NCC makes an owner of a
motor vehicle solidarity liable with the driver if, being in the vehicle at the time of the
mishap, he could have prevented it by the exercise of due diligence The traffic
conditions along EDSA at any time of day or night are such as to require the
observance of utmost care and total alertness in view of the large number of
vehicles running at great speed. Marcial was negligent in that he rendered himself
oblivious to the traffic hazards by reading a book Instead of focusing his attention on
the road and supervising the manner in which his car was being driven. Thus he
failed to prevent his driver from attempting to beat the traffic light at the junction of
Quezon Avenue and EDSA, which Marcial, without being a driver himself. could
have easily perceived as a reckless course of conduct.
16; Torts; primary liability vs subsidiary liability of employers
1980 No. X
(b) "MM", driver of "X" Bus Co. drove the bus recklessly and injured "NN", a
pedestrian, "NN" sued "X" Bus Co. for damages. The bus company is liable to "NN"
either under the Revised Penal Code or under the Civil Code.
Explain and distinguish the liability of the bus company under the Revised
Penal Code and under the Civil Code with respect to the nature of such liability and
the defense/s that may be interposed.
Answer
(b) Under the Revised Penal Code, the bus company is merely subsidiarily
liable to "NN". The latter, therefore, must proceed against "MM" criminally. He may
allow the civil action to be impliedly instituted in the criminal case or he may reserve
his right to institute a civil action separately. If "MM" is convicted but is insolvent, the
bus company is subsidiarily liable. May the company relieve itself of liability by
proving due diligence of a good father of family in the selection and supervision of
its drivers? It cannot. The reason is the very nature of the obligation itself. This is
well-settled.
Page 388 of 391
Under the Civil Code, however, the bus company is directly and primarily
liable to "NN" The reason is this. The negligence of "MM" is disputably presumed to
be the negligence of the company. "NN", therefore, may proceed against the
company alone. The basis will be a quasi-delict or culpa, aquiliana. May the
company relieve itself of liability by proving due diligence in the selection and
supervision of its drivers. According to the Civil Code, it may, in such a case, the
presumption of negligence on the part of the company has been overcome.
(NOTE: The above answer is based on Arts. 100 and 103 of the Revised
Penal Code, on Arts. 2176, 2177 and 2180 of the Civil Code and on a long line of
notable decisions rendered by the Supreme Court)
16; Torts; vicarious liability
2002 No XVII.
B. Explain the concept of vicarious liability in quasi-delicts. (1%)
SUGGESTED ANSWERS:
B. The doctrine of vicarious liability is that which renders a person liable
for the negligence of others for whose acts or omission the law makes him
responsible on the theory that they are under his control and supervision.
16; Torts; vicarious liability
2000 No XX.
a) Silvestre leased a car from Avis-Rent-A-Car Co. at the Mactan
International Airport. No sooner had he driven the car outside the airport when, due
to his negligence, he bumped an FX taxi owned and driven by Victor, causing
damage to the latter In the amount of P100,000.00. Victor filed an action for
damages against both Silvestre and Avis, based on quasi-delict. Avis filed a motion
to dismiss the complaint against it on the ground of failure to state a cause of action.
Resolve the motion. (3%)
SUGGESTED ANSWER:
The motion to dismiss should be granted, AVIS is not the employer of
Silvestre; hence, there is no right of action against AVIS under Article 2180 of the
Civil Code. Not being the employer. AVIS has no duty to supervise Silvestre. Neither
has AVIS the duty to observe due diligence in the selection of its customers.
Besides, it was given in the problem that the cause of the accident was the
negligence of Silvestre.
ALTERNATIVE ANSWER:
The motion should be denied. Under the Public Service Law, the registered
owner of a public utility is liable for the damages suffered by third persons through
the use of such public utility. Hence, the cause of action is based in law, the Public
Service Law,
16; Torts; vicarious liability
2001 No XVIII
After working overtime up to midnight, Alberto, an executive of an insurance
company drove a company vehicle to a favorite Videoke bar where he had some
drinks and sang some songs with friends to "unwind". At 2:00 a.m., he drove home,
but in doing so, he bumped a tricycle, resulting in the death of its driver. May the
insurance company be held liable for the negligent act of Alberto? Why?
SUGGESTED ANSWER
Page 389 of 391
The insurance company is not liable because when the accident occurred,
Alberto was not acting within the assigned tasks of his employment.
It is true that under Art. 2180 (par. 5), employers are liable for damages
caused by their employees who were acting within the scope of their assigned tasks.
However, the mere fact that Alberto was using a service vehicle of the employer at
the time of the injurious accident does not necessarily mean that he was operating
the vehicle within the scope of his employment. In Castilex Industrial Corp. v.
Vasquez Jr (321 SCRA393 [1999]). the Supreme Court held that notwithstanding the
fact that the employee did some overtime work for the company, the former was,
nevertheless, engaged in his own affairs or carrying out a personal purpose when he
went to a restaurant at 2:00 a.m. after coming out from work. The time of the
accident (also 2:00 a. m.) was outside normal working hours.
ALTERNATIVE ANSWER:
The insurance company is liable if Alberto was negligent in the operation of
the car and the car was assigned to him for the benefit of the insurance company,
and even though he was not within the scope of his assigned tasks when the
accident happened. In one case decided by the Supreme Court, where an executive
of a pharmaceutical company was given the use of a company car, and after office
hours, the executive made personal use of the car and met an accident, the
employer was also made liable under Art. 2180 of the Civil Code for the injury
caused by the negligent operation of the car by the executive, on the ground that the
car which caused the injury was assigned to the executive by the employer for the
prestige of the company. The insurance company was held liable even though the
employee was not performing within the scope of his assigned tasks when the
accident happened [Valenzuela v. CA, 253 SCRA 3O3 (1996)].
Page 390 of 391
17; Aleatory contracts; gambling
2004 No. I
A. Mr. ZY lost P100,000 in a card game called Russian poker, but he had no
more cash to pay in full the winner at the time the session ended. He promised to
pay PX, the winner, two weeks thereafter. But he failed to do so despite the lapse of
two months, so PX filed in court a suit to collect the amount of P50,000 that he won
but remained unpaid. Will the collection suit against ZY prosper? Could Mrs. ZY file
in turn a suit against PX to recover the P100,000 that her husband lost? Reason.
(5%)
17; Aleatory contracts; gambling
1977 No. I-c
A, a Crispa fan, and B, a Tanduay supporter, watching T.V. last night before
the Crispa-Tanduay match, agreed to give the other P500.00 if his team loses. Both
teams are so evenly matched in playing ability, in both offense and defense as well
as in import players and coaches, that they tied at the end of the regulation time and
the game went to a 5-minute extension period. Crispa finally won over Tanduay
ESQ, 133-110. What is this kind of obligation between A and B? Can it be enforced?
Decide with reasons.
Answer
The obligation between A and B is an obligation arising from a contract which
the Civil Code denominates as an aleatory contract of gambling or betting (Arts.
2013, et seq., Civil Code).
The obligation of B to pay his bet of P500 to A cannot be enforced. True,
under the Civil Code (Arts. 2019, 2020), since basketball is not a game of chance,
the obligation may be enforced, but this is possible only when there is no law which
prohibits betting therein. There is a law which prohibits it. Therefore, the obligation of
B to A is void.
18; Compromises and arbitration
1981 No. 18
(e) True or false? -- A stipulation that the arbitrator's award shall be final is
valid.
Answer
(e) True, According to the Civil Code, any stipulation that the arbitrator's
award or decision shall be final is valid, without prejudice to certain pertinent
provisions on compromise wherein the award may be revoked. (Art. 2044}
(Note: The Committee respectfully submits that an answer of False may also
be correct provided that the bar candidate states the above-stated reason.)
18; Compromises and arbitrations
1977 No. XV-c
What is a compromise? What are its effects? Specify five (5) instances where
it is prohibited by law.
Answer
A compromise is a contract whereby the parties, by making reciprocal
concessions, avoid a litigation or put an end to one already commenced (Art. 2028,
Civil Code).
Page 391 of 391
A compromise has upon the parties the effect and authority of res judicata;
but there shall be no execution except in compliance with a judicial compromise (Art.
2037, Civil Code).
There can be no compromise upon the following:
(1) Criminal liability of the accused in a criminal action;
(2) The civil status of persons;
(3) The validity of a marriage or a legal separation;
(4) Any ground for legal separation;
(5) Future support;
(6) The jurisdiction of courts;
(7) Future legitime. (Arts. 2034, 2035, Civil Code).

Вам также может понравиться